0% found this document useful (1 vote)
785 views292 pages

Krok I 2013-17 Explained

Vitamin B6 should be used to address symptoms of muscle weakness, decreased skin sensitivity, blurred vision, and impaired motor coordination in a patient taking isoniazid for tuberculosis treatment. These symptoms indicate peripheral neuropathy, a side effect of isoniazid that vitamin B6 can help with. Hypoglycemia in a diabetic patient taking insulin and the drug anaprilin is caused by anaprilin's inhibition of glycogenolysis. Pterin derivatives like aminopterin and methotrexate inhibit the enzyme dihydrofolate reductase, inhibiting DNA synthesis. A Mantoux test reaction of swelling, redness and pain is caused by mononuclear cells, T-lymphocytes and

Uploaded by

U1GoKu
Copyright
© © All Rights Reserved
We take content rights seriously. If you suspect this is your content, claim it here.
Available Formats
Download as PDF, TXT or read online on Scribd
0% found this document useful (1 vote)
785 views292 pages

Krok I 2013-17 Explained

Vitamin B6 should be used to address symptoms of muscle weakness, decreased skin sensitivity, blurred vision, and impaired motor coordination in a patient taking isoniazid for tuberculosis treatment. These symptoms indicate peripheral neuropathy, a side effect of isoniazid that vitamin B6 can help with. Hypoglycemia in a diabetic patient taking insulin and the drug anaprilin is caused by anaprilin's inhibition of glycogenolysis. Pterin derivatives like aminopterin and methotrexate inhibit the enzyme dihydrofolate reductase, inhibiting DNA synthesis. A Mantoux test reaction of swelling, redness and pain is caused by mononuclear cells, T-lymphocytes and

Uploaded by

U1GoKu
Copyright
© © All Rights Reserved
We take content rights seriously. If you suspect this is your content, claim it here.
Available Formats
Download as PDF, TXT or read online on Scribd
You are on page 1/ 292

KillingKrok2016 1

KROK I 2013, EXPLAINED


Introduction

This material aims to explain answers and sometimes, point out why the other options are not
the correct ones. We encourage the student preparing for Krok I to avoid memorizing answers
and instead set out to understand questions and the concepts behind them. It would also help a
great deal to seek out topics that occur repeatedly, and study them thoroughly so one is ready
for those questions in whatever form.

Note: All answers are A

1. A patient diagnosed with focal tuberculosis of the upper lobe of the right lung had been
taking isoniazid as a part of combination therapy. After some time, the patient reported of
muscle weakness, decreased skin sensitivity, blurred vision, and impaired motor
coordination. Which vitamin preparation should be used to address these phenomena?

A. Vitamin B6
B. Vitamin A C. Vitamin D
D. Vitamin B12
E. Vitamin C
Answer: The ‘muscle weakness, skin sensitivity and other symptoms are indicative of
peripheral neuropathy, a side effect of Isoniazid. Vitamin B6 (Pyridoxine)aids
neurotransmitter synthesis and Isoniazid also causes its hypovitaminosis.

2. A 60-year-old male patient has a 9-year history of diabetes and takes insulin Semilente
for the correction of hyperglycemia. 10 days ago he began taking Anaprilin for
hypertension. One hour after administration of the antihypertensive drug the patient
developed hypoglycemic coma. What is the mechanism of hypoglycemia in case of
Anaprilin use?
A. Inhibition of glycogenolysis
B. Reduction of glucagon half-life
C. Increase of insulin Semilente half-life
D. Increase of bioavailability of insulin Semilente
E. Decrease in glucose absorption

Answer: Inhibition of glycogenolysis._ Anapirilin is a non-selective beta blocker used both as


an anti-arrhythmic, anti-angina, anti-hypertensive drug. One of the side effects is causing
hypoglycemia in patients with insulin dependent diabetes by inhibiting glycogenolysis.

3. Pterin derivatives (aminopterin and methotrexate) are the inhibitors of dihydrofolate


reductase, so that they inhibit the regeneration of tetrahydrofolic acid from dihydrofolate.

NSA TSMU Academic Team Krok 1 2013


KillingKrok2016 2

These drugs inhibit the intermolecular transfer of monocarbon groups, thus suppressing
the synthesis of the following polymer:
A. DNA
B. Protein
C. Homopolysaccharides
D. Gangliosides
E. Glycosaminoglycans
Answer: DNA. Methotrexate inhibits synthesis of DNA.

4. A child with suspected tuberculosis was given Mantoux test. After 24 hours the site of
the allergen injection got swollen, hyperemic and painful. What are the main components
that determine such response of the body?
A. Mononuclear cells, T-lymphocytes and lymphokines
B. Granulocytes, T-lymphocytes and IgG
C. Plasma cells, T-lymphocytes and Lymphokines
D. B-lymphocytes, IgM
E. Macrophages, B-lymphocytes and monocytes
Answer: Mononuclear cells, T-lymphocytes and lymphokines aid in Antigen uptake.
Antigen here is M. Tuberculosis.

5. Hemoglobin catabolism results in release of iron which is transported to the bone


marrow by a certain transfer protein and used again for the synthesis of hemoglobin.
Specify this transfer protein:

A. Transferrin ( siderophilin )
B. Transcobalamin
C. Haptoglobin
D. Ceruloplasmin
E. Albumin
Answer: Transferrin. Transferrin is a plasma protein found in the blood that carries iron
compound round the body and controls the level of free iron in biological fluids

6. A 12-year-old boy has been hospitalized for suspected food poisoning. The fecal samples
were inoculated on the Endo agar, which resulted in growth of a large number of colorless
colonies. What microorganism is most likely to be EXCLUDED from the list of possible
causative agents of the disease?

A. Escherichia coli
B. Salmonella enteritidis
C. Proteus vulgaris
D. Pseudomonas aeruginosa
E. Yersinia enterocolitica

NSA TSMU Academic Team Krok 1 2013


KillingKrok2016 3

Answer: E. coli. This is because even though t is inoculated in Endo’s agar it forms pink
colonies unlike all other microbes listed.

7. A 23-year-old patient has been admitted to a hospital with a craniocerebral injury. The
patient is in a grave condition. Respiration is characterized by prolonged convulsive
inspiration followed by a short expiration. What kind of respiration is it typical for?
A. Apneustic
B. Gasping breath
C. Kussmaul’s
D. Cheyne-Stokes
E. Biot’s
Answer: Apneustic. This is an abnormal pattern of breathing characterized by deep, gasping
inspiration with a pause at full inspiration followed by brief insufficient.

8. It has been experimentally proven that the excitation of the motor neurons of flexor
muscles is accompanied by the inhibition of the motor neurons of extensor muscles. What
type of inhibition underlies this phenomenon?
A. Reciprocal
B. Inhibition after excitation
C. Pessimal
D. Feedback
E. Lateral

Answer: Reciprocal, Simple because excitation of one group of muscles (flexors) will inhibit
the Extensors.

9. A 3-year-old boy with pronounced hemorrhagic syndrome doesn’t have antihemophilic


globulin A (factor VIII) in the blood plasma. Hemostasis has been impaired at the following
stage:
A. Internal mechanism of prothrombinase activation
B. External mechanism of prothrombinase activation
C. Conversion of prothrombin to thrombin
D. Conversion of fibrinogen to fibrin E. Blood clot retraction
Answer: Factor VIII is involved in the Internal mechanism of Coagulation.

10. A patient got a gunshot wound of hip which damaged the sciatic nerve. Any impact on
the affected limb causes severe, excruciating pain. What mechanism of pain is most likely
in this case?
A. Causalgic

NSA TSMU Academic Team Krok 1 2013


KillingKrok2016 4

B. Reflex
C. Phantom
D. Endorphin hypofunction
E. Enkephalin hypofunction
Answer: Causalgic. It is a neuralgia distinguished by a burning pain along certain nerves.

11. A 60-year-old patient with a long history of stenocardia takes coronarodilator agents.
He has also been administered acetylsalicylic acid to reduce platelet aggregation. What is
the mechanism of antiplatelet action of acetylsalicylic acid?
A. It reduces the activity of cyclooxygenase B. It reduces the activity of phosphodiesterase
C. It enhances the activity of platelet adenylate cyclase
D. It enhances the synthesis of prostacyclin
E. It has membrane stabilizing effect
Answer: It reduces the activity of cyclooxygenase B. It reduces the activity of
phosphodiesterase. Acetlysalicyclic acid inhibit d activity of cyclooxygenase B.

12. A patient with bronchial asthma has developed acute respiratory failure. What kind of
respiratory failure occurs in this case?
A. Obstructive disturbance of alveolar
ventilation
B. Restrictive ventilatory defect
C. Perfusion
D. Diffusion
E. Dysregulation of alveolar ventilation
Answer: Bronchial asthma is an example of an obstructive form of respiratory disease.

13. On the fifth day after the acute blood loss a patient has been diagnosed with
hypochromic anemia. What is the main mechanism of hypochromia development?
A. Release of immature red blood cells from the bone marrow
B. Impaired iron absorption in the intestines
C. Increased destruction of red blood cells in the spleen
D Impaired globin synthesis
E. Increased excretion of body iron
Answer: Release of immature red blood cells from the bone marrow. Hypochromic
anemia is a form of anemia where RBC are paler than normal. Immature red blood cells lack
hemoglobin which is the responsible for the red pigment.

14. A patient with diabetes developed a diabetic coma due to the acid-base imbalance.
Specify the kind of this imbalance:

NSA TSMU Academic Team Krok 1 2013


KillingKrok2016 5

A. Metabolic acidosis
B. Metabolic alkalosis
C. Respiratory acidosis
D. Gaseous alkalosis
E. Non-gaseous alkalosis
Answer: Metabolic acidosis. Presence of ketone bodies causes ketoacidosis which leads to
increased acidosis of the blood.

15. A girl receives antibiotics of the penicillin group for acute bronchitis. On the third day of
treatment she developed allergic dermatitis. Which drug should be administered?
A. Loratadine
B. Cromolyn sodium
C. Beclomethasone
D. Ephedrine hydrochloride
E. Levamisole
Answer: Loratadine is an H1- anti histamine drug used to treat allergies.

16. A female patient has been diagnosed with cervical erosion, which is a precancerous
pathology. What defense mechanism can prevent the development of a tumor?

A. Increase in natural killer level (NK-


cells)
B. High-dose immunological tolerance C. Increase in the activity of lysosomal enzymes
D. Simplification of the antigenic structure of tissues
E. Low-dose immunological tolerance
Answer: Increase of natural killer level. Natural killer cells are cells which play major roles in
fighting both tumor cells and viral infected cells.

17. Microscopy of the coronary artery of a dead 53-year-old patient revealed luminal
occlusion due to a fibrous plaque with some lipids. The most likely form of atherosclerosis
in this case is:
A. Liposclerosis
B. Lipidosis
C. Prelipid stage
D. Atheromatosis
E. Ulceration
Answers: Liposclerosis. Occlusion was due to fibrous plague with LIPIDS.

NSA TSMU Academic Team Krok 1 2013


KillingKrok2016 6

18. Autopsy of the patient revealed bone marrow hyperplasia of tubular and flat bones
(pyoid marrow), splenomegaly (6 kg) and hepatomegaly (5 kg), enlargement of all lymph
node groups. What disease are the identified changes typical for?
A. Chronic myelogenous leukemia
B. Chronic lymphocytic leukemia
C. Multiple myeloma
D. Polycythemia vera
E. Hodgkin’s disease
Answer: CML, the characteristic feature of this disease is Pyoid Marrow(large pale nuclei
possessing neutrophilic qualities)

19. As a result of an injury a patient cannot extend his arm at the elbow. This may cause
abnormal functioning of the following muscle:

A. Musculus triceps brachii


B. Musculus infraspinatus
C. Musculus levator scapulae
D. Musculus teres major
E. Musculus subscapularis

Answer: Triceps muscle is responsible for extension of the arm.

20. A man sitting with his eyes closed, undergoes electroencephalography. What rhythm will
be recorded on the EEG if there is an audible signal?
A. Beta rhythm
B. Theta rhythm
C. Delta rhythm
D. Alpha rhythm
E. Gamma rhythm
Answers: Beta rhythm. This is the frequency of the brain wave normal waking consciousness
with eyes closed.

21. Electrophoretic study of a blood serum sample, taken from the patient with pneumonia,
revealed an increase in one of the protein fractions. Specify this fraction:
A. γ-globulins’
B. Albumins
C. α1-globulins
D. α2-globulins
E. β-globulins
Answer: γ-globulins. IgG also known as gamma globulins produced by plasma cells arise
during bacterial infection to fight foreign bodies.

NSA TSMU Academic Team Krok 1 2013


KillingKrok2016 7

22. Examination of an 18-year-old girl revealed the following features: hypoplasia of the
ovaries, broad shoulders, narrow pelvis, shortening of the lower extremities, "sphinx neck".
Mental development is normal. The girl was diagnosed with Turner’s syndrome. What kind
of chromosome abnormality is it?
A. Monosomy X
B. Trisomy X
C. Trisomy 13
D. Trisomy 18
E. Nullisomy X
Answer: Monosomy X. it is also known as Turner’s syndrome. With karyotype of 47XXX.

23. Hypertrichosis is the Y-linked character. The father has


hypertrichosis, and the mother is healthy. In this family, the probability of having a child
with hypertrichosis is:
A. 0 , 5
B. 0 , 25
C. 0,125
D. 0,625
E. 1
Answer: 0.5 , basically a 50% chance, they can have a girl.

24. A casualty has a fracture in the region of the inner surface of the left ankle. What is the
most likely site for the fracture?
A. Medial malleolus
B. Lower third of the fibula
C. Astragalus
D. Lateral malleolus
E. Calcaneus
Reason: The inner side of the left ankle is called the medial malleolus.

25. Some infectious diseases caused by bacteria are treated with sulfanilamides which
block the synthesis of bacteria growth factor. What is the mechanism of their action?
A. They are antivitamins of para-amino benzoic acid
B. They inhibit the absorption of folic acid
C. They are allosteric enzyme inhibitors
D. They are involved in redox processes
E. They are allosteric enzymes
Reason: Sulfanilamide is an anti-bacterial drug that inhibits then function of PABA (Para
Amino benzoic Acid).

NSA TSMU Academic Team Krok 1 2013


KillingKrok2016 8

26. A 42-year-old male patient with gout has an increased blood uric acid concentration. In
order to reduce the level of uric acid the doctor administered him allopurinol. Allopurinol
is the competitive inhibitor of the following enzyme:
A. Xanthine oxidase
B. Adenosine deaminase
C. Adenine phosphoribosyltransferase
D. Hypoxanthine-phosphoribosyltransferase
E. Guanine deaminase

27. A 40-year-old female patient diagnosed with acute pancreatitis has been delivered to
the admission department of a regional hospital. What drug should be administered the
patient in the first place?
A. Contrycal
B. Platyphyllin
C. Atropine
D. Metacin
E. Pirenzepine
Reason: Contrycal is a choice of drug for treating acute pancreatitis. JUST ACUTE.

28. A patient consulted a doctor about being unable to abduct his right arm after a past
trauma. Examination revealed that the passive movements were not limited. The patient
was found to have the atrophy of the deltoid muscle. What nerve is damaged?
A. Axillary
B. Radial
C. Ulnar
D. Median
E. Suprascapular
Reason: The axillary nerve innervates the posterior part of the deltoid muscle.

29. After a trauma of the upper third of the anterior forearm a patient exhibits difficult
pronation, weakening of palmar flexor muscles and impaired skin sensitivity of 1-3 fingers.
Which nerve has been damaged?

A. n. medianus
B. n. musculocutaneus
C. n. ulnaris
D. n. cutaneus antebrachii medialis
E. n. radialis

Reason: The median nerve innervates the first three fingers in the palmar surface.

NSA TSMU Academic Team Krok 1 2013


KillingKrok2016 9

30. A 38-year-old female patient complains of general weakness, cardiac pain, increased
appetite, no menstruation. Objectively: the height is 166 cm, weight 108 kg, the patient has
moon-shaped face, subcutaneous fat is deposited mainly in the upper body, torso and hips.
There are also blood-red streaks. Ps- 62/min, AP160/105 mm Hg. Which of the following
diseases is the described pattern of obesity most typical for?
A. Cushing pituitary basophilism
B. Alimentary obesity
C. Myxedema
D. Insulinoma
E. Babinski-Frohlich syndrome
Reason: Cushing pituitary basophilisim- caused by an ACTH (Adrenocorticotropic Hormone)
secreting basophil adenoma of the pituitary gland. One of the main clinical manifestation of
Cushing’s disease is MOON SHAPED FACE.

31. A 60-year-old patient with a long history of atherosclerosis and a previous myocardial
infarction developed an attack of retrosternal pain. 3 days later the patient was hospitalized
and then died of progressive cardiovascular insufficiency. At autopsy a white fibrous
depressed area about 3 cm in diameter with clear boundaries was found in the posterior
wall of the left ventricle and interventricular septum. The dissector evaluated these changes
as:
A. Focal cardiosclerosis
B. Myocardial ischemia
C. Myocardial infarction
D. Myocarditis
E. Myocardial degeneration
Reason: This characterized y a clear boundary and uniform distribution of fibrous tissue in a
defined area.

32. Measurements of the arterial pCO2 and pO2 during an attack of bronchial asthma
revealed hypercapnia and hypoxemia respectively. What kind of hypoxia occurred in this
case?

A. Respiratory
B. Hemic
C. Circulatory
D. Tissue
E. Histotoxic
Reason: Only Respiratory hypoxia can occur in Bronchial Asthma.

33. A female patient with bronchial asthma had taken prednisolone tablets (1 tablet 3
times a day) for 2 months. Due to a significant improvement of her condition the patient
suddenly stopped taking it. What complication is likely to develop in this case?

NSA TSMU Academic Team Krok 1 2013


KillingKrok2016 10

A. Withdrawal syndrome
B. Cushing’s syndrome
C. Gastrorrhagia
D. Upper body obesity
E. Hypotension
Reason: also called a discontinuation syndrome is a set of symptoms occurring in
discontinuation or dosage reduction of some types of medications.

34. A patient with suspected dysentery has been admitted to the infectious diseases
hospital. Which basic method of laboratory diagnosis must be applied in the first place?
A. Bacteriological
B. Serological
C. Allergic
D. Biological
E. Microscopic
Reason: To diagnose Dysentery caused by S. dysentery, a Bacteriological exam should be
conducted (culture or smear).

35. During a surgery with the use of hygronium the patient had an abrupt fall in blood
pressure. Blood pressure can be normalized by the representatives of the following drug
group:
A. α-adrenergic agonists
B. α- blockers C. Ganglionic blockers
D. M-cholinergic agents
E. N-cholinergic agents
Reason: Hygronium provide short term ganglion-blocking action and is suitable for
anesthetic practice for controlled hypotension.

36. A patient with respiratory failure has blood pH of 7,35. pCO2 test revealed hypercapnia.
Urine pH test revealed an increase in the urine acidity. What form of acid-base imbalance is
the case?
A. Compensated respiratory acidosis
B. Compensated metabolic acidosis
C. Decompensated metabolic acidosis
D. Compensated respiratory alkalosis
E. Decompensated respiratory alkalosis
Reason: Hypercapnia shows Respiratory acidosis, a pH of 7.35 shows compensation.

NSA TSMU Academic Team Krok 1 2013


KillingKrok2016 11

37. On examination a patient was found to have medial strabismus, the inward deviation of
the eyeball and inability to abduct the eyeball outwards. What nerve is damaged?
A. Abducent
B. Oculomotor
C. Ocular
D. Trochlear
E. Visual
Reason: Easy, Abducens nerve is responsible for abduction of eyeball.

38. A patient with a dislocated shoulder had been admitted to a hospital. With the purpose
of skeletal muscle relaxation he was given an injection of relaxant dithylinum acting
normally 5-7 minutes. However, the effect of dithylinum in this patient lasted up to 8
hours. What is the most likely cause of the prolonged effect of dithylinum in this patient?
A. Genetic deficiency of blood cholinesterase
B. Reduced activity of microsomal liver enzymes C. Reduced drug excretion
D. Material accumulation of the drug
E. Potentiation by another drug
Reason: Dithyilinum is a myorelaxant which destroys pseudocholinesterase and breaks down
into choline and succinic acid.

39. As a result of an injury of the knee joint a patient shows a drawer sign, that is the
anterior and posterior displacement of the tibia relative to the femur. What ligaments are
damaged?
A. Cruciate ligaments
B. Arcuate popliteal ligaments
C. Oblique popliteal ligament
D. Interosseous membrane
E. Collateral ligaments
Reason: “drawer sign”- the ability to move tibia forward when the femur is fixed is a positive
drawer sign which indicates rupture of only cruciate ligaments.

40. The neurosurgical department has admitted a 54-year-old male complaining of no


sensitivity in the lower eyelid skin, lateral surface of nose, upper lip. On examination the
physician revealed the inflammation of the second branch of the trigeminal nerve. This
branch comes out of the skull through the following foramen:
A. Round foramen
B. Lacerated foramen
C. Oval foramen
D. Spinous foramen
E. Superior orbital fissure

NSA TSMU Academic Team Krok 1 2013


KillingKrok2016 12

Answer: The Maxillary nerve leaves the cranium through the Round Foramen.

41. Bacteriologicalexamination of purulent discharges from the urethra


revealed some gram-negative beanshaped bacteria located in the leukocytes. They
can be identified as the causative agent of the following disease:
A. Gonorrhea
B. Syphilis
C. Venereal lymphogranulomatosis
D. Chancroid
E. Trichomoniasis
Reason: Neissera gonorrhea is a gram – bacteria, bean shaped and normally inhabits the
Urinary tract.

42. A male patient is 28 years old. Histological study of a cervical lymph node revealed a
change of its pattern due to the proliferation of epithelioid, lymphoid cells and
macrophages having nuclei in form of a horseshoe. In the center of some cell clusters there
were non-structured lightpink areas with fragments of nuclei. What disease are these
changes typical for?
A. Tuberculosis
B. Hodgkin’s disease
C. Actinomycosis
D. Tumor metastasis
E. Syphilis
Reason: Pirogov-Langhans cells are formed by a fusion of epitheloid cells and are arranged
in a horseshoe pattern, this is specific for granulomatous diseases e.g Tuberculosis,
Sarcoidosis.

43. A 35-year-old male patient has been referred by an andrologist for the genetic
counselling for the deviations of physical and mental development. Objectively: the patient
is tall, has asthenic constitution, gynecomastia, mental retardation. Microscopy of the oral
mucosa cells revealed sex chromatin (single Barr body) in 30 % of cells. What is the most
likely diagnosis?
A. Klinefelter syndrome
B. DiGeorge syndrome
C. Down syndrome
D. Recklinghausen’s disease
E. Cushing pituitary basophilism
Reason: Klinefelter’s syndrome- 47 XXY. A condition in which there is an addition of a barr
body in the male karyotype.

NSA TSMU Academic Team Krok 1 2013


KillingKrok2016 13

44. A patient with jaundice has high total bilirubin that is mainly indirect
(unconjugated), high concentration of stercobilin in the stool and urine. The level of direct
(conjugated) bilirubin in the blood plasma is normal. What kind of jaundice can you think
of?
A. Hemolytic
B. Parenchymal ( hepatic )
C. Mechanical
D. Neonatal jaundice
E. Gilbert’s disease
Reason: Hemolytic jaundice- blood protein ALBUMIN carries unconjugated bilirubin from the
spleen to the liver where it is converted to conjugated bilirubin. So the destruction will lead
to an increased number of unconjugated bilirubin.

45. A male with a lesion of one of the CNS parts has asthenia, muscular dystonia, balance
disorder. Which CNS part has been affected?
A. Cerebellum
B. Black substance
C. Reticular formation
D. Red nuclei
E. Vestibular nuclei

Reason: The Cerebellum is responsible for balance and co-ordination of movements.

46. A 50-year-old patient has been administered laevomycetin for the treatment of typhoid
fever, but on the next day the patient’s condition worsened, the temperature rose to
39,60C. The deterioration of the patient’s condition can be explained by:
A. Effects of endotoxins of the causative agent
B. Allergic reaction
C. Insensitivity of the pathogen to levomycetin
D. Secondary infection
E. Re-infection
Reason: Laevomycetin is a Bactericidal drug, when it kills the bacteria, its cell wall breaks
down and the endotoxins contained in the bacteria are released into the blood therefore
causing such reaction.

47. A 12-year-old patient has been admitted to a hospital for hemarthrosis of the knee
joint. From early childhood he suffers from frequent bleedings. Diagnose the boy’s disease:
A. Hemophilia
B. Hemorrhagic vasculitis
C. Hemolytic anemia
D. B12 (folic acid)-deficiency anemia

NSA TSMU Academic Team Krok 1 2013


KillingKrok2016 14

E. Thrombocytopenic purpura
Reason: Haemophilia is a group of hereditary genetic disorders that impairs the body's
ability to control blood clotting, which is used to stop bleeding when a blood vessel is broken.
Characteristic sign is Hemarthrosis(bleeding in joints).

48. Examination of a patient with ischemic heart disease revealed the impaired venous
blood flow in the territory of the cardiac vein running in the anterior interventricular
sulcus of heart. What vein is it?

A. V. cordis magna
B. V. cordis media
C. V. cordis parva
D. V. posterior ventriculi sinistri
E. V. obliqua atrii sinistri

49. For the direct injection of medications into the liver surgeons use the round ligament of
liver. This manipulation involves bougienage (lumen dilatation) of the following vessel:

A. V. umbilicalis
B. A. umbilicalis
C. Ductus venosus
D. V. porta
E. A. hepatica propria

Reason: The round ligament is a remnant of vena umbilicus in the fetal stage of life.

50. A patient with lobar pneumonia has had body temperature of 39oC with daily
temperature fluctuation of no more than 1oC for 9 days. This fever can be characterized by
the following temperature curve:
A. Persistent
B. Hectic
C. Remittent
D. Hyperpyretic
E. Recurrent
Reason: Persistent or Continuous Fever increases not more than 10C for days. In Febris
recurrens, there are periods of rising temp (5-8days), and periods of normal temp. In Febris
hectica, there is daily fluctuation of up to 3-4oC. In Febris remittens (indulgence fever),
fluctuation between morning and evening temps is 1-2oC.

51. The temperature in a production room is 36oC. Relative air humidity is 80 %. Under
these conditions the human body transfers heat mainly through:
A. Sweat evaporation

NSA TSMU Academic Team Krok 1 2013


KillingKrok2016 15

B. Heat conduction
C. Radiation
D. Convection
E. –
Reason: A Human body in an Empty room will only lose heat through evaporation.

52. A hospitalized patient bitten by a rabid animal has an avulsive wound of shin. What
kind of vaccine must be given to prevent rabies?
A. Anti-rabies vaccine
B. DTaP
C. Td
D. BCG
E. TABte
Reason: Anti-rabies vaccine is a vaccine administered to prevent rabies.

53. At autopsy the occipital lobe of brain was found to have a cavity 2,5x1,5 cm large filled
with a transparent liquid. The cavity had smooth brownish walls. What process had
developed in the brain?
A. Cyst on the site of a hemorrhage
B. Softening of the cerebrocortical grey matter
C. Brain abscess
D. Paracephalia
E. A cyst on the site of the softening of the cerebrocortical grey matter
Reason: Cyst is a sac-like pocket of tissue that contains clear fluid, air or other substances.

54. A child entering the school for the first time was given Mantoux test in order to
determine if there was a need for revaccination. The reaction was negative. What is the
meaning of this test result?
A. No cell-mediated immunity to tuberculosis
B. Availability of cell-mediated immunity to tuberculosis
C. No antibodies to the tuberculosis bacteria
D. No anti-toxic immunity to tuberculosis
E. Presence of antibodies to the tuberculosis bacteria

Reason: A positive test would indicate that the boy has antibodies to M. Tuberculosis but the
child had a negative reaction.

55. Study of the biopsy material revealed a granuloma consisting of lymphocytes, plasma
cells, macrophages with foamy cytoplasm (Mikulicz cells), many hyaline globules. What
disease can you think of?

NSA TSMU Academic Team Krok 1 2013


KillingKrok2016 16

A. Rhinoscleroma
B. Leprosy
C. Syphilis
D. Tuberculosis
E. Actinomycosis
Reason: Mikulicz cells are large vacuolated histiocytes that contain Klebsiella
rhinoscleromatis.

56. Autopsy of a 78-year-old patient revealed that retroperitoneal tissue was soaked with
blood, the abdominal aorta had a sacciform protrusion including a defect with irregular
edges. The wall of the aorta was here and there of stone-like density. This is the
complication of the following disease:
A. Atherosclerosis
B. Essential hypertension
C. Systemic vasculitis
D. Visceral syphilis
E. Marfan syndrome
Reason: Atherosclerosis is the accumulation of lipids in the Aorta and Arteries.

57. Glycogen polysaccharide is synthesized from the active form of glucose. The immediate
donor of glucose residues during the glycogenesis is:
A. UDP-glucose
B. Glucose-1-phosphate
C. ADP-glucose
D. Glucose-6-phosphate
E. Glucose-3-phosphate
Reason: UDP glucose is a precursor of glycogen and can be converted into UDP galactose
and UDP glucaronic acid which can be used as substrates by the enzyme that makes
polysaccharide.

58. After the diagnostic tests a 40-yearold male has been referred for the
lymphography of the thoracic cavity. The surgeon revealed that the tumor had affected an
organ whose lymphatic vessels drain directly into the thoracic duct. Specify this organ:
A. Esophagus
B. Trachea
C. Left main bronchus
D. Heart
E. Pericardium

NSA TSMU Academic Team Krok 1 2013


KillingKrok2016 17

59. A patient with biliary dyskinesia and constipations has been prescribed a cholagogue
having also a laxative effect. What drug has been administered?
A. Magnesium sulfate
B. Allochol
C. Cholosas
D. Cholenzyme
E. Nicodinum
Reason: Magnesium sulfate (commonly called ‘mixture of magnesium’ or ‘MixMag’ is one of
the few drugs which has both a laxative and a cholagogue effect.

60. It is known that individuals with genetically caused deficiency of glucose6-phosphate


dehydrogenase may develop RBC hemolysis in response to the administration of some
antimalarial drugs. Manifestation of adverse reactions to drugs is called:
A. Idiosyncrasy
B. Allergy
C. Sensibilization
D. Tachyphylaxis
E. Tolerance
Reason: Drug idiosyncrasy is untoward reaction to drug that occur in a small fraction of
patients and have no obvious relationship to dose or duration.

61. A 40-year-old patient with the progressing staphylococcal purulent periodontitis


developed purulent inflammation of bone marrow spaces of the alveolar process, and then
of the body of mandible. Microscopy revealed thinning of bone trabeculae, foci of necrosis,
bone sequesters surrounded by the connective tissue capsule. What is the most likely
diagnosis?
A. Chronic osteomyelitis
B. Acute osteomyelitis
C. Parodontome
D. Chronic fibrous periostitis
E. Purulent abscess
Reason: The diagnosis is Osteomyelitis, and it is Chronic because because bone sequesters
have appeared and it has been a progressing disease.

62. Curariform substances introduced into a human body cause the relaxation of all
skeletal muscles. What changes in the neuromuscular synapse cause this phenomenon?
A. Blockade of N-cholinergic receptors of the synaptic membrane
B. Impaired acetylcholine release
C. Blockade of Ca2+ channels of the presynaptic membrane
D. Impaired cholinesterase synthesis

NSA TSMU Academic Team Krok 1 2013


KillingKrok2016 18

E. Depolarization of the postsynaptic membrane


Reason: N- Cholinergic receptors are found in sympathetic nervous system (skeletal muscle).

63. A number of diseases can be diagnosed by evaluating activity of blood transaminases.


What vitamin is one of cofactors of these enzymes?

A. B6
B. B2 C. B1
D. B8
E. B5
Reason: Vitamin B6 one of the co factors of blood transaminase.

64. After a car accident a 23-year-old male presented to the hospital with a cut wound of
the anteromedial region of shoulder and arterial bleeding. Which artery was damaged?
A. A. brachialis
B. A. radialis
C. A. axillaris
D. A. subscapularis
E. A. profunda brachi

Reason: This is the route of the brachial artery along the upper arm.

65. During the operation on the small intestine the surgeon revealed an area of the mucous
membrane with a single longitudinal fold among the circular folds. Which portion of the
small intestine is this structure typical for?

A. Pars descendens duodeni


B. Pars horizontalis duodeni
C. Pars ascendens duodeni
D. jejunum
E. Distal ileum

Reason: The longitudinal fold occurs as a result of the invagination of the bile duct running
along the wall of the descending part of the duodenum.

66. 14 days after quinsy a 15-year-old child presented with morning facial swelling, high
blood pressure, "meat slops" urine. Immunohistological study of a renal biopsy sample
revealed deposition of immune complexes on the basement membranes of the capillaries
and in the glomerular mesangium. What disease developed in the patient?
A. Acute glomerulonephritis
B. Acute interstitial nephritis
C. Lipoid nephrosis
D. Acute pyelonephritis

NSA TSMU Academic Team Krok 1 2013


KillingKrok2016 19

E. Necrotizing nephrosis
Reason: In acute glomerulonephritis urine is meat slop color, increased blood pressure,
edema.

67. A diseased child has a high fever, sore throat, swelling of submandibular lymph nodes.
Objectively: pharyngeal mucosa is edematous, moderately hyperemic, the tonsils are
enlarged, covered with grayish membrane tightly adhering to the tissues above. Attempts to
remove the membrane produce the bleeding defects. What disease are these presentations
typical for?
A. Diphtheria
B. Catarrhal tonsillitis
C. Scarlet fever
D. Meningococcal disease
E. Measles
Reason: Diptheria has the characteristics symptoms above and has grayish membranes that
adhere to the palate and bleeds when they are removed.

68. Study of the biopsy material of an embryo revealed a zone of developmental


abnormality in a somite. The zone was located close to the endoderm and the notochord.
What formations may have abnormal development in case of pregnancy continuation?
A. Skeletal tissues
B. Genito-urinary system
C. Skeletal striated muscle tissue
D. Cardiac striated muscle tissue
E. Fibrous connective tissue of skin
Reason: The zone being asked about is the Mesoderm which is located between the
Endoderm and notochord. The mesoderm forms the skeletal muscles through a process
called Myogenesis.

69. A smear of sputum from the patient with suspected lobar pneumonia was stained with
the use of the following stains and reagents: solution of gentian violet, Lugol’s solution,
96o alcohol, water magenta. What staining method was applied in this case?
A. Gram
B. Ziehl-Nielsen
C. Romanovsky
D. Neisser
E. Leffler
Reason: The staining method described above is Gram stain.

NSA TSMU Academic Team Krok 1 2013


KillingKrok2016 20

70. A patient has normally coloured stool including a large amount of free fatty acids. The
reason for this is a disturbance of the following process:
A. Fat absorption
B. Fat hydrolysis
C. Biliary excretion
D. Choleresis
E. Lipase secretion
Reason: When fatty stool occurs, there’s a problem with the absorption of fat. Can be as a
result of Pancreatitis or Cholecystitis.

71. Examination of the removed stomach revealed a deep roundish defect with regular
edges at the lesser curvature of the antrum. The defect reached the muscular tunic and
was 1,5 cm in diameter. Within the defect floor there was a translucent dense area
resembling of a hyaline cartilage. What process had developed in the floor of the stomach
defect?
A. Local hyalinosis
B. Amyloidosis
C. Mucoid swelling
D. Fibrinoid alterations
E. General hyalinosis
Reason: Hyalinosis occurs as a result of deposits of hyaline membranes. It isn’t general
hyalinosis because the process is occurring in the antrum.

72. By the decarboxylation of glutamate in the CNS an inhibitory mediator is formed. Name
it:
A. GABA
B. Glutathione
C. Histamine
D. Serotonin
E. Asparagine
Reason: Glutamate is converted to GABA by glutamate decarboxylase.

73. Thermometry revealed that the temperature of the exposed skin is by 11,5 o lower than
the temperature of the adjacent areas covered with clothing from natural fabrics. The
reason for this is that the clothes reduce the heat loss through:
A. Convection
B. Radiation
C. Conduction
D. Evaporation
E. –

NSA TSMU Academic Team Krok 1 2013


KillingKrok2016 21

Reason: Convection is heat loss through fluids, the parts covered with fabrics will keep the
moisture in thereby preventing heat loss.

74. A specimen of pia mater includes a vessel whose wall doesn’t have the tunica media,
the tunica externa is adherent to the surrounding tissues, the intima is composed of a
basement membrane and endothelium. What vessel is it?
A. Nonmuscular vein
B. Muscular vein with underdeveloped muscular elements
C. Muscular artery
D. Arteriole
E. Artery of mixed type
Reason: Veins do not have tunica Media because they do not contract. Tunica media usually
contain the muscular layer of arteries.

75. A patient with extensive burns of torso skin exhibits signs of severe intoxication. What
stage of the burn disease is this typical for?
A. Burn toxemia
B. Burn shock
C. Burn infection
D. Burn emaciation
E. Terminal
Reason: Decomposed burned tissues cause septic toxemia.

76. As a result of a craniocerebral injury a patient has a decreased skin sensitivity. What area
of the cerebral cortex may be damaged?
A. Posterior central gyrus
B. Occipital region
C. Cingulate gyrus
D. Frontal cortex
E. Anterior central gyrus
Reason: The lateral postcentral gyrus is a prominent structure in the parietal lobe of the
human brain. It is the location of the primary somatosensory cortex, the main sensory
receptive area for the sense of touch.

77. A histological specimen of the eyeball shows a biconvex structure connected to the
ciliary body by the fibers of the Zinn’s zonule and covered with a transparent capsule.
Name this structure:
A. Crystalline lens
B. Vitreous body

NSA TSMU Academic Team Krok 1 2013


KillingKrok2016 22

C. Ciliary body
D. Cornea
E. Sclera
Reason: Like duhh! What else could it be?

78. A comatose patient was taken to the hospital. He has a history of diabetes mellitus.
Objectively: Kussmaul breathing, low blood pressure, acetone odor of breath. After the
emergency treatment the patient’s condition improved. What drug had been administered
to the patient?
A. Insulin
B. Adrenaline
C. Isadrinum
D. Glibenclamide
E. Furosemide
Reason: Patient is in a hyperglycemic coma and should be give insulin to reduce blood sugar.

79. In order to stimulate breathing in a child born with asphyxia, the doctor gave him a
drug injection into the umbilical vein. What drug might have been injected?
A. Aethimizolum
B. Corazolum
C. Cordiaminum
D. Sulfocamphocainum
E. Coffeinum
Reason: Aethimizolum is an analeptic drug used to stimulate breathing.

80. A patient complains of pain in the right lateral abdomen. Palpation revealed a dense,
immobile, tumor-like formation. A tumor is likely to be found in the following part of the
digestive tube:

A. Colon ascendens
B. Colon transversum
C. Colon descendens
D. Colon sigmoideum
E. Caecum

Reason: This part of the large intestine is located on the right lateral part of the abdomen.

81. A patient underwent biopsy of the soft palate arches for a suspected tumor
(macroscopy revealed an ulcer with a dense floor). Study of the biopsy material revealed
mucosal necrosis with infiltration of lymphocytes, epithelioid cells, plasma cells, single

NSA TSMU Academic Team Krok 1 2013


KillingKrok2016 23

neutrophils in the submucosa. There were also apparent signs of endovasculitis and
perivasculitis. The described changes are typical for:
A. Primary syphilis
B. Aphthous stomatitis
C. Ulcerative stomatitis
D. Necrotizing ulcerative Vincent stomatitis
E. Pharyngeal diphtheria
Reason: The bolded sentence describes a “Hard Chancre” which is the characteristic sign of
Primary Syphilis.

82. Healthy parents with unremarkable family history have the child with multiple
developmental defects. Cytogenetic analysis revealed the trisomy 13 in the somatic cells
(Patau syndrome). What phenomenon has caused the defects?
A. Abnormal gametogenesis
B. Somatic mutation
C. Recessive mutation
D. Dominant mutation
E. Chromosomal mutation
Reason: Patau syndrome is a syndrome caused by a chromosomal abnormality, in which
some or all of the cells of the body contain extra genetic material from chromosome 13.

83. A specimen shows an organ covered with the connective tissue capsule with trabeculae
radiating inward the organ. There is also cortex containing some lymph nodules, and
medullary cords made of lymphoid cells. What organ is under study?
A. Lymph node
B. Thymus
C. Spleen
D. Red bone marrow
E. Tonsils
Reason: lol! By now if you don’t know it’s a lymph node, you might need EPP!

84. A 25-year-old patient consulted a doctor about dysmenorrhea and infertility.


Examination revealed that the patient was 145 cm high and had underdeveloped secondary
sex characteristics, alar folds on the neck. Cytological study didn’t reveal any Barr bodies in
the somatic cells. What diagnosis was made?
A. Turner’s syndrome
B. Klinefelter syndrome
C. Morris syndrome
D. Trisomy X syndrome
E. –

NSA TSMU Academic Team Krok 1 2013


KillingKrok2016 24

Reason: Turners syndrome- 47X0 Seen in females. Patient has underdeveloped secondary
characteristics due to the absence of one X chromosome.

85. To prevent attacks of acute pancreatitis a doctor prescribed the patient trasylol
(contrycal, gordox), which is an inhibitor of:
A. Trypsin
B. Elastase
C. Carboxypeptidase
D. Chymotrypsin
E. Gastricsin
Reason: Contrycal inhibits trypsin, kallikrein and plasmin

86. A patient died from progressive heart failure. Autopsy revealed that the heart was
enlarged in diameter, flabby. The muscle section exhibited irregular blood supply.
Histological study of myocardium revealed hyperemia, the stroma was found to have
lymphohistiocytic infiltrates with degeneration of cardiomyocytes. The revealed
morphological changes are indicative of:
A. Non-purulent interstitial myocarditis
B. Venous plethora
C. Cardiomyoliposis
D. Cardiosclerosis
E. Myocardial infarction
Reason: First of all, We can see it’s Myocarditis, enlarged heart, degeneration of
cardiomyocytes …blah blah blah… it’s non purulent because its infiltrated with lymphocytes.
In a purulent course, neutrophils will show prevalence in the infiltrates.

87. A 13-year-old teenager underwent X-ray examination of the hip joint. Examination
revealed a 3 mm wide radiolucent zone between the head and the shaft of femur. This
situation should be evaluated as:
A. Normal (incomplete process of ossification)
B. Fracture of the femoral neck
C. Fissured fracture of the femoral neck
D. Dislocation of the femoral head
E. Radiographic film artifact
Reason: Normal Ossification between Head and shaft of femur and the three pelvic bones
occurs at the age of 14-16 years.

88. A surgeon examined the patient and found the injury of the upper third of the kidney.
Considering the syntopy of the left kidney, the intactness of the following organ should be
checked at the same time:

NSA TSMU Academic Team Krok 1 2013


KillingKrok2016 25

A. Stomach
B. Liver
C. Small intestine
D. Transverse colon
E. Descending colon
Reason: The other options are not close to the left kidney…it’s that clear..I hope.

89. A patient with urolithiasis has unbearable spasmodic pain. To prevent pain shock, he
has been given an injection of atropine along with a narcotic analgesic having
antispasmodic effect. What drug was it?
A. Promedol
B. Nalorphine
C. Tramadol
D. Ethylmorphine hydrochloride
E. Morphine hydrochloride
Reason: Promedol is a drug which has a narcotic analgesic effect, an antispasmodic effect
and can be administered together with atropine.

90. Despite the administration of cardiotonics and a thiazide diuretic a patient with chronic
heart failure has persistent edemata, there is a risk of ascites. What medication should be
administered in order to enhance the diuretic effect of the drugs used?
A. Spironolactone
B. Furosemide
C. Amiloride
D. Clopamide
E. Manithol
Reason: Spironolactone is an inhibitor of aldosterone and aids in the elimination of fluid.

91. A patient with a pathology of the cardiovascular system developed edemata of the
lower extremities. What is the mechanism of cardiac edema development?
A. Increased hydrostatic pressure at the venous end of the capillary
B. Increased oncotic pressure
C. Increased hydrostatic pressure at the arterial end of the capillary
D. Reduced osmotic pressure
E. Lymph efflux disorder
Reason: Hydro(water), Stasis(stagnation) occurs mostly when the blood in veins can’t go
back to the heart due to a Cardiovascular disease e.g Right-sided Heart failure, Inferior Vena
Cava Syndrome e.t.c.

NSA TSMU Academic Team Krok 1 2013


KillingKrok2016 26

92. During the fight, a man had a cardiac arrest due to the strong blow to the upper region
of the anterior abdominal wall. Which of the following mechanisms has led to the cardiac
arrest?
A. Parasympathetic unconditioned reflexes
B. Sympathetic unconditioned reflexes
C. Parasympathetic conditioned reflexes
D. Sympathetic conditioned reflexes
E. Peripheral reflexes
Reason: First of all, you should note that this was an unconditioned reflex since one cannot
have control over cardiac activity …Secondly, A sympathetic response would cause increase
of heart activity not reduction or arrest.

93. A pregnant woman underwent AB0 blood typing. Red blood cells were agglutinated
with standard sera of the I and II blood groups, and were not agglutinated with the III
group serum. What is the patient’s blood group?

A. B(III)
B. 0(I)
C. A(II)
D. AB(IV)
E. –
Reason: Absence of agglutination in the III blood group indicates the blood group B

94. Amniocentesis revealed two sex chromatin bodies (Barr bodies) in each cell of the
sample. What disease is this character typical for?
A. Trisomy X
B. Klinefelter syndrome
C. Turner’s syndrome
D. Down’s syndrome
E. Patau syndrome
Reason: Trisomy X =47, XX
95. A hospital has admitted a patient complaining of abdominal bloating, diarrhea,
flatulence after eating protein foods. These signs are indicative of the impaired digestion of
proteins and their increased degradation. Which of the following compounds is the product
of this process?
A. Indole
B. Bilirubin
C. Cadaverine
D. Agmatine
E. Putrescine

NSA TSMU Academic Team Krok 1 2013


KillingKrok2016 27

Reason: Indole can be produced by bacteria as a degradation of amino acid Tryptophan, it


occurs in human feces and has an intense fecal odor.

96. An attack of tachycardia that occurred in a patient was stopped by pressing on his
eyeballs. Which of the following reflexes underlies this phenomenon?
A. Aschner
B. Goltz
C. Bainbridge
D. Hering
E. Bernard’s
Reason: Aschner reflex is a decrease in pulse rate associated with traction on extra ocular
muscles or compression of the eyeballs.

97. A male patient has been diagnosed with acute post-streptococcal glomerulonephritis. It
is most likely that the lesion of the basement membrane of renal corpuscles was caused by
the following allergic reaction:
A. Immune complex
B. Anaphylactic
C. Cytotoxic
D. Delayed
E. Stimulating
Reason: This is the 3rd type of allergy reaction. The complex was formed by the Streptococcal
antigen and the human antibody.

98. An unconscious patient was taken by ambulance to the hospital. On objective


examination the patient was found to have no reflexes, periodical convulsions, irregular
breathing. After laboratory examination the patient was diagnosed with hepatic coma.
Disorders of the central nervous system develop due to the accumulation of the following
metabolite:
A. Ammonia
B. Urea
C. Glutamine
D. Bilirubin
E. Histamine
Reason: In Hepatic coma, ammonia would not be converted to urea. Excess ammonia then
crosses blood-brain barrier and causes HEPATIC ENCEPHALOPATHY.

99. A 20-year-old male patient complains of general weakness, rapid fatigability, irritability,
decreased performance, bleeding gums, petechiae on the skin. What vitamin deficiency
may be a cause of these changes?

NSA TSMU Academic Team Krok 1 2013


KillingKrok2016 28

A. Ascorbic acid
B. Riboflavin
C. Thiamine
D. Retinol
E. Folic acid

Reason: Deficiency of Vitamin C leads to the following illnesses.

100. It is known that the monoamine oxidase (MAO) enzyme plays an important part in the
metabolism of catecholamine neurotransmitters. In what way does the enzyme inactivate
these neurotransmitters (norepinephrine, epinephrine, dopamine)?
A. Oxidative deamination
B. Addition of an amino group
C. Removal of a methyl group
D. Carboxylation
E. Hydrolysis
Reason: MAO only catalyze Oxidative deamination of monoamines( Oxygen is used to
remove an amine group from a molecule, resulting in aldehyde and ammonia)

101. The cellular composition of exudate largely depends on the etiological factor of
inflammation. What leukocytes are the first to get into the focus of inflammation caused by
pyogenic bacteria?
A. Neutrophil granulocytes
B. Monocytes
C. Myelocytes
D. Eosinophilic granulocytes
E. Basophils
Reason: Neutrophils are markers for inflammation due to the fact that they increase in the
blood during inflammation caused by a bacteria

102. At the end of the working day a worker of a hot work shop has been delivered to a
hospital. The patient complains of a headache, dizziness, nausea, general weakness.
Objectively: the patient is conscious, his skin is hyperemic, dry, hot to the touch. Heart rate
is of 130/min. Respiration is rapid, superficial. What disorder of thermoregulation is most
likely to have occurred in this patient?
A. Reduced heat transfer
B. Increased heat transfer and reduced heat production
C. Increased heat transfer and heat production
D. Increased heat production with no changes to the heat transfer
E. Reduced heat production with no changes to the heat transfer

Reason: The symptoms show that there was no heat loss from the patient. No heat transfer
occurred.

NSA TSMU Academic Team Krok 1 2013


KillingKrok2016 29

103. Alveolar space of the acinus was invaded by some bacteria which interacted with the
surfactant. This led to the activation of the cells that are localized in the alveolar walls and
on the surface. What cells are these?
A. Alveolar macrophages
B. Alveolocytes type I
C. Endothelial cells
D. Clara cells
E. Alveolocytes type II
Reason: These are the names of macrophages found in the lungs.

104. A 35-year-old male developed acute heart failure while running for a long time. What
changes in the ionic composition can be observed in the cardiac muscle?

A. Accumulation of Na+ and Ca2+ ions in the myocardium cells


B. Accumulation of K+ and Mg2+ ions in the myocardium cells
C. Reduction of Na+ and Ca2+ ions in the myocardium cells
D. Reduction of K+ and Mg2+ ions in the extracellular space
E. Reduction of Na+ and Ca2+ ions in the
extracellular space
Reason: Hypercalcemia will cause inhibition of depolarization, it increases the threshold of
depolarization and causes abnormal rhythm in contraction, which leads to acute heart
failure.

105. Workers of a conveyor workshop received recommendations for the effective


organization of working time and higher working efficiency. What peculiarity of work in this
workshop causes the greatest stress for the workers?
A. Monotony of work
B. State of "operating rest"
C. Increased intellectual component
D. Increased responsibility
E. Social inefficiency of labor
Reason: Monotony of work is the quality of wearisome consistency, routine and lack of
variety. Same thing everyday

106. Mother of a boy who had recently returned from a summer camp found some small
whitish insects up to 3 mm long on the child’s clothing. Specify the parasite:
A. Pediculus humanus humanus
B. Phtirus pubis
C. Pulex irritans
D. Cimex lectularius
E. Blattella germanica

NSA TSMU Academic Team Krok 1 2013


KillingKrok2016 30

Reason: Only pediculus humanus would adhere to the body of humans, that’s why it is also
called Body louse.

107. Histological examination of the removed skin neoplasm revealed clusters and cords of
atypical cells of stratified squamous epithelium, growing into the underlying tissue. What
diagnosis can be assumed?
A. Non-keratinizing squamous cell carcinoma
B. Keratinizing squamous cell carcinoma
C. Carcinoma in situ
D. Papilloma
E. Adenoma
Reason: Skin neoplasms are poorly differentiated squamous carcinomas, they contain more
pleomorphic cells and have no keratinization.

108. An 18-year-old male has been diagnosed with Marfan syndrome. Examination
revealed a developmental disorder of connective tissue and eye lens structure,
abnormalities of the cardiovascular system, arachnodactylia. What genetic phenomenon
has caused the development of this disease?
A. Pleiotropy
B. Complementarity
C. Codominance
D. Multiple allelism
E. Incomplete dominance
Reason: Marfan syndrome is a genetic disorder of connective tissue. Pleiotropy is when one
gene influences 2 or more unrelated phenotypic traits. Basically a multi-system affection
caused by mutation of one gene.

109. A patient has severe catarrhal symptoms. Material growth on BordetGengou agar
showed mercury-droplike colonies. Examination of the blood smears revealed some
small ovoid grampositive bacilli sized 1-3 microns. What microorganisms were isolated?
A. Bordetella
B. Corynebacteria
C. C. Mycobacteria
D. Meningococcus
E. Brucella
Reason: Bordetella on agar has the characteristic mecury-droplike colony.

110. A 66-year-old patient with Parkinson’s disease shows an improvement in locomotor


activity after prolonged use of a certain drug which is converted to dopamine by the
decarboxylation. What drug has the patient taken?

NSA TSMU Academic Team Krok 1 2013


KillingKrok2016 31

A. Levodopa
B. Naloxone
C. Celecoxib
D. Droperidol
E. Chlorpromazine
Reason: Parkinson’s disease occurs when there is death of cells in the substantia nigra which
leads to decrease amount of dopamine. Levodopa is a precursor to dopamine used to treat
this disease.

111. Enzymatic jaundices are accompanied by abnormal activity of UDPglucurony


transferase. What compound is accumulated in blood serum in case of these pathologies?
A. Unconjugated bilirubin
B. Conjugated bilrubin
C. Dehydrobilirubin
D. Hydrobilirubin
E. Choleglobin
Reason: UDP glucuronyl transferase is in charge of converting unconjugated bilirubin to
conjugated bilirubin in the liver. So an abnormal activity of this enzyme will lead to an
increase of unconjugated bilirubin in the blood serum.

112. For the study of serum proteins various physical and physicochemical
methods can be used. In particular, serum albumins and globulins can be separated by this
method:
A. Electrophoresis
B. Polarography
C. Dialysis
D. Spectrography
E. Refractometry
Reasons: Electrophoresis is a technique used in laboratories in order to separate
macromolecules based on size such as proteins.

113. Negative environmental factors have caused the dysfunction of myosatellite cells.
What function of the whole muscle fibre is likely to be changed in this case?
A. Regeneration
B. Contraction
C. Trophism
D. Contractile thermogenesis
E. Relaxation
Reason: Myosatellite cells are responsible for the regeneration of muscle fibres.

NSA TSMU Academic Team Krok 1 2013


KillingKrok2016 32

114. The laboratory for especially dangerous infections conducts


microscopic examination of pathological material from a patient with
suspected plague. The sample was stained by BurriGins technique. What property of the
causative agent can be identified by this technique?
A. Capsule formation
B. Spore formation
C. Acid resistance
D. Alkali resistance
E. Presence of volutin granules
Reason: Capsule formation is one of the most pathogenic effect of a bacteria. Burri-Gins
method is used to identify Capsule of bacteria.

115. Autopsy of a 62-year-old woman revealed a dense well-circumscribed node of 6 cm in


diameter in the cranial cavity. The node was attached to the dura mater and histologically
consisted of clusters and micro-concentric structures of endothelial cells, psammoma
bodies. What kind of tumor was found at autopsy?
A. Meningioma
B. Glioblastoma
C. Medulloblastoma
D. Melanoma
E. Cancer metastasis
Reason: Meningoma is a tumor usually benign arising from meningeal tissues of the brain.
Psammoma bodies are found in the cells.

116. Inherited diseases, such as mucopolysaccharidoses, are manifested in metabolic


disorders of connective tissue, bone and joint pathologies. The sign of this disease is the
excessive urinary excretion of the following substance:
A. Glycosaminoglycans
B. Amino acids
C. Glucose
D. Lipids
E. Urea
Reason: Mucopolysaccharidoses are a group of metabolic disorders in which abnormal
accumulations of glycosaminoglycans occur because of enzyme deficiencies.
Glycosaminoglycans are polar and attract water, they will be revealed in urine.

117. An animal has an increased tonus of extensor muscles. This is the result of enhanced
information transmission to the motoneurons of the spinal cord through the following
descending pathways:

NSA TSMU Academic Team Krok 1 2013


KillingKrok2016 33

A. Vestibulospinal
B. Medial corticospinal
C. Reticulospinal
D. Rubrospinal
E. Lateral corticospinal
Reason: The vestibulospinal tract is responsible for upright posture and head stabilization by
sending signals to the extensor muscles of the legs.

118. A specimen of a parenchymal organ shows poorly delineated hexagonal lobules


surrounding a central vein, and the interlobular connective tissue contains embedded
triads (an 0artery, a vein and an excretory duct). What organ is it?
A. Liver
B. Pancreas
C. Thymus
D. Spleen
E. Thyroid
Reason: The liver is the organ with embedded triads an artery, a portal vein and an excretory
duct.

119. A patient has been admitted to the infectious diseases department for malaise, fever
up to 38oC, jaundice. A few months ago, the patient underwent blood transfusion. The
doctor suspected viral hepatitis B. What are the principal methods of laboratory diagnosis
of hepatitis B?

A. Serological and gene diagnostics


B. Virus isolation in cell culture and its identification by the cytopathic effects
C. Detection of virions in blood by electron microscopy
D. Isolation of the virus in laboratory animals (neutralization reaction)
E. Isolation of the virus in chicken embryos
Reason: The gold standard of Hepatitis B Diagnosis is serology, which is checking Antibody
Titre against the Antigens of the virus.

120. After resection of the middle third of the femoral artery obliterated by a thrombus the
limb is supplied with blood through the bypasses. What artery plays the main part in the
restoration of the blood flow?
A. Deep femoral artery
B. Superficial iliac circumflex artery
C. Descending genicular artery
D. Superficial epigastric artery
E. External pudendal artery

NSA TSMU Academic Team Krok 1 2013


KillingKrok2016 34

Reason: Resection in the middle part of femoral artery means that the Superficial femoral
artery was resected so in order to restore blood flow, it’s up to the deep femoral artery.
Please remember that the common femoral artery gives 2 branches, superficial and deep
femoral artery.

121. During the intravenous transfusion of the saline the patient’s condition deteriorated
dramatically, and the patient died from asphyxiation. Autopsy revealed acute venous
congestion of internal organs with the dramatic right heart dilatation. When the right
ventricle was punctured underwater, the bubbles escaped. What pathological process
occurred in the patient?
A. Air embolism
B. Gaseous embolism
C. Adipose embolism
D. Tissue embolism
E. Thromboembolism
Reason: One of the dangers of giving Intravenous injections is Air Embolism. Note that
bubbles escaped.

122. At the post-mortem examination the stomach of a patient with renal failure was
found to have a yellow-brown coating on the thickened mucosa. The coating was firmly
adhering to its surface and had significant thickness. Microscopy revealed congestion and
necrosis of mucosal and submucosal layers, fibrin presence. What is the most likely
diagnosis?
A. Fibrinous gastritis
B. Croupous gastritis
C. Gastric abscess
D. Esogastritis
E. Corrosive gastritis
Reason: Fibrinous gastritis is characterized by small brown-like yellow brown deposits on
mucosa.
123. Infectious diseases are treated with antibiotics (streptomycin, erythromycin, and
chloramphenicol). They inhibit the following stage of protein synthesis:
A. Translation
B. Transcription
C. Replication
D. Processing
E. Splicing
Reason: These antibiotics inhibit the Translation stage of protein formation.

124. Diseases of the respiratory system and circulatory disorders impair the transport of
oxygen, thus leading to hypoxia. Under these conditions the energy metabolism is carried

NSA TSMU Academic Team Krok 1 2013


KillingKrok2016 35

out by anaerobic glycolysis. As a result, the following substance is generated and


accumulated in blood:
A. Lactic acid
B. Pyruvic acid
C. Glutamic acid
D. Citric acid
E. Fumaric acid
Reason: Lactic acid is the end product of anaerobic glycolysis

125. A patient has been hospitalized for a suspected tumor of the prostate. During the
surgery, it was revealed that the tumor invaded the bladder. Which part of the bladder was
affected?
A. Cervix
B. Apex
C. Bottom
D. Body
E.

Reason: The neck of the urinary bladder is the portion of the urinary bladder adjacent to
the prostate gland in men.

126. A casualty with an injury of the temporal region has been diagnosed with epidural
hematoma. Which of the arteries is most likely to be damaged?
A. Medial membranous artery
B. Medial cerebral artery
C. Superficial temporal artery
D. D. Anterior membranous artery
E. Posterior auricular artery

Reason: The medial membranous artery runs along the lateral wall of the tympanic cavity so
it will most likely cause an epidural hematoma seeing as it’s so close to be visible when
injured.

127. A 19-year-old male was found to have an elevated level of potassium in the secondary
urine. These changes might have been caused by the increase in the following hormone
level:
A. Aldosterone
B. Oxytocin
C. Adrenaline
D. Glucagon
E. Testosterone

NSA TSMU Academic Team Krok 1 2013


KillingKrok2016 36

Reason: Aldosterone is a steroid hormone, the main mineralocorticoid hormone. It increases


reabsorption of ions and water in the kidney, to cause the conservation of sodium, secretion
of potassium, increase in water retention, and increase in blood pressure and blood volume.

128. Analysis of the ECG revealed the missing of several PQRST cycles. The remaining
waves and complexes are not changed. Specify the type of arrhythmia:

A. Sinoatrial block
B. Atrial fibrillation
C. Atrioventricular block
D. Atrial premature beat
E. Intra-atrial block
Reason: Absence of a whole cycle of heart waves starting with P wave can only indicate a
sinoatrial block.

129. ECG of a patient displays an abnormally long R wave (up to 0,18 s). This is caused by a
decrease in the conduction velocity of the following heart structures:
A. Ventricles
B. Atria
C. Atrio-ventricular node
D. Right ventricle
E. Left ventricle
Reason: R wave indicates Depolarization of the Ventricles. Atria would’ve been P, AV-node
would’ve been Q.

130. 6 hours after the myocardial infarction a patient was found to have elevated level of
lactate dehydrogenase in blood. What isoenzyme should be expected in this case?

A. LDH1
B. LDH2
C. LDH3
D. LDH4 E. LDH5
Reason: This is an isoenzyme which is normally elevated in the blood in myocardial
infarction.

131. A 46-year-old female is scheduled for a maxillofacial surgery. It is known that the
patient is prone to high blood coagulation. What natural anticoagulant can be used to
prevent blood clotting?
A. Heparin

NSA TSMU Academic Team Krok 1 2013


KillingKrok2016 37

B. Hirudin
C. Sodium citrate
D. Fibrinolysin
E. None of the above-listed substances
Reason: Heparin is an anticoagulant.

132. A 50-year-old patient with food poisoning is on a drip of 10% glucose solution. It not
only provides the body with necessary energy, but also performs the function of
detoxification by the production of a metabolite that participates in the following
conjugation reaction:
A. Glucuronidation
B. Sulfation
C. Methylation
D. Glycosylation
E. Hydroxylation
Reason: The body uses Glucoronidation to make substances water-soluble in order to make
their elimination or detoxification easier through urine or feces.

133. To assess the effectiveness of breathing in patients, the indicator of functional residual
capacity is used. It includes the following volumes:
A. Expiratory reserve volume and residual volume
B. Inspiratory reserve volume and residual volume
C. Inspiratory reserve volume, tidal volume, residual volume
D. Expiratory reserve volume and tidal volume
E. Inspiratory reserve volume and tidal volume
Reason: Functional reserve volume is the sum of Expiratory Reserve Volume (ERV) and
Residual Volume (RV)

134. It is required to evaluate the level of tissue excitability. For this purpose one should
determine:
A. Depolarization threshold
B. Resting potential
C. Critical level of depolarization
D. Action potential amplitude
E. Action potential duration
Reason: Depolarization causes Contraction of muscles, so therefore shows level of
excitability.

NSA TSMU Academic Team Krok 1 2013


KillingKrok2016 38

135. Due to the use of poor-quality measles vaccine for preventive vaccination, a 1-year-
old child developed an autoimmune renal injury. The urine was found to contain
macromolecular proteins. What process of urine formation was disturbed?
A. Filtration
B. Reabsorption
C. Secretion
D. Reabsorption and secretion
E. Secretion and filtration
Reason: Macromolecules are normally not filtered in the glomerulus. Proteinuria can only
occur when there’s problem with filtration, especially injury to glomerulus.

136. A patient has been administered an anti-inflammatory drug that blocks the action of
cyclooxygenase. Specify this anti-inflammatory agent:
A. Aspirin
B. Analgene
C. Allopurinol
D. Thiamin
E. Creatine
Reason: Aspirin also known as acetylsalicylic acid is an anti-inflammatory drug that inhibits
the action of the enzyme COX.

137. A pneumonia patient has been administered acetylcysteine as a part of complex


therapy. What principle of therapy was taken into consideration when applying this drug?
A. Pathogenetic
B. Symptomatic
C. Etiotropic
D. Antimicrobial
E. Immunomodulatory
Reason: Pathogenetically in Pneumonia, Neutrophiles, bacteria and fluid from surrounding
blood vessels fill the alveoli in form of mucous, Acetylcysteine is a mucolytic and expectorant.
So use of this is pathogenetic tx.

138. A 26-year-old female patient with bronchitis has been administered a broad spectrum
antibiotic as a causal treatment drug. Specify this drug:
A. Doxycycline
B. Interferon
C. BCG vaccine
D. Ambroxol
E. Dexamethasone

NSA TSMU Academic Team Krok 1 2013


KillingKrok2016 39

Reason: Doxycycline is a broad spectrum antibiotic used in the treatment of a number of


types of infections caused by bacteria and protozoa.

139. A 65-year-old male suddenly lost the vision in one eye due to the retinal detachment.
The patient underwent enucleation. Histological examination of the removed eye retina
and choroid revealed clusters of atypical cells with marked polymorphism of cells and
nuclei, with a moderate number of mitoses including the pathological ones. The cell
cytoplasm and intercellular medium contained brown pigment giving a positive DOPA
reaction. Perls’ reaction was negative. What is the most likely diagnosis?
A. Melanoma
B. Pigmented mole
C. Hemorrhage
D. Cysticercosis
E. Wilson’s disease
Reason: The Signs highlighted are typical for Melanoma.

140. A child cut his leg with a piece of glass while playing and was brought to the clinic for
the injection of tetanus toxoid. In order to prevent the development of anaphylactic shock
the serum was administered by Bezredka method. What mechanism underlies this method
of desensitization of the body?
A. Binding of IgE fixed to the mast cells
B. Blocking the mediator synthesis in the mast cells
C. Stimulation of immune tolerance to the antigen
D. Stimulation of the synthesis of antigenspecific IgG
E. Binding of IgE receptors to the mast cells

Reason: Anaphylactic shock occurs when IgE is released in the blood which causes
immediate type hypersensitivity, Bezredka method of administering the toxoid would bind it
to the mast cells thereby reducing its release.

141. Microscopy of the myocardium of a patient who had died from heart failure revealed
foci of fibrinoid necrosis located diffusely in the interstitial stroma, and often around the
vessels. Such foci were surrounded by lymphocytes, macrophages, histiocytes. Pericardium
was found to have signs of sero-fibrinous pericarditis. What is the most likely diagnosis?
A. Rheumatic heart disease
B. Myocardial infarction
C. Cardiomyopathy
D. Cardiosclerosis
E. –
Reason: Rheumatic heart disease is a chronic heart condition caused by rheumatic fever
caused by Streptococcus. Macrophages will only be found when there’s something to be
engulfed. The other diseases aren’t caused by a pathogen.

NSA TSMU Academic Team Krok 1 2013


KillingKrok2016 40

142. One of the factors that cause obesity is the inhibition of fatty acids oxidation due to:
A. Low level of carnitine
B. Impaired phospholipid synthesis
C. Excessive consumption of fatty foods
D. Choline deficiency
E. Lack of carbohydrates in the diet
Reason: Carnitine is a fat transporter, if there’s obesity most likely cause out of all the other
options would be its reduction.

143. The genetic defect of pyruvate carboxylase deficiency is the cause of delayed physical
and mental development and early death in children. This defect is characterized by
lacticemia, lactaciduria, disorder of a number of metabolic pathways. In particular, the
following process is inhibited:
A. Citric acid cycle and gluconeogenesis
B.Glycolysis and glycogenolysis
C. Glycogenesis and glycogenolysis
D. Lipolysis and lipogenesis
E.Pentose phosphate pathway and glycolysis

Reason: Pyruvate is first converted by pyruvate carboxylase to Oxaloacetate which is an


intermediate in the Citric Acid Cycle. Pyruvate carboxylase is involved in the synthesis of
phosphoenolpyruvate during Gluconeogenesis.

144. Deficiency of linoleic and linolenic acids in the body leads to the skin damage, hair
loss, delayed wound healing, thrombocytopenia, low resistance to infections. These
changes are most likely to be caused by the impaired synthesis of the following substances:
A. Eicosanoids
B. Interleukins
C. Interferons
D. Catecholamines
E. Corticosteroids
Reason: Eicosanoids exert complex control over many bodily systems, mainly in
inflammation or immunity and as messengers in CNS , growth during and after physical
activity
145. During ventricular systole, the cardiac muscle does not respond to additional
stimulation because it is in the phase of:
A. Absolute refractoriness
B. Relational refractoriness
C. Hyperexcitability
D. Subnormal excitability
E. There is no correct answer

NSA TSMU Academic Team Krok 1 2013


KillingKrok2016 41

Reason: Refractory period is recovery time of an excitable membrane to be ready for a


second stimulus once it returns to its resting state. That’s why the muscles do not respond to
additional stimulus in a normal state.

146. A mother had taken synthetic hormones during pregnancy. Her daughter was born
with hirsutism formally resembling of adrenal syndrome. Such manifestation of variability
is called:
A. Phenocopy
B. Mutation
C. Recombination
D. Heterosis
E. Replication
Reason: Phenocopy is a type of variation that is caused by Environmental conditions during
an organism development.

147. Since a patient has had myocardial infarction, atria and ventricles contract
independently from each other with a frequency of 60-70 and 35-40 per minute. Specify the
type of heart block in this case:
A. Complete atrioventricular
B. Partial atrioventricular
C. Sino-atrial
D. Intra-atrial
E. Intraventricular
Reason: The frequency of beats in the ventricles is reduced which suggest that impulses are
not travelling through the AV-node regularly and that is a sign of complete AV block.

148. A 67-year-old male patient consumes eggs, pork fat, butter, milk and meat. Blood test
results: cholesterol - 12 , 3 mmol/l, total lipids - 8,2 g/l, increased low density lipoprotein
fraction (LDL). What type of hyper-lipoproteinemia is observed in the patient?
A. Hyperlipoproteinemia type IIa
B. Hyperlipoproteinemia type I
C. Hyperlipoproteinemia type IIb
D. Hyperlipoproteinemia type IV
E. Cholesterol, hyperlipoproteinemia
Reason: Hyperlipoproteinemia type IIa is observed in patient with increased LDL.

149. A 12-year-old child has a viral infection complicated by obstructive bronchitis.


Bronchospasm can be eliminated by inhalations of a drug from the following
pharmacological group:

NSA TSMU Academic Team Krok 1 2013


KillingKrok2016 42

A. β2-agonists
B. M-anticholinergics
C. N- cholinomimetics
D. β2-adrenergic blockers
E. Analeptics
Reason: Bronchospasm can be eliminated by beta-agonists e.g Salbutamol, Other options
cause bronchospasm.

150. In course of an experiment there has been an increase in the nerve conduction velocity.
This may be caused by an increase in the concentration of the following ions that are
present in the solution around the cell:
A. Na+
B. K+ and C. K+ and
D. Ca2+ and Cl−
E. Ca2+

Reason: Sodium ions cause depolarization of cells, since there is increase in nerve
conduction, it can only mean that there’s increase of Sodium ions around the cell.

151. A male working as a blacksmith has been tested for auditory acuity. The tests revealed
50% hearing loss in the low frequency range and a near-normal auditory acuity in the high-
frequency range. This condition has been caused by the damage to the following structures
of the auditory system:
A. Corti’s organ - closer to helicotrema
B. Corti’s organ - closer to the oval foramen
C. Median part of the Corti’s organ
D. Muscles of the middle ear
E. Eardrum
Reason: The helicotrema is the part of the cochlear labyrinth where the scala vestibule and
tympani meet. The hair cells near this area best detect low frequency sounds.

152. In our country, routine preventive vaccinations against poliomyelitis involve using live
vaccine that is administered orally. What immunoglobulins are responsible for the
development of local post-vaccination immunity in this case?
A. Secretory IgA
B. IgM
C. IgG
D. Serum IgA
E. IgE

NSA TSMU Academic Team Krok 1 2013


KillingKrok2016 43

Reason: Secretory IgA are found in bodily fluids, please remember polio vaccine is given
orally, so in the saliva, stomach juice etc, this immunoglobulin will develop immunity for the
body.

153. An experiment proved that UV irradiated skin cells of patients with xeroderma
pigmentosum restore the native structure of DNA slower than the cells of healthy people
due to the defect in repair enzyme. What enzyme takes part in this process?
A. Endonuclease
B. RNA ligase
C. Primase
D. DNA polymerase
E. DNA gyrase
Reason: Xeroderma pigmentosaum is a rare deficiency in endonuclease which us usually
repair DNA after damage from UV light.

154. A patient who has recently come from an endemic area presents with elevated body
temperature, headache, chills, and malaise. These are symptoms which are typical for a
common cold. What laboratory tests are necessary to confirm or to refute the diagnosis of
malaria?
A. Microscopy of blood smears
B. Study of lymph node punctate
C. Urinalysis
D. Study of cerebrospinal fluid
E. Microscopy of bone marrow punctuate
Reason: The best way to diagnose Malaria is through Thick Blood drop Smear.

155. What condition may develop 15-30 minutes after re-administration of the antigen as a
result of the increased level of antibodies, mainly IgE, that are adsorbed on the surface of
target cells, namely tissue basophils (mast cells) and blood basophils?
A. Anaphylaxis
B. Antibody-dependent cytotoxicity
C. Delayed-type hypersensitivity
D. Immune complex hyperresponsiveness
E. Serum sickness
-Reason: Immediate type hypersensitivity, characterized by release of IgE, basophils, even
eosinophils.

156. 10 days after having quinsy caused by beta-hemolytic streptococcus a 6year-old child
exhibited symptoms of glomerulonephritis. What mechanism of glomerular lesion is most
likely in this case?

NSA TSMU Academic Team Krok 1 2013


KillingKrok2016 44

A. Immunocomplex
B. Cellular cytotoxicity
C. Anaphylaxis
D. Atopy
E. Antibody-dependent cell-mediated cytolysis
Reason: Immunocomplex hypersensitivity occurs when the Antigen of Streptococcus in this
case form a complex with an antibody against it.

157. A 22-year-old woman ate some seafood. 5 hours later the trunk and the distal parts of
limbs got covered with small itchy papules which were partially fused together. After one
day, the rash disappeared spontaneously. Specify the hypersensitivity mechanism
underlying these changes:

A. Atopy (local anaphylaxis)


B. Systemic anaphylaxis
C. Cellular cytotoxicity
D. Immune complex hypersensitivity
E. E. Antibody-dependent cell-mediated cytolysis
Reason: Firstly, this is an anaphylactic reaction because of its immediate reaction, then the
symptom of itchy rashes which disappeared after one day shows that it wasn’t systemic.

158. A hypertensive patient had been keeping to a salt-free diet and taking
antihypertensive drugs together with hydrochlorothiazide for a long time. This resulted in
electrolyte imbalance. What disorder of the internal environment occurred in the patient?
A. Hypochloremic alkalosis
B. Metabolic acidosis
C. Hyperkalemia
D. Hypermagnesemia
E. Increase in circulating blood volume
Reason: Hypocholermic alkalosis result from either low chloride intake or excessive chloride
wasting due to diuretic therapy.

159. A miner consulted a physician about the appearance of body rash followed by a loss of
appetite, bloating, duodenal pain, frequent bowel movements, and dizziness. Ovoscopic
probes of feces and duodenal contents revealed some eggs covered with a transparent
membrane through which 4-8 germinal cells could be seen. What disease is likely to have
occurred in the patient?
A. Ancylostomiasis
B. Strongyloidiasis
C. Trichocephaliasis
D. Hymenolepiasis

NSA TSMU Academic Team Krok 1 2013


KillingKrok2016 45

E. Enterobiasis
Reason: Ancyclostomiasis or miners anemia occur when hook worm in their large number
causes iron deficiency anemia.

160. Children with Lesch-Nyhan syndrome have a severe form of hyperuricemia


accompanied by the formation of tophi, urate calculi in the urinary tracts, as well as serious
neuro-psychiatric disorders. The cause of this disease is the reduced activity of the following
enzyme:
A. Hypoxanthine-guanine phosphoribosyltransferase
B. Xanthine oxidase
C. Dihydrofolate reductase
D. Thymidylate synthase
E. Karbamoyl phosphate synthetase
Reason: Simple! Lesch-Nyhan syndrome is caused by a deficiency of enzyme HGPRT located
on the X chromosome.

161. In a car accident a man got injured and lost a lot of blood. What changes in peripheral
blood are most likely to occur on the 2nd day after the injury?
A. Erythropenia
B. Hypochromia
C. Anisocytosis
D. Microplania
E. Significant reticulocytosis
Reason: Erythropenia is seen due to excessive loss of erythrocytes.

162. In the surgical ward, the dressing material was undergoing sterilization in an
autoclave. Through an oversight of a nurse the mode of sterilization was changed and the
temperature in the autoclave reached only 100oC instead of the due 1200C. What
microorganisms can stay viable under these conditions?
A. Bacilli and clostridia
B. Staphylococci and streptococci
C. Mold and yeast fungi
D. Salmonella and klebsiella
E. Corynebacteria and mycobacteria
Reason: Bacilli and Clostridia are resistant to heat, they require a higher temperature to be
destroyed.

163. As a result of a mechanical injury an over 10 cm long portion of a peripheral nerve was
damaged. This led to the impairment of the upper limb activity. The patient was offered

NSA TSMU Academic Team Krok 1 2013


KillingKrok2016 46

nerve transplantation. What glial cells will participate in regeneration and provide the
trophism of the injured limb?
A. Schwann cells
B. Fibrous cells
C. Protoplasmic cells
D. Microglia
E. Ependymal cells
Reason: Schwann cells are known for their roles in regeneration of damaged nerves. They
participate in phagocytosis of axons of damged cells and then guide regeneration by forming
the Schwann tunnel.

164. A 26-year-old woman at 40 weeks pregnant has been delivered to the maternity ward.
Objectively: the uterine cervix is opened, but the contractions are absent. The doctor has
administered her a hormonal drug to stimulate the labor.
Name this drug:
A. Oxytocin
B. Hydrocortisone
C. Estrone
D. Testosterone
E. ACTH
Reason: Oxytocin as a medication, is used to cause contraction of the uterus in order to start
labor

165. A patient has recurrent attacks of epileptic seizures and stays unconscious between
them. In order to stop convulsions the drugs of the following group should be used in the
first place:
A. Tranquilizers
B. Neuroleptics
C. Muscle relaxants
D. Sedatives
E. Analeptics
Reason: Tranquilizers are used to treat Epileptic seizures.

166. A patient with arthritis and varicose veins has been taking a non-steroidal
antiinflammatory drug for a long time, which caused thrombosis of skin veins. Which of the
following drugs might have caused this complication?
A. Celecoxib
B. Indomethacin
C. Aspirin
D. Phenylbutazone
E. Ibuprofen

NSA TSMU Academic Team Krok 1 2013


KillingKrok2016 47

Reason: One of the side effects of Celecoxib is thrombosis.

167. Students study the stages of gametogenesis. They analyze a cell having a
haploid number of chromosomes, and each chromosome consists of two chromatids. The
chromosomes are located in the equatorial plane of the cell. Such situation is typical for the
following stage of meiosis:
A. Metaphase of the second division
B. Metaphase of the first division
C. Anaphase of the first division
D. Anaphase of the second division
E. Prophase of the first division

Reason: In metaphase 2, the chromosomes arrange themselves on the metaphase plate


which is what is being described in the question.

168. A 35-year-old female patient underwent biopsy of the breast nodules. Histological
examination revealed enhanced proliferation of the small duct epithelial cells and acini,
accompanied by the formation of glandular structures of various shapes and sizes, which
were located in the fibrous stroma. What is the most likely diagnosis?
A. Fibroadenoma
B. Adenocarcinoma
C. Cystic breast
D. Invasive ductal carcinoma
E. Mastitis
Reason: Fibroadenoma is a benign tissue of both the fibrous tissue (fibroma) and the
glandular structure of the breast (adenoma).

169. Examination of the duodenal contents revealed some pear-shaped protozoa with two
nuclei and four pairs of flagella. The organisms had also two axostyles between the nuclei
and a ventral adhesive disc. What protozoan representative was found in the patient?
A. Lamblia
B. Toxoplasma
C. Leishmania
D. Intestinal trichomonad
E. Trypanosome
Reason: Lamblia is what is being described. Looks like a pear with eyes and two antennas.

170. A specimen of an onion rootlet includes a cell in which the fully condensed
chromosomes are located in the equatorial plane making the monaster. What phase of the
mitotic cycle is the cell in?
A. Metaphase

NSA TSMU Academic Team Krok 1 2013


KillingKrok2016 48

B. Early telophase
C. Prophase
D. Interphase
E. Late telophase
Reason: Metaphase- In this phase the cell's chromosomes align themselves in the middle of
the cells equator.
171. When examining a patient, the doctor revealed a tumor of the bronchus which
borders on the aorta. Which bronchus is affected?
A. Left principal
B. Right principal
C. Right upper lobar
D. Left upper lobar
E. Middle lobar
Reason: The Aorta arises from the left part of heart, the Left main bronchus too arises from
the left lung. A tumor would affect them both as they are located on the same side.

172. A 54-year-old female was brought to the casualty department after a car accident. A
traumatologist diagnosed her with multiple fractures of the lower extremities. What kind
of embolism is most likely to develop in this case?
A. Adipose
B. Tissue
C. Thromboembolism
D. Gaseous
E. Air
Reason: The long tubular bones are one of the places for fat storage. Hence the fracture of
these bones can lead to adipose embolism.

173. Microscopy of the bronchial wall revealed atrophy of the mucosa, metaplastic change
from columnar to squamous epithelium, an increase in the number of goblet cells, diffuse
infiltration of the bronchial wall with lymphoplasmacytic elements with a large number of
neutrophilic granulocytes, pronounced sclerosis. Specify the morphological form of
bronchitis:
A. Chronic purulent bronchitis
B. Acute bronchitis
C. Polypoid chronic bronchitis
D. Acute purulent bronchitis
E. Chronic bronchitis
Reason: Atrophy of the mucosa shows that the disease has occurred for a while i.e it is a
chronic disease. Then the presence of neutrophilic granulocytes shows that the course of the
disease is purulent.

NSA TSMU Academic Team Krok 1 2013


KillingKrok2016 49

174. Due to the blockage of the common bile duct (which was radiographically confirmed),
the biliary flow to the duodenum was stopped. We should expect the impairment of:
A. Fat emulsification
B. Protein absorption
C. Carbohydrate hydrolysis
D. Secretion of hydrochloric acid
E. Salivation inhibition
Reason: The blockade of the common bile duct causes the stoppage of bile acids whose main
function is the emulsification of fat in the small intestine.

175. Typical manifestations of food poisoning caused by C. botulinum are double vision,
abnormal functioning of the swallowing and breathing. These symptoms develop as a
result of:
A. Exotoxin effects
B. Enterotoxin effects
C. Enterotoxic shock development
D. Activation of adenylate cyclase
E. Pathogen adhesion to the enterocyte receptors
Reason: Clostridium botulinum contains exotoxins A, B, E which affect humans.

176. At the stage of translation in the rough endoplasmic reticulum, the ribosome moves
along the mRNA. Amino acids are joined together by peptide bonds in a specific sequence,
and thus polypeptide synthesis takes place. The sequence of amino acids in a polypeptide
corresponds to the sequence of:
A. mRNA codons
B. tRNA nucleotides
C. tRNA anticodons
D. rRNA nucleotides
E. rRNA anticodons
Reason: The sequence of amino acids always corresponds to mRNA codons which consist of
4 bases including uracil, cytosine, guanine and adenine.

177. After the prolonged vomiting a pregnant 26-year-old woman was found to have the
reduced volume of circulating blood. What change in the total blood volume can be the
case?
A. Polycythemic hypovolemia
B. Simple hypovolemia
C. Oligocythemic hypovolemia
D. Polycythemic hypervolemia

NSA TSMU Academic Team Krok 1 2013


KillingKrok2016 50

E. Oligocythemic hypervolemia
Reason: Polycythemia hypovolemia decrease plasma, increase blood viscosity and
hematocrit level due to diarrhea, prolonged vomiting, shock, and hyperventilation.

178. On allergological examination a patient has been diagnosed with pollinosis. Specific
desensitization can be performed by:
A. Intermittent administration of allergen
B. Antihistamines
C. Glucocorticoids
D. Administration of saline
E. –
Reason: Desensitization is a method used to reduce or eliminate an organism’s negative
reaction to a substance or stimulus.

179. A patient consulted a physician about chest pain, cough, fever. Roentgenography of
lungs revealed eosinophilic infiltrates which were found to contain the larvae. What kind of
helminthiasis are these presentations typical for?
A. Ascariasis
B. Echinococcosis
C. Fascioliasis
D. Cysticercosis
E. Trichinosis
Reason: Ascariasis is a helminthes which spends the larvae stage in the lungs where they
later move on to the small intestine.

180. A patient with signs of osteoporosis and urolithiasis has been admitted
to the endocrinology department. Blood test revealed hypercalcemia and
hypophosphatemia. These changes are associated with abnormal synthesis of the following
hormone:
A. Parathyroid hormone
B. Calcitonin
C. Cortisol
D. Aldosterone
E. Calcitriol
Reason: Increased PTH( Hyperparathyroidism) causes increased removal of Calcium from
bones to blood which causes Osteoporosis and Hypercalcemia and also increases movement
of phosphate from blood to bones which causes Hypophosphatemia.

181. Histological examination of the biopsy material obtained from the lower third of the
esophagus of a 57- year-old male with the symptoms of continuous reflux revealed the

NSA TSMU Academic Team Krok 1 2013


KillingKrok2016 51

change of the stratified squamous epithelium to the single layer columnar glandular
epithelium with signs of mucus production. Specify the pathological process in the mucous
membrane:
A. Metaplasia
B. Hyperplasia
C. Hypertrophy
D. Organization
E. Regeneration
Reason: Any change of normal cell epithelia to another variant or type of epithelium is
known as Metaplasia.

182. A 30-year-old female exhibits signs of virilism (growth of body hair, balding temples,
and menstrual disorders). This condition can be caused by the overproduction of the
following hormone:
A. Testosterone
B. Oestriol
C. Relaxin
D. Oxytocin
E. Prolactin

Reason: The female is beginning to exhibit the features of a man which can only be caused
by overproduction of Testosterone.

183. A patient with bacterial periodontitis has been administered iontophoresis with the
use of iodine solution. Specify the mechanism of therapeutic action of this agent:
A. Substitution of hydrogen atoms when the protein amino group contains a nitrogen
atom
B. Reduction of the nitro group under the effect of nitroreductase
C. Albumin formation
D. Changing the surface tension of the bacterial cell membrane
E. Inhibition of the cell wall formation
Reason: Iodine is reduced to hydrogen iodide through substitution.

184. A patient with extensive myocardial infarction has developed heart failure. What
pathogenetic mechanism contributed to the development of heart failure in the patient?
A. Reduction in the mass of functioning myocardiocytes
B. Pressure overload
C. Volume overload
D. Acute cardiac tamponade
E. Myocardial reperfusion injury

NSA TSMU Academic Team Krok 1 2013


KillingKrok2016 52

Reason: Infarction occurs as a result of decreased blood flow to the myocardium, after
infarction a scar(fibrous tissue) is present on affected part. These parts can no longer
function i.e contract.

185. A patient who had been continuously taking drugs blocking the production of
angiotensin II developed bradycardia and arrhythmia. A likely cause of these disorders is:
A. Hyperkalemia
B. Hypokalemia
C. Hypernatremia
D. Hypocalcemia
E. Hypercalcemia
Reason: In summary, Aldosterone causes hypokalemia by excreting potassium and
reabsorbing sodium. Angiotensin Receptor blockers antagonize Aldosterone so therefore
cause Hyperkalemia, that is they now cause decreased excretion of potassium which will
now be excess in blood.

186. A patient has arterial hypertension. What long-acting drug from the group of calcium
channel blockers should be prescribed?
A. Amlodipine
B. Octadine
C. Pyrroxanum
D. Atenolol
E. Reserpine
Reason: This drug is a calcium channel blocker used to treat high blood pressure, chest pain
and other conditions caused by coronary artery disease.

187. Human skin has a high breaking strength. It is known that the skin consists of epithelial
tissue and two kinds of connective tissue. Which of the following tissues provides the skin
strength?
A. Unformed dense connective tissue
B. Stratified squamous epithelium
C. Loose connective tissue
D. Single-layer epithelium
E. Transitional epithelium

Reason: Dense connective tissue is composed of type 1 collagen which are rows of
fibroblasts, they form strong ropelike structures such as tendons and ligaments, they provide
strength.

NSA TSMU Academic Team Krok 1 2013


KillingKrok2016 53

188. As a result of a home injury, a patient suffered a significant blood loss, which led to a
fall in blood pressure. Rapid blood pressure recovery after the blood loss is provided by the
following hormones:
A. Adrenaline, vasopressin
B. Cortisol
C. Sex hormones
D. Oxytocin
E. Aldosterone
Reason: Adrenalin will increase heart contraction, which means more blood will be pumped.
Secretion of Vasopressin will increase fluid in the body which can help for adequate perfusion
in cells. Fluid is necessary when blood is being lost.
189. A patient with constant headaches, pain in the occipital region, tinnitus, and dizziness
has been admitted to the cardiology department. Objectively: AP- 180/110 mm Hg, heart
rate - 95/min. Radiographically, there is a stenosis of one of the renal arteries.
Hypertensive condition in this patient has been caused by the activation of the following
system:
A. Renin-angiotensin
B. Hemostatic
C. Sympathoadrenal
D. Kinin
E. Immune
Reason: When renal blood flow is reduced, the juxtaglomerular cells in kidneys covert
prorenin in rennin and secrete it directly into the blood. Plasma renin then carries out the
conversion of angiotensinogen released by liver to angiotensin I, which is now converted to
angiotensin II by angiotensin-converting enzyme found in lungs. It then causes
Vasoconstriction and increases blood pressure.

190. A patient complains that at the bare mention of the tragic events that once occurred
in his life he experiences tachycardia, dyspnea and an abrupt rise in blood pressure. What
structures of the CNS are responsible for these cardiorespiratory reactions in this patient?
A. Cerebral cortex
B. Cerebellum
C. Lateral hypothalamic nuclei
D. Specific thalamic nuclei
E. Quadrigemina of mesencephalon
Reason: Cerebral cortex is the part of the brain responsible for cardiorespiratory reactions.

191. A patient consulted a dentist about limited mouth opening (trismus). He has a history
of a stab wound of the lower extremity. What infection may cause these symptoms?
A. Tetanus
B. Brucellosis

NSA TSMU Academic Team Krok 1 2013


KillingKrok2016 54

C. Whooping cough
D. Wound anaerobic infection
E. Tularemia
Reason: Trismus is the spasm of the jaw muscles causing the mouth to remain tightly close
as a symptom of tetanus.

192. Anatomical dead space is the portion of the air that is left in the airways after
expiration. The reduction of the anatomical dead space is typical for the following
situation:
A. Tracheostomy
B. Forward flexion of head
C. Turning the lying patient on his left side
D. Turning the lying patient on his right side
E. Breathing through the mouth

Reason: This is a surgical procedure used to reduce anatomical dead space.


193. Analysis of the experimental spirogram of a 55-year-old person revealed a decrease in
tidal volume and respiratory amplitude compared to the situation of ten years ago. The
change in these indicators is caused by:
A. Decreased force of respiratory muscle contraction
B. Gas composition of the air
C. Physical build of a person
D. Height of a person
E. Body mass of a person
Reason: Easy! Old age causes decrease in muscle force and contraction so with time tidal
volume will be decreased.

194. A patient underwent a course of treatment for atherosclerosis. Laboratory tests


revealed an increase in the antiatherogenic lipoprotein fraction in the blood plasma. The
treatment efficacy is confirmed by the increase in:
A. HDL
B. VLDL
C. IDL
D. LDL
E. Chylomicrons
Reason: HDL is a lipoprotein that carries cholesterol from the tissues to the liver. It is an
antiatherogenic lipoprotein.

195. A 65-year-old female patient has chronic constipations due to the colon hypotonia.
What drug should be chosen in this case?
A. Bisacodyl

NSA TSMU Academic Team Krok 1 2013


KillingKrok2016 55

B. Castor oil
C. Magnesium sulfate
D. Neostigmine methylsulfate
E. Metoclopramide
Reason: It is a drug used for the treatment of chronic constipation.

196. A female patient complains of vision impairment. On examination she was found to
have obesity, fasting hyperglycemia. What complication of diabetes can cause vision
impairment?
A. Microangiopathy
B. Macroangiopathy
C. Atherosclerosis
D. Neuropathy
E. Glomerulopathy
Reason: Microangiopathy is one of the other complications of diabetes mellitus.

197. Analysis of the family history of children with Van der Woude syndrome revealed that
in their families one of the parents had the typical for this syndrome defects (cleft lip and
palate, lip pits regardless of gender). What is the type of inheritance of this syndrome?
A. Autosomal dominant
B. X-linked recessive
C. X-linked dominant
D. Autosomal recessive
E. Multifactorial
Reason: Van der Woude syndrome is an autosomal dominant disease.

198. Administration of doxycycline hydrochloride caused an imbalance of the


symbiotic intestinal microflora. Specify the kind of imbalance caused by the antibiotic
therapy:
A. Dysbacteriosis
B. Sensibilization
C. Idiosyncrasy
D. Superimposed infection
E. Bacteriosis
Reason: Dysbacetriosis is microbial disbalance in the human body normally caused due to a
long term use of antibiotics.

199. A 3-year-old child had eaten some strawberries. Soon he developed a rash and itching.
What was found in the child’s leukogram?

NSA TSMU Academic Team Krok 1 2013


KillingKrok2016 56

A. Eosinophilia
B. Hypolymphemia
C. Neutrophilic leukocytosis
D. Monocytosis
E. Lymphocytosis
Reason: This because eosinophils are white blood cells which binds with mast cells causing
release of cell mediators. They are released during allergies or when you have helminthes.

200. A 12-year-old patient was found to have blood serum cholesterol at the rate of 25
mmol/l. The boy has a history of hereditary familial hypercholesterolemia, which is caused
by the impaired synthesis of the following protein receptors:
A. Low density lipoproteins
B. High density lipoproteins
C. Chylomicrons
D. Very low density lipoproteins
E. Intermediate density lipoproteins

Reason: Familial hypercholesterolemia is a genetic disorder. It is caused by a defect on


chromosome 19. The defect makes the body unable to remove low density lipoprotein (LDL,
or "bad") cholesterol from the blood. This results in a high level of LDL in the blood.

SUCCESS!!!
…and may the odds be ever in your favour.

NSA TSMU Academic Team Krok 1 2013


2014 Krok I, Explained
INTRODUCTION

For Krok past questions, the answers are usually the first option. We felt this encourages cramming.
The answers in this material have been shuffled, and then below each question is an explanation for
the correct option. In some explanations, we used elimination method (explaining why all the other
options are not the answer).

Also, some frequently occurring concepts are explained so that even if a question is ‘tweaked’,
students can still handle them.

1. The infectious diseases department of a hospital admitted a patient with nausea, liquid stool with
mucus and blood streaks, fever, and weakness. Dysentery was suspected. What method of
laboratory diagnostics should be applied to confirm the diagnosis?

A. Serological

B. Mycological

C. Bacteriological

D. Microscopic

E. Protozoological

EXP: The causative agent of dysentery is Shigella dysenteriae which is a bacteria so the correct
answer is C. This involves testing the ‘biologic’ materials i.e stool for the bacteria.

2. Malaria is treated with structural analogs of vitamin B2 (riboflavin). These drugs disrupt the
synthesis of the following enzymes in plasmodium:

A. Cytochrome oxidase

B. FAD-dependent dehydrogenase

C. Peptidase

D. NAD-dependent dehydrogenase

E. Aminotransferase

EXP: The answer is B because that is the coenzyme of vitamin B2 along with FMN (Flavin-
Mononucleotide).

3. A 6-year-old child with suspected active tuberculous process underwent the diagnostic Mantoux
test. What immunobiological preparation was injected?

Krok 1: 2014 Past Questions, Answers Explained NSA-TSMU Academic Team


A. BCG vaccine

B. DTP vaccine

C. Tularinum

D. Td vaccine

E. Tuberculin

EXP: Answer is E. Tuberculin is the intradermal injection for mantoux test to confirm tuberculosis
because TB is still suspected. BCG vaccine is vaccination against TB.

4. In a young man during exercise, the minute oxygen uptake and carbon dioxide emission equalled
to 1000 ml. What substrates are oxidized in the cells of his body?

A. Proteins

B. Fats

C. Carbohydrates

D. Carbohydrates and fats

E. Carbohydrates and proteins

EXP: Answer is C. During exercise, glucose is used up and it’s a product of carbohydrate

5. A sportsman spontaneously held breath for 40 seconds, which resulted in an increase in heart rate
and systemic arterial pressure. Changes of these indicators are due to activation of the following
regulatory mechanisms:

A. Unconditioned parasympathetic reflexes

B. Conditioned sympathetic reflexes

C. Conditioned parasympathetic reflexes

D. Unconditioned sympathetic reflexes

EXP; Answer is D. Changes of heart rate and blood pressure are unconditioned responses. The
‘increase’ here is due to action of the sympathetic nervous system through production of epinephrine
(flight or fight reflex). Parasympathetic NS reduces HR and BP via acetylcholine.

6. A 53-year-old male patient is diagnosed with Paget’s disease. The concentration of oxyproline in
daily urine is sharply increased, which primarily means intensified disintegration of:

A. Collagen

B. Keratin

C. Albumin

D. Hemoglobin

Krok 1: 2014 Past Questions, Answers Explained NSA-TSMU Academic Team


E. Fibrinogen

HINT: Oxyproline and Hydroxyproline play a key role in stability of Collagen. Increased excretion of
Oxyproline will reduce stability of collagen, therefore it leads to Paget’s disease which is a pathology
of the bone. Answer is A.

7. A patient has hoarseness of voice. During laryngoscopy a gray-white larynx tumor with PAPILLARY
surface has been detected. Microscopic investigation has shown the following: growth of connective
tissue covered with multilayer, strongly keratinized pavement epithelium, no cellular atypia. What is
the most likely diagnosis?

A. Fibroma

B. Polyp

C. Angioma

D. Papilloma

E. Angiofibroma

EXP: A papilloma is a benign epithelial tumor. The absence of atypia confirms the tumor as benign.
Papillary surface of the epithelial tumor nails it. Answer is D .

8. During autopsy approximately 2,0 liters of pus have been found in the abdominal cavity of the
corpse. Peritoneum is lusterless and has grayish shade, serous tunic of intestines has grayish-colored
coating that is easily removable. Specify the most likely type of peritonitis in the patient:

A. Hemorrhagic peritonitis

B. Fibrinopurulent peritonitis

C. Serous peritonitis

D. Tuberculous peritonitis

EXP; Grayish shade of peritoneum indicates purulence. The grayish-colored coating points to
presence of fibrin. Answer is B

9. Autopsy of a dead patient revealed bone marrow hyperplasia of tubular and flat bones (pyoid
marrow), splenomegaly (6 kg) and hepatomegaly (5 kg), enlargement of all lymph node groups.
What disease are the identified changes typical for?

A. Chronic lymphocytic leukemia

B. Multiple myeloma

C. Polycythemia Vera

D. Hodgkin’s disease

E. Chronic myelogenous leukemia

Krok 1: 2014 Past Questions, Answers Explained NSA-TSMU Academic Team


EXP; Pyoid marrow is characteristic for Myelogenous leukemia. Answer is E.

10. Autopsy of the dead patient who died from pulmonary edema revealed a large yellow-grey nidus
in the myocardium, and a fresh thrombus in the coronary artery. What is the most likely diagnosis?

A. Cardiosclerosis

B. Myocarditis

C. Amyloidosis

D. Myocardial infarction

E. Cardiomyopathy

REASON; The large yellow-grey area or nidus indicates infarction or death of the myocytes in that
area. This is seen to have been caused by the thrombus in the coronary artery that blocked blood
flow to that area of the heart. Answer is D.

11. An animal experiment is aimed at studying the cardiac cycle. All the heart valves are closed.
What phase of the cycle is characterized by this status?

A. Isometric contraction

B. Asynchronous contraction

C. Protodiastolic period

D. Rapid filling

E. Reduced filling

REASON; Systole is the part of the cardiac cycle when the ventricles contract. In early systole, all the
valves are closed, and the ventricles contract without a change in volume, this is known as
‘Isovolumetric/Isometric Contraction’. So the answer is A.

12. Cyanide is a poison that causes instant death of the organism. What enzymes found in
mitochondria are affected by cyanide?

A. Cytochrome oxidase (aa3)

B. Flavin enzymes

C. Cytochrome 5

D. NAD+-dependent dehydrogenase

E. Cytochrome P-450

HINT; Answer is A. Cyanide ion stops cellular respiration by inhibiting the enzyme cytochrome c
oxidase and disrupting the electron transport chain.

Krok 1: 2014 Past Questions, Answers Explained NSA-TSMU Academic Team


13. Increased HDL levels decrease the risk of atherosclerosis. What is the mechanism of HDL anti-
atherogenic action?

A. They supply tissues with cholesterol

B. They are involved in the breakdown of cholesterol

C. They activate the conversion of cholesterol to bile acids

D. They promote absorption of cholesterol in the intestine

E. They remove cholesterol from tissues

HINT; High Density Lipoproteins (HDL) removes cholesterol from tissues so that they do not
accumulate in these tissues, therefore preventing process of atherosclerosis. This is why it is known
as good cholesterol. Answer is E.

14. It has been found out that one of a pesticide components is sodium arsenate that blocks lipoic
acid. Which enzyme activity is impaired by this pesticide?

A. Microsomal oxidation

B. Pyruvate dehydrogenase complex

C. Methemoglobin reductase

D. Glutathione peroxidase

E. Glutathione reductase

EXP; Pyruvate dehydrogenase converts pyruvate to acetyl-CoA in the CA cycle. It acts on pyruvate
and lipoic acid. So sodium arsenate has to impair this enzyme complex. The answer is B.

15. Stool culture test revealed in a 6- month-old bottle-fed baby the strain of intestinal rod-shaped
bacteria of antigen structure 0-111. What diagnosis can be made?

A. Gastroenteritis

B. Choleriform disease

C. Food poisoning

D. Colienteritis

E. Dysentery-like disease

EXP; The antigen strain 0-111 is specific to E.coli which is a rod shaped bacteria and this bacteria
affects intestine since it is also a normal flora of the intestine. Hence, it can cause colienteritis, which
is inflammation of the intestines. Answer is D

16. A boy referred to a genetics clinic was found to have 1 drumstick in blood neutrophils. The boy is
likely to have the following syndrome:

Krok 1: 2014 Past Questions, Answers Explained NSA-TSMU Academic Team


A. Down’s

B. Turner’s

C. Edwards’

D. Klinefelter’s

E. Trisomy X

EXP; Normally, boys should have no barr body. (46XY). Drumstick means ‘barr body’. Klinefelter
syndrome has 1 barr body i.e XXY (47XXY). Answer is D

17. A drycleaner’s worker has been found to have hepatic steatosis. This pathology can be caused by
the disruption of synthesis of the following substance:

A. Phosphatidylcholine

B. Tristearin

C. Urea

D. Phosphatidic acid

E. Cholic acid

EXP; Steatosis, also called fatty degeneration is the abnormal retention of lipids. Phosphatidylcholine
acts as a surfactant that prevents lipid droplets from forming large droplets. So disorder of its
synthesis will cause deposition of fats in the liver. Answer is D

18. Ascarid eggs have been detected during stool analysis. What drug should be prescribed?

A. Nystatin

B. Mebendazole

C. Chloramphenicol

D. Tetracycline

E. Furazolidone

HINT; Ascaris is a helminth and its treated by an anti-helminthic drug. Most of them ends in
‘’dazole’’. Answer is B. Nystatin is antifungal. C, D and E are Antibiotics.

19. Bacteriological examination of the urine of the patient with pyelonephritis revealed
microorganisms that produced yellow-green pigment and a characteristic odor in meat-peptone
agar. What are they called?

A. Pseudomonas

B. Escherichia

C. Proteas

Krok 1: 2014 Past Questions, Answers Explained NSA-TSMU Academic Team


D. Klebsiella

E. Azotobacter

EXP; Pseudomonas secretes a pigment pyoverdine which is responsible for the yellow-green pigment.
Answer is A

20. Feces of a patient contain high amount of undissociated fats and have grayish-white color.
Specify the cause of this phenomenon:

A. Hypoactivation of pepsin by hydrochloric acid

B. Hypovitaminosis

C. Obturation of bile duct

D. Enteritis

E. Irritation of intestinal epithelium

EXP: Obturation or blockage of bile duct prevents bile from being released thereby causing fats to be
undigested. Answer is C

21. A 46-year-old female patient consulted a doctor about pain in the small joints of the upper and
lower limbs. The joints are enlarged and shaped like thickened nodes. Serum test revealed an
increase in urate concentration. This might be caused by a disorder in metabolism of:

A. Carbohydrates

B. Lipids

C. Purines

D. Pyrimidines

E. Amino acids

EXP; This patient has gout which is caused by disorder of purine metabolism because the end product
of this metabolism is uric acid, so its disorder will lead to uric acid increase. Uric acid in the form of
urate will deposits in joints forming those nodules (also called “tophi”). Ans = C

22. A 26-year-old female consulted a doctor about having stool with white flat moving organisms
resembling noodles. Laboratory analysis revealed proglottids with the following characteristics:
long, narrow, with a longitudinal channel of the uterus with 17-35 lateral branches on each side.
What kind of intestinal parasite was found?

A. Taenia solium

B. Hymenolepis nana

C. Taeniarhynchus saginatus

D. Diphyllobothrium latum

Krok 1: 2014 Past Questions, Answers Explained NSA-TSMU Academic Team


E. Echinococcus granulosus

HINT; Proglottids are found in tapeworms. T. saginata has uterus of 17-35 while T. solium with uterus
of 7-13. Ans = C

23. A male patient is 28 years old. Histological study of a cervical lymph node revealed a change of
its pattern due to the proliferation of epithelioid, lymphoid cells and macrophages having nuclei in
form of a horseshoe. . In the center of some cell clusters there were non-structured light-pink areas
with fragments of nuclei. What disease are these changes typical for?

A. Hodgkin’s disease

B. Actinomycosis

C. Tumor metastasis

D. Syphilis

E. Tuberculosis

HINT; TB occurs mainly in immunocompromised patients. So these epitheloid cells (granuloma) are
immune cells (histiocytes) which are suppose to fight the bacteria but the patient is
immunocomprised so they accumulate and form large giant cells which are arranged in horseshoe
pattern. Ans = E

24. A biochemical urine analysis has been performed for a patient with progressive muscular
dystrophy. In the given case muscle disease can be confirmed by the high content of the following
substance in urine:

A. Creatine

B. Porphyrin

C. Urea

D. Hippuric acid

E. Creatinine

HINT; In a normal situation, creatine is broken down to creatinine before excretion…but in PMD, the
destruction of muscles results in a high quantity of creatine being excreted, so it will be seen in urine.
Ans = A

25. While examining foot blood supply a doctor checks the pulsation of a large artery running in the
separate fibrous channel in front of articulatio talocruralis between the tendons of long extensor
muscles of hallux and toes. What artery is it?

A. A. tibialis anterior

B. A. dorsalis pedis

C. A. tarsea medialis

Krok 1: 2014 Past Questions, Answers Explained NSA-TSMU Academic Team


D. A. tarsea lateralis

E. A. fibularis

HINT: Dorsal pedis is the largest artery in the foot where pulse is able to be checked. Ans = B

26. A patient with chronic heart failure with edema has increased level of blood aldosterone. What
diuretic would be most effective in this case?

A. Triamterene

B. Spironolactone

C. Acetazolamide

D. Hydrochlorothiazide

E. Furosemide

HINT; Retention of sodium & water contributes to development of edema and Aldosterone reabsorbs
H2O and Na. So aldosterone levels are increased and have to be blocked thus sparing potassium and
excreting Na. Spirinolactone and Triamterene are both K sparing agents but Spirinolactone has direct
antagonistic effect on aldosterone but triamterene just affects electrolyte transport but does not
block aldosterone. Ans = B

27. During autopsy the following has been revealed: the meninges of the upper cerebral
hemispheres are extremely plethoric, of yellow green color and are soaked with purulent effluent.
What kind of meningitis is characterized by such clinical presentations?

A. Meningococcal meningitis

B. Tuberculous meningitis

C. Grippal meningitis

D. Anthrax-induced

E. Epidemic typhus-induced

HINT: Meningococcus is the only causative agent in the options that can cause purulent effluent type
of meningitis. Ans = A

28. A 41-year-old male patient has a history of recurrent attacks of heartbeats (paroxysms), profuse
sweating, and headaches. Examination revealed hypertension hyperglycemia, increased basal
metabolic rate, and tachycardia. These clinical presentations are typical for the following adrenal
pathology:

A. Hyperfunction of the medulla

B. Hypofunction of the medulla

C. Hyperfunction of the adrenal cortex

Krok 1: 2014 Past Questions, Answers Explained NSA-TSMU Academic Team


D. Hypofunction of the adrenal cortex

E. Primary aldosteronism

EXP; If it’s C, we’ll see symptoms like moon face. If E, muscle spasm, numbness, and polyuria will be
seen. So A is correct because adrenal medulla secretes epinephrine and we know its function.

29. A comminuted fracture of infraglenoid tubercle caused by shoulder joint injury has been
detected during X-ray examination of a patient. What muscle tendon attached at this site has been
damaged?

A. Long head of m. biceps brachii

B. Medial head of m. triceps brachii

C. Lateral head of m. triceps brachii

D. Long head of m. triceps brachii

E. Short head of m. biceps brachii

HINT: Infraglenoid tubercle is part of scapula from which long head of triceps brachii originates.
Ans = D.

30. A patient has increased thickness of alveolar-capillary membrane caused by a pathologic


process. The direct consequence will be reduction of the following value:

A. Diffusing lung capacity

B. Oxygen capacity of blood

C. Respiratory minute volume

D. Alveolar ventilation of lungs

E. Expiratory reserve volume

HINT: Alveolar capillary membrane is thin which allows transfer of gases so its thickness will do
otherwise. DLC measures the ability of lungs to transfer gas from alveolar space to RBC in pulmonary
vessel, so when the alveolar membrane becomes thick, the DLC reduces as gases are not really
transferred. Ans = A

31. What drug will be most appropriate for the patient who has chronic gastritis with increased
secretion?

A. Pancreatine

B. Pepsin

C. Pirenzepine

D. Aprotinin

Krok 1: 2014 Past Questions, Answers Explained NSA-TSMU Academic Team


E. Chlorphentermine

HINT; Pepsin is an enzyme produced by stomach. Aproptinin is a pancreatic trypsin inhibitor.


Pancreatinine Is produced by exocrine cells of pancreas. Chlorphentermine acts as a selective
serotonin releasing agent. So C is correct as it reduces gastric acid secretion.

32. As a result of careless handling of an iron, a 34-year-old female patient has got acute pain,
redness, swelling of her right index finger. A few minutes later, there appeared a blister filled with a
transparent liquid of straw yellow color. The described changes are a manifestation of the following
pathological process:

A. Traumatic edema

B. Exudative inflammation

C. Vacuolar degeneration

D. Proliferative inflammation

E. Alternative inflammation

HINT; First of all, with the signs, its an inflammation. (so A and C are out). Exudate is any fluid
produced during an inflammation and here, a fluid was produced. Ans = B

33. A 42-year-old male with a lesion of the ulnar nerve is unable to flex the II and V fingers to the
midline. Which muscle function is impaired in this case?

A. Dorsal interosseous muscle

B. Fidicinales

C. Short palmar muscle

D. Palmar interosseous muscles

E. Abductor muscle of little finger

HINT; All interosseous muscles are innervated by ulnar. The dorsal interosseous muscles control
adduction while palmar interosseous control abduction. Ans = D

34. A smear from the tonsillar coating of a patient with suspected diphtheria was found to contain
blue bacilli with a thickening at the poles. What method of smear staining was used?

A. Leffler

B. Burri

C. Hins

D. Gram

E. Neisser

Krok 1: 2014 Past Questions, Answers Explained NSA-TSMU Academic Team


HINT; C. diphtheria, is also known as KLEBS-LEFFLER BACCILUS( the inventors) and leffler stains blue.
Ans = A

35. A child was born asphyxiated. What drug must be administered to the newborn to stimulate
breathing?

A. Proserin

B. Lobeline

C. Prazosin

D. Atropine

E. Aethimizolum

HINT; Proserin is a cholinesterase inhibitor, it stimulates muscles by improving nerve impulse


transmission. Lobeline acts on nicotinic receptors. Prazosin is an alpha1 blocker. Atropine is an
anticholinesterase inhibitor. So E, Aethimizolum is correct because its an ANALEPTIC(it stimulates
breathing center.

36. A 40-year-oldpatient with the progressing staphylococcal purulent periodontitis developed


purulent inflammation of bone marrow spaces of the alveolar process, and then of the body of
mandible. Microscopy revealed thinning of bone trabeculae, foci of necrosis, bone sequesters
surrounded by the connective tissue capsule. What is the most likely diagnosis?

A. Acute osteomyelitis

B. Parodontome

C. Chronic osteomyelitis

D. Chronic fibrous periostitis

E. Purulent abscess

HINT: The highlighted problems can only occur in Chronic course of Osteomyelitis. Ans = C

37. Human red blood cells do not contain mitochondria. What is the main pathway for ATP
production in these cells?

A. Anaerobic glycolysis

B. Aerobic glycolysis

C. Oxidative phosphorylation

D. Creatine kinase reaction

E. Cyclase reaction

HINT: Ans = A. Because all the other options require the mitochondria, or oxygen.

Krok 1: 2014 Past Questions, Answers Explained NSA-TSMU Academic Team


38. A 40-year-old female patient has undergone thyroidectomy. Histological study of thyroid gland
found the follicles to be of different size and contain foamy colloid, follicle epithelium is high and
forms papillae, there is focal lymphocytic infiltration in stroma. Diagnose the thyroid gland disease:

A. Basedow’s disease

B. Hashimoto’s thyroiditis

C. Riedel’s thyroiditis

D. De Quervain’s disease

E. Nodular goiter

HINT; Histologically, the different size and amount of follicles will present clinically as diffused goiter
which is a clinical symptom of GRAVES DIESEASE(another name is BASEDOW DISEASE). Ans = A

39. A 43-year-old-patient has arterial hypertension caused by an increase in cardiac output and
general peripheral resistance. Specify the variant of hemodynamic development of arterial
hypertension in the given case:

A. Eukinetic

B. Hyperkinetic

C. Hypokinetic

D. Combined

E. –

HINT: In Eukinetic Arterial Hypertension, both systolic and diastolic BP will be increased i.e there will
be increase in cardiac output. Ans = A

40. A patient has been hospitalized with provisional diagnosis of virus B hepatitis. Serological
reaction based on complementation of antigen with antibody chemically bound to peroxidase or
alkaline phosphatase has been used for disease diagnostics. What is the name of the applied
serological reaction?

A. Immune-enzyme analysis

B. Radioimmunoassay technique

C. Immunofluorescence test

D. Bordet-Gengou test

E. Antigen-binding assay

EXP: The binding of the antibody-antigen complement to enzymes makes option A the correct
answer.

Krok 1: 2014 Past Questions, Answers Explained NSA-TSMU Academic Team


41. A patient has been hospitalized with provisional diagnosis of botulism. What serological reaction
should be used to reveal botulinum toxin?

A. Agglutination reaction

B. Neutralization reaction

C. Bordet-Gengou test

D. Precipitation reaction

E. Immunofluorescence test

HINT: Clostridium botulinum excretes neurotoxin, Neutralization reaction with anti-toxin is the only
way to reveal the exotoxin. Answer is B

42. Examination of a 52-year-old female patient has revealed a decrease in the amount of red blood
cells and an increase in free hemoglobin in the blood plasma (hemoglobinemia). Color index is 0,85.
What type of anemia is being observed in the patient?

A. Acquired hemolytic

B. Hereditary hemolytic

C. Acute hemorrhagic

D. Chronic hemorrhagic

E. Anemia due to diminished erythropoiesis

HINT; No history of bleeding (so C and D). No family history of it ( so B is out). E is also out because
there is no reticulocytopenia on peripheral smear. So A is correct.

43. The processes of heat transfer in a naked person at room temperature have been studied. It was
revealed that under these conditions the greatest amount of heat is transferred by:

A. Heat conduction

B. Convection

C. Heat radiation

D. Evaporation

HINT: Heat Radiation doesn’t require body contact or flow of fluid to lose heat unlike Conduction and
Convection respectively. So the answer is B.

44. Urine analysis has shown high levels of protein and erythrocytes in urine. This can be caused by
the following:

A. Effective filter pressure

B. Hydrostatic blood pressure in glomerular capillaries

Krok 1: 2014 Past Questions, Answers Explained NSA-TSMU Academic Team


C. Hydrostatic primary urine pressure in capsule

D. Oncotic pressure of blood plasma

E. Renal filter permeability

HINT; Normally, protein and RBC in large quantities should not be seen in urine. Proteins are large
molecules that cannot normally pass through the glomerular filtration system. Their presence in urine
indicates increase in permeability. Ans = E

45. The development of both immune and allergic reactions is based upon the same mechanisms of
the immune system response to an antigen. What is the main difference between the immune and
allergic reactions?

A. Amount of released antigen

B. Antigen structure

C. Development of tissue lesion

D. Routes by which antigens are delivered into the body

E. Hereditary predisposition

HINT: There’ll be development of lesion in allergic reactions, e.g rashes. Ans = C

46. After a craniocerebral injury a patient is unable to recognize objects by touch. What part of
brain has been damaged?

A. Occipital lobe

B. Temporal lobe

C. Precentral gyrus

D. Postcentral gyrus

E. Cerebellum

HINT; Occipital is mainly for vision. Temporal for memory, speech. Cerebellum for Coordination of
limbs and extremity. Precentral gyrus Commands voluntary movements of skeletal muscles. That
leaves D, but it is also for touch or tactile sensitivity. So the answer is D.

47. A patient complaining of rapid pulse, dyspnea and bluish color of mucosa has been admitted to
the cardiological department. The objective symptoms are as follows: edema of lower extremities,
ascites. Which of the given medicines should be prescribed for intravenous administration to
improve the patient’s general state?

A. Cordiamin

B. Adrenalin hydrochloride

C. Digitoxin

Krok 1: 2014 Past Questions, Answers Explained NSA-TSMU Academic Team


D. Corglyconum

E. Drotaverine

EXP: This patient is in Heart Failure. Adrenalin is both an alpha and beta agonist which is
contraindicated in heart failure. Cordiamin is an Analeptic (stimulates breathing center) which is not
really useful here. Drotaverine is an Antispasmodic drug for childbirth. So Digitoxin and Corglyconum
are both cardiac glycosides but corglyconum has rapid action and due to digitoxin’s high level of
toxicity, it is hardly used. Hence, the answer is D.

48. As a result of a continuous chronic encephalopathy, a patient has developed spontaneous


motions and a disorder of torso muscle tone. These are the symptoms of the disorder of the
following conduction tract:

A. Tractus corticospinalis

B. Tractus rubrospinalis

C. Tractus corticonuclearis (for muscles of face, head and neck)

D. Tractus spinothalamicus (pain, touch)

E. Tractus tectospinalis (head and eye)

HINT: Rubrospinal tract (from red nucleus to spinal cord) is responsible for large muscle movements
in upper part of body. Corticospinal tract involves lower limbs. So the answer is B

49. A 10-year-old child has painful swallowing, neck edema, temperature rise up to 39.0oC, the
whole body is covered with bright-red petechial rash. Back of the throat and tonsils are hyperemic,
the tongue is crimson colored. Tonsillar surface is covered with isolated grayish-colored necrosis
nidi. What disease is it?

A. Meningococcal nasopharyngitis

B. Scarlet fever

C. Diphtheria

D. Influenza

E. Measles

HINT; Tongue is crimson (red) colored (generally, called scarlet fever). And also the age matters (it
affects children). It’s a diagnostic for SCARLET FEVER. The answer is B

50. In a patient elevation of body temperature takes turns with drops down to normal levels during
the day. The rise in temperature is observed periodically once in four days. Specify the type of
temperature curve:

A. Febris intermittent

B. Febris continua

Krok 1: 2014 Past Questions, Answers Explained NSA-TSMU Academic Team


C. Febris recurrent

D. Febris hectica

E. Febris remittent

HINT; in Febris continua the daily fluctuation does not exceed 1oC. In Febris recurrens, there are
periods of rising temp (5-8days), and periods of normal temp. In Febris hectica, there is daily
fluctuation of up to 3-4oC. In Febris remittens (indulgence fever), fluctuation between morning and
evening temps is 1-2oC.

Ans = A. The temp curve here is the febris quartana type of Febris Intermittens, where temp rises
every 4th day and then falls. F. intermittens could also be every 3 days (febris tertiana) or daily
(quotidiana).

51. As a result of a craniocerebral injury, a patient has a decreased skin sensitivity. What area of the
cerebral cortex is likely to be damaged?

A. Posterior central gyrus

B. Occipital region

C. Cingulate gyrus

D. Frontal cortex

E. Anterior central gyrus

HINT; Same as 46. ( for skin sensitivity and touch). The answer is A

52. Diabetic nephropathy with uremia has developed in a patient with pancreatic diabetes. The
velocity of glomerular filtration is 9 ml/min. What mechanism of a decrease in glomerular filtration
velocity and chronic renal failure development is most likely in the case of this patient?

A. Tissue acidosis

B. Decrease in systemic arterial pressure

C. Obstruction of nephron tubules with hyaline casts

D. Reduction of active nephron mass

E. Arteriolar spasm

HINT; Normal GFR is 90 – 120ml/min. Diabetic nephropathy damages kidney glomeruli, thereby
reducing the number of nephrons that is supposed to filter uremia and waste. The answer is D

53. A 40-year-old patient has ulcer perforation in the posterior wall of stomach. What anatomical
structure will blood and stomach content leak to?

A. Bursa hepatica

B. Bursa praegastrica

Krok 1: 2014 Past Questions, Answers Explained NSA-TSMU Academic Team


C. Right lateral channel (canalis lateralis dexter)

D. Left lateral channel (canalis lateralis sinister)

E. Bursa omentalis

EXP; The answer is E

54. A patient is diagnosed with hereditary coagulopathy that is characterized by factor VIII
deficiency. Specify the phase of blood clotting during which coagulation will be disrupted in the
given case:

A. Fibrin formation

B. Thrombin formation

C. Thromboplastin formation

D. Clot retraction

E. –

EXP’; The deficiency of factor VIII (Hemophilia A) makes it impossible to make thromboplastin.
Thrombin is factor II. Fribrin is factor 1a. The answer is C

55. Angiocardiography of a 60-yearold male patient revealed constriction of a vessel located in the
left coronary sulcus of the heart. What is the pathological vessel called?

A. Ramus circumflexus

B. Ramus interventricularis posterior

C. A. coronaria dextra

D. V.cordis parva

E. Ramus interventricularis anterior

HINT; Left coronary gives rise to A,B,E. but only A, is in sulcus. So, the answer is A

56. Those organisms which in the process of evolution failed to develop protection from H2O2 can
exist only in anaerobic conditions. Which of the following enzymes can break hydrogen peroxide
down?

A. Flavin-dependent oxidase

B. Oxygenase and hydroxylase

C. Cytochrome oxidase, cytochrome B5

D. Oxygenase and catalase

E. Peroxidase and catalase

HINT; E is an enzyme that breaks down H2O2 (Hydrogen peroxide) to H2O and O2. So that’s the
answer.

Krok 1: 2014 Past Questions, Answers Explained NSA-TSMU Academic Team


57. A patient complains of pain in the right lateral abdomen. Palpation revealed a dense, immobile,
tumor like formation. A tumor is likely to be found in the following part of the digestive tube:

A. Colon sigmoideum

B. Colon transversum

C. Colon descendens

D. Colon ascendens

E. Caecum

EXP: The ascending colon is located on the right lateral part of the abdomen so it will be the part of
the large intestine. The answer is D

58. A patient complaining of pain in the left shoulder-blade region has been diagnosed with
miocardial infarction. What kind of pain does the patient have?

A. Radiating

B. Visceral

C. Phantom

D. Protopathic

E. Epicritic

Exp: When pain felt moves from one location to another it is known to as radiated, Just like question
the pain started from the the heart(myocardial infarction) and then radiated to the left shoulder-
blade region. The answer is A

59. A patient has a critical impairment of protein, fat and hydrocarbon digestion. Most likely it has
been caused by low secretion of the following digestive juice:

A. Saliva

B. Pancreatic juice

C. Gastric juice

D. Bile

E. Intestinal juice

HINT; B because it contains all the enzymes that breaks down protein, fats and hydrocarbon. So
that’s the answer.

Krok 1: 2014 Past Questions, Answers Explained NSA-TSMU Academic Team


60. A female patient has facial neuritis that has caused mimetic paralysis and hearing impairment.
Hearing impairment results from the paralysis of the following muscle:

A. Superior auricular muscle

B. Anterior auricular muscle

C. Stapedius muscle

D. Posterior auricular muscle

E. Nasal muscle

HINT; Stapedius muscle is innervated by a branch of facial nerve, so any paralysis of the muscle will
cause injury to the nerve. So the answer is C.

61. A 22-year-old female student consulted a physician about fever up to 38oC, weakness, sore
throat. Objectively: there is white coating of the tongue. What histological structures of the tongue
are involved in the formation of this coating?

A. Epithelium of the circumvallate papillae

B. Epithelium of the foliate papillae

C. Epithelium of the fungiform papillae

D. Epithelium of the filiform papillae

E. Connective-tissue base of all the lingual papillae

HINT: Oral Candidiasis is mostly seen on the filiform papillae. So the answer is D

62. A patient has a traumatic injury of sternocleidomastoid muscle. This has resulted in a decrease
in the following value:

A. Inspiratory reserve volume

B. Expiratory reserve volume

C. Respiratory volume

D. Residual volume

E. Functional residual lung capacity

HINT; The sternocleidomastoid muscle (along with the scalene muscle), are accessory muscle for
breathing. It increases IRV. So, any injury will decrease IRV. So the answer is A

Krok 1: 2014 Past Questions, Answers Explained NSA-TSMU Academic Team


63. The receptors under study provide transfer of information to the cortex without thalamic
involvement. Specify these receptors:

A. Gustatory

B. Tactile

C. Olfactory

D. Visual

E. Auditory

HINT; Tactile- 1st and 2nd neuron is in spinal cord and 3rd is VPM (ventral posterior medial nucleus) of
thalamus. Gustatory- Terminate in medial half of VPM. Visual- About 90% of axons in optic nerve go
to lateral geniculate nucleus of thalamus. Auditory passes through medial geniculate body which is
part of the thalamic system. So we are left with OLFACTORY which is the correct answer.

64. Prolonged fasting causes hypoglycemia which is amplified by alcohol consumption, as the
following process is inhibited:

A. Lipolysis

B. Glycolysis

C. Glycogenolysis

D. Gluconeogenesis

E. Proteolysis

EXP; Alcohol consumption causes the accumulation of NADH. Due to this, pyruvate is converted to
lactate instead of acetyl CoA which is supposed to be the first enzyme of gluconeogenesis. So this is
how the process is impaired. The answer is D

65. A 39-year-old female patient with a history of diabetes was hospitalized in a precomatose state
for diabetic ketoacidosis. This condition had been caused by an increase in the following metabolite
level:

A. Acetoacetate

B. Citrate

C. Alpha-ketoglutarate

D. Malonate

E. Aspartate

HINT; In diabetic ketoacidosis, ketone bodies are increased. But ketone bodies is not here but KB have
3 types- Acetoacetate, Acetone and B- hydroxybutyrate. So the answer is A

Krok 1: 2014 Past Questions, Answers Explained NSA-TSMU Academic Team


66. A 37-year-old female patient complains of headache, vertigo, troubled sleep, numbness of limbs.
For the last 6 years she has been working at the gas-discharge lamp-producing factory in the lead-
processing shop. Blood test findings: low hemoglobin and RBC level, serum iron concentration
exceeds the norm by several times. Specify the type of anemia:

A. Metaplastic anemia

B. Iron-deficiency anemia

C. Minkowsky-Shauffard disease

D. Hypoplastic anemia

E. Iron refractory anemia

Exp: iron refractory anemia is an anemia in which the serum iron concentration is increased. In Iron
def, Fe is reduced. Minkowsky-shauffard (hereditary spherocytosis) is just when the RBC is spherical
in shape. Hypoplastic anemia is a disorder in bone marrow characterized by pancytopenia.
Metaplastic is vit B12 def. So the answer is E

67. During an animal experiment, surgical damage of certain brain structures has caused deep
prolonged sleep. What structure is most likely to cause such condition, if damaged?

A. Reticular formation

B. Basal ganglion

C. Red nuclei

D. Hippocampus

E. Cerebral cortex

HINT; B- controls voluntary motor nucleus as eye movement. C- controls muscle of shoulder (such as
swinging arms). D- controls formation of memories. E- controls all 4 lobes and their function. A-
controls sleep and alertness so injury to it will prolong sleep and possibly coma. So the answer is A.

68. A patient takes cholagogues. What other process besides biliary excretion will be stimulated?

A. Gastric juice secretion

B. Intestinal motility

C. Pancreatic juice secretion

D. Gastric motor activity

E. Water absorption

EXP; Cholagogues is a medicinal agent that promotes the discharge of bile. Bile works in the small
intestine. So it will make sense for cholagogues to work due to intestinal mobility. So the answer is B

69. A 3-year-old child with meningeal symptoms died. Postmortem macroscopy of the pia matter
revealed miliary nodules which were microscopically represented by a focus of caseous necrosis with

Krok 1: 2014 Past Questions, Answers Explained NSA-TSMU Academic Team


masses of epithelioid and lymphoid cells with some crescent-shaped large cells inbetween having
peripheral nuclei. Specify the type of meningitis in the child:

A. Tuberculous

B. Syphilitic

C. Brucellar

D. Grippal

E. Meningococcal

HINT; Same as 23. So the answer is A

70. A patient with homogentisuria has signs of arthritis, ochronosis. In this case, the pain in the joints
is associated with the deposition of:

A. Carbonates

B. Urates

C. Phosphates

D. Oxalates

E. Homogentisates

HINT; Homogentisuria is accumulation of homogentisic acid in urine (alkaptonuria) due to


homogentisate 1,2 dioxygenase that helps in catabolism of phenylalanine and tyrosine. So the
answer is E

71. A patient with hereditary hyperammonemia due to a disorder of ornithine cycle has developed
secondary orotaciduria. The increased synthesis of orotic acid is caused by an increase in the
following metabolite of ornithine cycle:

A. Urea

B. Citrulline

C. Ornithine

D. Carbamoyl phosphate

E. Argininosuccinate

HINT; Lack of ornithine carbamoyl transferase that is suppose to covert carbamoyl + ornithine to
citrulline gives room for the build up of carbamoyl phosphate which is a substrate for the orotic
cycle.( Orotic cycle;- carbamoyl phosphate + aspartate is converted to N- carbamoylaspartate (by
ASPARTATE TRANSCARBAMYLASE) which is converted to dihydroorotate (by dihydroorotase) which is
converted finally to orotic acid (by dihydroorotate dehydrogenase). Simply put, carbamoyl phosphate
is required in both cycle. So the answer is D

Krok 1: 2014 Past Questions, Answers Explained NSA-TSMU Academic Team


72. Amniocentesis revealed two sex chromatin bodies (Barr bodies) in each cell of the sample. What
disease is this character typical for?

A. Trisomy X

B. Klinefelter syndrome

C. Turner’s syndrome

D. Down’s syndrome

E. Patau syndrome

HINT; Barr body is the inactive X chromosome. A female has only one barr body, while male has
none. So A, is correct.

73. A 49-year-old male patient with acute pancreatitis was likely to develop pancreatic necrosis,
while active pancreatic proteases were absorbed into the blood stream and tissue proteins broke
up. What protective factors of the body can inhibit these processes?

A. α2-macroglobulin, α1-antitrypsin

B. Immunoglobulin

C. Cryoglobulin, interferon

D. Ceruloplasmin, transferrin

E. Hemoplexin, haptoglobin

HINT; According to the keyword, we need a protease inhibitor and that happens to be A which is the
correct answer. B- are antibodies. Cryoglobulins are abnormal antibody proteins. Interferons acts as
defense against viral infections. Ceruloplasmin carries copper. Transferrin transfers Fe. Hemoplexin
binds heme with any protein. Haptoglobin carries free Hb back to the liver.

74. A 16-year-old female patient has fainted after quickly changing her body position from horizontal
to vertical one. Which process from the ones listed below has caused the loss of consciousness in
the first place?

A. Decreasing oncotic pressure of blood plasma

B. Increasing venous return

C. Increasing central venous pressure

D. Decreasing venous return

E. Increasing arterial pressure

HINT: Sudden change of body position vertically decreases venous blood return to the heart due to
gravity. The body loses consciousness so that by the fall and consequent return to horizontal position,
blood return will be easier. Remember that’s why we get head rushes. ANSWER IS D

Krok 1: 2014 Past Questions, Answers Explained NSA-TSMU Academic Team


75. A 53-year-old male patient complains of acute pain in the right hypochondrium. Objective
examination revealed scleral icterus. Laboratory tests revealed increased ALT activity, and stercobilin
was not detected in the stool. What disease is characterized by these symptoms?

A. Cholelithiasis

B. Hemolytic jaundice

C. Hepatitis

D. Chronic colitis

E. Chronic gastritis

HINT; Location of pain removes E. Increased ALT removes D. Age of patient and absence of signs of
hemolysis removes B. Absence of stercobilin removes C, so we are left with A, because the gallstone
blocks the bile duct hindering the flow of the stercobilin which is the correct answer.

76. A child patient has dry cough. What non-narcotic antitussive drug will relieve the patient’s
condition?

A. Althaea officinalis root extract

B. Potassium iodide

C. Morphine hydrochloride

D. Codeine phosphate

E. Glaucine hydrochloride

HINT; A and B are expectorants. C and D are narcotic (opiods). So we are left with E which is right
answer.

77. A patient complains of acute pain attacks in the right lumbar region. During examination the
nephrolithic obturation of the right ureter in the region between its abdominal and pelvic segments
has been detected. What anatomical boundary exists between those two segments?

A. Linea semilunaris

B. Linea terminalis

C. Linea arcuata

D. Linea transversa

E. Linea inguinalis

EXP; linea terminalis includes linea arcuta and it lies in abdominal and pelvic segments. Transverse
line lies between iliac spines. Semi-lunar line lies on either side of rectus abdominis muscle. Inguinal
line lies from pubic tubercle to superior iliac spine. ANSWER IS B

Krok 1: 2014 Past Questions, Answers Explained NSA-TSMU Academic Team


78. When measuring total muscle action potential it was revealed that it was subject to the power-
law relationship. The reason for this is that individual muscle fibers differ in:

A. Depolarization threshold

B. Diameter

C. Conduction velocity

D. Resting potential

E. Critical level of depolarization

EXP; ANSWER IS A. No comment

79. A patient has insufficient blood supply to the kidneys, which has caused the development of
pressor effect due to the constriction of arterial resistance vessels. This is the result of the vessels
being greately affected by the following substance:

A. Renin

B. Angiotensinogen

C. Angiotensin II

D. Catecholamines

E. Norepinephrine

HINT; The only strong direct vasoconstrictor is angiotensin II. Renin activates the renin-angiostensin
system. Angiotensinogen is not active in vessels. Catecholamines has nothing to do with the kidneys.
Answer is c

80. A patient is diagnosed with iron deficiency sideroachrestic anemia, progression of which is
characterized by skin hyperpigmentation, pigmentary cirrhosis, heart and pancreas affection. Iron
level in the blood serum is increased. What disorder of iron metabolism causes this disease?

A. Failure to assimilate iron leading to iron accumulation in tissues

B. Excessive iron intake with food

C. Disorder of iron absorption in bowels

D. Increased iron assimilation by body

E. –

HINT; A is correct because several organs are affected indicating that Fe is been accumulated in the
tissues because its not been used.

81. In a village, a case of anthrax had been registered. Medical services began epidemiologically
indicated specific prophylaxis of population against anthrax. What preparation was used for this
purpose?

Krok 1: 2014 Past Questions, Answers Explained NSA-TSMU Academic Team


A. Genetically engineered vaccine

B. Inactivated vaccine

C. Chemical vaccine

D. Live vaccine

E. Anatoxin

EXP; D is correct because anthrax is a live vaccine.

82. Experimental stimulation of the sympathetic nerve branches that innervate the heart caused an
increase in force of heart contractions because the membrane of typical cardiomyocytes permitted
an increase in:

A. Calcium ion exit

B. Calcium ion entry

C. Potassium ion exit

D. Potassium ion entry

E. Calcium and potassium ion exit

HINT; Ca levels increase HR & contractility (influx of Ca). Potassium does the opposite. ANSWER IS B

83. According to the results of glucose tolerance test, the patient has no disorder of carbohydrate
tolerance. Despite that, glucose is detected in the patients’s urine (5 mmol/l). The patient has been
diagnosed with renal diabetes. What renal changes cause glucosuria in this case?

A. Decreased activity of glucose reabsorption enzymes

B. Increased activity of glucose reabsorption enzymes

C. Exceeded glucose reabsorption threshold

D. Increased glucose secretion

E. Increased glucose filtration

HINT; The problem here is glucose wasn’t reabsorbed along with Na because Sodium- glucose
transport (SGLT) enzymes activity is reduced. ANSWER IS A

84. As a result of past encephalitis, a male patient has developed an increase in cerebrospinal fluid
pressure in the right lateral ventricle. What can be the cause of this condition?

A. Closure of right interventricular foramen

B. Closure of left interventricular foramen

C. Atresia of tubus medullaris

Krok 1: 2014 Past Questions, Answers Explained NSA-TSMU Academic Team


D. Atresia of sylvian aqueduct

E. Atresia of the fourth ventricle foramina

EXP; It will make sense for B to be the correct answer. The increased pressure is in the right ventricle
due to the closure of the right interventricular foramen

85. Parents of 5-year-old child report him o have frequent colds that develop into pneumonias,
presence of purulent rashes on the skin. Laboratory tests have revealed the following: absence of
immunoglobulins of any type, and naked cells are absent from the lymph nodes punctate. What kind
of immune disorder is it?

A. X-linked hypogammaglobulinemia (Bruton type agammaglobulinemia)

B. Autosomal recessive agammaglobulinaemia (Swiss type)

C. Hypoplastic anemia

D. Agranulocytosis

E. Louis-Barr syndrome

EXP;- Absence of Ig removes C, D and E. Bruton’s type is just deficiency of humoral immunity (B
lymphocytes). So the symptoms are less severe than Swiss type which is deficiency of both humoral
and cellular immunity. The patient will start showing symptoms from like 5 months.

86. A 3-year-old child has continuous fever, lymph nodes are enlarged, the amount of lymphocytes
in blood is significantly increased. Enzyme linked immunosorbent assay (ELISA) revealed antigen of
Epstein-Barr virus. What diagnosis can be made based on the information given above?

A. Cytomegalovirus infection

B. Burkitt’s lymphoma

C. Herpetic lymphadenopathy

D. Generalized infection caused by herpes-zoster

E. Infectious mononucleosis

HINT; Causative agent of infectious mononcleosis is Epstein- Barr which is also HERPES 4. RIGHT
ANSWER IS E

87. A microslide contains the preparation of a gland composed of several secretory saccule-shaped
parts that open in the common excretory duct. What gland is it?

A. Simple branched alveolar gland

B. Compound branched alveolar gland

C. Simple unbranched alveolar gland

Krok 1: 2014 Past Questions, Answers Explained NSA-TSMU Academic Team


D. Compound unbranched alveolar gland

E. Simple branched tubular gland

HINT; Alveolar glands are sac like (saccule). So E is out. There are several secretory parts which
means its BRANCHED (C &D is out), but they empty into ONE duct. Compound (means many) won’t
empty into one duct. That leaves A which is the right answer.

88. A patient intending to undergo a gender reassignment surgery has been admitted to a
specialized clinic. In the course of examination both male and female gonads have been revealed,
with male structure of external genitals. What kind of genital maldevelopment has the patient?

A. Female pseudo hermaphroditism

B. Male pseudo hermaphroditism

C. True hermaphroditism

D. Accessory ovary

E. Ectopia of testis

HINTS;. Both male and female gonads was revealed so D & E is out. In pseudo hermaphroditism, we
won’t be able to differentiate which gentalia is developed. So C, is correct because male genitalia
was revealed

89. An 18-year-old male has been diagnosed with Marfan syndrome. Examination revealed a
developmental disorder of connective tissue and eye lens structure, abnormalities of the
cardiovascular system, arachnodactylia. What genetic phenomenon has caused the development of
this disease?

A. Multiple allelism

B. Complementarity

C. Codominance

D. Pleiotropy

E. Incomplete dominance

HINT: Pleiotropy is when one gene affects multiple systems…different systems are affected in
Marfan’s syndrome. So the right answer is D

90. In case of alkaptonuria, homogentisic acid is excreted in urine in large amounts. The
development of this disease is associated with a disorder of metabolism of the following amino acid:

A. Asparagine

B. Phenylalanine

C. Alanine

Krok 1: 2014 Past Questions, Answers Explained NSA-TSMU Academic Team


D. Methionine

E. Tyrosine

HINT; In alkaptonuria, the enzyme (homogentisate 1,2- dioxygenase) is involved in metabolism of


phenylalanine and tyrosine. And also tyrosine is a product of phenylalanine. So, there’s excess
tyrosine. So the RIGHT ANSWER IS E

91. A male patient complains of skin insensitivity of inferior eyelid, external lateral surface of nose
and upper lip. A doctor in the course of examination has revealed inflammation of the second
branch of trigeminal nerve. What cranial foramen does this branch go through?

A. Lacerum

B. Supraorbital

C. Superior orbital fissure

D. Spinosum

E. Oval

HINT; Trigeminal nerve- (1st branch is ophthalmic (foramen is superior orbital fissure). 2nd branch is
maxillary (foramen rotundum). 3rd branch is mandibular (foramen ovale). ANSWER IS B

92. An patient with insomnia induced by allergic rash and itch has been prescribed the drug that has
antihistamine and hypnotic effect. Specify this drug:

A. Benadryl

B. Loratadine

C. Prednisolone

D. Acetylsalicylic acid

E. Analgin

HINT; B- is H2 antagonist. C- is an oral, synthetic corticosteroid. D- its also called ASPIRIN which is an
NSAID. E- analgesic, antipyretic. That leaves A which is antihistamine and also have hypnotic effect
and is correct.

93. In a cat with decerebrate rigidity the muscle tone is to be decreased. This can be achieved by:

A. Stimulation of the vestibulocochlear nerve

B. Stimulation of the otolithic vestibular receptors

C. Stimulation of the vestibular nuclei of Deiters

D. Destruction of the vestibular nuclei of Deiters

E. Stimulation of the ampullar vestibular receptors

Krok 1: 2014 Past Questions, Answers Explained NSA-TSMU Academic Team


HINT: Decerebrate rigidity is caused by lesion on brainstem. Vestibular nuclei of deiters is located on
the brainstem. RIGHT ANSWER IS D

94. When studying the signs of pulmonary ventilation, reduction of forced expiratory volume has
been detected. What is the likely cause of this phenomenon?

A. Increase of functional residual lung capacity

B. Increase of respiratory volume

C. Increase of inspiratory reserve volume

D. Increase of pulmonary residual volume

E. Obstructive pulmonary disease

HINT: Expiratory Volume is always reduced in all Obstructive Pulmonary disease. E.g Asthma, COPD.
ANSWER IS E

95. X-ray examination of a patient allowed to diagnose a tumor in the superior lobe of the right lung.
There is a probability of metastases spread to the following lymph nodes:

A. Sternal

B. Inferior mediastinum

C. Anterior mediastinum

D. Axillary

E. Deep lateral cervical

ANSWER IS A. NO HINT

96. A 55-year-old patient with a characteristic rash, fever, dizziness has been admitted to a hospital.
He has been provisionally diagnosed with typhus. No similar cases have been reported. In his youth
(15 years old) the patient suffered typhus in a boarding school. What disease is it?

A. Rubella

B. Typhoid fever

C. Measles

D. Brill’s disease

E. Cholera

HINT; Typhus is caused by RICKETTSIAE PROWAZEKII. This bacteria can remain latent and reactivate
after months or even years. When it reactivates, it is known as BRILL’S DISEASE. ANSWER IS D.

97. A young woman suddenly developed fever up to 39oC accompanied by a strong headache.
Examination revealed marked nuchal rigidity. Spinal puncture was performed. Gram-stained smear

Krok 1: 2014 Past Questions, Answers Explained NSA-TSMU Academic Team


of cerebrospinal fluid contained many neutrophils and Gram-positive diplococci. What bacteria
could be the cause of this disease?

A. Neisseria meningitidis

B. Streptococcus pneumonia

C. Haemophilus influenzae

D. Staphylococcus aureus

E. Pseudomonas aeruginosa

HINT; A IS CORRECT and it can cause meningitis and it has 3 signs; Nuchal rigidity, brudzinski and
kernig sign.

98. In the course of an experiment adenohypophysis of an animal has been removed. The resulting
atrophy of thyroid gland and adrenal cortex has been caused by deficiency of the following
hormone:

A. Somatotropin

B. Thyroid hormones

C. Tropic hormones

D. Cortisol

E. Thyroxin

HINT: All tropic hormones are produced in the Adenohypophysis. Since thyroid gland and adrenal
cortex are affected, it removes all other options that focus on only one hormone. RIGHT ANSWER IS C

99. Degenerative changes in posterior and lateral columns of spinal cord (funicular myelosis) caused
by methylmalonic acid accumulation occur in patients with B12-deficiency anemia. This results in
synthesis disruption of the following substance:

A. Acetylcholine

B. Myelin

C. Norepinephrine

D. Dopamine

E. Serotonin

HINT: B12 is responsible for cell regeneration, Therefore deficiency of it causes degeneration of cells.
Note that Myelin sheath is the covering of Spinal Cord. ANSWER IS B

100. A patient with suspected tumor of lung had been admitted to the oncological department.
Examination revealed localized pathology in the inferior lobe of the left lung. How many
bronchopulmonary segments does this lobe have?

Krok 1: 2014 Past Questions, Answers Explained NSA-TSMU Academic Team


A. 5

B. 6

C. 4

D. 3

E. 2

HINT: Each Lung has 10 bronchopulmonary segments. Right lung has 3 lobes and its segments are
upper lobe- 3, middle-2, inferior- 5. For LEFT lung, just 2 lobes are present so 5 segments each in
upper and lower lobes. ANSWER IS A

101. During local anesthetization the patient has gone into anaphylactic shock. What drug must be
administered to the patient?

A. Propranolol
B. Diazepam
C. Atropine sulfate
D. Epinephrine hydrochloride
E. Nitroglycerin

EXP: Epinephrine hydrochloride (epinephrine is given for anaphylactic reactions) Epinephrine acts
quickly to improve breathing, stimulate the heart, raise a dropping blood pressure, reverse hives, and
reduce swelling of the face, lips, and throat. ANSWER IS D

102. As a result of an injury, the integrity of the anterior spinal cord root was broken. Specify the
neurons and their processes that had been damaged:

A. Motor neuron dendrites


B. Axons of motor neurons
C. Axons of sensory neurons
D. Dendrites of sensory neurons
E. Dendrites of association neurons

EXP: the anterior root of the spinal cord is made up of the axons of motor neurons (posterior is
sensory neurons) so a damage of anterior root leads to damage of axons of motor neurons. ANSWER
IS B

103. A patient who had been taking diclofenac sodium for arthritis of mandibular joint developed an
acute condition of gastric ulcer. Such side effect of this medicine is caused by inhibition of the
following enzyme:

A. Cyclooxygenase-1(COX-1)
B. Cyclooxygenase-2(COX-2)
C. Lipoxygenase
D. Phosphodiesterase
E. Monoamine oxidase

Krok 1: 2014 Past Questions, Answers Explained NSA-TSMU Academic Team


Exp: Cyclooxygenase-1 (COX-1) is known to be present in most tissues. In the gastrointestinal tract,
COX-1 maintains the normal lining of the stomach so its inhibition could lead to ulcer. ANSWER IS A

104. In one of Polessye regions there was an outbreak of helminthias is manifested by cramps and
facial edmata. The developed preventive measures in particular included ban for eating infested
pork even after heat processing. What helminthiasis was the case?

A. Trichinosis
B. Taeniarhynchosis
C. Teniasis
D. Echinococcosis
E. Alveococcosis

EXP: Trichinosis and taeniasis are both for pork but length of uterus is not described here for
taeniasis. Taeniarhynchosis (taenia saginata) is for pork. Both echinococcosis and alveococcosi is for
dog. ANSWER IS A

105. A female patient sought medical genetic consultation. Physical examination revealed
pterygium coli deformity (webbed-neck), broad chest, under-developed breasts. Study of buccal
epithelium cells revealed no X-chromatin in the nuclei. This indicates that the patient has the
following syndrome:

A. Turner’s
B. Klinefelter’s
C. Patau’s
D. Down’s
E. Edwards’

EXP: Turner syndrome ( 45 X0 no barr body) is a chromosomal condition that affects development in
females. The most common feature of Turner syndrome is an early loss of ovarian function
(ovarian hypofunction or premature ovarian failure) such as broad chest, underdeveloped breasts
ANSWER IS B

106. During blood transfusion a patient has developed intravascular erythrocyte hemolysis. What
kind of hypersensitivity does the patient have?

A. II type (antibody-dependent)
B. I type (anaphylactic)
C. III type (immune complex)
D. IV type (cellular cytotoxicity)
E. IV type (granulomatosis)

EXP: type 2 AKA Complement(Antibody) dependent reactions: Antibody is directed against antigen on
cells (such as circulating red blood cells) or extracellular materials (basement membrane).
The resulting Ag-Ab complexes activate complement (via the classic pathway), leading to cell lysis or
extracellular tissue damage. ANSWER IS A

Krok 1: 2014 Past Questions, Answers Explained NSA-TSMU Academic Team


107. Patients with erythropoietic porphyria (Gunther’s disease) have teeth that fluoresce with
bright red colour when subjected to ultraviolet radiation; their skin is light-sensitive, urine is red-
colored. What enzyme can cause this disease, when it is deficient?

A. Ferrochelatase
B. Uroporphyrinogen I synthase
C. Delta-aminolevulinate synthase
D. Uroporphyrinogen decarboxylase
E. Uroporphyrinogen III cosynthase

Exp: (Gunther's disease, GD) is a rare autosomal recessive disease. It results from the deficiency of
uroporphyrinogen III synthase, the fourth enzyme on the metabolic pathway of heme synthesis.
ANSWER IS E

108. During the air and bone conduction tests it was revealed that in the left ear the tones were
louder by bone conduction. This might be associated with the disease of:

A. Left middle ear


B. Right middle ear
C. Left inner ear
D. Right inner ear
E. Right external ear

Exp : During infection of the middle ear there is accumulation of fluid in the Eustachian tube, this
reduces air conduction of sounds. Left because its in the left ear. ANSWER IS A

109. In course of invasive abdominal surgery a surgeon has to locate the origin of the mesenteric
root. Where is it normally localized?

A. Right mesenteric sinus


B. Right flexure of colon
C. Left flexure of colon
D. Sigmoid colon
E. Duodenojejunal flexure

Exp: The mesenteric root is located at the duodeno-jejunal flexure anatomically. ANSWER IS E

110. A 55-year-old male had been delivered to the resuscitation unit unconscious. Relatives reported
him to have mistakenly drunk an alcoholic solution of unknown origin. On examination the patient
was diagnosed with methanol intoxication. What antidote should be used in this case?

A. Naloxone
B. Teturamum
C. Ethanol
D. Protamine sulfate
E. Acetylcysteine

Krok 1: 2014 Past Questions, Answers Explained NSA-TSMU Academic Team


Exp : Antidote therapy, often using ethanol, is directed towards delaying methanol metabolism until
the methanol is eliminated from the patient’s system either naturally or via dialysis. ANSWER IS C

111. Due to the use of poor quality measles vaccine for preventive vaccination, a 1-year-old child has
developed an autoimmune renal injury. The urine was found to contain macromolecular proteins.
What process of urine formation has been disturbed?

A. Reabsorption and secretion


B. Reabsorption
C. Secretion
D. Filtration
E. Secretion and filtration

EXP- Presence of macromolecular (large molecules) proteins shows that filtration process has been
distorted. ANSWER IS D

112. As a result of a road accident a 37-year-old female victim developed urinary incontinence. What
segments of the spinal cord had been damaged?
A. S2 - S4
B. T h1 - T h5
C. L1 - L2
D. T h2 - T h5
E. T h1 - L1

EXP; -S2 - S4 nerves have both afferent and efferent fibers, thus they are responsible for part of the
sensory perception and their parasympathetic fibers whose electrical potential supply the descending
colon and rectum, urinary bladder and genital organs. ANSWER IS A

113. An 8-week-pregnant woman with acute respiratory disease and temperature rise up to 39, 0oC
has called in a doctor. The doctor insisted on her avoiding taking paracetamol, because in this period
of pregnancy there is a risk of its:

A. Teratogenicity
B. Embryotoxicity
C. Fetotoxicity
D. Hepatotoxicity
E. Allergenicity

EXP- Teratogenicity is the property or capability of producing congenital malformations.


it usually occurs within the first trimester. ANSWER IS A

114. A pneumonia patient has been administered acetylcysteine as a part of complex therapy. What
principle of therapy has been taken into consideration when applying this drug?

A. Etiotropic
B. Symptomatic
C. Pathogenetic
D. Antimicrobial

Krok 1: 2014 Past Questions, Answers Explained NSA-TSMU Academic Team


E. Immunomodulatory

EXP; Acetylcysteine is used to loosen thick mucus and that’s the effect of pneumonia pathogenically
on bronchioles. ANSWER IS C

115. A 26-year-old female patient with bronchitis has been administered a broad spectrum antibiotic
as a causal treatment drug. Specify this drug:

A. Dexamethasone
B. Interferon
C. BCG vaccine
D. Ambroxol
E. Doxycycline

EXP; Doxycycline is a broad spectrum antibiotic. Interferon is antiviral. BCG vaccine for TB. Ambroxol
is a mucolytic agent. Dexamethasone is a corticosteroid. ANSWER IS E

116. When defining blood group according to the AB0 system, using salt solutions of monoclonal
antibodies, agglutination didn’t occur with any of the solutions. What blood group is it?

A. 0 (I)
B. ? (II)
C. ? (III)
D. ?? (IV)
E. -

EXP; Group O – has neither A nor B antigens on red cells (but both A and B antibody are in the
plasma) so when in contact with monoclonal antibiodies it had no agglutinations because there the
absence of antigens on red cells. ANSWER IS A

117. Examination of a 56-year-old female patient with a history of type 1 diabetes revealed a
disorder of protein metabolism that is manifested by aminoacidemia in the laboratory blood test
values, and clinically by the delayed wound healing and decreased synthesis of antibodies. Which of
the following mechanisms causes the development of aminoacidemia?

A. Increase in low-density lipoprotein level


B. Albuminosis
C. Decrease in the concentration of amino acids in blood
D. Increase in the oncotic pressure in the blood plasma
E. Increased proteolysis

EXP;- Proteolysis is the breakdown of proteins into smaller polypeptides or amino acids. An increased
proteolysis results in an increase in amino acids in the blood. ANSWER IS E

118. Symptoms of pellagra (vitamin PP deficiency) is particularly pronounced in patients with low
protein diet, because nicotinamide precursor in humans is one of the essential amino acids, namely:
A. Tryptophan

Krok 1: 2014 Past Questions, Answers Explained NSA-TSMU Academic Team


B. Threonine
C. Arginine
D. Histidine
E. Lysine

EXP; Tryptophan is the only nicotiamide precusor present here. Threonine is a homoserine precursor.
Arginine is a citrulline precursor. Histidine is a precursor for histamine. Lysine is a precursor for
glutamate. ANSWER IS A

119. A 68-year-old patient consults a cardiologist, complaining of high arterial blood pressure, pain
in the heart egion, intermittent pulse. Prescribe the ß1-adrenoreceptor blocker for the treatment of
the described pathology:

A. Fenoterol
B. Morphine hydrochloride
C. Nootropil
D. Metoprolol
E. Benzylpenicillin

EXP;- Metoprolol is a ß1-adrenoreceptor blocker. Morphine is an opiod. Nootropil is a cerebral


stimulant. Fenoterol is a ß1 agonist. Benzylpenicillin is an antibiotic. ANSWER IS D

120. A female suffered rubella during pregnancy. The child was born with developmental
abnormalities, namely cleft lip and palate. The child’s genotype is normal. These malformations
are a manifestation of:

A. Aneuploidy
B. Polyploidy
C. Combinative variability
D. Chromosomal mutation
E. Modification variability

EXP;-Non-hereditary(phenotype) variability-these are variabilities which can not be inherited.


They are caused by special conditions and environment.They are also called modifications. ANSWER
IS E

121. An electron micrograph shows a cell-to-cell adhesion consisting, in each cell, of an attachment
plaque. The intercellular space is filled with electron-dense substance including transmembrane
fibrillar structures. Specify this adhesion:

A. Synapse
B. Desmosomes
C. Tight junction
D. Nexus
E. Adherens junction

EXP; Desmosomes form links between cells, and provide a connection between intermediate
filaments of the cell cytoskeletons of adjacent cells and its intercellular space is filled with electron-

Krok 1: 2014 Past Questions, Answers Explained NSA-TSMU Academic Team


dense substances such as transmembrane fibrillar structures. This structure gives strength to tissues.
Synapse allows passage of signal to another neuron. Tight junctions are two cells whose membrane
join together. Nexus and adherens junctions are cell jnctions. ANSWER IS B

122. For biochemical diagnostics of myocardial infarction it is necessary to measure activity of


number of enzymes and their isoenzymes. What enzymatic test is considered to be the best to prove
or disprove the diagnosis of infarction in the early period after the chest pain is detected?

A. Creatine kinase isoenzyme CK-MM


B. Creatine kinase isoenzyme CK-MB
C. LDH1 lactate dehydrogenase isoenzyme
D. LDH2 lactate dehydrogenase isoenzyme
E. Aspartate aminotransferase cytoplasmic isoenzyme

EXP- Creatine kinase (CK) is an enzyme found in your muscles. CK-MB, found in the heart and rises
when heart muscle is damaged. while CK-MM, found in your skeletal muscle and heart. LDH 1 and 2
is found in the heart only. ANSWER IS B

123. A patient complains of pain in the heart area during acute attack of gastric ulcer. What
vegetative reflex can cause this painful feeling?

A. Viscerovisceral reflex
B. Viscerodermal reflex
C. Visceromotor reflex
D. Dermatovisceral reflex
E. Motor-visceral reflex

EXP- Viscerovisceral reflex (heart and gastric [stomach] are both visceral hence 2 visceral). ANSWER
IS A

124. A patient is diagnosed with acute morphine hydrochloride intoxication. Prescribe an oxidizing
agent for gastric lavage:

A. Potassium permanganate
B. Chloramine
C. Sulfocamphocainum (Procaine + Sulfocamphoric acid)
D. Cerigel
E. Chlorhexidine (bi) gluconate

EXP; Potassium permanganate is an oxidizing agent.

125. In the course of puncture biopsy of transplanted kidney the following has been revealed:
diffuse infiltration of stroma by lymphocytes and plasmocytes and necrotic arteritis. What
pathological process has developed in the transplant

A. Immune rejection
B. Ischemic kidney failure

Krok 1: 2014 Past Questions, Answers Explained NSA-TSMU Academic Team


C. Glomerulonephritis
D. Tubular necrosis
E. Pyelonephritis

EXP- (Diffuse infiltration of stroma by lymphocytes and plasmocytes) this are typical signs for
autoimmune disease (i.e. when the body's immune system attacks and destroys healthy body tissue).
Since this occurs after transplantation of kidney. It shows that the body's immune system rejected the
kidney. ANSWER IS A

126. During cell division, DNA replication occurs by a signal from the cytoplasm, and a certain
portion of the DNA helix unwinds and splits into two individual strains. What enzyme facilitates this
process?

A. Helicase
B. RNA polymerase
C. Ligase
D. Restrictase
E. DNA polymerase

EXP- DNA helicases separates double-stranded DNA into single strands allowing each strand to be
copied during replication. RNA polymerase is an enzyme that produces primary transcript RNA.
Ligase is an enzyme that join two large molecules. Restrictase is an enzyme that cuts DNA. DNA
polymerase is used to synthesize DNA. ANSWER IS D

127. A female patient, having visited the factory premises with lots of dust in the air for the first
time, has got cough and burning pain in the throat. What respiratory receptors, when irritated,
cause this kind of reaction?

A. Irritant receptors
B. Juxtacapillary (J) receptors
C. Stretch receptors of lungs
D. Proprioceptors of respiratorymuscles
E. Thermoreceptors

EXP- Pulmonary Irritant Receptors are sensors present within the respiratory epithelium which can
sense and respond to a variety of chemical irritants. These sensory cells may initiate coughing in
response to a variety of inhaled irritants and might induce bronchoconstriction. ANSWER IS A

128. Since a patient has had myocardial infarction, his atria and ventricles contract independently
from each other with a frequency of 60-70 and 35-40 per minute. Specify the type of heart
block in this case:

A. Intraventricular
B. Partial atrioventricular
C. Sino-atrial
D. Intra-atrial
E. Complete atrioventricular

Krok 1: 2014 Past Questions, Answers Explained NSA-TSMU Academic Team


EXP; This is a complete atrioventricular block because both atria and ventricles contract
independently showing that there’s a block between them. ANSWER IS E

129. A 35-year-old female patient has been hospitalised with acute intoxication caused by salts of
high-density metals (lead, most probably). As a part of complex therapy the antidote that
contains two active sulfhydric groups has been prescribed. Specify this antidote:

A. Metamizole
B Dimercaprol
C. Mannitol
D. Nalorphine hydrochloride
E. Calcium chloride

EXP- Dimercaprol is a chelating agent that is used to remove a heavy metal (such as lead or mercury)
from the blood. Metamizole acts as an antispasmodic and antipyretic. Mannitol is a diuretic.
Nalorphine is an opoid. CaCl is used for Mg toxicity. ANSWER IS B

130. A 60-year-old male patient has type II diabetes. A doctor has prescribed him synthetic
hypoglycemic long acting drug that is sulfonylurea derivative. What drug is it?

A. Glibenclamide
B. Butamide
C. Metformin
D. Actrapid (soluble insulin)
E. Acarbose

EXP- Glibenclamide is an antidiabetic drug in a class of medications known as sulfonylureas.


other sulfonylureas drugs are glipizide (glucotrol), gliclazide etc. ANSWER IS A

131. A patient has been given atropine sulfate for rapid relief of spastic colon symptoms. The use of
this drug is contraindicated during the following disease:

A. Hypotension
B. Bronchial asthma
C. Bradycardia
D. Glaucoma
E. Gastric ulcer

EXP- Moderate doses of atropine sulfate dilate pupils and increase intraocular pressure so its
contraindicate for glaucoma. ANSWER IS D

132. As an example of specific human parasites one can name Plasmodium falciparum, human
pinworm and some others. The source of parasite invasion is always a human. Such specific human
parasites cause the diseases that are called:

A. Anthroponoses
B. Zoonoses

Krok 1: 2014 Past Questions, Answers Explained NSA-TSMU Academic Team


C. Anthropozoonoses
D. Infections
E. Multifactorial diseases

EXP- Anthroponosis, is an infectious disease in which a disease causing agent carried by humans is
transferred to other animals. ANSWER IS E

133. In an experiment a dog had been conditioned to salivate at the sight of food and a flash of light.
After conditioning the reflex, the light was then paired with the bell. The dog didn’t start to salivate.
What type of inhibition was observed?

A. Extinctive
B. Differential
C. External
D. Persistent
E. Protective

EXP- External inhibition is the sudden decrement of a conditioned response due to the presence of a
non important stimulus (change of context). ANSWER IS C

134. A newborn baby has numerous hemorrhages. Blood coagulation tests reveal increased
prothrombin time. The child is most likely to have a disorder of the following biochemical process:
A. Production of gammacarboxyglutamate
B. Conversion of homocysteine to methionine
C. Conversion of methylmalonyl CoA to succinyl CoA
D. Degradation of glutathione
E. Hydroxylation of proline

EXP- Gammacarboxyglutamate is an essential substrate in several proteins, including coagulation


factors X, VII, IX, and XIV, vitamin K-dependent protein S and Z, prothrombin, transthyretin. ANSWER
IS D

135. A 63-year-old male patient with bladder atony had been prescribed a medication, which he had
been arbitrarily taking at a higher dose. The patient developed hypehydration, salivation diarrhea,
muscle spasms. The prescribed drug relates to the following group:

A. Cholinomimetics
B. Cholinesterase reactivators
C. Adrenergc blockers
D. Tocolytics
E. Ganglionic blockers

EXP- Cholinomimetics aka parasympathomimetic drug, stimulates the parasympathetic nervous


system by mimicing the effects of acetylcholine (ACh) as the neurotransmitter. On over
administration of the drug the parasympathetic nervous system is over stimulated the clinical signs
appear such as; hyperhydration, salivation, diarrhea, muscle spasms. ANSWER IS A

Krok 1: 2014 Past Questions, Answers Explained NSA-TSMU Academic Team


136. A 67-year-old male patient consumes eggs, pork fat, butter, milk and meat. Blood test results:
cholesterol - 12,3 mmol/l, total lipids - 8,2 g/l, increased low-density lipoprotein fraction (LDL). What
type of hyper-lipoproteinemia is observed in the patient?

A. Hyperlipoproteinemia type I
B. Hyperlipoproteinemia type IIa
C. Hyperlipoproteinemia type IIb
D. Hyperlipoproteinemia type IV
E. Cholesterol, hyperlipoproteinemia

EXP- Hyperlipoproteinemia type IIa is characterized by increased levels of LDL cholesterol in the blood
due to the lack of uptake of LDL particles. ANSWER IS B

137. Tissue sampling of a 37-year-old male patient with chronic renal disease has revealed the
following: sclerosis, lymphocytic and plasmocytic infiltration of renal pelvis and calices walls,
dystrophy and atrophy of tubules. Remaining tubules are enlarged and stretched with colloid
masses, epithelium is flattened out ("scutiform"or "shield-shaped"kidney). What is the
most likely diagnosis?

A. Chronic pyelonephritis
B. Tubular interstitial nephritis
C. Acute pyelonephritis
D. Glomerulonephritis
E. Nephrosclerosis

EXP- No haematuria that removes D. No history of drug or anything that could harm the interstitial,
that removes B. It is chronic, that removes C. No history of hypertension, so E is out. There was
sclerosis, lymphocytic and plasmocytic infiltration on renal (pelvis) suggests pyelonephritis (chronic
renal disease) shows the disease is chronic. ANSWER IS A.

138. During the hystological study of cortical shaft, basophilic cells with developed synthesis
organelles can be seen on the bone surface under the layer of fibers. These cells take part in
bone tissue regeneration. What shaft layer are they located in?

A. Bone
B. Periosteum
C. Osteon layer
D. Outer lamellae of compact bone tissue
E. Inner lamellae of compact bone tissue

EXP;- The periosteum in one is made up of osteocytes and osteoblasts(all basophilic), they are
responsible for bone tissues regeneration. ANSWER IS B

139. Autopsy of a 50-year-old male who had tuberculosis revealed a dense grey-white nidus in form
of a nodule 2 cm in diameter in the sub pleural portion of the upper right lobe. The pleura in this
region was thickened, in the pleural cavity there was a small amount of serous haemorrhagic fluid.
Histological study of the region revealed some glandular structures with signs of cellular atypia and

Krok 1: 2014 Past Questions, Answers Explained NSA-TSMU Academic Team


abnormal mitoses, which were found within the fibrous connective tissue. What other pathology
had developed in the lungs?

A. Fibro sarcoma
B. Squamous cell carcinoma
C. Adenoma
D. Adenocarcinoma
E. Fibroma

EXP;- Adenocarcinoma is the cancer of epithelial tissue that has glandular origin, glandular
characteristics, or both. ANSWER IS D

140. In course of an experiment there has been an increase in the nerve conduction velocity. This
may be caused by an increase in the concentration of the following ions that are present in the
solution around the cell:

A. Ca2+
B. K+ and Cl-
C. K+ and Na+
D. Ca2+ and Cl-
E. Na+

EXP;- Neurotransmitters interact specifically with sodium channels (or gates). So sodium ions flow
into the cell, reducing the voltage across the membrane and increasing their velocity. ANSWER IS E

141. Hepatitis B is diagnosed through laboratory tests that determine the presence of HBA-DNA in
blood serum of the patient. What reference method is applied for this purpose?

A. Ligase chain reaction method


B. Hybridization method
C. Hybridization signal amplification method
D. Polymerase chain reaction
E. ELISA diagnostic method

EXP;- Polymerase chain reaction is a tool that you can use to focus in on a segment of DNA and copy
it billions of times over. PCR is used every day to diagnose diseases, identify bacteria and viruses,
match criminals to crime scenes, and in many other ways. ANSWER IS D

142. On the 24th day since the onset of disease, a male patient diagnosed with typhoid fever and
undergoing treatment in an infectious diseases hospital has suddenly developed clinical
presentations of acute abdomen leading to the death of the patient. During autopsy peritonitis has
been revealed, with numerous ulcers covering the colon mucosa and reaching as deep as muscular
and, in places, serous tunic. The ulcers have smooth edges and even floor. The intestinal wall is
perforated. What stage of typhoid fever has the lethal complication arisen at?

A. Necrosis
B. Medullary swelling
C. Clean ulcer

Krok 1: 2014 Past Questions, Answers Explained NSA-TSMU Academic Team


D. Dirty ulcer
E. Regeneration

EXP;- The word here is ulcer, so every other option is out. The dilemma is if it’s clean or dirty. The
ulcer does not have a bloody discharge covering the grey slough and it’s not swollen. Ulcer has
smooth edges, so it’s clean. ANSWER IS C

143. An HIV-positive patient’s cause of death is acute pulmonary insufficiency resulting from
pneumonia. Pathohistological investigation of lungs has revealed interstitial pneumonia,
alveolocyte desquamation and their metamorphoses: alveolocyte enlargement, large intranuclear
Inclusions surrounded by lightly coloured areas. Transformed cells resemble owl’s eye. Name the
pneumonia causative agent:

A. Candida fungi
B. Pneumococcus
C. Influenza virus
D. Cytomegalovirus
E. Toxoplasma

EXP;- Cytomegalovirus (CMV) produces typical owl's eye intranuclear inclusion bodies in infected
cells. ANSWER IS D

144. The organisms to be identified have a nucleus surrounded by a nuclear membrane. Genetic
material is concentrated predominantly in the chromosomes which consist of DNA strands and
protein molecules. These cells divide mitotically. Identify these organisms:

A. Bacteria
B. Bacteriophages
C. Prokaryotes
D. Viruses
E. Eukaryotes

EXP;- Eukaryotic cells may contain several other types of organelles such as; mitochondria, nucleus,
chloroplasts, the endoplasmic reticulum, the Golgi apparatus, and lysosomes. While other do not
have these organelles. DNA is located in the nucleus. ANSWER IS E

145. A 37-year-old male patient developed pseudo-arthrosis after a closed fracture of the femur.
Specify the type of regeneration in the patient:

A. Pathological hypo-regeneration
B. Pathological hyper-regeneration
C. Reparative
D. Physiological
E. –
EXP;- In pseudo-arthrosis, the body perceives bone fragments as separate bones and do not attempt
to unite them, so the bone would not regenerate. It will make sense to go with A, WHICH IS THE
RIGHT ANSWER

Krok 1: 2014 Past Questions, Answers Explained NSA-TSMU Academic Team


146. Nuclear organizers of the 13-15, 21, 22 human chromosomes contain about 200 cluster genes
that synthesize RNA. These regions of chromosomes bear the information on the following type of
RNA:

A. snRNA
B. tRNA
C. mRNA
D. rRNA
E. tRNA + Rrna

EXP;- Genes synthesizing RNA should be rRNA. tRNA transfers information. mRNa serves as
messenger between DNA and protein synthesis. SnRNA processes heterogenous nuclear RNA.
ANSWER IS D

147. The resuscitation unit has admitted a patient in grave condition. It is known that he had
mistakenly taken sodium fluoride which blocks cytochrome oxidase. What type of hypoxia
developed in the patient?

A. Cardiovascular
B. Hemic
C. Tissue
D. Hypoxic
E. Respiratory

EXP;- An oxidizing enzyme that contains iron and a porphyrin and is found in the mitochondrial
membrane of tissue, where it catalyzes the transfer of electrons to oxygen as part of the electron
transport chain, ultimately leading to the formation of ATP. ANSWER IS C

148. Steatosis is caused by the accumulation of triacylglycerols in hepatocytes. One of the


mechanisms of this disease development is a decrease in the utilization of VLDL neutral fat. What
lipotropics prevent the development of steatosis?

A. Methionine, B6, B12


B. Arginine, B2, B3
C. Alanine, B1, P P
D. Valine, B3, B2
E. Isoleucine, B1, B2

EXP;- Steatosis occurs when the breakdown of fats in the liver is disrupted. Methionine, an essential
amino acid, is a major lipotropic compound in humans. When estrogen levels are high, the body
requires more methionine. Estrogens reduce bile flow through the liver and increase bile cholesterol
levels. Methionine helps deactivate estrogens. ANSWER IS A

149. A 2-year-old boy is diagnosed with Down syndrome. What chromosomal changes may be the
cause of this disease?

A. Trisomy 13
B. Trisomy 21

Krok 1: 2014 Past Questions, Answers Explained NSA-TSMU Academic Team


C. Trisomy X
D. Trisomy 18
E. Monosomy X

EXP;- Down syndrome is a genetic disorder caused when abnormal cell division results in extra
genetic material from chromosome 21. Patau is trisomy 13. Edward is trisomy 18. ANSWER IS B

150. A patient who has recently arrived from an endemic area presents with elevated body
temperature, headache, chills, malaise that is with the symptoms which are typical for a common
cold. What laboratory tests are necessary to prove or to disprove the diagnosis of malaria?

A. Urinalysis
B. Study of lymph node punctate
C. Microscopy of blood smears
D. Study of cerebrospinal fluid
E. Microscopy of bone marrow punctate

EXP;- Malaria parasites can be identified by examining under the microscope a drop of the patient's
blood, spread out as a "blood smear" on a microscope slide. ANSWER IS C

151. A patient has severe blood loss caused by an injury. What kind of dehydration will be observed
in this particular case?

A. Iso-osmolar
B. Hyposmolar
C. Hyperosmolar
D. Normosmolar
E. -

EXP;- Iso-osmolar(isotonic) dehydration is when proportionally the same amount of water and
sodium is lost from the body (e.g.; Repeated vomiting, Severe bleeding) the sodium concentration of
the extracellular fluid and hence its tonicity will not change. ANSWER IS A

152. A 30-year-old patient has dyspnoea Fits, mostly at night. He has been diagnosed with bronchial
asthma. What type of allergic reaction according to the Gell-Coombs classification is most likely in
this case?

A. Cytotoxic
B. Anaphylactic
C. Stimulating
D. Immune complex
E. Delayed-type hypersensitivity

EXP;- Gell–Coombs conception, disorders, such as hay fever or allergic asthma, are classic examples
of Type I hypersensitivity which leads to anaphylatic. ANSWER IS B

Krok 1: 2014 Past Questions, Answers Explained NSA-TSMU Academic Team


153. Decarboxylation of glutamate induces production of gamma-aminobutyric acid (GABA )
neurotransmitter. After breakdown, GABA is converted into a metabolite of the citric acid cycle, that
is:

A. Fumarate
B. Citric acid
C. Malate
D. Succinate
E. Oxaloacetate

EXP;- GABA transaminase enzyme catalyzes the conversion of 4-aminobutanoic acid (GABA) and 2-
oxoglutarate (α-ketoglutarate) into succinic semialdehyde and glutamate. Succinic semialdehyde is
then oxidized into succinic acid by succinic semialdehyde dehydrogenase and as such enters the citric
acid cycle. ANSWER IS D

154. An outbreak of an intestinal infection occurred in a kindergarten on the eve of New Year
holidays. Bacteriological examination of patients’ faeces didn’t reveal any pathogenic bacteria.
Electron microscopy revealed roundish structures with clear outer edges and a thick core resembling
a wheel. Specify the most likely causative agent of this infection:

A. Adenovirus
B. Rotavirus
C. Coxsacki-virus
D. E.coli
E. P.vulgaris

EXP;- On electron microscopy of rotavirus, intact virus particles resemble a wheel, with short spokes
and a well-defined rim. ANSWER IS B

155. A smear of streptobacillus preparation stained by Ozheshko method has been studied
microscopically with oil immersion. What structural feature of the bacteria has been studied?

A. Spores
B. Capsule
C. Flagella
D. Inclusions
E. Structure of cell wall

EXP;- On examination of bacillus by Ozheshko method it is usually to check for the presence of
spores. ANSWER IS D

156. After a road accident a victim has tachycardia, arterial blood pressure 130/90 mm Hg,
tachypnoe, the skin is pale and dry, excitation of central nervous system is observed. What
shock stage is the patient most likely in?

A. Agony
B. Terminal
C. Torpid

Krok 1: 2014 Past Questions, Answers Explained NSA-TSMU Academic Team


D. Preshock (compensation stage)
E. Erectile

EXP;- The erectile phase proceeds from several minutes to a half an hour. For it the expressed
reaction is characteristic from a CNS and Sympatico adrenal system. During this period, especially if
the trauma was preceded by a strong nervous overstrain, sensitivity rising to external stimuli,
impellent and speech excitation, fluctuations of arterial and venous pressure, pallor of integuments,
increase and quite often arrhythmia of pulse, breath. ANSWER IS E

157. Autopsy has revealed shrunken kidneys weighing 50 mg, with fine grained surface and
uniformly thinned substance. Microscopic investigation has shown the thickening of arteriole walls
due to accumulation of homogeneous anhistic pink-coloured masses in them. Glomerulus were
Undersized, sclerotic, with atrophied tubules. What disease are these changes characteristic of?

A. Renal amyloidosis
B. Pyelonephritis with kidney shrinkage
C. Essential hypertension
D. Acute glomerulonephritis
E. Membranous nephropathy

EXP;- Essential hypertension causes damages to the artery walls, it makes artery walls thick and stiff
and also causes kidney damages. ANSWER IS C

158. A fixed-run taxi passenger has a sudden and expressed attack of tachycardia. A doctor travelling
by the same taxi has managed to slow down his heart rate by pressing upon the eyeballs and thus
causing the following reflex:

A. Bainbridge reflex
B. Dagnini-Aschner reflex
C. Holtz’s reflex
D. Hering-Breuer reflex
E. Frank-Starling mechanism

-Dagnini-Aschner reflex; Slowing of the heart rate after pressure is applied to the eye or the carotid
sinus. It was formerly used to slow the heart in patients with supraventricular tachycardia or angina
pectoris. ANSWER IS B

159. Histological examination of biopsy samples taken from the thickened edges of a gastric ulcer
revealed small clusters of small, markedly atypical hyperchromatic epithelial cells that were localized
in the overdeveloped stroma. Specify the tumour:

A. Scirrhous undifferentiated carcinoma


B. Medullary carcinoma
C. Adenocarcinoma
D. Undifferentiated sarcoma
E. Adenoma
EXP;- Scirrhous undifferentiated cancer is of epithelial origin that is atypical and hyperchromatic.
ANSWER IS A

Krok 1: 2014 Past Questions, Answers Explained NSA-TSMU Academic Team


160. A 10-year-old child was found to have a congenital hypoplasia of the left kidney. Ultrasound
examination revealed that the right kidney was markedly enlarged and had regular shape. No
functional disorders were revealed. Specify the process that developed in the right kidney:

A. Hypertrophic growth
B. Working hypertrophy
C. Vicarious hypertrophy
D. Pseudohypertrophy
E. Metaplasia

EXP;- Vicarious hypertrophy is the hypertrophy of an organ following failure of another organ
because of a functional relationship between them. ANSWER IS C

161. In allergic diseases, a dramatic increase in basophilic leukocyte number in patients’ blood is
observed. This phenomenon is due to the following basophil function:

A. Participation of heparin and histamine in metabolism


B. Phagocytosis of microorganisms and small particles
C. Immunoglobulin synthesis
D. Phagocytosis of immune complexes
E. Participation in blood clotting

EXP;- Basophilic leukocytes are phagocytic leukocytes of the blood characterized by numerous
basophilic granules containing heparin, histamine, and leukotrienes. ANSWER IS A

162. A 47-year-old male patient developed intestinal colic against the background of essential
hypertension. In this situation it would be most efficient to arrest the colic by using drugs of the
following group:

A. Anticholinesterase agents
B. Myotropic antispasmodics
C. Sympathomimetic
D. M-cholinomimetics
E. Adrenomimetics

EXP;- Intestinal colic occurs due to smooth muscular contractions(spasms) of a hollow tube
(intestines, gall bladder or ureter). Best method of treatment is to give antispasmodics. ANSWER IS B

163. A patient has been admitted to the contagious isolation ward with signs of jaundice caused by
hepatitis virus. Which of the symptoms given below is strictly specific for hepatocellular jaundice?
A. Increase of ALT, AST level
B. Hyperbilirubinemia
C. Bilirubinuria
D. Cholemia
E. Urobilinuria

Krok 1: 2014 Past Questions, Answers Explained NSA-TSMU Academic Team


EXP;- The AST:ALT( aspartate aminotransferase /alanine aminotransferase) ratio can be a good
indicator because, when the liver cells are damaged or diseased, it releases ALT/AST into the
bloodstream, which makes ALT/AST levels go up. ANSWER IS A

164. An underage patient has signs of achondroplasia (dwarfism). It is known that this is a
monogenic disease and the gene that is responsible for the development of such abnormalities is
a dominant one. The development of that child’s brother is normal. Specify the genotype of the
healthy child:

A. AaBb
B. AA
C. Aa
D. aa
E. AABB

EXP;-. Achondroplasia is an autosomal dominant disorder, therefore a normal child from parents
with achondroplasia will have to get the good recessive gene from both (aa). ANSWER IS D

165. A patient has acute bronchitis. The fever up to 38,5oC had lasted for a week, presently there is
a decrease in temperature down to 37,0oC. Specify the leading mechanism in the 3rd stage of fever:

A. Increased respiratory rate


B. Increased heat production
C. Development of chill
D. Increased diuresis
E. Peripheral vasodilation

EXP;- In this stage The hypothalamus detects the blood as being too hot and initiates heat loss
(vasodilation). ANSWER IS E

166. A 35-year-old female patient has undergone biopsy of the breast nodules. Histological
examination has revealed enhanced proliferation of the small duct and acini epithelial cells,
accompanied by the formation of glandular structures of various shapes and sizes, which were
located in the fibrous stroma. What is the most likely diagnosis?

A. Adenocarcinoma
B. Fibroadenoma
C. Cystic breast
D. Invasive ductal carcinoma
E. Mastitis

EXP;- Fibroadenoma has rounded contour, overgrowth of fibrous and glandular tissue. ANSWER IS B

168. Examination of the duodenal contents revealed some pear-shaped protozoa with two nuclei
and four pairs of flagella. The organisms had also two axostyles between the nuclei and a ventral
adhesive disc. What protozoan representative was found in the patient?
A. Intestinal trichomonads

Krok 1: 2014 Past Questions, Answers Explained NSA-TSMU Academic Team


B. Toxoplasma
C. Leishmania
D. Lamblia
E. Trypanosome

EXP- Giardia lamblia is trophozoite which is pear shaped, with a broad anterior and much attenuated
posterior. It is 10-12µm long and 5-7µm wide, has 4 flagella, bilaterally symmetrical, and has two
nuclei. ANSWER IS D

169. A 36-year-old female patient has a history of B2-hypovitaminosis. The most likely cause of
specific symptoms (epithelial, mucosal, cutaneous, corneal lesions) is the deficiency of:

A. Cytochrome A1
B. Flavin coenzymes
C. Cytochrome oxidase
D. Cytochrome B
E. Cytochrome C

EXP;- Riboflavin (vitamin B2) deficiency is mostly caused by the deficiency of riboflavin coenzymes
FAD and FMN. this leads to the symptoms above. ANSWER IS B

170. A 54-year-old female was brought to the casualty department after a car accident.
Traumatologist diagnosed her with multiple fractures of the lower extremities. What kind of
embolism is most likely to develop in this case?

A. Air
B. Tissue
C. Thromboembolism
D. Gaseous
E. Fat

EXP;- A fat embolism is a type of embolism that is often caused by physical trauma such as fracture
of long bones, soft tissue trauma, and burns. ANSWER IS E

171. A blood drop has been put into a test tube with 0,3% solution of NaCl. What will happen to
erythrocytes?

A. Osmotic haemolysis
B. Shrinkage
C. Mechanical haemolysis
D. Any changes will be observed
E. Biological haemolysis

EXP;- When the red blood cell has been subjected to a hypotonic solution it undergoes osmosis. this
cause stress to the RBC cell wall, making it to rupture. ANSWER IS A

Krok 1: 2014 Past Questions, Answers Explained NSA-TSMU Academic Team


172. There are cortical and medullary substances separated by connective tissue layer in the
endocrine gland specimen. Parenchyma cells make up three zones in cortical substance, with
rounded masses in the superficial zone, parallel chords in the middle one, reticular structure of cell
chords in the deep one. What gland is it?

A. Pituitary gland
B. Thyroid gland
C. Adrenal gland
D. Epiphysis
E. Hypothalamus

EXP;- Adrenal glands is made up of two layers (cortex, medullar). The adrenal cortex itself is divided
into Three zones: zona glomerulosa, the zona fasciculata and the zona reticularis. ANSWER IS C

173. Histological specimen of a 10-day human embryo represents 2 contacting sacs (amniotic and
yolk sacs). Specify the structure that separates the amniotic cavity from the yolk sac:

A. Embryonic shield
B. Amniotic stalk
C. Floor of the amniotic sac
D. Roof of the yolk sac
E. Extraembryonic mesoderm

EXP;- The embryoblast forms an embryonic disc (embryonic shield) which is a bilaminar disc of two
layers, an upper layer the epiblast (primitive ectoderm), and a lower layer the hypoblast (primitive
endoderm). The disc is stretched between what will become the amniotic cavity and the yolk sac.
ANSWER IS A

174. An electron micrograph shows a cell of neural origin. The terminal portion of the cell dendrite
has cylindrical shape and consists of 1000 closed membrane disks. What cell is represented by the
micrograph?

A. Spinal node neuron


B. Cone receptor cell
C. Rod receptor cell
D. Neuron of the cerebral cortex
E. Neuron of the anterior horns of the spinal cord

EXP;- It comes down between cone and rod. Rod is cylindrical while cones is conical in shape.
ANSWER IS C

175. Disruption of nerve fiber myelinogenesis causes neurological disorders and mental retardation.
These symptoms are typical for hereditary and acquired alterations in the metabolism of:

A. Sphingolipids
B. Neutral fats
C. Higher fatty acids
D. Cholesterol

Krok 1: 2014 Past Questions, Answers Explained NSA-TSMU Academic Team


E. Phosphatidic acid

EXP;- Sphingolipids, a family of membrane lipids, are bioactive molecules that participate in diverse
functions controlling fundamental cellular processes such as cell division, differentiation, and cell
death. Sphingolipidoses, or disorders of sphingolipid metabolism, have particular impact on neural
tissue. ANSWER IS A

176. A patient underwent surgical removal of a cavitary liver lesion 2 cm in diameter. It was revealed
that the cavity wall was formed by dense fibrous connective tissue; the cavity contained muddy,
thick, yellowish greenish fluid with an unpleasant odor. Microscopically, the fluid consisted mainly of
polymorphonuclear leukocytes. What pathological processare these morphological changes typical
for?

A. Chronic abscess
B. Acute abscess
C. Phlegmon
D. Empyema
E

EXP;- In chronic abscess there are formations of cavity while there is none in acute.
The size and other signs (dense fibrous connective tissue wall) shows it is an abscess(i.e. both
empyema and phlegmon are much larger in size). ANSWER IS B

177. A patient complains of photoreception disorder and frequent acute viral diseases. He has been
prescribed a vitamin that affects photoreception processes by producing rhodopsin, the
photosensitive pigment. What vitamin is it?
A. Retinol acetate
B. Tocopherol acetate
C. Pyridoxine hydrochloride
D. Cyanocobalamin
E. Thiamine

EXP;- Retinyl Acetate is a naturally-occurring fatty acid ester form of retinol (vitamin A) with
potential antineoplastic and chemo preventive activities such as production of rhodopsin. Tocopherol
is vit. E. pyridoxine is vit. B6. Cyanocobalamin is vit. B12. Thiamine is vit. B2. ANSWER IS A

178. A 7-year-old boy got ill with diphtheria. On the third day he died of asphyxiation. At autopsy the
mucosa of the larynx, trachea and bronchi had thickened, edematous, lustreless appearance and
was covered with gray films which could be easily removed. Specify the type of laryngeal
inflammation:

A. Catarrhal
B. Diphtheritic
C. Purulent
D. Croupous
E. Intermediate

Krok 1: 2014 Past Questions, Answers Explained NSA-TSMU Academic Team


EXP; In corpus laryngitis, there is exudation of fibranous materials which leads to development of
respiratory problems. On autopsy presence of grey films which could be the fibrinous materials
produced. ANSWER IS D

179. A child has a history of hepatomegaly, hypoglycaemia, seizures, especially on an empty stomach
and in stressful situations. The child is diagnosed with Gierke disease. This disease is caused by the
genetic defect of the following enzyme:

A. Amyloid-1,6-glycosidase
B. Glucose-6-phosphatase
C. Phosphoglucomutase
D. Glycogen phosphorylase
E. Glucokinase

EXP;- Gierke's disease, is the most common of the glycogen storage diseases. This genetic disease
results from deficiency of the enzyme glucose-6-phosphatase, it impairs the ability of the liver to
produce free glucose from glycogen and from gluconeogenesis. ANSWER IS B

180. A public utility specialist went down into a sewer well without protection and after a while lost
Consciousness. Ambulance doctors diagnosed him with hydrogen sulphide intoxication. What type
of hypoxia developed?

A. Respiratory
B. Overload
C. Tissue
D. Circulatory
E. Hemic

EXP;- Basis of this hypoxia(hemic) type is decrease of blood oxygen capacity. There are Two
variants: anaemic and toxic. Toxic form arises in case of hemic toxins poisoning. The general content
of haemoglobin in blood remains normal, but the contents of functionally active haemoglobin
decreases. The part of hemoglobin turns into such compounds, which are not capable to execute
oxygen transport function. ANSWER IS E

181.A child with a normal karyotype is diagnosed with cleft lip and hard palate, defects of the
cardiovascular system, microcephaly. The child’s mother suffered rubella during pregnancy. This
pathology in the child may be an example of:

A. Genocopy
B. Trisomy
C. Phenocopy
D. Monosomy
E. -

EXP;- Genocopy refers to situation when identical phenotype is produced by two different genes /
genotypes. ANSWER IS A

Krok 1: 2014 Past Questions, Answers Explained NSA-TSMU Academic Team


182. A 28-year-old patient undergoing treatment in the pulmonological department has been
diagnosed with pulmonary emphysema caused by splitting of alveolar septum by tissular tripsin. The
disease is cased by the congenital deficiency of the following protein:

A. α1-proteinase inhibitor
B. α2-macroglobulin
C. Cryoglobulin
D. Haptoglobin
E. Transferrin

EXP;- Alpha 1-antitrypsin is also referred to as alpha-1 proteinase inhibitor (A1PI) because it inhibits a
wide variety of proteases. It protects tissues from enzymes of inflammatory cells, especially
neutrophil elastase and tripsin. ANSWER IS A

183. A patient with signs of osteoporosis and urolithiasis has been admitted to the endocrinology
department. Blood test has revealed hypercalcemia and hypophosphatemia. These changes are
associated with abnormal synthesis of the following hormone:

A. Aldosterone
B. Calcitonin
C. Cortisol
D. Parathyroid hormone
E. Calcitriol

EXP;- PTH (parathyriod hormone) increases blood calcium levels by stimulating osteoclasts to break
down bone and release calcium. It is also an inhibitor of proximal tubular reabsorption of
phosphorus. When there is of production of PTH, there hypercalcemia and hypophosphatemia.
ANSWER IS D

184. During a surgery for femoral hernia a surgeon operates within the boundaries of femoral
trigone. What structure makes up its upper margin?

A. Lig.inguinale
B. Arcus iliopectineus
C. Lig.lacunare
D. Lig. pectinale
E. Fascia lata

EXP;- Superior border-Formed by the inguinal ligament, a ligament that runs from the anterior
superior iliac spine to the pubis tubercle. Lateral border – Formed by the medial border of the
sartorius muscle. Medial border – Formed by the medial border of the adductor longus muscle.
ANSWER IS A

185. A 19-year-old victim has been delivered to the casualty department with a cut wound of the
trapezius muscle. Which of the cervical fasciae forms a sheath for this muscle?

A. Muscular part of the pretacheal layer of cervical fascia


B. Investing layer of cervical fascia

Krok 1: 2014 Past Questions, Answers Explained NSA-TSMU Academic Team


C. Visceral part of the pretracheal layer of cervical fascia
D. Prevertebral layer of cervical fascia
E. Carotid sheath of cervical fascia

EXP;- The Investing layer of deep cervical fascia is the most superficial part of the deep cervical
fascia, and it encloses the whole neck. Where it meets the trapezius and sternocleidomastoid
muscles. ANSWER IS B

186. A patient with acne has been prescribed doxycycline hydrochloride. What recommendations
should be given to the patient, while he is taking this drug?

A. Avoid long stay in the sun


B. Take with large quantity of liquid, preferably milk
C. Take before meal
D. The course of treatment should not exceed 1 day
E. Do not take with vitamins

EXP;- A IS CORRECT because the sun's UVrays has direct negative influences on patients with acne.

187. A 30-year-old patient with a past history of virus B hepatitis complains of prolonged
nosebleeds. What drug will be most efficient in remedying this condition?

A. Dipiridamol
B. Fraxiparine
C. Folic acid
D. Vicasolum
E. Asparcam

EXP;- Vicasol is a synthetic, water-soluble analog of vitamin K, which promotes the increase of
prothrombin in the blood and improves blood coagulation. ANSWER IS D

188. A patient has arterial hypertension. What long-acting drug from the group of calcium channel
blockers should be prescribed?

A. Reserpine
B. Octadine
C. Pyrroxanum
D. Atenolol
E. Amlodipine

EXP;- Amlodipine is the only long-acting drug of calcium channel blockers present in question.
more long-acting calcium channel blockers are( felodipine). Octadine is a ganglion blocker.
Pyrroxanum is an antianxiety drug. Atenolol is beta blocker. Reserpine is a sympatholytic. So,
ANSWER IS E

Krok 1: 2014 Past Questions, Answers Explained NSA-TSMU Academic Team


189. A patient has been diagnosed with ARVI. Blood serum contains immunoglobulin M. What is the
stage of infection in this case?

A. Reconvalescence
B. Prodromal
C. Incubation
D. Acute
E. Carriage

EXP;- IG and M are found in blood and lymph fluid and are the first type of antibody made in
response to an infection. ANSWER IS D

190. In a dysentery patient undergoing treatment in the contagious isolation ward, a significant
increase in packed cell volume has been observed (60%). What other value will be affected by
this change?

A. Leukopenia
B. Increasing volume of blood circulation
C. Increasing blood viscosity
D. Thrombocytopenia
E. Increasing erythrocyte sedimentation rate (ESR)

EXP: Increase in PCV shows that there is an abnormal increase in erythrocytes level. This can lead to
increased viscosity as there space for RBCs to flow normally. ANSWER IS C

191. A patient complains of palpitation after stress. The pulse is 104bpm, P-Q=0,12 seconds, there
are no changes of QRS complex. What type of arrhythmia does the patient have?

A. Sinus arrhythmia
B. Sinus bradycardia
C. Sinus tachycardia
D. Ciliary arrhythmia
E. Extrasystole

EXP: ECG of sinus tachycardia shows P–R interval: Between 0.12–0.20 seconds and shortens with
increasing heart rate. ANSWER IS C

192. A 30-year-old patient has undergone keratoplasty in the transplantation center, cornea has
been taken fron a donor, who died in a road accident. What kind of transplantation was performed?

A. Heterotransplantation
B. Autotransplantation
C. Xenotransplantation
D. Explantation
E. Allotransplantation

EXP;- Allotransplantation is the transplantation of cells, tissues, or organs, to a recipient from a


genetically non-identical donor of the same species. Auto is transplantation of tissues from one part

Krok 1: 2014 Past Questions, Answers Explained NSA-TSMU Academic Team


of body to another of the same person. Xeno is of different species. Hetero is from one individual to
another. ANSWER IS E

193. A 29-year-old male with a knife wound of neck presents with bleeding. During the initial
debridement of the wound the surgeon revealed the injury of a vessel found along the lateral edge
of the sternocleidomastoid muscle. Specify this vessel:

A. V. jugularis externa
B. V. jugularis anterior
C. A. carotis externa
D. A. carotis interna
E. V. jugularis interna

EXP;- The external jugular vein runs along the lateral edge of the sternocleidomastoid. Anterior
jugular runs along the median line and anterior border of sternocleidomastoid. Internal carotid runs
vertically upward in carotid sheath and enters the skull. External carotid runs behind the mandible.
Internal jugular runs laterally along with internal carotid. ANSWER IS A

194. A histologic specimen represents an organ with walls comprised of mucous, submucous, fibrous
cartilaginous and adventitial membranes. Epithelium is multirowed and ciliated, muscular layer of
mucous membrane is absent, submucous membrane contains serous-mucous glands, hyaline
cartilage forms open circles. What organ has the described morphological features?

A. Secondary bronchi (lobar bronchi)


B. Tertiary bronchi (segmental bronchi)
C. Trachea
D. Terminal bronchiole
E. Larynx.

EXP;- Histology of the trachea show all listed qualities, most especially hyaline cartilages which forms
open circles. ANSWER IS C

195. In cancer patients who have been continuously receiving methotrexate, the target cells of
tumor with time become insensitive to this drug. In this case, gene amplification of the following
enzyme is observed:

A. Dihydrofolate reductase
B. Thiaminase
C. Deaminase
D. Thioredoxin reductase
E. -

EXP;- DHFR (dihydrofolate reductase) can be targeted in the treatment of cancer. DHFR is
responsible for the levels of tetrahydrofolate in a cell, and the inhibition of DHFR can limit the
growth and proliferation of cells that are characteristic of cancer. Methotrexate, a competitive
inhibitor of DHFR, is one such anticancer drug that inhibits DHFR. ANSWER IS A

Krok 1: 2014 Past Questions, Answers Explained NSA-TSMU Academic Team


196. Pancreas is known as a mixed gland. Endocrine functions include production of insulin by beta
cells. This hormone affects the metabolism of carbohydrates. What is its effect upon the activity of
glycogen phosphorylase GP) and glycogen synthase (GS)?

A. It inhibits both GP and GS


B. It activates both GP and GS
C. It inhibits GP and activates GS
D. It activates GP and inhibits GS
E. It does not affect the activity of GP and GS

EXP;- Glycogen phosphorylase breaks up glycogen into glucose subunits AND glycogen synthase (GS)
takes glucose and converts them into long polymers of glycogen. Insulin has a negative effect on GP
and a positive effect on GS. ANSWER IS C

197. A patient has the oxyhemoglobin dissociation curve shifted to the left. What blood changes
induce this condition?

A. Alkalosis, hypocapnia, temperature drop


B. Acidosis, hypercapnia, temperature rise
C. Acidosis, hypercapnia, temperature drop
D. Acidosis, hypocapnia, temperature rise
E. -

EXP;- A shift to the left implies an increased oxygen affinity and hence, tighter binding due to the
higher oxygen saturation in relation to the pO2. This in turn leads to alkalosis, hypercapnia and
decrease in temperature. ANSWER IS A

198. Administration of doxycycline hydrochloride caused an imbalance of the symbiotic intestinal


microflora. Specify the kind of imbalance caused by the antibiotic therapy:

A. Sensibilization
B. Dysbacteriosis
C. Idiosyncrasy
D. Superimposed infection
E. Bacteriosis

EXP;- Doxycycline hydrochloride is an antibiotic which kills of bacteria in the body, including the
essential bacterial in gut system need for digestion. This leads to microbial imbalance on or inside the
body (dysbacteriosis). ANSWER IS B

199. A patient with signs of emotional lability that result in troubled sleep has been prescribed
nitrazepam. Specify the sleep-inducing mechanism of this drug:

A. Suppression of serotonergic neurotransmission


B. Blockade of opiate receptors
C. Inhibition of stimulating amino acids
D. H1-histamine receptors stimulation
E. GABA- ergic system activation

Krok 1: 2014 Past Questions, Answers Explained NSA-TSMU Academic Team


EXP;- Nitrazepam is a nitro-benzodiazepine. It acts on benzodiazepine receptors in the brain which
are associated with the GABA receptors, causing an enhanced binding of GABA to GABAA receptors.
GABA is a major inhibitory neurotransmitter in the brain. ANSWER IS E

200. A patient has been found to have a marked dilatation of saphenous veins in the region of
anterior abdominal wall around the navel. This is symptomatic of pressure increase in the following
vessel:

A. V. cava inferior
B. V. cava superior
C. V. portae hepatis
D. V. mesenterica inferior
E. V. mesenterica superior

EXP;- Probably a mistake(i.e. the saphenous), we should pay more attention to the dilation of veins in
anterior abdominal wall, most veins in abdominal region (i.e. Superior mesenteric vein, Splenic vein)
drain to the hepatic portal vein. An increase in pressure in H.P.vein would lead to dilation in most
veins in anterior abdominal wall. ANSWER IS C

Krok 1: 2014 Past Questions, Answers Explained NSA-TSMU Academic Team


KillingKrok2016 1

KROK 1 2015, ANSWERS EXPLAINED


Introduction
This material aims to explain answers and sometimes, point out why the other options are not
the correct ones. We encourage the student preparing for Krok 1 to avoid memorizing answers
and instead set out to understand questions and the concepts behind them. It would also help
a great deal to seek out topics that occur repeatedly, and study them thoroughly so one is
ready for those questions in whatever form.

Note: All answers are A

1. Characteristic sign of glycogenosis is muscle pain during physical work. Blood examination
usually reveals hypoglycemia. This pathology is caused by congenital deficiency of the
following enzyme:

A. Glycogen phosphorylase
B. Glucose 6-phosphate dehydrogenase
C. α-amylase
D. γ-amylase
E. Lysosomal glycosidase

EXP: Mcardle's Disease is a congenital disease caused by a deficiency in glycogen


phosphorylase in skeletal tissues. This phenomenon leads to muscle cramping during
exertion. This deficiency causes hypoglycemia since glycogen cannot be converted back to
glucose.

2. Histologic specimen of a kidney demonstrates cells closely adjoined to the renal corpuscle
in the distal convoluted tubule. Their basement membrane is extremely thin and has no folds.
These cells sense the changes in sodium content of urine and influence renin secretion
occurring in juxtaglomerular cells. Name these cells:
A. Macula densa cells
B. Juxtaglomerular cells
C. Mesangial cells
D. Podocytes
E. Glomerular capillary endothelial cells

EXP: These peculiarities are typical for Macula Densa cells; closely packed cells which are
part of the juxtaglomerular apparatus.

3. A 46-year-old female patient has continuous history of progressive muscular (Duchenne’s)


dystrophy. Which blood enzyme changes will be of diagnostic value in this case?

A. Creatine phosphokinase

B. Lactate dehydrogenase

C. Pyruvate dehydrogenase

NSA-TSMU Academic Team Krok I 2015 Questions


KillingKrok2016 2

D. Glutamate dehydrogenase

E. Adenylate cyclase

EXP: Creatine phosphokinase is a specific enzyme found in the skeletal muscles, heart
muscles and brain tissue. On destruction of any of these tissues, there is release of this
specific marker.

4. A laboratory experiment on a dog was used to study central parts of auditory system. One
of the mesencephalon structures was destroyed. The dog has lost the orienting response to
auditory signals. What structure was destroyed?

A. Inferior colliculi of corpora quadrigemina

B. Superior colliculi of corpora quadrigemina

C. Substantia nigra

D. Reticular formation nuclei

E. Red nucleus

EXP: Inferior colliculi of corpora quadrigemina contains the auditory nucleus and several
auditory pathways. Note that the Superior Colliculi has to do with vision.

5. A patient has decreased concentration of magnesium ions that are required for ribosomes
connection to granular endoplasmic reticulum. This condition is known to disrupt the process
of protein biosynthesis. Disruption occurs at the following stage:

A. Translation

B. Transcription

C. Replication

D. Amino acids activation

E. Processing

EXP: Translation occurs when the ribosome connects to the granular endoplasmic reticulum
which leads to protein biosynthesis, disruption of this process leads to disruption in
translation.

6. A 6-year-old child with suspected active tuberculous process has undergone diagnostic
Mantoux test. What immunobiological preparation was injected?

A. Tuberculin

B. BCG vaccine

C. DTP vaccine
NSA-TSMU Academic Team Krok I 2015 Questions
KillingKrok2016 3

D. Tularinum

E. Td vaccine

EXP: Mantoux test is a test done to diagnose TB and tuberculin is a purified protein derivative
essential for the test. BCG is the vaccine for TB.

7. During postembryonal haemopoiesis in red bone marrow the cells of one of the cellular
differons demonstrate gradual decrease in cytoplasmic basophilia as well as increase in
oxyphilia, the nucleus is being forced out. Such morphological changes are typical for the
following haemopoiesis type:

A. Erythropoiesis

B. Lymphopoiesis

C. Neutrophil cytopoiesis

D. Eosinophil cytopoiesis

E. Basophil cytopoiesis

EXP: Erythropiesis is the formation of red blood cells. The reticulocytes (young red blood
cells) do contain a nucleus but after they mature the nucleus is forced out and they gain more
affinity for oxygen(oxyphilia).

8. When taking exams students often have dry mouth. The mechanism that causes this state
results from the following reflexes:

A. Conditioned sympathetic

B. Unconditioned parasympathetic

C. Conditioned parasympathetic

D. Unconditioned sympathetic

E. Unconditioned peripheral

EXP: Conditioned sympathetic reflex consists of dryness of mouth, increased heart rate,
urinary retention. Please note that it is conditioned because the cause (exams) is not a
naturally occurring i.e the students had to learn it.

9. A 67-year-old patient with clinical diagnosis of chronic bronchitis, pneumosclerosis, and


cardiopulmonary decompensation has the biopsy material taken from the suspicious area in
his right bronchus mucosa. Cellular and tissue atypism along with Pearly bodies can be
histologically detected. What pathologic process is characterized by the described histological
changes?

A. Squamous cell carcinoma of bronchus with keratinization

NSA-TSMU Academic Team Krok I 2015 Questions


KillingKrok2016 4

B. Polypoid chronic bronchitis

C. Bronchiectasis

D. Acute bronchitis

E. Squamous cell metaplasia of bronchial mucosa

EXP: Cellular and tissues atypsim is a sign of malignzation and Pearly bodies are formed due
to keratinization. Also, squamous epithelium lines the walls of the bronchus.

10. A microspecimen of heart shows rectangular cells from 50 to 120 micrometers large with
central position of nucleus and developed myofibrils. The cells are connected by intercalated
discs. These cells are responsible for the following function:

A. Function of heart contractions

B. Function of impulse conduction

C. Endocrine

D. Protective

E. Regeneratory

EXP: Heart muscles are composed of tubular cells called myocytes, and a myofibril is the
basic unit of the myocyte. Notice the size of the cells and the intercalated discs (smooth
muscles).

11. Untrained people often have muscle pain after sprints as a result of lactate accumulation.
This can be caused by the intensification of the following biochemical process:

A. Glycolysis

B. Gluconeogenesis

C. Pentose phosphate pathway

D. Lipogenesis

E. Glycogenesis

EXP: Anaerobic glycolysis leads to production of lactate and this tends to accumulate in
muscle tissue and causes pain and cramps.

12. Poisoning caused by botulinum toxin that prevents calcium ions from entering axon
nerve endings of motor neurons is life-threatening because it can lead to:

A. Respiratory arrest

B. Cardiac arrest

NSA-TSMU Academic Team Krok I 2015 Questions


KillingKrok2016 5

C. Vasotonic disorder

D. Vomiting

E. Diarrhea

EXP: When there is prevention of calcium ions from entering the axon nerve endings of
motorneurons, there is prevention of the exocytosis of acetylcholine into the synaptic cleft.
This leads to paralysis of major respiratory muscles eg. diaphragm, intercostal muscles.

13. Increased HDL levels decrease the risk of atherosclerosis. What is the mechanism of HDL
anti-atherogenic action?

A. They remove cholesterol from tissues

B. They supply tissues with cholesterol

C. They are involved in the breakdown of cholesterol

D. They activate the conversion of cholesterol to bile acids

E. They promote absorption of cholesterol in the intestine HDL has the ability to remove
cholesterol from tissues.

EXP: High Density Lipoproteins (HDL) removes cholesterol from tissues so that they do not
accumulate in these tissues, therefore preventing process of atherosclerosis. This is why it is
known as good cholesterol.

14. It has been found out that one of a pesticide components is sodium arsenate that blocks
lipoic acid. Which enzyme activity is impaired by this pesticide?

A. Pyruvate dehydrogenase complex

B. Microsomal oxidation

C. Methemoglobin reductase

D. Glutathione peroxidase

E. Glutathione reductase

EXP: Sodium arsenate binds to the sullfhydryl groups in dihydrolipoic acid leading to an
impairement of the pyruvate dehydrogenase complex.

15. A drycleaner’s worker has been found to have hepatic steatosis. This pathology can be
caused by the disruption of synthesis of the following substance:

A. Phosphatidylcholine

B. Tristearin

NSA-TSMU Academic Team Krok I 2015 Questions


KillingKrok2016 6

C. Urea

D. Phosphatidic acid

E. Cholic acid

EXP: Steatosis, also called fatty degeneration is the abnormal retention of lipids.
Phosphatidylcholine acts as a surfactant that prevents lipid droplets from forming large
droplets. So disorder of its synthesis will cause deposition of fats in the liver.

16. A 35-year-old man with peptic ulcer disease has undergone antrectomy. After the surgery
secretion of the following gastrointestinal hormone will be disrupted the most:

A. Insulin

B. Glucagon

C. Thyroxin

D. Aldosterone

E. Adrenalin

EXP: Postoperative pancreatitis occurs after the antrectomy leading to a disruption of insulin.

17. Atretic bodies and developed yellow body can be observed along with follicles of various
orders in an ovary specimen. What stage of ovarian and menstrual cycle is characterized by
the described ovary condition?

A. Premenstrual

B. Menstrual

C. Postmenstrual

D. Regeneration

E. Follicle growth

EXP: Premenstrual stage is the stage between menstrual cycles when the uterus is preparing
for menstruation (growing follicles). It is characterized by the presence of atretic bodies and
developed yellow body.

18. A 16-year-old adolescent is diagnosed with hereditary UDP (uridine diphosphate)


glucuronyltransferase deficiency. Laboratory tests revealed hyperbilirubinemia caused mostly
by increased blood content of the following substance:

A. Unconjugated bilirubin

B. Conjugated bilirubin

NSA-TSMU Academic Team Krok I 2015 Questions


KillingKrok2016 7

C. Urobilinogen

D. Stercobilinogen

E. Biliverdine

EXP: UDP glucuronyltransferase deficiency leads to an increase in unconjugated bilirubin


since the enzyme converts unconjugated bilirubin to conjugated bilirubin.

19. Prior to glucose utilization in cells it is transported inside cells from extracellular space
through plasmatic membrane. This process is stimulated by the following hormone:

A. Insulin

B. Glucagon

C. Thyroxin

D. Aldosterone

E. Adrenalin

EXP: Insulin helps in the transportation and utilization of glucose in various tissues.

20. After implantation of a cardiac valve a young man systematically takes indirect
anticoagulants. His state was complicated by hemorrhage. What substance content has
decreased in blood?

A. Prothrombin

B. Haptoglobin

C. Heparin

D. Creatin

E. Ceruloplasmin

EXP: Indirect Anticoagulants (e.g warfarin) act by inhibiting Prothrombin in blood leading to
decreased clotting of blood and increased bleeding time. Note that direct anticoagulants (e.g
heparin) inhibit thrombin.

21. A 12-year-old patient has white non-pigmented spots on the skin. The spots appeared
after the patient became 10 years old, and they constantly grow. This spots appeared due to
the lack of the following skin cells:

A. Melanocytes

B. Adipocytes

C. Fibrocytes

NSA-TSMU Academic Team Krok I 2015 Questions


KillingKrok2016 8

D. Plasmocytes

E. Labrocytes

EXP: Melanocytes are responsible for the pigmentation of skin and a lack of it leads to
depigmentation.

22. A group of Ukrainian tourists returning from Samarqand was bringing with them gerbils.
During examination in customs office, ulcers were detected on the skin of the animals. What
protozoa is the most likely to cause the disease in the animals, if mosquitos are the carriers?

A. Leishmania tropica major

B. Balantidium coli

C. Plasmodium falciparum

D. Trypanosoma cruzi

E. Toxoplasma gondii

EXP: Leishmania tropica major causes leishmaniasis, a disease that causes ulcers in the skin
of its host and is transmitted by the mosquito.

23. A 5-year-old child has been diagnosed with acute right distal pneumonia. Sputum
innoculation revealed that the causative agent is resistant to penicillin and sensitive to
macrolides. What drug should be prescribed?

A. Azithromycin

B. Tetracycline

C. Gentamycin

D. Streptomycin

E. Ampicillin

EXP: Azithromycin is a macrolide that is broad spectrum and used to treat diseases and
infections that do not respond to penicillin medication. Note that most macrolides (antibiotics)
have the –ycin ending.

24. To an emergency ward a 7-year-old child was delivered in the condition of allergic shock
caused by a bee sting. High concentration of histamine is observed in blood. Production of this
amine is the result of the following reaction:

A. Decarboxylation

B. Hydroxylation

C. Dehydrogenation
NSA-TSMU Academic Team Krok I 2015 Questions
KillingKrok2016 9

D. Deaminization

E. Reduction

EXP: Production of Histamine leads to decarboxylation which is the removal of the carboxyl
group and release of carbon dioxide leading to respiratory insufficiency.

25. A 65-year-old man suffering from gout complains of pain in his kidneys. Ultrasonic
examination revealed kidney stones. A certain substance in increased concentration can
cause kidney stones formation. Name this substance:

A. Uric acid

B. Cholesterol

C. Bilirubin

D. Urea

E. Cystine

EXP: High quantities of uric acid cause it to crystallize, and accumulate to form stones. It
crystallizes easily due to very low solubility.

26. Sex chromatin was detected during examination of a man’s buccal epithelium. It is
characteristic of the following chromosome disease:

A. Klinefelter’s syndrome

B. Down’s disease

C. Turner’s syndrome

D. Triple X syndrome

E. Hypophosphatemic rickets

EXP: Sex chromatin is dna material only found in nuclei of female cells (esp as Barr body) and
believed to represent the X chromosome. Finding it in a man indicates Klinefelter’s syndrome
with karyotype is XXY.

27. Pure culture of microorganisms was obtained from pharynx of a child with suspected
diphtheria. Morphologic, tinctorial, cultural, and biochemical properties of the microorganisms
were studied and revealed to be characteristic of diphtheria agents. What investigation should
be additionally performed to make a conclusion, that these microorganisms are pathogenic
diphtheria bacilli?

A. Determine toxigenic properties

B. Determine proteolytic properties

NSA-TSMU Academic Team Krok I 2015 Questions


KillingKrok2016 10

C. Determine urease activity

D. Determine cystinase activity

E. Determine amylolytic activity

EXP: Diphtheria bacilli undergoes phage conversion and becomes toxigenic. The action of
toxins is the pathophysiology of disease so determining toxigenic properties is a sure way to
confirm diagnosis of diphtheria.

28. Autopsy of a man who had tuberculosis revealed a 3x2 cm large cavity in the superior
lobe of the right lung. The cavity was interconnected with a bronchus, its wall was dense and
consisted of three layers: the internal layer was pyogenic, the middle layer was made of
tuberculous granulation tissue and the external one was made of connective tissue. What is
the most likely diagnosis?

A. Fibrous cavernous tuberculosis

B. Fibrous focal tuberculosis

C. Tuberculoma

D. Acute focal tuberculosis

E. Acute cavernous tuberculosis

EXP: Fibrous cavernous tuberculosis is normally found in the superior lobe of lungs, fibrous
meaning there is formation of connective tissue, Cavernous meaning there is formation of a
cavity and of tuberculous origin.

29. A 7-year-old child has acute onset of disease: temperature rise up to 38oC, rhinitis, cough,
lacrimation, and large-spot rash on the skin. Pharyngeal mucosa is edematous, hyperemic,
with whitish spots in the buccal area. What kind of inflammation causes the changes in the
buccal mucosa?

A. Catarrhal inflammation

B. Suppurative inflammation

C. Fibrinous inflammation

D. Hemorrhagic inflammation

E. Serous inflammation

EXP: Catarrhal inflammation normally occurs in respiratory tracts accompanied by edema,


increased production of mucous and inflammation of the mucosa. Also there is no purulent
discharge (suppurative), connective tissue (fibrinous) or blood (hemorrhagic).

NSA-TSMU Academic Team Krok I 2015 Questions


KillingKrok2016 11

30. Analysis of sputum taken from a patient with suspected pneumonia revealed slightly
elongated gram-positive diplococci with pointed opposite ends. What microorganisms
were revealed in the sputum?

A. Streptococcus pneumoniae

B. Staphylococcus aureus

C. Klebsiella pneumoniae

D. Neisseria meningitidis

E. Neisseria gonorrhoeae

EXP: Gram positive diplococci with pointed ends is characteristic of Streptococcus


pneumoniae.

31. Serological diagnostics of infectious diseases is based upon specific interaction with
antigens. Specify the serological reaction that underlies adhesion of microorganisms when
they are affected by specific antibodies in presence of an electrolyte:

A. Agglutination reaction D. Hemadsorption reaction

B. Precipitation reaction E. Neutralization reaction

C. Complement-binding reaction

EXP: The options are all antibody-antigen reactions. But Agglutination is characterized by the
adhesion or clumping together of the antigens (microorganisms) after reacting with specific
antibodies in presence of electrolyte.

32. A 4-year-old child was admitted to an orthopaedic department with displaced shin
fracture. Bone fragments reposition requires analgesia. What drug should be chosen?

A. Promedol C. Morphine hydrochloride

B. Analgin D. Panadol

EXP: All the drugs have analgesic effect. However… Panadol is a mild analgesic and can’t
be used alone in this case, Analgin (also Metamizole) can cause serious blood-related
toxicity and is rarely used nowadays, Morphine also has a lot of side-effects.

Promedol (also Trimeperidine), is a very potent analgesic with sedative effect and is the best
choice in this case.

33. Doctors make mostly radial incisions during mammary gland surgery. What particulars of
anatomical organization make such surgical technique preferable?

A. Lobe apexes converge towards nipples C. Transversal position of gland lobes

B. Lobe bases radiate from nipples D. Vertical position of gland lobes


NSA-TSMU Academic Team Krok I 2015 Questions
KillingKrok2016 12

EXP: The female adult breast contains 14–18 irregular lactiferous lobes that converge at the
nipple. The apex of the lobes converge.

34. A 41-year-old man has a history of recurrent attacks of heartbeats (paroxysms), profuse
sweating, headaches. Examination revealed hypertension, hyperglycemia, increased basal
metabolic rate, and tachycardia. These clinical presentations are typical for the following
adrenal pathology:

A. Hyperfunction of the medulla D. Hypofunction of the adrenal cortex

B. Hypofunction of the medulla E. Primary aldosteronism

C. Hyperfunction of the adrenal cortex

EXP: Hyperfunction of the medulla leads to an increase in secretion of corticosteroids leading


to hyperfunction of the sympathetic nervous system (tachycardia, increased BMR, sweating
etc).

35. A 12-year-old child has developed nephritic syndrome (proteinuria, hematuria, cylindruria)
2 weeks after the case of tonsillitis, which is a sign of affected glomerular basement
membrane in the kidneys. What mechanism is the most likely to cause the basement
membrane damage?

A. Immune complex D. Reaginic

B. Granulomatous E. Cytotoxic

C. Antibody-mediated

EXP: Post streptococcal glomerulonepritis is characterized by the accumulation of immune


complex (antibody-antigen) in the basement membrane leading to its damage.

36. Several minutes after a dentist administered novocaine for local anaesthesia of a
patient’s tooth, the following symptoms sharply developed in the patient: fatigue, skin itching.
Objectively the following can be observed: skin hyperemia, tachycardia, BP dropped down to
70/40 mm Hg. What kind of allergic reaction is this pathology?

A. Anaphylactic B. Cytotoxic

NSA-TSMU Academic Team Krok I 2015 Questions


KillingKrok2016 13

C. Stimulating E. Immune complex

D. Cell-mediated immune reaction

EXP: The patient is allergic to novocaine and this leads to anaphylaxis which is characterized
by skin itching, skin hyperemia, tachycardia and drop in blood pressure. Note that it took only
a few minutes to develop.

37. A patient with probable liver abscess was delivered to a surgical department. The patient
for a long time had been on an assignment in an African country and had recurrent cases of
acute gastrointestinal disturbance. What protozoan disease can it be?

A. Amebiasis D. Malaria

B. Trypanosomiasis E. Toxoplasmosis

C. Leishmaniasis

EXP: Amoebiasis leads to the formation of liver abscesses and this leads to acute gastro-
intestinal disturbances.

38. Work in a mine is known to cause inhalation of large amounts of coal dust. Inhaled coal
dust can be detected in the following pulmonary cells:

A. Alveolar macrophages D. Capillary endothelial cells

B. Respiratory epithelial cells E. Pericapillary cells

C. Secretory epithelial cells

EXP: Alveolar macrophages ingest foreign materials in the lungs.

39. Electrical activity of neurons is being measured. They fire prior to, and at the beginning of
inhalation. Where are these neurons situated?

A. Medulla oblongata D. Spinal cord

B. Diencephalon E. Cerebral cortex

C. Mesencephalon

EXP: Medulla Oblangata contains the respiration center, therefore controls breathing
mechanism.

40. Electrocardiogram analysis demonstrates that cardiac cycle of a human equals 1 second.
It means that heart rate per minute equals:

A. 60 B. 50

NSA-TSMU Academic Team Krok I 2015 Questions


KillingKrok2016 14

C. 70 E. 100

D. 80

EXP: In a second the heart beats once, meaning in one minute the heart beats 60 times.

41. Glomerular filtration of a person, who has been starving for a long time, has increased
by 20%. The most likely cause of filtration changes in the given conditions is:

A. Decrease of blood plasma oncotic C. Increase of renal filter permeability


pressure
D. Increase of filtration factor
B. Increase of systemic blood pressure
E. Increase of renal plasma flow

EXP: Starvation leads to a use up of carbohydrates, fats and proteins. Protein deficiency
leads to reduction of substances like albumin(heavy molecules) that are responsible of
keeping blood plasma oncotic pressure normal, and lack of these heavy molecules will make
filtration faster.

42. Cells of healthy liver actively synthesize glycogen and proteins. What organelles are the
most developed in them?

A. Granular and agranular endoplasmic C. Lysosomes


reticulum
D. Mitochondria
B. Cell center
E. Peroxisomes

EXP: Granular and agranular endoplasmic reticulum are the cell organelles responsible for
the synthesis of glycogen and proteins.

43. Atria of an experimental animal were super distended with blood, which resulted in
decreased reabsorption of Na+ and water in renal tubules. This can be explained by the
influence of the following factor on kidneys:

A. Natriuretic hormone D. Angiotensin

B. Aldosterone E. Vasopressin

C. Renin

EXP: Natriuretic hormone is hormone released from the heart muscles when there is high
blood pressure, its release leads to increased release of sodium and water to reduce the high
blood pressure.

N.b: Natriuretic: Causing natriuresis, the excretion of an excessively large amount of sodium
in the urine.

A hybrid of the Latin natrium = sodium and the Greek ouresis = a making water.

NSA-TSMU Academic Team Krok I 2015 Questions


KillingKrok2016 15

44. A patient with hypertensive crisis has increased content of angiotensin II in blood.
Angiotensin pressor effect is based on:

A. Contraction of arteriole muscles

B. Activation of biogenic amine synthesis

C. Prostaglandin hyper-production

D. Vasopressin production stimulation

E. Activation of kinin–kallikrein system

EXP: ‘Pressor effect’ means to raise blood pressure usually by vasoconstriction. Angiotensin II
is a vasoconstricting agent which is found in the blood, its pressor effect is characterized by
the contraction of the arteriole muscles.

45. A 43-year-old-patient has arterial hypertension caused by increase in cardiac output and
general peripheral resistance. Specify the variant of hemodynamic development of arterial
hypertension in the given case:

A. Eukinetic C. Hypokinetic

B. Hyperkinetic D. Combined

EXP: The hemodynamic devt. of arterial hypertension may be Hypokinetic, Hyperkinetic or


Eukinetic.

Hyperkinetic type usually is accompanied by increasing of heart outflow with increased


heart index more than 4.5 l per min in m² and normal general peripheral resistance.

Hypokinetic type is accompanied by decreasing of heart outflow with decreased heart


index to 2.8 l per min in m² and high general peripheral resistance.

Eukinetic type is accompanied by normal or slightly increased heart outflow and


slightly increased general peripheral resistance.

46. A patient has been hospitalised with provisional diagnosis of virus B hepatitis. Serological
reaction based on complementation of antigen with antibody chemically bound to
peroxidase or alkaline phosphatase has been used for disease diagnostics. Name this
serological reaction:

A. Immune-enzyme analysis D. Bordet-Gengou test

B. Radioimmunoassay technique E. Antigen-binding assay

C. Immunofluorescence test

NSA-TSMU Academic Team Krok I 2015 Questions


KillingKrok2016 16

EXP: Note the enzymes used. ELISA (enzyme-linked immune-sorbent assay) or immune-
enzyme analysis is the only test here that uses enzymes.

47. A surgeon has to find the common hepatic duct during operative intervention for
treatment of concrements in the gall ducts. The common hepatic duct is located between the
leaves of:

A. Hepatoduodenal ligament D. Round ligament of liver

B. Hepatogastric ligament E. Venous ligament

C. Hepatorenal ligament

EXP: Common hepatic duct is found between the leaves of the hepatoduodenal ligament.

48. It is known that the gene responsible for development of blood groups according to AB0
system has three allele variants. Existence of the IV blood group can be explained by the
following variability form:

A. Combinative D. Genocopy

B. Mutational E. Phenocopy

C. Phenotypic

EXP: The fourth (IV) group known as AB blood group includes the combination of both A and
B therefore variability form is combinative.

49. When measuring power inputs of a person by the method of indirect calorimetry the
following results were obtained: oxygen consumption is 1000 ml and carbon dioxide
production is 800 ml per minute. The person under examination has the following respiratory
coefficient:

A. 0,8 D. 0,84

B. 1,25 E. 1,0

C. 0,9

EXP: The respiratory coefficient or quotient (RQ) is the ratio of CO 2 produced to O2 consumed
while food is being metabolized: RQ = CO 2 eliminated/O2 consumed i.e 800/1000 = 0.8

50. On examination of a newborn boy’s external genitalia a fissure in the urethra opening on
the inferior surface of his penis is detected. What maldevelopment is it?

A. Hypospadias C. Epispadia

B. Hermaphroditism D. Monorchism

NSA-TSMU Academic Team Krok I 2015 Questions


KillingKrok2016 17

E. Cryptorchidism

EXP: Hypospadias is when the urethral fissure opens on the inferior surface of the penis.
Epispadiais when it opens on the superior surface.

51. Poisoning caused by mercury (II) chloride (corrosive sublimate) occurred in the result of
safety rules violation. In 2 days the patient’s diurnal diuresis became 620 ml. The patient
developed headache, vomiting, convulsions, dyspnea; moist crackles are observed in the
lungs. Name this pathology:
A. Acute renal failure
B. Chronic renal failure
C. Uremic coma
D. Glomerulonephritis
E. Pyelonephritis
EXP: Poisoning with mercury leads to renal failure(but because it occurs within 2 days it is
acute)as there is damage to the kidneys causing a reduction in diurnal diuresis.

52. For people adapted to high external temperatures profuse sweating is not accompanied
by loss of large volumes of sodium chloride. This is caused by the effect the following
hormone has on the perspiratory glands:
A. Aldosterone
B. Vasopressin
C. Cortisol
D. Tgyroxin
E. Natriuretic
EXP: Aldosterone increases the reabsorption of sodium and water from sweat glands.

53. Along with normal hemoglobin types there can be pathological ones in the organism of an
adult. Name one of them:
A. HbS
B. HbF
C. HbA1
D. HbA2
E. HbO2

EXP: Hemoglobin S is a type of hemoglobin inherited from one's parents and is known to
have the sickle cell trait. HbF is fetal hemoglobin, HbO 2 oxygenated hemoglobin, while Hb A1
and A2 are hemoglobin proteins.

54. Emotional stress causes activation of hormone-sensitive triglyceride lipase in the


adipocytes. What secondary mediator takes part in this process?
A. Cyclic adenosine monophosphate
B. Cyclic guanosine monophosphate
C. Adenosine monophosphate
D. Diacylglycerol
E. Ions of __2+

EXP: Cyclic adenosine monophosphate cAMP is an important secondary messenger, and it


takes part in activation of hormone-sensitive triglyceride lipase and other processes.

NSA-TSMU Academic Team Krok I 2015 Questions


KillingKrok2016 18

55. A patient has been diagnosed with Alkaptonuria. Choose an enzyme that can cause this
pathology when deficient:
A. Homogentisic acid oxidase
B. Phenylalanine hydroxylase
C. Glutamate dehydrogenase
D. Pyruvate dehydrogenase
E. Dioxyphenylalanine decarboxylase

EXP: Alkaptonuria is caused by inherited deficiency of the liver enzyme homogentisic acid
deoxygenase. This leads to accumulation of homogentisic acid, which is rapidly cleared in the
kidney and excreted.

56. As a result of a continuous chronic encephalopathy a patient has developed spontaneous


motions and disorder of torso muscle tone. These are the symptoms of the disorder of the
following conduction tract:
A. Tractus rubrospinalis
B. Tractus corticospinalis
C. Tractus corticonuclearis
D. Tractus spinothalamicus
E. Tractus tectospinalis

EXP: The tractus rubospinalis is an extrapyramidal motor tract that runs from the motor Red
Nucleus to the spinal cord, mainly responsible for movement in upper extremities, this
includes the torso.
Rudimentary knowledge of Latin will help you a lot when it comes to the nervous system.
Finding out the literal English translations of the structures is a very good idea.

57. A child is 10 years old. The following presentations have developed: sharp pain during
swallowing, swollen neck, body temperature rise up to 39.0oC, bright-red fnely papular
rash all over the body. Pharynx and tonsils are sharply hyperemic ("faming pharynx"),
"crimson tongue". On the tonsils surface there are isolated greyish necrosis focuses. What
disease it might be?
A. Scarlet fever
B. Meningococcal nasopharyngitis
C. Diphtheria
D. Influenza
E. Measles
EXP: Apart from the ‘crimson tongue’ and ‘flaming pharynx’ which are pathognomonic for
Scarlet fever, the presence of papular rash all over the body, with high-temperature confirms
diagnosis.

58. A patient suffers from intermittent fevers and normalizations of body temperature that
occur during the day. The temperature rise is observed regularly every fourth day. Specify the
type of temperature curve:
A. Febris intermittens
B. Febris continua
C. Febris reccurens
D. Febris hectica
E. Febris remitens
NSA-TSMU Academic Team Krok I 2015 Questions
KillingKrok2016 19

EXP: Febris intermittens temperature curve is characterized by rising periods of temperature


(paroxysms) that alternate with the periods of normal temperature (apirrhexions).
in Febris Continua the daily fluctuation does not exceed 1oC. In Febris Recurrens, there
are periods of rising temp (5-8days), and periods of normal temp. In Febris Hectica, there is
daily fluctuation of up to 3-4oC. In Febris Remittens (indulgence fever), fluctuation between
morning and evening temps is 1-2oC.
The temp curve here is the febris quartana type of Febris Intermittens, where temp rises
every 4th day and then falls. F. intermittens could also be every 3 days (febris tertiana) or
daily (quotidiana).

59. A woman with the III (В), Rh- blood group gave birth to a child with the II (А) blood group.
The child is diagnosed with hemolytic disease of newborn caused by rhesus incompatibility.
What blood group and Rh can the father have?
A. II (А), Rh+
B. I (0), Rh+
C. III (B), Rh+
D. I (0), Rh−
E. II (A), Rh−
EXP: The child has hemolytic disease caused by Rhesus incompatibility. This means the child
is Rh(+) since the mother is Rh(-.) Rh is an inherited trait, so the father has to be Blood group
II(A) with Rh(+).

60. A patient is diagnosed with hereditary coagulopathy that is characterised by factor VIII
deficiency. Specify the phase of blood clotting during which coagulation will be disrupted in
the given case:
A. Thromboplastin formation
B. Thrombin formation
C. Fibrin formation
D. Clot retraction

EXP: The deficiency of factor VIII( haemophilia A) makes it impossible for the body to produce
thromboplastin.

61. Angiocardiography of a 60-year-old man revealed constriction of the vessel located in the
left coronary sulcus of his heart. Name this pathological vessel:
A. Ramus circumflexus
B. Ramus interventricularis posterior
C. A. coronaria dextra
D. V. cordis parva
E. Ramus interventricularis anterior

EXP: The Circumflex Artery is a ‘ramus’ or branch of the Left Coronary Artery that branches
off to loop around the heart via the left coronary sulcus (the coronary sulci are circular
grooves that delimit the atria and ventricles).

62. A patient complains of pain in the right lateral abdomen. Palpation revealed a dense,
immobile, tumor-like formation. The tumor is likely to be found in the following part of the
digestive tube:
A. Colon ascendens
B. Colon transversum
NSA-TSMU Academic Team Krok I 2015 Questions
KillingKrok2016 20

C. Colon descendens
D. Colon sigmoideum
E. Caecum
EXP: The ascending colon is located at the right lateral part of the abdomen.

63. During regular check-up a child is detected with interrupted mineralization of the bones.
What vitamin deficiency can be the cause?
A. Calciferol
B. Riboflavin
C. Tocopherol
D. Folic acid
E. Cobalamin
EXP: Calciferol is a derivative of Vitamin D which helps in development and strengthening of
children bones.

64. During histological analysis of the lymph node situated in the posterior neck triangle of an
18-year-old patient a morphologist detected a cluster of cells including the following: isolated
multinucleate Reed-Sternberg cells, large and small Hodgkin’s cells and numerous
lymphocytes, isolated plasma cells, eosinophils. What disease has developed in the patient?
A. Lymphogranulomatosis
B. Nodular lymphoma
C. Burkitt’s lymphoma
D. Lymphocytic lymphoma
E. Chronic lymphocytic leukemia

EXP: Reed-Sternberg cells are typical for Hodgkin’s lymphoma. This is a cancer of the
lymphatic system aka Lymphogranulomatosis.

65. An infant has pylorospasm, weakness, hypodynamia, convulsions as a result of frequent


vomiting. What kind of acid-base disbalance is it?
A. Excretory alkalosis
B. Excretory acidosis
C. Metabolic acidosis
D. Exogenous nongaseous acidosis
E. Gaseous alkalosis

EXP: In metabolic or excretory alkalosis, the main sign is vomiting (or other forms of
excretion like sweating/dehydration), which leads to loss in hydrogen ions which further leads
to weakness, pylorospasm, hopodynamia etc….

66. A 39-year-old man who had been operated for the stomach ulcer died 7 days after the
surgery. Autopsy revealed that peritoneal leaves were dull, plethoric, covered with massive
yellow-greenish films. The peritoneal cavity contained about 300ml of thick yellow-greenish
liquid. What pathologic process was revealed in the peritoneal cavity?
A. Fibrinous suppurative peritonitis
B. Serous peritonitis
C. Fibrinous serous peritonitis
D. Peritoneal commissures
E. Fibrinous haemorrhagic peritonitis

NSA-TSMU Academic Team Krok I 2015 Questions


KillingKrok2016 21

EXP: Fibrinous suppurative peritonitis is a secondary peritonitis mainly caused by perforation


of (e.g. peptic ulcer, diverticular disease, Crohn's disease, appendix or gall bladder), infection
(e.g. appendix abscess, pyosalpinx), it is characterized by yellow-greenish films and liquids.

67. Monoamine oxidase inhibitors are widely used as psycho-pharmacological drugs. They
change the level of nearly all neurotransmitters in synapses, with the following
neurotransmitter being the exception:
A. Acetylcholine
B. Noradrenaline
C. Adrenaline
D. Dopamine
E. Serotonin

EXP: This is because all enzymes here can be broken down in the body by both MAO-A and
MAO-B (Monoamine oxidase A&B) except acetylcholine.

68. A patient with urolithiasis has developed severe pain attacks. For pain shock prevention
he was administered an antispasmodic narcotic analgesic along with atropine. Name this
drug:
A. Promedol
B. Nalorphine
C. Tramadol
D. Ethylmorphine hydrochloride
E. Morphine hydrochloride

EXP: Promedol with both analgesic and spasmolytic effect is used either in adults, or in
children older than 2 years old on moderate and intensive pains, mainly of traumatic origin,
before, during and after surgery.
The preparation is effective on a pain syndrome related to the smooth muscle spasm in the
inner organs (ureter in this case) in combination with atropine-like and spasmolytic
preparations).

69. A patient with acute myocardial infarction has been administered heparin as a part of
complex therapy. Sometime after heparin injection the patient developed hematuria. What
heparin antagonist should be injected to remove the complication?
A. Protamine sulfate
B. Vicasol
C. Aminocaproic acid
D. Neodicumarin
E. Fibrinogen

EXP: Protamine sulfate is a drug that reverses the anticoagulant effects of heparin by binding
to it.

70. A 37-year-old woman complains of headache, vertigo, troubled sleep, numbness of limbs.
For the last 6 years she has been working at a gas-discharge lamp-producing factory in a
lead-processing shop. Blood test findings: low hemoglobin and RBC level, serum iron
concentration exceeds the norm by several times. Specify the type of anemia:
A. Iron refractory anemia
B. Iron-deficiency anemia
NSA-TSMU Academic Team Krok I 2015 Questions
KillingKrok2016 22

C. Minkowsky-Shauffard disease
D. Hypoplastic anemia
E. Metaplastic anemia

EXP: The underlined symptoms are neurological effects of Lead poisoning. Lead poisoning
also causes anemia by causing hemolysis of red blood cells… hence the increased iron
content in blood.

71. Despite the administration of cardiotonics and thiazide diuretic a patient with chronic
heart failure has persistent edemas and the risk of ascites arose. What medication should be
administered to enhance the diuretic effect of the administered drugs?
A. Spironolactone
B. Furosemide
C. Amiloride
D. Clopamide
E. Manithol

EXP: Thiazide diuretics are Potassium wasting diuretics. Spironolactone (Triamterene) is a


Potassium-sparing diuretic that also relieves pulmonary and systemic edema. It
reduces blood pressure and pre-load helping the heart failure.

72. Acute renal impairment caused death of a patient with hemorrhage. Autopsy revealed
enlarged kidneys with broad pale-pink cortical layer expressively demarcated from dark-red
renal pyramids. Macroscopic examination revealed lack of epithelial nuclei of convoluted
tubules, tubulorrhexis, phlebostasis. The cell nuclei of choroid glomus and straight tubules
were present. What pathology is it?
A. Necronephrosis
B. Infarction
C. Glomerulonephritis
D. Pyelonephritis
E. Nephrosis

EXP: In Necronephrosis (also called Acute Tubular Necrosis), there is usually tubulorrhexis,
that is, localized necrosis of the epithelial lining in renal tubules, with focal rupture or loss of
basement membrane.

73. A 3-year-old child with meningeal symptoms died. Postmortem macroscopy of the pia
matter revealed miliary nodules which were microscopically represented by a focus of
caseous necrosis with masses of epithelioid and lymphoid cells with large cells containing
crescent-shaped peripheral nuclei situated between them. Specify the type of meningitis in
the child:
A. Tuberculous
B. Syphilitic
C. Brucellar
D. Grippal
E. Meningococcal

NSA-TSMU Academic Team Krok I 2015 Questions


KillingKrok2016 23

EXP: This occurs most commonly shortly after a primary infection in childhood or as part of
miliary tuberculosis. The usual local source of infection is a caseous focus in the meninges or
brain substance adjacent to the CSF pathway.
The brain is covered by a greenish, gelatinous exudate, especially around the base, and
numerous scattered tubercles are found on the meninges.

74. A 66-year-old woman had intravenous injection of magnesium sulfate solution to stop
hypertensive crisis. However her arterial pressure did not decrease and after repeated
introduction of the same preparation she developed sluggishness, slow response to stimuli;
the patient is unconsciousness and her respiration is inhibited. What preparation is antagonist
of magnesium sulfate and can remove the symptoms of its overdose?
A. Calcium chloride
B. Potassium chloride
C. Sodium chloride
D. Activated carbon
E. Potassium permanganate

EXP: Calcium chroride IV can be used to treat magnesium intoxication.

75. A patient working at a pig farm complains of paroxysmal abdominal pain, liquid feces
with mucus and blood, headache, weakness, fever. Examination of large intestine revealed
ulcers from 1 mm up to several cm in diameter, feces contained oval unicellular organisms
with cilia. What disease can be suspected?
A. Balantidiasis
B. Amebiasis
C. Toxoplasmosis
D. Lambliasis
E. Trichomoniasis

EXP: Balantidium coli are oval unicellular organisms with cilia around them. It is acquired by
humans via the feco-oral route from the normal host, the pig.

76. An unconscious patient was delivered by ambulance to the hospital. On objective


examination the patient was found to present no reflexes, periodical convulsions, irregular
breathing. After laboratory examination the patient was diagnosed with hepatic coma.
Disorders of the central nervous system develop due to accumulation of the following
metabolite:
A. Ammonia
B. Urea
C. Glutamine
D. Bilirubin
E. Histamine

EXP: The ammonia accumulates in the brain tissues, some of which is converted to
glutamine, by way of the glutamine synthetase pathways, leading to localized swelling of
astrocytes (brain cells). This ammonia-induced glutamine accumulation may cause
dysfunction of astrocytes that leads to impairment of vascular reactivity.

NSA-TSMU Academic Team Krok I 2015 Questions


KillingKrok2016 24

77. When playing a child received a hit to the presternum region. As a result of this trauma an
organ located behind the presternum was damaged. Name this organ:
A. Thymus
B. Thyroid gland
C. Heart
D. Pericardium
E. Larynx

EXP: Presternal region is the region of the thorax overlying the sternum, the thymus is
located here.
p.s Do you know what the Thymus is named after?

78. A child suffers from dry cough. What non-narcotic antitussive drug will relieve the
patient’s condition?
A. Glaucine hydrochloride
B. Codeine phosphate
C. Morphine hydrochloride
D. Potassium iodide
E. Althaea officinalis root extract

EXP: Glaucine hydrochloride is the only non-narcotic anti-tussive here. Potassium iodide and
Extract of Althaea are not even anti-tussives.

79. A patient complains of acute pain attacks in the right lumbar region. During examination
a nephrolithic obturation of the right ureter in the region between its abdominal and pelvic
segments has been detected. What anatomical boundary exists between those two
segments?
A. Linea terminalis
B. Linea semilunaris
C. Linea arcuata
D. Linea transversa
E. Linea inguinalis

EXP: The linea teminalis is an imaginary anatomical boundary line that is between the
abdominal and lumbar regions.

80. A patient has insufficient blood supply to the kidneys, which has caused the development
of pressor effect due to constriction of arterial resistance vessels. This condition results from
the vessels being strongly affected by the following substance:
A. Angiotensin II
B. Angiotensinogen
C. Renin
D. Catecholamines
E. Norepinephrine

EXP When renal blood flow is reduced; juxtaglomerular cells in the kidneys convert the
Prorenin already present in the blood to Renin. Plasma Renin then converts Angiotensinogen,
released by the liver, to Angiotensin I.

NSA-TSMU Academic Team Krok I 2015 Questions


KillingKrok2016 25

Angiotensin I is subsequently converted to Angiotensin II by the enzyme angiotensin-


converting enzyme (ACE) found in the lungs. Angiotensin II is a potent vaso-active peptide
that causes blood vessels to constrict, resulting in increased blood pressure.

81. In a village a case of anthrax has been registered. Medical services began
epidemiologically indicated specific prophylaxis of population against anthrax. What
preparation was used for this purpose?
A. Live vaccine
B. Inactivated vaccine
C. Chemical vaccine
D. Genetically engineered vaccine
E. Anatoxin

EXP: Specific Prophylaxis means a vaccine. Anthrax vaccine is always live vaccine i.e. the
weakened (attenuated) form of the virus.

82. Experimental stimulation of the sympathetic nerve branches that innervate the heart
caused an increase in the force of heart contractions because the membrane of typical
Cardiomyocytes permitted an increase in:
A. Calcium ion entry
B. Calcium ion exit
C. Potassium ion exit
D. Potassium ion entry
E. Calcium and potassium ion exit

EXP: Sympathetic nervous system causes an increase in Inotropic effect (heart contraction)
by increasing the rate of calcium entry into the cardiomyocytes.

83. According to the results of glucose tolerance test a patient has no disorder of
carbohydrate tolerance. Despite that glucose is detected in the patient’s urine (5mmol/l). The
patient has been diagnosed with renal diabetes. What renal changes cause glucosuria in this
case?
A. Decreased activity of glucose reabsorption enzymes
B. Increased activity of glucose reabsorption enzymes
C. Exceeded glucose reabsorption threshold
D. Increased glucose secretion
E. Increased glucose filtration

EXP: On the decrease activity of glucose reabsorption enzymes on kidney, it makes it difficult
for the kidney to reabsorb glucose back into the blood after filtration, leading to glucosuria.

84. Alveolar space of acinus was invaded by bacteria that interacted with the surfactant. This
led to the activation of the cells that are localized in the alveolar walls and on the surface.
Name these cells:
A. Alveolar macrophages
B. Alveolocytes type I
C. Endothelial cells
D. Clara cells
E. Alveolocytes type II

NSA-TSMU Academic Team Krok I 2015 Questions


KillingKrok2016 26

EXP: Alveolar macrophages are localized on the alveolar wall. Alveolar macrophages are
phagocytes that play a critical role in homeostasis, host defence, and response to foreign
substances.

85. Parents of a 5-year-old boy report him to have frequent colds that develop into
pneumonias, presence of purulent rashes on the skin. Laboratory tests have revealed the
following: absence of immunoglobulins of any type, and naked cells are absent from the
lymph nodes punctate. What kind of immune disorder is it?
A. X-linked hypogammaglobulinemia (Bruton type agammaglobulinemia)
B. Autosomal recessive agammaglobulinaemia (Swiss type)
C. Hypoplastic anemia
D. Agranulocytosis
E. Louis-Barr syndrome

EXP: This question is about immune deficiencies and how to differentiate them.
The answer is Bruton’s Type Hypo-/Agammaglobuinemia, which is the reduction or complete
absence of immunoglobulins (IgA, IgD, IgE, IgM, IgG) due to a disorder of the B cells that
produce them. It affects humoral immunity and we can see the patient has recurrent
infections.
DiGeorge Syndrome will have absence of only T Cells (cellular immunity) and hypocalcemia
due to development disorder of the Thymus and Parathyroid glands respectively.
Swiss Type Agammaglobulinaemia will have absence of both T Cells and B
Cells/Immunoglobulins.
Louis–Bar syndrome also called Ataxia telangiectasia (A-T) is a rare, neurodegenerative,
autosomal recessive disease causing severe disability. Ataxia refers to poor coordination and
telangiectasia to small dilated blood vessels.
In Agranulocytosis, the concentration of granulocytes (neutrophils, basophils, & eosinophils)is
low or absent, Hypoplastic Anemia is well… Hypoplastic Anemia.

86. Examination of a 42-year-old patient revealed a tumour of adenohypophysis.


Objectively: the patient’s weight is 117kg; he has moon-like hyperemic face, red-blue striae of
skin distension on his belly. Osteoporosis and muscle dystrophy are present. AP is 210/140
mm Hg. What is the most probable diagnosis?
A. Cushing’s disease
B. Cushing’s syndrome
C. Conn’s disease
D. Diabetes mellitus
E. Essential hypertension

EXP: Moon –like hyperemic face, skin distension on belly, osteoporosis, muscle dystrophy and
high arterial blood pressure are key signs of Cushing’s.
The difference between Cushing Disease and Syndrome is that, Cushing's syndrome occurs
when the body produces too much cortisol while Cushing's disease occurs when a tumor on
the pituitary gland causes the gland to produce too much ACTH, the hormone responsible for
cortisol production.

87. 2 days after labour a woman developed shock along with DIC syndrome that caused her
death. Autopsy revealed purulent endomyometritis, regional purulent lymphangitis,
lymphadenitis and purulent thrombophlebitis. There were also dystrophic alterations and
interstitial inflammation of parenchymal organs. What is the most likely diagnosis?
NSA-TSMU Academic Team Krok I 2015 Questions
KillingKrok2016 27

A. Septicemia
B. Syphilis
C. Tuberculosis of genital organs
D. Chorioadenoma destruens
E. Hydatid mole

EXP: Purulent inflammation of heart, blood vessels, lymph nodes, and even parenchymal
organs, this shows generalized inflammation all over the body mostly due to septicaemia
(sepsis in blood).

88. In case of alkaptonuria, homogentisic acid is excreted in urine in large amounts. The
development of this disease is associated with metabolic disorder of the following amino acid:
A. Tyrosine
B. Phenylalanine
C. Alanine
D. Methionine
E. Asparagine
EXP: Alkaptonuria, also called Black Urine/Black Bone Disease is caused by build-up of
Homogentisic Acid due to lack of the enzyme Homogentisic Acid 1, 2, Dioxygenase to convert
it.

Phenylalanine-----------------------> Tyrosine---------------> Homogentisic Acid-------------------


>MalylAcetoacetate
Phenylketonuria
Alkaptonuria

89. When blood circulation in the damaged tissue is restored, lactate accumulation stops and
glucose consumption decelerates. These metabolic changes are caused by activation of the
following process:
A. Aerobic glycolysis
B. Anaerobic glycolysis
C. Lipolysis
D. Gluconeogenesis
E. Glycogen biosynthesis

EXP: When the blood supply comes back to damaged tissue, oxygen returns back which in
turn instead of lactate accumulation (because of anaerobic glycolysis) glucose is converted to
pyruvate (aerobic glycolysis).

90. A doctor examined a patient, studied the blood analyses, and reached a conclusion, that
peripheral immunogenesis organs are affected. What organs are the most likely to be
affected?
A. Tonsils
B. Thymus
C. Kidneys
D. Red bone marrow
E. Yellow bone marrow

EXP: The peripheral immunogenetic organs are the tonsils while the central immunogenesis
organs are the red bone marrow, thymus, lymph nodes and spleen.
NSA-TSMU Academic Team Krok I 2015 Questions
KillingKrok2016 28

91. A doctor asked a patient to make a deep exhalation after a normal inhalation. What
muscles contract during such exhalation?
A. Abdominal muscles
B. External intercostal muscles
C. Diaphragm
D. Trapezius muscles
E. Pectoral muscles

EXP: Deep or forced exhalation is achieved by the abdominal and the internal intercostal
muscles. During this process air is forced or exhaled out. The Diaphragm and External
Intercostal muscles contract for inhalation.

92. A 4-year-old child with hereditary renal lesion has signs of rickets; vitamin D
concentration in blood is normal. What is the most probable cause of rickets development?
A. Impaired synthesis of calcitriol
B. Increased excretion of calcium
C. Hyperfunction of parathyroid glands
D. Hypofunction of parathyroid glands
E. Lack of calcium in food

EXP: In this child, Vitamin D is normal in blood.


Calcitriol (dihydroxycholecalciferol) is produced in the kidneys, and it increases blood calcium
level… the renal lesion leading to reduced calcitriol and reduced blood calcium is the primary
cause of this rickets.

93. In a cat with decerebrate rigidity the muscle tone is to be decreased. This can be
achieved by:
A. Destruction of the vestibular nuclei of Deiters
B. Stimulation of the otolithic vestibular receptors
C. Stimulation of the vestibular nuclei of Deiters
D. Stimulation of the vestibulocochlear nerve
E. Stimulation of the ampullar vestibular Receptors

EXP: The lateral vestibular nucleus (Deiter’s nucleus) is responsible for muscle tone.

94. Autopsy of a 5-year-old child revealed in the area of vermis of cerebellum a soft greyish-
pink node 2 cm in diameter with blurred margins and areas of haemorrhage. Histologically
this tumour consisted of atypical monomorphous small round cells with large
polymorphous nuclei. What tumour is it?
A. Medulloblastoma
B. Meningioma
C. Glioblastoma
D. Astrocytoma
E. Oligodendroglioma

EXP: Medulloblastoma is a pediatric malignant primary brain tumor (cancer), histologically


it is seen
atypical monomorphous small round cells with large polymorphous nuclei.

NSA-TSMU Academic Team Krok I 2015 Questions


KillingKrok2016 29

95. Surface with an intact toad on it was inclined to the right. Tone of extensor muscles
became reflectory higher due to the activation of the following receptors:
A. Vestibuloreceptors of utricle and saccule
B. Vestibuloreceptors of semicircular ducts
C. Mechanoreceptors of foot skin
D. Photoreceptors of retina
E. Proprioreceptors

EXP: Vestibuloreceptors of utricle and saccule are responsible for tone of extensor muscles in
the toad.

96. A patient with high-titer antinuclear antibodies died from progressing renal
impairment. Autopsy revealed mesangioproliferative glomerulonephritis and abacterial
polypous endocarditis. Periarterial bulbar sclerosis was detected in spleen and productive
proliferative vasculitis in skin. What is the most likely diagnosis?
A. Systemic lupus erythematosus
B. Nephrotic syndrome
C. Rheumatism
D. Dermatomyositis
E. Periarteritis nodosa

EXP: Systemic lupus erythematous (SLE) is a chronic autoimmune disease that can affect
almost any organ system. It is characterized by butterfly rash, it can affect the kidneys, heart,
vessels etc. on biochemical blood examination it presents with high-titre antinuclear
antibodies.

97. When studying the signs of pulmonary ventilation, reduction of forced expiratory volume
has been detected. What is the likely cause of this phenomenon?
A. Obstructive pulmonary disease
B. Increase of respiratory volume
C. Increase of inspiratory reserve volume
D. Increase of pulmonary residual volume
E. Increase of functional residual lung capacity

EXP: The reduction of FEV1 (forced expiratory volume in one second) is typical for obstructive
pulmonary diseases. Think about it, there is an ‘obstruction’ to exhaled/outgoing air.

98. A specimen of a parenchymal organ shows poorly delineated hexagonal lobules


surrounding a central vein, and the interlobular connective tissue contains embedded triads
(an artery, a vein and an excretory duct).What organ is it?
A. Liver
B. Pancreas
C. Thymus
D. Spleen
E. Thyroid

EXP: Histologically the Liver is divided into lobules. The center of the lobule is the central
vein. At the peripheries of the lobule are portal triads (an artery, a vein and an excretory
duct).

NSA-TSMU Academic Team Krok I 2015 Questions


KillingKrok2016 30

99. A patient had a trauma that caused dysfunction of motor centres regulating activity of
head muscles. In what parts of cerebral cortex can the respective centre normally be located?
A. Inferior part of Precentral gyrus
B. Superior part of precentral gyrus
C. Supramarginal gyrus
D. Superior parietal lobule
E. Angular gyrus

EXP: The Precentral gyrus controls motor activities. The superior part is for the lower limbs,
the middle part for the trunk and arms, and the inferior part for the head.

100. During intravenous saline transfusion a patient’s condition deteriorated drastically, and
the patient died from asphyxiation. Autopsy revealed acute venous congestion of internal
organs with sharp right heart dilatation. When the right ventricle was punctured underwater,
the bubbles escaped. What pathological process occurred in the patient?
A. Air embolism
B. Gaseous embolism
C. Adipose embolism
D. Tissue embolism
E. Thromboembolism

EXP: An air embolism, also called a gas embolism, occurs when one or more air bubbles enter
a vein or artery and block it. This air was caused by intravenous saline transfusion which had
air (bubbles) in it. The air caused the dilation of right side of the heart.

101. Diseases of respiratory system and circulatory disorders impair the transport of oxygen,
thus causing hypoxia. Under these conditions the energy metabolism is carried out by
anaerobic glycolysis. As a result, the following substance is generated and accumulated in
blood:
A. Lactic acid*
B. Pyruvic acid
C. Glutamic acid
D. Citric acid
E. Fumaric acid

EXP: In absence of oxygen, pyruvate is converted to Lactate by influence of Lactate


dehydrogenase enzyme. In prolonged oxygen deprivation, Lactic acid accumulates in blood.

102. Examination of a 6-month-old child revealed a delay in closure of the occipital


Fontanelle. When should it normally close?
A. Before 3 months*
B. Before the child is born
C. Before 6 months
D. Before the end of the first year of life
E. Before the end of the second year of life

EXP: Occipital fontanelle closes within first 2 to 3 months of life.

NSA-TSMU Academic Team Krok I 2015 Questions


KillingKrok2016 31

103. Sex chromosomes of a woman didn’t separate and move to the opposite poles of a cell
during gametogenesis (meiosis). The ovum was impregnated with a normal spermatozoon.
Which chromosomal disease can be found in her child?
A. Turner’s syndrome*
B. Down’s syndrome
C. Patau’s syndrome
D. Edwards’ syndrome
E. Cat cry syndrome

EXP: Failure of X chromosome separation leads to the presence of only 1 X chromosome in a


female child XO, thus; Turner Syndrome.

104. A patient was prescribed Loratadine to treat allergic cheilitis. What is the mechanism of
action of this drug?
A. Blockade of H1-histamine receptors*
B. Blockade of adrenergic receptors
C. Increases activty of monoamine oxidase
D. Suppresses activity of Na+/K+-ATPase
E. Suppresses activity of choline esterase

EXP: Loratadine is an Anti-histamine drug which blocks H1 histamine receptors. Note that
most anti-histamines have the ‘-ine’ ending (Acrivastine, Clemastine, Cetirizine, Cimetidine,
Famotidine)

105. During introduction of local anaesthesia a patient has gone into anaphylactic shock.
What drug must be administered to the patient?
A. Epinephrine hydrochloride*
B. Diazepam
C. Atropine sulfate
D. Propranolol
E. Nitroglycerin

EXP: In Anaphylactic shock, The Internal Organs enter into a shut-down phase. As a result,
Epinephrine/Adrenaline hydrochloride, a non-selective adrenomimetic vigorously acts on all
adrenergic receptors preventing and reversing the Shut- down phase.

106. A patient has been diagnosed with influenza. His condition drastically worsened after
taking antipyretic drugs. He is unconscious, AP is 80/50 mm Hg, Ps is 140/m, body
temperature dropped down to 35,8oC. What complication developed in this patient?
A. Collapse*
B. Hyperthermia
C. Hypovolemia
D. Acidosis
E. Alkalosis

EXP: In Influenza, administration of Antipyretics leads to appearance of a complication called


Collapse (note the sharply reduced blood pressure and temperature, increased pulse, and
unconsciousness).

NSA-TSMU Academic Team Krok I 2015 Questions


KillingKrok2016 32

107. An alcoholic has alcoholic psychosis with evident psychomotor agitation. What
neuroleptic drug should be administered for emergency aid?
A. Aminazine*
B. Diazepam
C. Sodium bromide
D. Reserpine
E. Halothane

EXP: Aminazine is a Neuroleptic; psychotropic drug from the group PHENOTHIAZINES used in
psychosis treatment.

108. During blood transfusion a patient has developed intravascular erythrocyte hemolysis.
What kind of hypersensitivity does the patient have?
A. II type (antibody-dependent)*
B. I type (anaphylactic)
C. III type (immune complex)
D. IV type (cellular cytotoxicity)
E. IV type (granulomatosis)

EXP: In blood transfusion, intravascular erythrocyte hemolysis belongs to Type II antibody


dependent hypersensitivity and it occurs in transfusion of Incompatible Blood Groups.

109. In the course of an experiment thalamocortical tracts of an animal were cut. What type
of sensory perception remained intact?
A. Olfactory*
B. Auditory
C. Exteroreceptive
D. Visual
E. Nociceptive

EXP: The Olfactory tract is the only tract among these that does not pass through Thalamus
or have Collateral connections with thalamus.

110. A 4-year-old child presents with general weakness, sore throat and deglutitive problem.
After his examination a doctor suspected diphtheria and sent the material to a
bacteriological laboratory. To determine the diphtheria causative agent the material should be
inoculated into the following differential diagnostic medium:
A. Blood tellurite agar*
B. Endo’s agar
C. Ploskyrev’s agar
D. Sabouraud’s agar
E. Levenshtein-Yessen agar

EXP: Diagnostic culture media for diphtheria is blood tellurite agar.

111. When treating a patient with chronic cardiac failure a doctor detected bradycardia and
deterioration of the patient’s general state. Such condition is caused by cumulative effect of a
drug. Which drug of those listed below has cumulative action?
A. Digoxin*
B. Diphenhydramine (Dimedrol)
NSA-TSMU Academic Team Krok I 2015 Questions
KillingKrok2016 33

C. Hydrochlorothiazide
D. Isosorbide
E. Retinol acetate

EXP: Cardiac glycosides; e.g Digoxin have an accumulated toxic effect. Thus, deterioration of
the patient’s state.

112. A doctor was addressed by a 30 year-old man. There is a probability of the patient being
HIV-positive. To clarify the diagnosis the doctor proposed to perform polymerase chain
reaction. The basic process in this kind of investigation is:
A. Gene amplification*
B. Transcription
C. Genetic recombination
D. Genomic mutation
E. Chromosome mutation

EXP: P.C.R is an enzymatic method for repeated copying of two strands of DNA of a gene
sequence. It amplifies minute/small quantities of Biologic material for laboratory studies.

113. Due to the use of poor-quality measles vaccine for preventive vaccination, a 1- year-old
child developed an autoimmune renal injury. The urine was found to contain macromolecular
proteins. What process of urine formation was disturbed?
A. Filtration*
B. Reabsorption
C. Secretion
D. Reabsorption and secretion
E. Secretion and filtration

EXP: Auto-Immune renal injuries mainly affect Glomerulus of the kidney distorting its
permeability. Thus, there is disorder of filtration that is bigger molecules can pass through the
membrane.

114. A 26-year-old female patient with bronchitis has been administered a broad spectrum
antibiotic as a causal treatment drug. Specify this drug:
A. Doxycycline*
B. Interferon
C. BCG vaccine
D. Ambroxol
E. Dexamethasone

EXP: Doxycycline is the only antibiotic among the options provided.

115. A 65-year-old man suddenly lost vision in one eye due to the retinal detachment. The
patient underwent enucleation. Histological examination of the removed eye retina and
choroid revealed clusters of atypical cells with marked polymorphism of cells and
nuclei, with a moderate number of mitoses including the pathological ones. The cell
cytoplasm and intercellular medium contained brown pigment resulting in positive DOPA
reaction. Perls’ reaction was negative. What is the most likely diagnosis?
A. Melanoma*
B. Pigmented mole
NSA-TSMU Academic Team Krok I 2015 Questions
KillingKrok2016 34

C. Hemorrhage
D. Cysticercosis
E. Wilson’s disease

EXP: Enucleation is the removal of entire structure of eyeball without rupture usually
occurring in the presence of Tumours. In this case; Melanoma (note the presence of brown
pigment).

116. During determining the blood group according to the AB0 system with salt solutions of
monoclonal antibodies agglutination did not occur with any of the solutions. What blood group
is it?
A. 0 (I)*
B. А (II)
C. В (III)
D. АВ (IV)

EXP: Group I(O) or group zero is called the Universal donor because it has neither anti-A nor
anti-B antibodies and is compatible with all other groups.

117. One of the factors that cause obesity is inhibition of fatty acids oxidation due to:
A. Low level of carnitine*
B. Impaired phospholipid synthesis
C. Excessive consumption of fatty foods
D. Choline deficiency
E. Lack of carbohydrates in the diet

EXP: Fatty acids’ Beta oxidation is impaired in Carnitene (a transport protein) deficiency as
fatty acids can’t be transported into inner mitochondrial membrane.

118. During ventricular systole the cardiac muscle does not respond to additional stimulation
because it is in the phase of:
A. Absolute refractoriness*
B. Relational refractoriness
C. Hyperexcitability
D. Subnormal excitability
E. There is no correct answer

EXP: In absolute refractory period, Excitability is depressed and as a result, action potentials
of threshold intensity cannot stimulate heart any further.

119. A patient is diagnosed with acute morphine hydrochloride intoxication. Prescribe the
oxidizing agent for gastric lavage:
A. Potassium permanganate*
B. Chloramine
C. Sulfocamphocainum (Procaine + Sulfocamphoric acid)
D. Cerigel
E. Chlorhexidine (bi)gluconate
EXP: Potassium permanganate (purple crystals) is a strong oxidising agent used for gastric
lavage.
NSA-TSMU Academic Team Krok I 2015 Questions
KillingKrok2016 35

120. During cell division DNA replication occurs after a signal is received from the cytoplasm,
then a certain portion of the DNA helix unwinds and splits into two individual strains. What
enzyme facilitates this process?
A. Helicase*
B. RNA polymerase
C. Ligase
D. Restrictase
E. DNA polymerase
EXP: Unwinding of DNA helix is facilitated by the enzyme Helicase

121. A patient has been given atropine sulfate for rapid relief of spastic colon symptoms. The
use of this drug is contraindicated during the followng disease:
A. Glaucoma*
B. Bronchial asthma
C. Bradycardia
D. Hypotension
E. Gastric ulcer

EXP: Atropine sulfate increases intraocular pressure, as a result is contraindicated in


Glaucoma patients.

122. As an example of specific human parasites one can name Plasmodium falciparum,
human pinworm and some others. The source of parasite invasion in these cases is always a
human. Such specific human parasites cause the diseases that are called:
A. Anthroponoses*
B. Zoonoses
C. Anthropozoonoses
D. Infections
E. Multifactorial diseases

EXP: Diseases caused by human parasites are called ANTHRO-PONOSES. By animals is


Zoo-noses

123. In the course of an experiment there has been increase in nerve conduction velocity.
This may be caused by increase in concentration of the following ions that are present in the
solution around the cell:
A. Na+*
B. K+ and Cl−
C. K+ and Na+
D. Ca2+ and Cl−
E. Ca2+

EXP: Cells are bags of potassium ions surrounded by sodium ions. Nerve conduction
occurs when sodium enters and potassium leaves.

124. An HIV-positive patient’s cause of death is acute pulmonary insufficiency resulting from
pneumonia. Patho-histological investigation of lungs has revealed interstitial Pneumonia,
alveolocyte desquamation and methamorphoses: alveolocyte Enlargement, large intranuclear

NSA-TSMU Academic Team Krok I 2015 Questions


KillingKrok2016 36

inclusions surrounded by lightly-coloured areas. Transformed cells resemble owl’s eye. Name
the causative agent of pneumonia:
A. Cytomegalovirus*
B. Pneumococcus
C. Influenza virus
D. Candida fungi
E. Toxoplasma

EXP: Cytomegalovirus is synonymous with pneumonia in H.I.V with Owl’s eye cells.

125. The organisms to be identified have a nucleus surrounded by a nuclear membrane.


Genetic material is concentrated predominantly in the chromosomes that consist of DNA
strands and protein molecules. These cells divide mitotically. Identify these organisms:
A. Eukaryotes*
B. Bacteriophages
C. Prokaryotes
D. Viruses
E. Bacteria

EXP: Prokaryotes have their DNA scattered in the cell. The nucleus and mitosis indicate
eukaryotes

126. A 2-year-old boy is diagnosed with Down syndrome. What chromosomal changes may be
the cause of this disease?
A. Trisomy 21*
B. Trisomy 13
C. Trisomy X
D. Trisomy 18
E. Monosomy X

EXP: No comment :P

127. After a road accident a victim has tachycardia, arterial blood pressure 130/90 mm Hg,
tachypnoe, the skin is pale and dry, excitation of central nervous system is observed. What
shock stage is the patient most likely in?
A. Erectile*
B. Terminal
C. Torpid
D. Preshock (compensation stage)
E. Agony

EXP: Victim is in a stage of Erectile shock or agitation as the Vital signs are increased.

128. As a result of a mechanical injury an over 10 cm long portion of a peripheral nerve was
damaged. This caused an impairment of the upper limb activity. The patient was offered nerve
transplantation. What glial cells will participate in regeneration and provide the trophism of
the injured limb?
A. Schwann cells*
B. Fibrous cells
C. Protoplasmic cells
NSA-TSMU Academic Team Krok I 2015 Questions
KillingKrok2016 37

D. Microglia
E. Ependymal cells

EXP: Glial cells are support cells with different functions. Schwann cells provide help
with trophism and regeneration.

129. A 47-year-old man developed intestinal colic against the background of essential
hypertension. In this situation it would be most efficient to arrest the colic by administering
drugs of the following group:
A. Myotropic antispasmodics* B. Anticholinesterase agents
C. Sympathomimetics D. M-cholinomimetics
E. Adrenomimetics

EXP: Intestinal Colics are best treated using spasmolytics such as no-spa. Thus;
Myotropic Antispasmodics

130. Microscopy of a female patient’s swabs made from vaginal secretion revealed gram-
negative bean-shaped diplococci. What provisional diagnosis can be made?
A. Gonorrhoea* D. Mycoplasmosis
B. Syphilis E. Toxoplasmosis
C. Clamidiosis

EXP: Description above matches that of gram negative Neisseria Gonorrhoea.

131. A 54-year-old woman was brought to a casualty department after a car accident. A
traumatologist diagnosed her with multiple fractures of the lower extremities. What kind of
embolism is most likely to develop in this case?
A. Adipose* D. Gaseous
B. Tissue E. Air
C. Thromboembolism

EXP: In Multiple fractures, subcutaneous fat is dislodged into vessels leading to


obstruction of blood flow called Adipose Embolism.

132. A therapeutist has an appointment with a 40-year-old patient complaining of recurrent


pain attacks in his hallux joints and their swelling. Urine analysis revealed its marked acidity
and pink colour. What substances can cause such changes in the urine?
A. Uric acid salt* D. Calcium phosphate
B. Chlorides E. Magnesium sulfate
C. Ammonium salts

EXP: Acidic, pink fluid causing swelling of joints of fingers and toes is seen in Gout.
Identified as Uric Acid Salts

133. A 30-year-old man with diabetes mellitus type I was hospitalised. The patient is
comatose. Laboratory tests revealed hyperglycemia and ketonemia. What metabolic disorder
can be detected in this patient?
A. Metabolic acidosis* B. Metabolic alkalosis
NSA-TSMU Academic Team Krok I 2015 Questions
KillingKrok2016 38

C. Respiratory acidosis E. Normal acid-base balance


D. Respiratory alkalosis

EXP: Ketoacidosis is the most likely complication that will develop in Diabetes mellitus
manifested as Metabolic acidosis.

134. A 15-year-old patient has fasting plasma glucose level 4,8 mmol/l, one hour after
glucose challenge it becomes 9,0 mmol/l, in 2 hours it is 7,0 mmol/l, in 3 hours it is 4,8
mmol/l. Such parameters are characteristic of:
A. Subclinical diabetes mellitus* D. Healthy person
B. Diabetes mellitus type 1 E. Cushing’s disease
C. Diabetes mellitus type 2

EXP: The fasting glucose is normal before and after. However, in healthy persons, it
goes back to normal 2 hours after the glucose challenge not 3. This suboptimal glucose
control is typical for Subclinical Diabetes Mellitus.

135. A patient has undergone surgical removal of a cavitary liver lesion 2 cm in diameter. It
was revealed that the cavity wall was formed by dense fibrous connective tissue; the cavity
contained murky thick yellowish-green fluid with an unpleasant odor. Microscopically the fluid
consisted mainly of polymorphonuclear leukocytes. What pathological process are these
morphological changes typical for?
A. Chronic abscess* D. Empyema
B. Acute abscess E
C. Phlegmon

EXP: In Chronic Abscess of the Liver, yellowish fluid progresses to yellowish green fluid
and Odourless fluid becomes very unpleasant.

136. A patient consulted a physician about chest pain, cough, and fever. Roentgenography of
lungs revealed eosinophilic infltrates which were found to contain the larvae. What kind
of helminthiasis are these presentations typical for?
A. Ascariasis* D. Cysticercosis
B. Echinococcosis E. Trichinosis
C. Fascioliasis

EXP: Helminth with Larvae and Eosinophilic infiltrate is typical for Ascaris

137. During appendectomy a patient had the a. appendicularis ligated. This vessel branches
from the following artery:
A. A. Ileocolica* D. A. sigmoidea
B. A. colica dextra E. A. mesenterica inferior
C. A. colica media

EXP: Appendicular artery is a branch of Ileocolic artery. It descends posterior to the


terminal Ileum and supplies Appendix.

NSA-TSMU Academic Team Krok I 2015 Questions


KillingKrok2016 39

138. A 28-year-old patient undergoing treatment in a pulmonological department has been


diagnosed with pulmonary emphysema caused by splitting of alveolar septum by tissular
tripsin. The disease is caused by the congenital deficiency of the following protein:
A. α1-proteinase inhibitor* D. Haptoglobin
B. α2-macroglobulin E. Transferrin
C. Cryoglobulin

EXP: Trypsin is a Proteinase, an enzyme which helps to hydrolyse proteins. Disease


here is a congenital deficiency of Alpha 1 Proteinase inhibitor leading to no
suppression/inhibition of tissular trypsin.

139. A patient, who has been suffering for a long time from intestine dysbacteriosis, has
increased hemorrhaging caused by disruption of posttranslational modification of blood-
coagulation factors II, VII, IХ, and Х in the liver. What vitamin deficiency is the cause of this
condition?
A. К* D. С
B. _12 E. Р
C. _9
EXP: Vitamin K aids blood clotting and its deficiency leads to increased haemorrhage.

140. During a surgery for femoral hernia a surgeon operates within the boundaries of femoral
trigone. What structure makes up its upper margin?
A. Lig. Inguinale* D. Lig. pectinale
B. Arcus iliopectineus E. Fascia lata
C. Lig. lacunare

EXP: Femoral Triangle is formed with the inguinal ligament superiorly and the adductor
longus and Sartorius muscle forming the 2 sides.

141. An obstetrician-gynecologist measures pelvis size of a pregnant woman. A caliper was


used to measure the distance between the two iliac crests. What measurement of large pelvis
was made?
A. Distantia cristarum* D. Conjugata vera
B. Distantia throchanterica E. Conjugata anatomica
C. Distantia spinarum

EXP: Cristaric Distance/ Distance between iliac crests is the Ideal measurement for the
large pelvis in a pregnant woman.

142. A patient has arterial hypertension. What long-acting drug from the group of calcium
channel blockers should be prescribed?
A. Amlodipine* D. Atenolol
B. Octadine E. Reserpine
C. Pyrroxanum

EXP: Although they are all used to treat hypertension, Amlodipine is the only Ca+
channel blocker here; Atenolol is a beta blocker, Reserpine and Octadine are adrenergic
inhibitors.
NSA-TSMU Academic Team Krok I 2015 Questions
KillingKrok2016 40

143. A patient has been diagnosed with URTI. Blood serum contains immunoglobulin M. What
stage of infection is it?
A. Acute* D. Reconvalescence
B. Prodromal E. Carriage
C. Incubation

EXP: IgM is the 1st antibody released in infection. In blood serum this confirms acute
stage in Upper respiratory tract infection.

144. A 43-year-old patient suffers from acute pancreatitis with disrupted common bile duct
patency. What condition can develop in this case?
A. Mechanical jaundice* D. Hepatic coma
B. Hemolytic jaundice E. Portal hypertension
C. Hepatocellular jaundice

EXP: Common bile duct is oriented medially to Pancreatic duct. In acute pancreatitis,
edema of pancreatic duct leads to compression of Common bile duct and thus;
Mechanical Jaundice.

145. A patient has a tumor of the eye socket tissues behind the eyeball. Disruption of
accomodation and pupil constriction is observed. What anatomical structure is damaged?
A. Ganglion ciliare D. N. opticus
B. N. nasociliaris E. N. trochlearis
C. N. lacrimalis

EXP: Ciliary Ganglion receives parasympathetic fibers from the Edinger Westphal
nucleus of Oculomotor nerve which in turn supplies Sphincter pupillae and Ciliaris
muscles. Thus; disruption of accommodation and pupil constriction.

146. Lymphocytes and other cells of our body synthesize universal antiviral agents as a
response to viral invasion. Name these protein factors:
A. Interferon* D. Interleukin - 4
B. Interleukin - 2 E. Tumor necrosis factor
C. Cytokines

EXP: Lymphocytes synthesise Interferons as a response to Viral Infection.

147. A patient consulted a dentist about restricted mouth opening (trismus). Anamnesis
states a stab wound of the lower extremity. What infection may cause these symptoms?
A. Tetanus* D.Wound anaerobic infection
B. Brucellosis E. Tularemia
C. Whooping cough

EXP: Trismus is a symptom of progressing Tetanus Infection.

148. A patient has damaged spinal cord white matter in the middle area of the posterior
white column, disrupted proprioceptive sensibility of the lower limb joints and muscles. What
fibres are affected?
A. Fasciculus gracilis* B. Tr. spinothalamicus lateralis
NSA-TSMU Academic Team Krok I 2015 Questions
KillingKrok2016 41

C. Tr. spinocerebellaris anterior E. Fasciculus cuneatus=


D. Tr. spinocerebellaris posterior

EXP: Posterior white Column of spinal cord contains Dorsal column tract of spinal cord
responsible for Deep sensations, Propriception and Fine touch. Medial Affect is present
in Fasciculus Gracilis.

149. In an elderly person the change in heart force and vessels physical properties were
detected; they can be clearly observed on graphic recording of carotid pulse waves. What
method was applied?
A. Sphygmography* D. Myography
B. Plethysmography E. Phlebography
C. Rheography

EXP: Sphygmography is the use of a sphygmomanometer to measure force of heart


and blood pressure generally.

150. A patient has developed paroxysmal ventricular tachycardia against the background of
cardiac infarction. What antiarrhythmic drug should be chosen to avoid lowering cardiac
output?
A. Lidocaine hydrochloride* D. Propranolol
B. Procainamide E. Potassium chloride
C. Verapamil

EXP: Lidocaine hydrochloride is the antiarrhythmic drug capable of preserving cardiac


output.

151. Electrocardiogram of a young man reveals deviation of his electrical axis of heart to the
left. This phenomenon can be caused by:
A. Hypersthenic body type* D. Dilation of the right ventricle
B. Asthenic body type E. Dilation of the left atrium
C. Dilation of the right atrium

EXP: Hypersthenic/Excessively built body type has highest predisposition to ECG


deviation to the left.

152. A 6-year-old child suffers from delayed growth, disrupted ossification processes,
decalcification of the teeth. What can be the cause?
A. Vitamin D deficiency* D. Hyperthyroidism
B. Decreased glucagon production E. Vitamin C deficiency
C. Insulin deficiency
EXP: Vitamin D is responsible for Bone ossification, prevention of rickets in children.

153. A 30-year-old patient’s blood test revealed the following: erythrocyte count is 6 · 1012/l,
hemoglobin is 10,55ммоль/l. Vaquez’s disease was diagnosed. Name the leading part of
pathogenesis:
A. Neoplastic erythroid hyperplasia* D. Hypoxia
B. Iron-deficiency E. Acidosis
C. B12-deficiency

NSA-TSMU Academic Team Krok I 2015 Questions


KillingKrok2016 42

EXP: Vaquez’ disease is a chronic form of polycythemia usually characterised by Bone


marrow hyperplasia, RBC increase.

154. A 25-year-old patient complains of increasing pain in his leg muscles occurring during
walking and forcing him to make frequent stops. Objectively: skin of legs is pale, no hair-
covering, toenails are with trophic changes, no pulsation of pedal arteries. The most probable
cause of these changes is:
A. Ischemia* D. -
B. Venous hyperemia E. Embolism
C. Arterial hyperemia

EXP: These features are typical for hypoxia, oxygen deficiency leading to Tissue
Ischemia.

155. This year influenza epidemic is characterised by patients’ body temperature varying
from 36.9oC to 37,9oC. Such fever is called:
A. Subfebrile* D. Apyretic
B. High E. Moderate
C. Hyperpyretic

EXP: Subfebrile fever is characterised by temperature 36.9 – 37.9 oC

156. A woman complains of visual impairment. Examination revealed obesity in the patient
and her fasting plasma glucose level is hyperglycemic. What diabetes complication can cause
visual impairment/blindness?
A. Microangiopathy* D. Neuropathy
B. Macroangiopathy E. Glomerulopathy
C. Atherosclerosis

EXP: Vessels of the Retina are very small. Therefore, angiopathy of small vessels occur
called Microangiopathy.

157. Prolonged treatment of hypothyroidism has caused general dystrophy, dental caries,
tachycardia, tremor of extremities. What drug is the cause of these side effects?
A. L-thyroxin* D. Thyrocalcitonin
B. Humulin (Human insulin) E. Prednisolone
C. Parathyreoidinum

EXP: L-Thyroxin is used to treat hypothyroidism and it presents these side effects in
prolonged treatment.

158. A 3-year-old child has eaten some strawberries. Soon he developed a rash and itching.
What was found in the child’s leukogram?
A. Eosinophilia* D. Monocytosis
B. Hypolymphemia E. Lymphocytosis
C. Neutrophilic leukocytosis

EXP: Child has allergy to strawberries, in allergic reaction, Eosinophilia is seen in


Leukogram

NSA-TSMU Academic Team Krok I 2015 Questions


KillingKrok2016 43

159. A patient has been found to have a marked dilatation of the saphenous veins in the
region of anterior abdominal wall around the navel. This is symptomatic of pressure increase
in the following vessel:
A. V. portae hepatis*
B. V. cava superior
C. V. cava inferior
D. V. mesenterica inferior
E. V. mesenterica superior

EXP: Patient has symptoms of Portal Hypertension; hepatic portal vein is affected.

160. Cholesterol content in blood serum of a 12-year-old boy is 25 mmol/l. Anamnesis states
hereditary familial hypercholesterolemia caused by synthesis disruption of receptor-related
proteins for:
A. Low-density lipoproteins*
B. High-density lipoproteins
C. Chylomicrons
D. Very low-density lipoproteins
E. Middle-density lipoproteins

EXP: Low density lipoproteins take cholesterol from the Liver to target organs e.g
Adrenal gland for steroid hormone synthesis. In deficiency of LDL, hypercholesteremia
occurs.

161. Obesity is a common disease. The aim of its treatment is to lower content of neutral fats
in the body. What hormone sensitive enzyme is the most important for intracellular lipolysis?
A. Triacylglycerol lipase*
B. Protein kinase
C. Adenylate kinase
D. Diacylglycerol lipase
E. Monoacylglycerol lipase

EXP: T.A.G lipase helps to break down Triacylglycerol into free fatty acids for beta
oxidation.

162. A 40-year-old woman was diagnosed with glomerulonephritis based on her clinical
symptoms and the results of urine analysis. Anamnesis states chronic tonsillitis. What
microorganisms are the most likely cause for her kidney damage?
A. Streptococci*
B. Staphylococci
C. Escherichia
D. Mycoplasma
E. Meningococci

EXP: Streptococci have highest possibility to cause glomerulonephritis after previous


throat infection.

NSA-TSMU Academic Team Krok I 2015 Questions


KillingKrok2016 44

163. A man is suffering from diarrhea. In summer he spent his vacation in the south at the
sea coast. Bacteria with the following properties were detected in his feces: gram negative
curved mobile monotrichous bacilli that do not produce spores or capsules. They are
undemanding to nutrient medium but require alkaline reaction (рН 8,5-09,5). Described are
the agents of the following enteric infection:
A. Cholera*
B. Shigellosis
C. Typhoid fever
D. Colienteritis
E. Pseudotuberculosis

EXP: Vibrio is the one with curve. Chronic diarrhoea with such epidemiology and
clinical picture is typical for Cholera.

164. A patient is diagnosed with chronic atrophic gastritis attended by deficiency of Castle’s
intrinsic factor. What type of anemia does the patient have?
A. B12-deficiency anemia*
B. Iron refractory anemia
C. Hemolytic anemia
D. Iron-deficiency anemia
E. Protein-deficiency anemia

EXP: Intrinsic factor of stomach helps absorb Vitamin B-12. Its deficiency leads to B-12
deficiency anemia.

165. A pregnant woman was detected with IgM to rubella virus. An obstetrician/gynecologist
recommended therapeutic abortion due to the high risk of teratogenic affection of the fetus.
Detection of IgM was of great importance as it is these specific immunoglobulins that:
A. Indicate recent infection*
B. Penetrate placental barrier
C. Have the largest molecular weight
D. Are associated with anaphylactic reactions
E. Are the main factor of antiviral protection

EXP: Ig M indicates recent infection, Memory antibodies in Rubella.

166. A patient is diagnosed with cardiac infarction. Blood test for cardiospecific enzymes
activity was performed. Which of the enzymes has three isoforms?
A. Creatine kinase*
B. Lactate dehydrogenase
C. Aspartate transaminase
D. Alanine transaminase
E. Pyruvate kinase

EXP: Creatine Kinase has 3 Isoforms; CK1, CK2, CK3.

NSA-TSMU Academic Team Krok I 2015 Questions


KillingKrok2016 45

167. A 50-year-old man, who has been suffering from chronic hepatic failure for several years,
has developed ascites. What is the main mechanism of this disorder development?
A. Increased pressure in portal vein system*
B. Decrease of albumin and globulin synthesis in liver
C. Increased content of low-density and very low-density lipoproteins in blood
D. Neurotoxins appearing in blood
E. Increase of blood oncotic pressure

EXP: Ascites in Chronic hepatitis develops from venous congestion, increased pressure
in Portal vein system.

168. A 30-year-old man has sustained an injury to his thorax in a traffic incident, which
caused disruption of his external respiration. What type of ventilatory difficulty can be
observed in the given case?
A. Restrictive extrapulmonary ventilator impairment*
B. Restrictive pulmonary ventilatory impairment
C. Obstructive ventilatory impairment
D. Impaired ventilation regulation dysfunction
E. Cardiovascular collapse
EXP: It is restrictive extra pulmonary ventilator impairment because it concerns
external respiration (exhalation) and it doesn’t affect the lungs (extrapulmonary).

169. A 53-year-old man suffering from diabetes mellitus has developed a painful conical
induration, bluish-red with yellow center, on the skin of his neck. Such changes are
characteristic of:
A. Furuncle*
B. Abscess
C. Carbuncle
D. Phlegmon
E. Empyema

EXP: Symptom above is typical for a furuncle in Diabetes mellitus.

170. Biochemical analysis of an infant’s erythrocytes revealed evident glutathione peroxidase


deficiency and low concentration of reduced glutathione. What pathological condition can
develop in this infant?
A. Hemolytic anemia*
B. Pernicious anemia
C. Megaloblastic anemia
D. Sicklemia
E. Iron-deficiency anemia

EXP: Glutathione peroxidase deficiency results in build-up of glutathione which causes


hemolysis with oxidative distress. It is a tripeptide of Glycine, Cysteine and Glutamate.

NSA-TSMU Academic Team Krok I 2015 Questions


KillingKrok2016 46

171. In winter a 3-year-old child has sharp rise of body temperature up to 40 oC. Hemorrhagic
rash is observed on the skin and mucosa. Bean-shaped gramnegative microorganisms
situated in pairs are detected in the blood. What provisional diagnosis can be made?
A. Meningococcosis* D. Influenza
B. Gonorrhea E. Diphtheria
C. Scarlet fever

EXP: Paired bean shaped gram negative organism is typical for Neisseria Meningitidis
causing Meningococcosis.

172. Microelectrode analysis of nerve fiber biolectrical activity revealed, that its membrane
potential equals 90 mV. Its initial rest potential was 85 mV. What process occurs in this case?
A. Hyperpolarization* D. Overshoot
B. Depolarization E. Supernormality
C. Repolarization

EXP: An increase in polarisation of membranes past the resting potential is called


Hyperpolarisation.

173. Parkinson’s disease is caused by disruption of dopamine synthesis. What brain structure
synthesizes this neurotransmitter?
A. Substantia nigra*
B. Globus pallidus
C. Corpora quadrigemina
D. Red nucleus
E. Hypothalamus

EXP: Dopamine is primarily synthesised in Substantia Nigra of the brain.

174. Determining a patient’s blood group with monoclonal test-reagents revealed positive
agglutination reaction to anti-A and anti-B reagents, and negative reaction to anti-D. What
blood group does this patient have?
A. IV (АВ) Rh−*
B. II (А) Rh+
C. III (В) Rh−
D. IV (АВ) Rh+
E. I (0) Rh+

EXP: Group I(0) has no antigens so there is no agglutination with either anti-A [group
III(B)], or anti-B [group II(A)] antibodies.
Group II (A) with A antigens will only agglutinate with anti-A antibodies (in group
B blood)
Group III(B) with B antigens will only agglutinate with anti-B antibodies (in group
A blood)
Group AB has both A & B antigens, so it will react with both anti-A and anti-B
antibodies.
Anti-D reaction is positive for Rh + and negative for Rh-
NSA-TSMU Academic Team Krok I 2015 Questions
KillingKrok2016 47

175. A patient visited a dentist to extract a tooth. After the tooth had been extracted,
bleeding from the tooth socket continued for 15 minutes. Anamnesis states that the patient
suffers from active chronic hepatitis. What phenomenon can extend the time of
hemorrhage?
A. Decrease of fibrinogen content in blood*
B. Thrombocytopenia
C. Hypocalcemia
D. Increased activity of anticoagulation system
E. Decrease of albumine content in blood

EXP: Thrombocytes and calcium aid clotting so…no. Albumin has nothing to do with it.
The key is the chronic hepatitis. The liver which synthesizes proteins (including
fibrinogen in option A & the anticoagulating agents in option D) is damaged. This
causes decrease in its synthesis of proteins.

176. Name the halogen-containing antiseptic with fungicidal properties, which is used to treat
dermatomycosis:
A. Iodine solution*
B. Formalin solution
C. Methylene blue
D. Brilliant green
E. Boric acid solution

EXP: Iodine solution is the ONLY halogen compound here.

177. Due to severe pain syndrome a patient was prescribed a narcotic analgesic. Name this
drug:
A. Morphine*
B. Metamizole (Analgin)
C. Nimesulide
D. Dimethyl sulfoxide
E. Indometacin

EXP: Morphine is a Narcotic analgesic used for very severe pain.Metamizole,


Nimesulide and Indomethacin are NSAIDs. DiMethy Sulfoxide is also a non-narcotic
drug with anti-inflammatory, analgesic and anti-oxidant effects.

178. During pathomorphological kidney investigation of a patient, who for a long time had
been suffering from osteomyelitis and died from progressing renal failure, the following was
revealed: deposits of homogeneous eosinophilic masses in glomerular mesangium,
arterial and arteriolar walls, and stroma, which became red when stained with Congo red.
What pathological process is this?
A. Amyloidosis*
B. Mucoid swelling
C. Calcinosis
NSA-TSMU Academic Team Krok I 2015 Questions
KillingKrok2016 48

D. Carbohydrate degeneration
E. Hyalinosis

EXP: Description is typical for pathological protein called Amyloid seen in degenerative
processes. Condition is called Amyloidosis. Stained by Congo red.

179. A microslide presents a tissue with spherical cells, each of them containing a large fat
drop covered with thin cytoplasm layer in its center. Nucleus is compressed and situated at
the cell periphery. What tissue is it?
A.White adipose tissue *
B. Brown adipose tissue
C. Mucous tissue
D. Pigmented tissue
E. Reticular tissue

EXP: We know it is adipose, now let us differentiate.


In white adipose tissue, the cells accumulate fat in the cytoplasm, small "droplets" of
fat grow into a single large droplet. This pushes the remaining cytoplasm and the
nucleus to one corner of the cell. During preparation of H & E slides the fat is dissolved
and the cell appears empty, with a very thin rim of cytoplasm and the nucleus along
the edge. This appearance is described as "signet ring" appearance. (If you have a
historical or biblical movie you may have seen how the ring of a king carries his seal
which can be "impressed" on writing material by punching it!)

Brown fat cells, on the other hand, contain multiple droplets of fat which do not push
the nucleus to one side. The nucleus remains in the centre and the multiple droplets
give an appearance of tiny soap bubbles or sponge within the cell.

180. To treat rheumatoid arthritis a 65-year-old woman was prescribed an


immunosuppressive hormonal drug as a part of her complex therapy. Name this drug:
A. Prednisolone*
B. Thymus cytomedins (Thymalin)
C. Chloropyramine (Suprastin)
D. Riboflavin
E. Fercovenum

EXP: Glucocorticoids (prednisolone) have Immunosuppressive, anti-inflammatory; anti-


shock effects and are hormonal preparations.

181. An experiment was aimed at testing flexor reflex in a spinal frog, which was initiated by
simultaneous stimulation with isolated pre-threshold electrical impulses. The frequency of
those impulses was such that the reflex occurred. What process in the nerve centers can be
observed during this experiment?
A. Temporal summation* D. Postsynaptic summation
B. Spatial summation E. Threshold summation
C. Presynaptic summation

EXP: Single pre-threshold impulses cannot stimulate flow of action potentials but when
transmitted from multiple sources or at high frequencies, they add up to produce a
NSA-TSMU Academic Team Krok I 2015 Questions
KillingKrok2016 49

Threshold Impulse capable of producing action potentials in a phenomenon called


Summation.
There are 2 types of Summation, Spatial (with multiple sources) and Temporal (single
or isolated source, repeatedly at a high frequency). Note the words we have made
bold.

182. A patient diagnosed with acute dysentery has been treated for 3 days in an infectious
diseases hospital. On admission there were complaints of high temperature, stomach ache
and fluid excrements with mucus as often as 8-10 times a day. What sample should be taken
for analysis?
A. Feces* D. Liquor
B. Urine E. Blood
C. Bile
EXP: In acute dysentery, fecal analysis is the most reliable source of diagnosis.

183. An 18-year-old woman has body disproportion; wing-like folds on the skin of her neck,
underdeveloped ovaries, nuclei of her buccal epithelium cells have no Barr bodies.
Dermatoglyphics method revealed that her adt angle is 66 o. What provisional diagnosis can be
made in this case?
A. Turner’s syndrome* D. Patau’s syndrome
B. Cri du chat (cat cry) syndrome E. Edwards’ syndrome
C. Kleinfelter’s syndrome

EXP: In healthy females, there is usually 1 Barr body. This is due to underdevelopment of 1 of
the 2 XX chromosomes. In Turner’s syndrome the karyotype is 45, X0. Note the
underdevelopment of sex organs.

184. A 27-year-old patient with injury to the neck has lost approximately 30% of the
blood volume. The patient’s condition is severe: blood pressure is 60/40 mmHg, heart rate is
140/min., and respiratory rate is 30/min., conscious. Characterize the condition of the
patient’s circulatory system:
A. Hypovolemic shock* D. Coma
B. Cardiogenic shock E. Arterial hypertension
C. Collapse

EXP: Due to massive loss of blood, Blood volume and pressure have drastically
decreased causing Hypovolemic Shock.
Check out Question 106 with similar parameters and the answer there is Collapse. The
difference is no loss of blood and unconsciousness.

185. A soldier with explosion-caused trauma was delivered to a hospital. Examination


revealed his tympanic membrane to be intact. What defense reflex prevented the tympanic
membrane from rupturing?
A. Contraction of m. tensor tympani D. Relaxation of m. auricularis arterior
B. Relaxation of m. tensor tympani E. Relaxation of m. stapedins
C. Contraction of m. auricularis arterior

NSA-TSMU Academic Team Krok I 2015 Questions


KillingKrok2016 50

EXP: Tensor veli tympani muscle provides contraction defence reflex for tympanic
membrane.

186. A person with the fourth blood group (genotype IAIB) has in erythrocytes both antigen A
controlled by allele IA and antigen B controlled by allele IB. This phenomenon is an example of
the following gene interaction:
A. Codominance* D. Polymery
B. Complementarity E. Epistasis
C. Semidominance

EXP: An equal degree of dominance of both genes is seen here. That is; genes A and B
are Co-dominant.

187. During narcosis a patient developed a risk of cerebral edema. What drug should be
administered in this case?
A. Furosemide D. Triamterene
B. Dopamine E. Sodium bromide
C. Phenazepam

EXP: Diuretics are used to treat and prevent edema and Furosemide is a strong loop
diuretic.

188. During surgery performed in abdominal cavity a surgeon located ligament of liver
stretching from anterior abdominal wall (navel) to inferior surface of liver. What ligament is it?
A. Round ligament of the liver* D. Venous ligament of the liver
B. Falciform ligament of the liver E. Triangular ligament of the liver
C. Coronary ligament of the liver

EXP: Round ligament of the liver ‘ligament teres’ extends from the navel to the inferior
surface of the liver.

189. A 9-year-old boy has acute onset of disease: sore throat, body temperature rise up to
39,5oC; on the second day diffuse skin rash was detected all over his skin except for
nasolabial triangle. On examination of oral cavity: crimson tongue, "faming pharynx", and
necrotic tonsillitis. What diagnosis is the most likely?
A. Scarlet fever* D. Influenza
B. Measles E. Meningococcemia
C. Diphtheria

EXP: Flaming pharynx, crimson tongue are pathognomonic for Scarlet fever.
Remember that Scarlet fever is caused by Streptococci (note the sore throat and
fever).

190. A 49-year-old man complains of pain in his metatarsophalangeal joints and joint
deformation. In blood hyperuricemia can be observed. X-ray has revealed
metatarsophalangeal joint space narrowing, erosion, periarticular calcification of the both
joints, osteoporosis. Microscopy has revealed inflammatory granulomatous reaction
surrounding necrotizing masses in the area of the first metatarsophalangeal joint. Choose the
most likely diagnosis:
NSA-TSMU Academic Team Krok I 2015 Questions
KillingKrok2016 51

A. Gout (podagra)* C. Rheumatoid arthritis


B. Pyrophosphate arthropathy D. Hyperparathyroidism
E. Urolithiasis

EXP: Hyperuricemia is typical for Gout. Also, the granulomatous inflammation of the
1st Metatarso-Phalangeal joint is very specific for gout (the patients usually have
swollen feet).

191. Granulomas containing lymphocytes and macrophages were detected during analysis of
skin biopsy material. Among macrophages there are large cells with fat inclusions, which
contain microorganisms in spherical packages (Virchow’s cells). The following disease is
based on the described type of hypersensitivity:
A. Leprosy* D. Rhinoscleroma
B. Syphilis E. Epidemic typhus
C. Tuberculosis
EXP: Virchow cells are pathognomonic for Leprosy.

192. A patient with suspected necrosis of the upper abdominal cavity organs was delivered to
a surgical department. This condition is associated with acute circulatory disturbance of the
following vessel:
A. Truncus coeliacus* D. A. iliaca communis
B. A. mesenterica inferior E. A. renalis
C. A. mesenterica superior

EXP: The Common Iliac Artery and Renal Artery supply the lower limbs and kidney
respectively.
The superior mesenteric artery supplies the midgut, i.e., from the second part of the
duodenum to the left part of the transverse colon. The inferior mesenteric artery
supplies the hindgut, i.e., from the left part of the transverse colon to the rectum.
Coeliac trunk supplies upper abdominal cavity organs (liver, spleen, stomach, gall
bladder, pancreas). In its violation, Ischemia and Necrosis occur.

193. Name the drug that inhibits excretory function of pancreas during treatment of acute
pancreatitis:
A. Contrykal (Aprotinin)* D. Pancreatin (Mezym
B. Allochol forte)
C. Раnzynorm E. Festal

EXP: Contrycal inhibits excretory function of pancreas, used in acute pancreatitis.


Pancreatin (also called Panzynorm and Festal) is used in chronic pancreatitis.
Allochol is a cholagogue, used for chronic hepatitis, cholangitis and cholecystitis.

194. An 18-year-old patient has developed candidiasis after the case of pneumonia treated
with β-lactam antibiotic. What antimycotic agent should be prescribed?
A. Fluconazole* D. Phthalylsulfathiazole
B. Streptomycin E. Trimethoprim/sulfamethoxazole
C. Ampicillin (Biseptol)

NSA-TSMU Academic Team Krok I 2015 Questions


KillingKrok2016 52

EXP: Candidiasis usually occurs due to dysbacteriosis as Normal flora have been killed
by antibiotics leading to increased amount of opportunistic flora such as candida.
Fluconazole is the only antifungal drug here, used in its treatment. The other drugs are
antibiotics/anti-bacterial.

195. During autopsy of a 9-month-old girl’s body, who died due to severe pneumonia
complicated with sepsis, lack of thymus is observed. In the lymph nodes the lymphoid
follicles and cortical substance are absent; follicles of spleen are reduced in size with no light
zones and plasma cells. What is the cause of such structural changes?
A. Thymus agenesis* D. Thymus atrophy
B. Accidental involution of thymus E. Thymus aplasia
C. Thymus hypoplasia

EXP: Well, lack of thymus is observed and ‘agenesis’ means total congenital absence.

196. A patient with femoral neck fracture, who for a long time had to remain in bed in a
forced (supine) position, has developed dark-brown lesions along the backbone; soft
tissues are swollen, in the areas of maceration there is a foul-smelling liquid. Name the clinic-
pathologic type of necrosis:

A. Bedsore* D. Coagulation necrosis


B. Infarction E. Dry gangrene
C. Sequestrum

EXP: Remaining in bed for a prolonged period of time leads to development of Bed-
Sores due to reduced circulation to the affected area.

197. A woman poisoned with unknown substance was hospitalised in a toxicological


department. What group of drugs can be administered to decrease absorption and
introduction of the poison to her body?
A. Adsorbents* D. Organic nitrates
B. Neuroleptics E. Cholinesterase inhibitors
C. Antioxidants

EXP: Effects of Poisons are best neutralised, decreased using Adsorbents such as
activated charcoal.

198. A patient after disrupted cerebral circulation has developed paralysis. Choose the
anticholinesterase drug to be prescribed in this case:
A. Proserin* D. Methacin
B. Cordiamin E. Hexamethonium (Benzohexonium)
C. Aceclidine

EXP: Proserin is the Anticholinesterase drug among these.


Cordiamnin is a CNS stimulant
Aceclidine is a choline-mimetic
Methacin is an NSAID
Benzohexonium is a non-depolarizing ganglionic blocker

NSA-TSMU Academic Team Krok I 2015 Questions


KillingKrok2016 53

199. A 50-year-old woman is being treated for shingles in a neurology unit. What reactivated
virus causes this disease?
A. Varicella zoster virus (chickenpox virus) D. Measles virus
B. Herpes simplex virus type 1 E. Cytomegalovirus
C. Herpes simplex virus type 2

EXP: Varicella Zoster virus causes Shingles. It is interesting to note that Varicella and
Cytomegalovirus are in the Herpes family.

200. During examination of a patient a doctor should use anatomical division of anterior
abdominal wall into regions for more precise diagnostics. How many such regions can the
abdomen be divided into?
A. 9* D. 5
B. 8 E. 4
C. 6

NSA-TSMU Academic Team Krok I 2015 Questions


EXP: 3 upper, 3 middle, and 3 lower. In total 9 regions based on projection of internal
organs on abdominal wall.
Krok 1 Medicine (англомовний варiант, iноземнi студенти) 2016 рiк 1

ALL ANSWERS ARE –A—KROK 2016

1) A 35-year-old man has been deli-vered into a surgical ward with a suppurati-ng wound in the
neck, anterior to trachea (previsceral space). If a surgical operation is not performed urgently,
there is a risk of infection spreading to:
A. Thoracic cavity - anterior mediastinum
B. Thoracic cavity - middle mediastinum
C. Thoracic cavity - posterior mediastinum
D. Retrovisceral space
E. Interaponeurotic suprasternal space

Trachea (previsceral space), the pretracheal portion of the visceral space extends to the anterior
mediastinum at the level of the arch of the aorta and fibrous pericardium, infections in the
periveseral space can rapidly extend to anterior mediastinum.

2. Characteristic sign of glycogenosis is muscle pain during physical work. Blood examination
usually reveals hypoglycemia. This pathology is caused by congenital defi-ciency of the following
enzyme:
A. Glycogen phosphorylase
B. Glucose 6-phosphate dehydrogenase
C. α-amylase
D. γ-amylase
E. Lysosomal glycosidase

The disease is caused by a genetic defect in the phosphorylase (also known as


myophosphorylase) enzyme, which affects the breakdown of glocogen, the stored form of
glucose (sugar). Phosphorylase deficiency (McArdle disease)

3. Histologic specimen of a kidney demonstrates cells closely adjoined to the renal corpuscle in the
distal convoluted tubule. Their basement membrane is extremely thin and has no folds. These cells
sense the changes in sodium content of uri-ne and influence renin secretion occurring in
juxtaglomerular cells. Name these cells:
A. Macula densa cells
B. Juxtaglomerular cells
C. Mesangial cells
A. Podocytes
B. Glomerular capillary endothelial cells

The macula densa participates in the regulation of renin release from juxtaglomerular granular
cells. Renin secretion depends on NaCl delivery to and reabsorption by the macula-densa cells.

4. Bacteriological analysis of tap water has resulted in the following: total bacterial count in 1,0 ml of
water is 80, coli index is 3. What would be the conclusion?
A. The water is safe for consumption
B. The water quality is doubtful
C. The water quality is extremely doubtful
D. The water is polluted
E. The water is extremely polluted

Colibacillus index (coli index) — is amount of bacteria of colibacillus group in 1 litre of water, ti
(coli titre) — is the minimal volume of analyzed water in ml, in which one bacterium of
colibacillus group is detected. Coli index norm is 0-10
5. The process of metabolism in the human body produces active forms of oxygen,
including superoxide anion radical −2. This anion is inactivated by the following enzyme:

A. Superoxide dismutase
B. Catalase
C. Peroxidase
D. Glutathione peroxidase
E. Glutathione reductase
Superoxide is produced as a by-product of oxygen metabolism and, if not regulated, causes
many types of cell damage. Thus, Superoxide Dismutase is an important antioxidant defence in
nearly all living cells exposed to oxygen.

6. What kind of muscle contraction occurs in an upper limb during an attempt to lift a load beyond
one’s strength?
A. Isometric
B. Isotonic
C. Auxotonic
D. Phasic
E. Single
Lifting a load beyond one’s strength denote the exertion of more strength and more usage of
more muscle force whereby there will be a form of distributed force throughout the body.e.g
imagine yourself lifting weight at the gym.

7. A patient suffers from high fever, apnoea, pain in the thorax on the right. Pleurocentesis yielded
700 ml of yellow-green viscous liquid. Make the diagnosis:
A. Pleural empyema
B. Bronchial pneumonia
C. Serous pleurisy
D. Hemorrhagic pleurisy
E. Pleural carcinomatosis

Pluerocentesis is the puncturing of the pleural cavity with a fine aspirating needle to evacuate
fluid or liquid lodged in it. The fluid can be pus (empyema) or blood (hemothorax)

A patient suffers from disrupted patency of the airways at the level of small and medium-sized
bronchial tubes. What changes of acid-base balance can occur in the patient?

A. Respiratory acidosis
B. Respiratory alkalosis
C. Metabolic acidosis
D. Metabolic alkalosis
Acid-base balance remains unchanged
the level of small and medium-sized bronchial tubes is the point of gas exchange, Especially the
removal of CO2 from the body. If this process is disrupted there will be build-up of CO2 in the
body leading to hypercapnea and this is Respiratory ACIDOSIS
Upon toxic damage of hepatic cells resulting in disruption of liver function the patient
developed edemas. What changes of blood plasma are the main cause of edema development?
A. Decrease of albumin content
B. Increase of globulin conten
C. Decrease of fibrinogen conten
D. Increase of albumin conten
E. Decrease of globulin conten

The liver is involved in the synthesis of proteins especially albumin that takes part in
maintenance of oncotic pressure which helps in the movement of fluids within and across the
lymphatic vessels. Damage to the Liver leads to reduction in the production of albumin thereby
causes imbalance to the movement of fluid and leads to EDEMA
A 6-year-old child with suspected acti-ve tuberculous process has undergone di-agnostic
Mantoux test. What immunobi-ological preparation was injected?
Tuberculin
BCG vaccine
DTP vaccine
Tularinum
Td vaccine
Mantoux test is a test for immunity to tuberculosis using intradermal injection of tuberculin.

A 15-year-old boy has been diagnosed with acute viral hepatitis. What blood value should be
determined to confirm acute affection of hepatic cells?
A. Aminotransferase activity (AST, ALT)
B. Unconjugated and conjugated bilirubin content
C. Erythrocytes sedimentation rate (ESR)
D. Cholesterol content
E. Protein fraction content

Cytolysis of hepatocyte gives rise to the increase of ALT and AST

12. A 53-year-old man is diagnosed wit


Paget’s disease. Concentration of oxyproli-ne in daily urine is sharply increased, which primarily
means intensified disintegration of:

Collagen
Keratin
Albumin
Hemoglobin
Fibrinogen
Hydroxyproline is a metabolite present in the collagen synthesis pathway

When taking exams students often have dry mouth. The mechanism that causes this state
results from the following reflexes:
A. Conditioned sympathetic
B. Unconditioned parasympathetic
C. Conditioned parasympathetic
D. Unconditioned sympathetic
E. Unconditioned peripheral

The exam is a situation or a condition that triggers the Dry mouth (a sign of sympathetic
nervousness). This scenario goes inline with the Pavlov theory of conditioned sympathetic
reflex
A patient has hoarseness of voice. Duri-ng laryngoscopy a gray-white larynx tumor with
papillary surface has been detected. Microscopic investigation has shown the following: growth of
connective tissue covered with multilayer, strongly kerati-nized pavement epithelium, no cellular
atypia. What is the most likely diagnosis?
Papilloma
Fibroma
Polyp
Angioma
Angiofibroma
The keypoint here is that the tumourhas PAPILLARY SURFACE. The gra-white colour
makes it differ from polyp which is always red
During autopsy approximately 2,0 liters of pus have been found in the abdominal cavity of the
body. Peritoneum is dull and of grayish shade, serous tunic of intestines has grayish-colored coating
that is easily removable. Specify the most likely type of peritonitis in the patient:
A. Fibrinopurulent peritonitis
B. Hemorrhagic peritonitis
C. Serous peritonitis
D. Tuberculous peritonitis
-

From the question there was 2,0 litres of PUS that indicates the presence of Purulent Exudate.
Autopsy of a body revealed bone marrow hyperplasia of tubular and flat bones (pyoid marrow),
splenomegaly (6 kg) and hepatomegaly (5 kg), enlargement of all lymph node groups. What
disease are the identified changes typical of?
A. Chronic myelogenous leukemia
B. Chronic lymphocytic leukemia
C. Multiple myeloma
D. Polycythemia vera
E. Hodgkin’s disease

It’s chronic because of the signs that are present which are splenomegaly and hepatomegaly.
Its myelogenous leukaemia instead of lymphocytic because there was bone marrow hyperplasia
with more tubular and flat bones which determines the origin of the disease. If it were to be
lymphatic leukaemia, there won’t be tubular or flat bones hyperplasia but increase in matured
lymphocyte alongside with enlargement of all lymph node
A bacteriological laboratory has been investigating a sample of homemade dri-ed fish that was
the cause of severe
food poisoning. Microscopy of the culture inoculated in Kitt-Tarozzi medium revealed
microorganisms resembling a tennis racket. What diagnosis can be made?
Botulism
Salmonellosis
Cholera
Dysentery
Typhoid fever
Kitt-tarozzi medium is used for anaerobic microorganism e.g. botulinum , clostridum. Teniss
racket is unique for botulism.
An infant has been diagnosed with mi-crocephaly. Doctors suspect that this brain disorder
developed due to the fact that the mother had been taking actinomycin D during her pregnancy.
What germinal layers have been affected by this teratogen?
Ectoderm
Entoderm
Mesoderm
Entoderm and mesoderm
All germinal layers
Microcephally -Microcephaly is a condition where the head (circumference) is smaller than
normal. Actinomycin acts on the Ectoderm
A patient demonstrates sharp decrease of pulmonary surfactant activity. This condi-tion can
result in:
A. Alveolar tendency to recede B. Decreased airways resistance
C. Decresed work of expiratory muscles D. Increased pulmonary ventilation
Hyperoxemia

Pulmonary surfactant is a mixture of lipids and proteins which is secreted into the alveolar
space by epithelial type II cells. The main function of surfactant is to lower the surface tension
at the air/liquid interface within the alveoli of the lung

A patient is diagnosed with diabetic coma. Blood sugar is 18,44 mmol/l. What glucose-
regulating drug should be prescri-bed in the given case?
A. Rapid-acting insulin
Intermediate-acting insulin
Long-acting insulin
Biguanide
Sulfonylurea derivative
With this level of glucose in blood ( norm is 3.3 – 5.5 mmol/L) insulin is the only hormone that
can normalize the condition

Initial inoculation of water in 1% peptone water resulted in growth of a thin film on the
medium surface in 6 hours. Such cultural properties are characteristic of causative agent of the
following disesase:
Cholera
Plague
Tuberculosis
Dysentery
Pseudotuberculosis

Inoculation In 1% peptone is unique for Cholera


An infant born prematurely 2 days ago presents with yellow coloring of skin and mucosa. Such
a condition in the infant is caused by temporary deficiency of the following enzymeA. UDP-
glucuronyl transferase
B. Aminolevulinate synthase
C. Heme oxygenase
D. Heme synthetase
E. Biliverdine reductase

Infant or new-born jaundice is caused by deficiency of UDP-glucoronyl transferase an


important enzyme necessary for the conjugation of bilirubin in the liver.

It has been determined that one of a pesticide components is sodium arsenate that blocks lipoic
acid. Enzyme activity can be impaired by this pesticide. Name this enzyme:

A. Pyruvate dehydrogenase complex B. Microsomal oxidation


C. Methemoglobin reductase
D. Glutathione peroxidase
E. Glutathione reductase
Lipoic acid is part of the metabolites present in the Pyruvate dehydrogenase complex needed
for the transition between glycolysis and kreb’s cycle. Especially in the conversation of
Pyruvate into Acetyl CoA.

A 50-year-old woman diagnosed with cardiac infarction has been delivered into an intensive
care ward. What enzyme will be the most active during the first two days?
A. Aspartate aminotransferase
B. Alanine aminotransferase
C. Alanine aminopeptidase
LDH4
LDH5
AST- Aspartate aminotransferase, Creatinine Kinase , Troponin , LDH1 and LDH2 are the enzymes that is
released in the heart whenever there is damage to the myocardial muscle.

Stool culture test of a 6-month-old bottlefed baby revealed a strain of intesti-nal rod-shaped
bacteria of antigen structure 0-111. What diagnosis can be made?
Colienteritis
Gastroenteritis
Choleriform disease
Food poisoning
Dysentery-like disease
This bacteria antigenic structure ==> 0-111 is unique for E.coli

Parents of a sick 5-year-old girl vi-sited a genetic consultation. Karyotype investigation revealed
46 chromosomes. One chromosome of the 15th pair was abnormally long, having a part of the
chromosome belonging to the 21st pair attached to it. What mutation occurred in this girl?
Translocation
Deletion
Inversion
Deficiency
Duplication
From the highlighted parts above, it’s obvious that the mutation that occurred was –
translocation
A patient consulted a doctor with complaints of dyspnea occurring after physical exertion.
Physical examination revealed anemia, paraprotein was detected among gamma globulins. What
value should be determined in the patient’s urine to confirm the diagnosis of myeloma?
A. Bence Jones protein
Bilirubin
Hemoglobin
Ceruloplasmin
Antitrypsin

The Bence-Jones protein urine test is used mainly to diagnose and monitor multiple myeloma,
a type of cancer. An abnormal Bence-Jones test result is also linked with malignant
lymphomas. These are cancers of the lymphatic system. Multiple myeloma is a blood cancer of
the plasma cells.

A patient complaining of dizziness, thi-rst, difficult swallowing, and impaired visi-on of close
objects has addressed a doctor. Objectively: respiratory rate is increased, pupils are dilated,
general agitation, talkati-veness, though the speech is indistinct. BP is 110/70 mm Hg, heart rate is
110/min. Given symptoms can indicate overdosage of the following drug:
Atropine
Morphine
Ephedrine
Aminazine
Caffeine
Atropine is an anticholinergic agent. The above signs and symptoms are signs of atropine
overdose

A dry-cleaner’s worker has been found to have hepatic steatosis. This pathology can be caused
by disruption of synthesis of the following substance:
Phosphatidylcholine
Tristearin
Urea
Phosphatidic acid
Cholic acid
Fatty liver is a reversible condition wherein large vacuoles of triglyceride fat accumulate
in liver cells via the process ofsteatosis (i.e., abnormal retention of lipids within a cell).

A 35-year-old man with peptic ulcer di-sease has undergone antrectomy. After the surgery
secretion of the following gastro-intestinal hormone will be disrupted the most:

Gastrin
Histamine
Secretin
Cholecystokinin
Neurotensin

Antrectomy is the removal of the antrum; This leads to the reduction of the production of the
gastric Hormones like Gastrin and also HCL by the parietal cells .

A 16-year-old adolescent is di-agnosed with hereditary UDP (uridine di-phosphate)


glucuronyltransferase defici-ency. Laboratory tests revealed hyperbi-lirubinemia caused mostly by
increased blood content of the following substance:
A. Unconjugated bilirubin B. Conjugated bilirubin
Urobilinogen
Stercobilinogen
Biliverdine
UDP (uridine di-phosphate) glucuronyltransferase is needed in the process of bilirubin
Conjugation.
A 60-year-old patient with a long hi-story of atherosclerosis and a previous myocardial
infarction developed an attack of retrosternal pain. 3 days later the patient was hospitalized and then
died of progressi-ve cardiovascular insufficiency. During autopsy a white fibrous depressed area
about 3 cm in diameter with clear margi-

ns was found within the area of posterior wall of the left ventricle and interventricular septum. The
dissector considered these changes to be:
A. Focal cardiosclerosis
B. Myocardial ischemia
C. Myocardial infarction
Myocarditis
Myocardial degeneration
Cardiosclerosis is mostly white fibrous with a clear margin

A patient with hypertension has developed headache, tinnitus, vomiting, hi-gh BP up to


220/160 mm Hg. On exami-nation: facial asymmetry on the right, voli-tional mobility is absent,
increased tendon reflexes and muscle tone of extremities on the right. What motor disorder of
nervous system occurred in this case?
Hemiplegia
Paraplegia
Tetraplegia
Hyperkinesis
Monoplegia
Hemiplegia- paresis or paralysis of one side of the body

A 7-year-old child in the state of allergic shock caused by a bee sting has been delivered into an
emergency ward. High concentration of histamine was observed in blood. Production of this
amine was the result of the following reaction:
Decarboxylation
Hydroxylation
Dehydrogenation
Deaminization
Reduction

Decarboxylation is the biochemical process In which CO2 is removed from amino acids e.g
Histidine = HISTAMINE + CO2
A 26-year-old woman consulted a doctor about having stool with white flat moving organisms
resembling noodles. Laboratory analysis revealed proglottids with the following characteristics:
long, narrow, wi-th a longitudinal channel of the uterus with 17-35 lateral branches on each
side. What kind of intestinal parasite was found?
A. Taeniarhynchus saginatus
B. Taenia solium
C. Hymenolepis nana
D. Diphyllobothrium latum
E. Echinococcus granulosus

This feature of the proglotid is unique to SAGINATUS _-- long, narrow, wi-th a longitudinal
channel of the uterus with 17-35 lateral branches on each side
A man is 28 years old. Histological investigation of the cervical lymph node revealed a change
of its pattern due to proli-feration of epithelioid, lymphoid cells and macrophages with horseshoe-
shaped nuclei. In the center of some cell clusters there were non-structured light-pink areas with
fragments of nuclei. What disease are these changes typical of?
Tuberculosis
Hodgkin’s disease
Actinomycosis
Tumor metastasis
Syphilis

This feature is unique to Tuberculosis - horseshoe-shaped nuclei.

Sex chromatin was detected during examination of a man’s buccal epitheli-um. It is


characteristic of the following chromosome disease:
A. Klinefelter’s syndrome
B. Down’s disease
C. Turner’s syndrome
D. Triple X syndrome
E. Hypophosphatemic rickets

Whenever sex chromatin is discovered in a man’s buccal epitheli-um it can only mean
KLINEFELTER’S SYNDROME FROM THE EARTH TO THE MOON

A patient, having suffered a thermal burn, developed painful boils filled with turbid liquid in
the skin. What morphologi-cal type of inflammation has developed in the patient?

Serous
Proliferative
Croupous
Granulomatous
Diphtheritic
THERMAL BURN FROM YOUR PERSONAL KNOWLEDGE THAT’S NOT INFECTION
SO IT CAN’T BE ANY OF THE OTHER OPTIONS BUT THERE WILL BE FORMATION
OF BULLA WHICH WILL CONTAIN SEROUS FLUID.

Autopsy of a man with tuberculosis revealed a 3x2 cm large cavity in the superi-or lobe of the
right lung. The cavity was interconnected with a bronchus, its wall was dense and consisted of three
layers: the internal layer was pyogenic, the middle layer was made of tuberculous granulation tissue
and the external one was made of connective tissue. What is the most likely diagnosis?
A. Fibrous cavernous tuberculosis B. Fibrous focal tuberculosis
Tuberculoma
Acute focal tuberculosis
Acute cavernous tuberculosis
FROM THE UNDERLINED PARTS ABOVE, THE TUBERCULOSIS CANNOT BE OTHER
OPTIONS. IT CAN’T BE ACUTE CAVERNOUS BECAUSE TUBERCULOSIS IS A
CHRONIC PROCESS. THE CONNCECTIVE TISSUE STREAMLINED THE ANSWER
NOT TO BE TUBERCULOMA
A 7-year-old child has acute onset of di-sease: temperature rise up to 38oC, rhinitis, cough,
lacrimation, and large-spot rash on the skin. Pharyngeal mucosa is edematous, hyperemic, with
whitish spots in the buccal area. What kind of inflammation caused the changes in the buccal
mucosa?
A. Catarrhal inflammation
B. Suppurative inflammation
C. Fibrinous inflammation
D. Hemorrhagic inflammation
E. Serous inflammation

RHINITIS IS OFTEN ASSOCITATED TO CATARRHAL INFLAMATION. DON’T


FORGET THIS AS A MEDICAL STUDENT.
After a traffic accident a 36-year-old patient has developed muscle paralysis of the extremitis on
the right, lost pain and thermal sensitivity on the left, and partially lost tactile sensitivity on both
sides. What part of the brain is the most likely to be

damaged?
Right-hand side of the spinal cord
Motor cortex on the left
Left-hand side of the spinal cord
Anterior horn of the spinal cord
Posterior horn of the spinal cord
N.B THE BRAIN SENSITIVITY IS CONTRALATERAL I.E THE RIGHT PART OF THE
BRAIN CONTROLS THE LEFT SIDE OF THE BODY AND VICE-VERSA. OTHER SIGNS
PRESENT ARE SIGNS OF LOCAL IMPACT
A 4-year-old child has been admitted to an orthopaedic department with displaced shin fracture.
Bone fragments reposition requires analgesia. What drug should be chosen?

Promedol
Analgin
Morphine hydrochloride
Panadol
-

Promedol – IS MOST PREFFERED IN THIS LIST FOR SUCH CONDITION


MORPHINE IS OFTEN USED IN CASES LIKE BONE CANCER (LUKAEMIA).
ANALGIN AND PANADOL AR OFTEN USED IN MILD PAIN E.G FOR HEADACHE

While examining foot blood supply a doctor checks the pulsation of a large artery running in the
separate fibrous channel in front of articulatio talocruralis between the tendons of long extensor
muscles of hallux and toes. What artery is it?
A. A. dorsalis pedis
B. A. tibialis anterior
C. A. tarsea medialis
D. A. tarsea lateralis
E. A. fibularis
CHECK THIS OUT ON GOOGLE --- articulatio talocruralis TO HAVE THE PICTURE IN
YOUR MIND

Representatives of a certain human population can be characterized by elongated body, height


variability, decreased volume of muscle mass, increased length of limbs, decreased size and volume
of rib cage, increased perspiration, decreased indi-ces of base metabolism and fat synthesis. What
type of adaptive evolution is it?
Tropical
Arctic
Moderate
Intermediate
Mountain

THE UNDERLINED INFO IS UNIQUE TO THE PEOPLE IN THE TROPICALS


A 59-year-old woman has been hospiali-zed in a surgical ward due to exacerbation of chronic
osteomyelitis of the left shin. Blood
test: leukocytes - 15, 0 · 109/l. Leukogram: myelocytes - 0%, metamyelocytes - 8%, stab
neutrophils - 28%, segmented neutrophils - 32%, lymphocytes - 29%, monocytes - 3%. Such
blood count would be called:
A. Regenerative left shift
B. Right shift
C. Hyperregenerative left shift
D. Degenerative left shift
Regenerative-degenerative left shift
IT’S REGENERATIVE LEFT SHIFT BECAUSE THE SEGMENTED NEUTROPHIL
(MATURED) OUTNUMBERS THE IMMATURE ONES (STAB NEUTROPHILS)
ALTHOUGH THERE IS INCREASE IN THE AMOUNT OF GENERAL LEUCOCYTE.

A 41-year-old man has a hi-story of recurrent attacks of heartbeats (paroxysms), profuse


sweating, headaches. Examination revealed hypertension, hyperglycemia, increased basal
metabolic rate, and tachycardia. These clinical presentations are typical of the following adrenal
pathology:

A. Hyperfunction of the medulla B. Hypofunction of the medulla


C. Hyperfunction of the adrenal cortex
D. Hypofunction of the adrenal cortex
E. Primary aldosteronism
THE UNDERLINED SYMPTOMS DENOTES THE ACTIVATION OF SYMPATHETIC
NERVOUS SYSTEM. CATHECOLAMINES (ADRENALINE AND NOREADRENALINE)
HAS BEEN RELEASED IN HIGH AMOUNT INTO THE BLOOD. THIS CAN ONLY BE
POSSIBLE WHEN THERE IS ADDISON’S DISEASE FROM THE OPTIONS AVAILABLE.
HYPERFUNTION

During autopsy of a man, who di-ed of acute transmural cardiac infarction, the following has
been detected on the pericardium surface: fibrous whitish-brown deposit connecting parietal and
visceral pericardial layers. What kind of inflammati-on occurred in the pericardium?
Croupous
Diphtheritic
Serous
Suppurative
Granulomatous
CROUPOUS INFLAMMATION. A HOMOGENEOUS LAYER OF EXUDATE LYING
CLOSE TO BUT DETACHED FROM THE UNDERLYING INFLAMED TISSUE, WHICH
IS COMPARATIVELY UNHARMED; MAY FORM A FIBRINOUS CAST.
A 12-year-old child developed nephri-tic syndrome (proteinuria, hematuria, cyli-ndruria) 2
weeks after a case of tonsilli-tis, which is a sign of affected glomerular basement membrane in the
kidneys. What mechanism is the most likely to cause the basement membrane damage?
A. Immune complex
Granulomatous
Antibody-mediated
Reaginic
Cytotoxic
THIS OCCURS MAJORLY IN CASE OF STREPTOCOCAL INFECTION OF THE
TONSILS. THE BODY PRODUCES ANTIBODIES AGAINST THE STREP. ANTIGEN,
THIS ANTIGEN ALSO HAVE STRUCTURES THAT ARE SIMILAR TO THE PROTEINS
PRESENT ON THE BASMEMENT MEMBRANE OF KIDNEY. AUTOIMMUNITY
OCCURS AT THE BASMEMENT MEMBRANE
A man arrived into a traumatologi-cal department with a trauma of the right shoulder.
Examination revealed a displaced humeral shaft fracture on the right in the middle one-third of the
humerus; the pati-ent cannot extend the fingers of his right hand. What nerve is damaged?
Radial
Ulnar
Median
Musculocutaneous
Axillary
THE RADIAL NERVE PASSES THROUGH THE MIDDLE ONE-THIRD OF THE
HUMERUS ANATOMICALLY.

Work in a mine is known to cause inhalation of large amounts of coal dust. Inhaled coal dust
can be detected in the following pulmonary cells:
A. Alveolar macrophages
B. Respiratory epithelial cells
C. Secretory epithelial cells
D. Capillary endothelial cells
E. Pericapillary cells
ALVEOLAR MACROPHAGES ARE THE PRIMARY PHAGOCYTES OF THE INNATE
IMMUNE SYSTEM, CLEARING THE AIR SPACES OF INFECTIOUS, TOXIC, OR
ALLERGIC PARTICLES THAT HAVE EVADED THE MECHANICAL DEFENSES OF
THE RESPIRATORY TRACT, SUCH AS THE NASAL PASSAGES, THE GLOTTIS, AND
THE MUCOCILIARY TRANSPORT SYSTEM

What drug will be the most appropriate for a patient suffering from chronic gastritis with
increased secretion?
Pirenzepine
Pancreatine
Pepsin
Aprotinin
Chlorphentermine

PIRENZEPINE (GASTROZEPIN), AN M1 SELECTIVE ANTAGONIST, IS USED IN THE


TREATMENT OF PEPTIC ULCERS AND GASTRITIS, AS IT REDUCES GASTRIC ACID
SECRETION AND REDUCES MUSCLE SPASM.

A 63-year-old man, who has been suffering from chronic fibrous-cavernous pulmonary
tuberculosis for 24 years, has been delivered to a nephrology department with uremia. Intravital
diagnostic test for amyloid in the kidneys was positive. What amyloidosis is it in this case?
A. Secondary systemic
B. Primary systemic
C. Localized (focal)
D. Hereditary (genetic)
Senile

THE DISORDER OF THE KIDNEY IS AS A RESULT OF THE PULMONARY


TUBERCULOSIS. THE ABOVE FORMATION OF AMYLOID IN THE KIDNEY IN A
TUBERCULOTIC PATIENT CAN BE DIAGNOSED AS KIDNEY TUBERCULOSIS

Cells of a healthy liver actively synthesi-ze glycogen and proteins. What organelles are the
most developed in them?
A. Granular and agranular endoplasmic reticulum
B. Cell center
Lysosomes
Mitochondria
Peroxisomes
THE ENDOPLASMIC RETICULUM IS OF TWO TYPES – 1) THE AGRANULAR THAT
SYNTHESIZES GLYCOGEN 2) GRANULAR TYPE THAT SYNTHESIZE PROTEIN
Immune-enzyme assay has detected HBs antigen in blood serum. What disease is it
characteristic of?
A. Viral hepatitis type B
B. Viral hepatitis type A
AIDS
Tuberculosis
Syphilis
FROM THE UNDERLINED HBs THE ALPHABET “B” DENOTES THE SPECIFIC
TYPE OF VIRAL HEPATITIS.

A patient has been diagnosed with gonorrhea. As fluoroquinolones are the drugs of choce for
treatment of gonorrhea the patient should be prescribed:
Ciprofloxacin
Furazolidone
Fluorouracil
Sulfacarbamide (Urosulfanum)
Cefazolin
HINT - FLUOROQUINOLONES ARE ANTIBIOTICS THAT ENDS WITH –FLOXACIN
E.G CIPROFLOXACIN, MEVOFLOXACIN, LEVOFLOXACIN, OFLOXACIN
Autopsy of a Middle-Eastern woman, who had been suffering from wasting fever for a long
time, revealed enlarged blackened liver and spleen. Bone marrow was hyperplastic and black-
colored as well. Cerebral cortex was smoky grey. What di-sease is it characteristic of?

Malaria
AIDS
Epidemic typhus
Sepsis
Hepatitis
SMOKEY GREY CEREBERAL CORTEX IS UNIQUE FOR MALARIA

Human red blood cells contain no mi-tochondria. What is the main pathway for ATP
production in these cells?
A. Anaerobic glycolysis
B. Aerobic glycolysis
C. Oxidative phosphorylation
D. Creatine kinase reaction
E. Cyclase reaction
AS IT IS KNOWN THAT MITOCHONDRIA IS MEANT FOR ATP PRODUCTION IN AN
AEROBIC CONDITION. IN ITS ABSENCE IN RBC THAT’S A CLUE THAT THE
PROCESS WILL BE ANAEROBIC
Atria of an experimental animal were superdistended with blood, which resulted
in decreased reabsorption of N a+ and water in renal tubules. This can be explained by the influence
of the following factor on ki-dneys:

A. Natriuretic hormone
Aldosterone
Renin
Angiotensin
Vasopressin
THE ATRIAL NATRIURETIC HORMONE (ANP) IS A CARDIAC HORMONE WHICH GENE AND
RECEPTORS ARE WIDELY PRESENT IN THE BODY. ITS MAIN FUNCTION IS TO LOWER
BLOOD PRESSURE AND TO CONTROL ELECTROLYTE HOMEOSTASIS
A woman gave birth to a stillborn baby with numerous malformations. What protozoan disease
could cause intrauterine death?
Toxoplasmosis
Leishmaniasis
Malaria
Amebiasis
Lambliasis
TOXOPLASMOSIS IS CAUSED BY TOXOPLASMA GONDII WHICH IS ONE OF THE
MAJOR “TORCH” INFECTIONS THAT CAN CAUSE STILL BIRTH BECAUSE THEY
CAN CROSS THROUGH THE PLACENTA.

A 42-year-old patient complains of pain in the epigastral area, vomiting; vomit masses have the
color of coffee-grounds; the patient suffers from melena. Anamnesis records gastric ulcer disease.
Blood formula:
erythrocytes - 2, 8 · 1012/l, leukocytes - 8 · 109/l, Hb- 90 g/l. What complication is it?

Hemorrhage
Penetration
Perforation
Canceration
Pyloric stenosis
THE COFFEE-GROUND COLOR IS UNIQUE FOR BLOOD IN VOMIT ,
ERYTHROCYTOPENIA I.E LOW AMOUNT OF RBC IS A GREAT CLUE OF
HAEMORAHHAGE

A patient has been hospitalised with provisional diagnosis of virus B hepatitis. Serological
reaction based on complementation of antigen with antibody chemically bound to peroxidase or
alkaline phosphatase was used for disease diagnosti-cs. What is the name of the applied serologi-
cal react
Immune-enzyme analysis
Radioimmunoassay
Immunofluorescence test
Bordet-Gengou test
Antigen-binding assay
PEROXIDASE OR ALKALINE PHOSPHATASE ARE ENZYMES THIS IS A FORM OF
HINT TO THE ANSWER.

A patient with insulin-dependent di-abetes mellitus has been administered insulin. After a
certain period of time the patient developed fatigue, irritabili-ty, excessive sweating. What is the
main mechanism of such presentations developi-ng?
A. Carbohydrate starvation of the brain
B. Increased glycogenolysis
C. Increased ketogenesis
D. Increased lipogenesis
E. Decreased glyconeogenesis
FATIGUE, IRRITABILI-TY ARE SYMPTOMS OF BRAIN STARVATION OF GLUCOSE

Examination of a 52-year-old woman has revealed a decrease in the amount of red blood cells
and an increase in free hemoglobin in the blood plasma (hemoglobinemia). Color index is 0,85.
What type of anemia is being observed in the patient?

A. Acquired hemolytic
B. Hereditary hemolytic
C. Acute hemorrhagic
D. Chronic hemorrhagic
E. Anemia due to diminished erythropoiesis
N.B THE COLOR INDEX IS WITHIN NORMAL RANGE OF 0.85 -1.15
Poisoning caused by mercury (II) chlori-de (corrosive sublimate) occurred in the result of
safety rules violation. In 2 days the patient’s diurnal diuresis was 620 ml. The patient developed
headache, vomiting, convulsions, dyspnea; moist crackles were observed in the lungs. Name
this pathology:
A. Acute renal failure
B. Chronic renal failure
C. Uremic coma
Glomerulonephritis
Pyelonephritis

THE CONDITION IS ACUTE BECAUSE OF THE POISOINING. THE MAIN


CRITERIA THAT WAS ALARMIC IS THE REDUCTION IN DIURESIS FROM THE
DAILY 1900-2000ML PER DAY TO 620ML. THIS SHOWS THAT THERE IS DAMAGE
TO THE URINE FORMATION PROCESS.

For people adapted to high external temperatures profuse sweating is not accompanied by loss
of large volumes of sodium chloride. This is caused by the effect the following hormone has on
perspiratory glands:
Aldosterone
Vasopressin
Cortisol
Tgyroxin
Natriuretic
THE ALDOSTERONE HORMONE IS A HORMONE PRODUCED BY THE ADRENAL
GLAND. THE HORMONE ACTS MAINLY IN THE FUNCTIONAL UNIT OF THE
KIDNEYS TO AID IN THE CONSERVATION OF SODIUM, SECRETION OF
POTASSIUM, WATER RETENTION AND TO STABILIZE BLOOD PRESSURE.

The processes of heat transfer in a naked person at room temperature have been studied. It was
revealed that under these conditions the greatest amount of heat is transferred by:

A. Heat radiation
B. Heat conduction
Convection
Evaporation
-
RADIATION IS A METHOD OF HEAT TRANSFER THAT DOES NOT RELY UPON ANY CONTACT
BETWEEN THE HEAT SOURCE AND THE HEATED OBJECT AS IS THE CASE WITH
CONDUCTION AND CONVECTION. HEAT CAN BE TRANSMITTED THROUGH EMPTY SPACE
BY THERMAL RADIATIONOFTEN CALLED INFRARED RADIATION.

Due to destruction of certain structures of the brainstem an animal has lost its ori-entation
reflexes in response to strong light stimuli. What structures were destroyed?
A. Anterior quadrigeminal bodies
B. Posterior quadrigeminal bodies
C. Red nuclei
D. Vestibular nuclei
E. Substantia nigra
THE ANTERIOR QUADRIGEMINAL BODIES CONTOLS LIGHT STIMULI

Urine analysis has shown high levels of protein and erythrocytes in urine. This can be caused
by the following:
A. Renal filter permeability B. Effective filter pressure
C. Hydrostatic blood pressure in glomerular capillaries
D. Hydrostatic primary urine pressure in capsule
E. Oncotic pressure of blood plasma
THIS CAN ONLY BE POSSIBLE IF THERE IS HIGH PERMEABILITY OF THE RENAL
FILTER. PROTEIN ESPECIALY ARE NOT MEANT TO BE PRESENT IN URINE. IT’S A
KEY CLUE TO DESTRUCTION OF THE BASMENT MEMBRANE E.G NEPHROTIC
SYNDROME

Along with normal hemoglobin types there can be pathological ones in the organi-sm of an
adult. Name one of them:
HbS
HbF
HbA1
HbA2
HbO2
THE HbS IS UNIQUE IN PEOPLE WITH SICKLE CELL ANAEMIA

Development of both immune and allergic reactions is based upon the same mechanisms of
immune system response to an antigen. What is the main difference between immune and allergic
reactions?
A. Development of tissue lesion
B. Amount of released antigen
C. Antigen structure
D. Routes by which antigens are delivered into the body
E. Hereditary predisposition
IN ALLERGIC REACTION THERE IS FORMATION A SKIN LESSION LIKE- HIVES
THIS IS ABSENT IN IMMUNE REACTION.

Histologic preparation stained wi-th orcein demonstrates from 40 to 60 fenestrated elastic


membranes within the middle coat of vessel. Name this vessel:
A. Elastic artery
B. Muscular artery
C. Mixed type artery
D. Muscular vein
E. Nonmuscular vein
ORCEIN IS A HISTOLOGICAL DYE USED MAINLY TO IDENTIFY ELSATIN OR
ELASTIC FIBRES. THEREFORE ORECEIN = ELASTIC FIBRES

Angiocardiography of a 60-year-old man revealed constriction of a vessel located inThe left


coronary sulcus of the heart. Name this pathological vessel:
Ramus circumflexus
Ramus interventricularis posterior
A. coronaria dextra
V.cordis parva
Ramus interventricularis anterior
-CHECK GOOGLE OR YOUR ANATOMY ATLAS FOR THE PICTURE OF THE
RAMUS CIRCUMFLEXUS.

A comatose patient was taken to the hospital. He has a history of diabetes melli-tus.
Objectively: Kussmaul breathing, low blood pressure, acetone odor of breath. After the
emergency treatment the pati-ent’s condition improved. What drug had been administered?
Insulin
Adrenaline
Isadrinum
Glibenclamide
Furosemide
FROM THE UNDERLINED INFO ABOVE, THE PATIENT IS IN COMA. THE ONLY …
I REPEAT THE ONLY DRUG TO ADMINISTER IS INSULIN
A patient complains of pain in the right lateral abdomen. Palpation revealed a dense, immobile,
tumor-like formation. The tumor is likely to be found in the following part of the digestive tube:
A. Colon ascendens
B. Colon transversum
C. Colon descendens
D. Colon sigmoideum
Caecum
CONSULT YOUR ANATOMY ATLAS ABOUT COLON ASCENDENS FOR MIND
REFRESHMENT!

A patient hospitalized due to mercury intoxication presents with the following processes in the
kidneys: focal necrotic changes of tubules of major renal regions, edema, leukocyte infiltration
and hemorrhages in the interstitial tissue, venous congestion. What condition developed in the
patient?
A. Acute necrotic nephrosis
B. Acute glomerulonephritis
C. Chronic renal failure
D. Acute pyelonephritis
E. Chronic pyelonephritis
THE MECURY INTOXICATION MAKES THE SITUATION AAACUUUUUTEE! WHICH
AUTOMATICALLY CAUSE FOCAL NECROSIS! DON’T FORGET WHENVER THE
KIDNEY IS EXPOSED TO HEAVY METAL THERE WILL BE NECROSIS AND
THEREFORE THERE WILL BE HAEMORAHGE, LOW DIURESIS AND ABOVE ALL
RENAL FAILUREEEE!

According to phenotypic diagnosis a female patient has been provisionally di-agnosed with X-
chromosome polysomia. This diagnosis can be confirmed by cytogenetic method. What karyotype
will confirm the diagnosis?
47(ХХХ)
48(XXXY)
48(XXYY)
47(XXY)
46(XX)
IT HAS BEEN NOTED IN THE QUESTION THAT THE PATIENT IS A FEMALE AND
SHE HAS X-CHROMOSOME POLYSOMIA I.E MORE THAN NORMAL
CHROMOSOMES. THEREFORE ALL YOU WILL SEE WILL BE PLENTY “X”
ALONE WITH NO “Y” AT ALL!

An uconscious young man in the state of morphine intoxication has been delivered into an
admission room. The patient’s respiration is slow and shallow due to suppressi-on of the
respiratory center. What kind of respiratory failure occurred in this case?
A. Ventilatory disregulation
B. Ventilatory obstruction
C. Ventilatory restriction
Perfusion
Diffusion

THIS IS UNIQUE SIDE-EFFECT OF MORPHINE. NOTE OF ALL THE OPTION


VENTILATORY DISREGULATION IS THE ONLY OPTION WITHOUT AN
ORGANIC/ INFECTIOUS CAUSE.
On histological examination of uterine mucosa the following is detected: sinuous glands,
serratiform and corkscrew-shaped elongated growths of stroma with cell proli-feration. Make
the diagnosis:
A. Glandular endometrial hyperplasia B. Acute endometritis
Leiomyoma
Vesicular mole
Placental polyp
sinuous glands, serratiform and corkscrew-shaped elongated growths of stroma with cell
proli-feration – THIS IS UNIQUE TO GLANDULAR ENDOMETRIAL HYPERPLASIA

10 minutes after the beginning of heavy physical work a person demonstrates increase of
erythrocyte number in blood from 4, 0 · 1012/l to 4, 5 · 1012/l. What is the cause of this
phenomenon?
A. Erythrocytes exit from depot
B. Suppression of erythrocyte destruction C. Erythropoiesis activation
D. Increase of cardiac output E. Water loss
THE INCREASE IN RBC IS AS A RESULT OF THE PHYSCAL EXERCISE.
THE EXIT IS FROM THE SPLEEN WHICH IS A NATURAL DEPOT OF
RBC. THEY ARE RELEASED WHEN THE BODY NEEDS MORE OXYGEN
DUE TO ITS HIGH DEMAND DURING PHYSICAL ACTIVITIES

80. A patient has a traumatic injury of sternocleidomastoid muscle. This has resulted in a decrease
of the following value:
A. Inspiratory reserve volume
B. Expiratory reserve volume
C. Respiratory volume
D. Residual volume
E. Functional residual lung capacity
STERNOCLEIDOMASTOID MUSCLE IS PART OF MUSCLES USED IN FORCED
INSPIRATION.******* Inspiratory reserve volume = THE MAXIMAL AMOUNT OF
ADDITIONAL AIR THAT CAN BE DRAWN INTO THE LUNGS BY DETERMINED
EFFORT/ FORCE AFTER NORMAL INSPIRATION.

Autopsy of a 40-year-old woman, who died of cerebral hemorrhage duri-ng hypertensic crisis,
revealed: upper-body obesity, hypertrichosis, hirsutism, stretchmarks on the skin of thighs and
abdomen. Pituitary basophil adenoma is detected in the anterior lobe. What di-agnosis is the
most likely?
A. Cushing’s disease
B. Essential hypertension
C. Alimentary obesity
D. Simmonds’ disease
E. Hypothalamic obesity
PITUITARY BASOPHIL ADENOMA MADE THE CONDITION WITH THE PRESENTED
SYMPTOMS UNDERLIED UINQUE FOR CUSHING’S DISEASE

A specimen shows an organ covered with the connective tissue capsule with radiating
trabeculae. There is also cortex containing lymph nodules, and medullary cords made of
lymphoid cells. What organ is under study?
A. Lymph node
Thymus
Spleen
Red bone marrow
Tonsils
TRABECULAE AND MEDULLARY CORDS, DIFFERENTIATE LYMPH NODE FROM
OTHER LYMPHATIC STRUCTURES IN THE OPTIONS
After a craniocerebral injury a patient has lost the ability to recognize shapes of objects by touch
(stereognosis). What area of cerebral cortex normally contains the relevant center?

A. Superior parietal lobule


B. Inferior parietal lobule
C. Supramarginal gyrus
D. Postcentral gyrus
E. Angular gyrus
STEREGNOSIS IS THE MENTAL PERCEPTION OF DEPTH OR THREE-
DIMENSIONALITY BY THE SENSES, USUALLY IN REFERENCE TO THE ABILITY TO
PERCEIVE THE FORM OF SOLID OBJECTS BY TOUCH. THE SUPERIOR PAREIETAL
LOBULE IS THE AREA WHERE STEREGNOSIS IS SENSED.

Monoamine oxidase inhibitors are widely used as psychopharmacological drugs. They change
the level of nearly all neurotransmitters in synapses, with the following neurotransmitter being the
exception:
Acetylcholine
Noradrenaline
Adrenaline
Dopamine
Serotonin
MONOAMINE OXIDASE INHIBITORS ACTS MAJORLY ON MONOAMINES LIKE
NOREADRENALINE, ADRENALINE, DOPAMINE, SEROTONIN BUT NOT ON
ACETYLCHOLINE. ALL OTHER OPTIONS ARE MADE FROM THE AMINE
CALLED TYROSINE WHILE ACETYCHOLINE IS MADE FROM CHOLINE A
PHOSPHOLIPID.

A worker of an agricultural enterprise had been suffering from an acute disease with aggravating
intoxication signs, which resulted in his death. On autopsy: the spleen is enlarged, flaccid, dark
cherry-red in the section, yields excessive pulp scrape. Soft meninges of fornix and base of the brain
are edematous and saturated with blood (”cardinal’s cap”). Microscopically: serous-hemorrhagic
inflammation of meninges and cerebral tissues. Make the diagnosis:
Anthrax
Tularemia
Plague
Cholera
Brucellosis
cardinal’s cap IS UNIQUE IN THE CASE OF ANTHRAX .

Autopsy of an 8-month-old boy, who di-ed of severe pneumonia complicated with sepsis,
revealed absence of thymus. Lymph nodes have no lymphoid follicles and corti-cal substance. In
the spleen the follicles are decreased in size and have no light centers. What is the cause of such
changes?
A. Thymus agenesis
B. Thymus aplasia
C. Thymus atrophy
D. Thymus hypoplasia
E. Accidental thymic involution
FROM THE UNDERLINED POINTS - ABSENCE OF THYMUS SIMPLY MEANS
ABSENCE OF THYMUS – EVERY OTHER HIGHLIGHTED POINTS ARE JUST
ADDITIONAL POINTS TO ESTABLISH THE FACT OF LYMPHOCYTES
(SPECIFICALLY THE T-LYMPHOCYTE). *** A-GE-NE-SIS SIMPLY MEANS
SOMETHING DIDN’T GENERATE OR GROW OR DEVELOP OR IS ABSENT
A patient has been delivered into a surgical ward with an incised wound of the anterior surface
of the shoulder in its lower One-third. Flexing function was disrupted in the shoulder and elbow
joints, which is caused by the damage to the:
A. Biceps muscle of the arm
B. Triceps muscle of the arm
C. Anconeus muscle
D. Deltoid muscle
E. Coracobrachial muscle
OF AL THE OPTIONS LISTED BICEPS IS THE ONLY ONE THAT FITS IN THIS
QUESTION ON THE BASIS OF FLEXING. JUST TO ADD- TRICEPS = EXTENSION,
ACONEUS = EXTENSION, DELTOID= ABDUCTION , CORACOBRACHIAL =
EXTENSION

A 40-year-old patient suffers from bronchial asthma and prolonged tachycardia. Choose the
optimal drug for rapid relief of bronchial spasm in the given case:
Salbutamol
Adrenalin hydrochloride
Ephedrine hydrochloride
Orciprenaline
Isoprenaline (Isadrinum)
SALBUTAMOL IS A SELECTIVE BETA AGONIST ( BETA 2 TO BE SPECIFIC) ITS
THE ONLY DRUG YOU CAN PRESCRIBE IN A PATIENT WITH TACHYCARDIA
BCOS OTHERS CAN WORSEN THE STATUS OF THE TACHYCARDIA BCOS THEY
ARE NON-SELECTIVE BETA AGONIST

A patient with urolithiasis has developed severe pain attacks. For pain shock prevention he was
administered an antispasmodic narcotic analgesic along with atropine. Name this drug:
Promedol
Nalorphine
Tramadol
Ethylmorphine hydrochloride
Morphine hydrochloride
TRAMADOL IS NOT A NARCOTIC ANALGESIC, OTHERS ARE NARCOTIC
ANALGESICS BUT PROMEDOL IS AN ANTISPASMODIC NARCOTIC ANALGESIC

A patient suffers from acute cardi-opulmonary failure with pulmonary edema. What diuretic
should be prescribed in the given case?
Furosemide
Triamterene
Spironolactone
Hydrochlorothiazide (Dichlothiazidum)
Acetazolamide (Diacarb)
FRUSEMIDE OR LASIX IS THE BEST DRUG TO USE IN THE CASE OF CARDIAC
RELATED PULMONARY EDEMA. NEVER FORGET AS A DOCTOR !!!

A patient with acute myocardial infarcti-on has been administered heparin as a part of complex
therapy. Some time after heparin injection the patient developed hematuria. What heparin antagonist
should be injected to remove the complication?
A. Protamine sulfate
Vicasol
Aminocaproic acid
Neodicumarin
Fibrinogen
N.B FROM THE EARTH TO THE MOON = THE ANTIDOTE OF HEPARIN IS
PROTAMINEEEEE SULLLLFATE!!!
The key reaction of fatty acid synthesis is production of malonyl-CoA. What metabolite is the
source of malonyl-CoA synthesis?
Acetyl-CoA
Succinyl-CoA
Acyl-CoA
Malonate
Citrate
MALONYL-COA IS FORMED BY CARBOXYLATING ACETYL-COA USING THE
ENZYME ACETYL-COA CARBOXYLASE. ONE MOLECULE OF ACETYL-COA
JOINS WITH A MOLECULE OF BICARBONATE,[1] REQUIRING ENERGY
RENDERED FROM ATP.

A family of healthy students, who have arrived from Africa, gave birth to a chi-ld with signs of
anemia. The child has di-ed shortly after. Examination has revealed that the child’s erythrocytes are
abnormally crescent-shaped. The disease is characreri-zed by autosomal recessive inheritance.
Determine the genotype of the child’s parents:
A. Аа х Аа
B. Аа х аа
C. АА х АА
D. аа х аа
E. Аа х АА
CRESENT-SHAPED RBC IS TALKING ABOUT SICKLE-CELL ANAEMIA. FOLLOWING
OUR MENDELIAN CROSSING PATTERN WITH THE KNOWLEDGE THAT THE
PARENT ARE BOTH HEALTHY AND THE STUDENT HIM/HERSELF. THE HEALTHY
PARENT CAN ONLY BE HOMEGENIOUSLY(I.E ONE GOOD GENE AND BAD ONE) A
CARIER OF THE GENE AS SEEN IN THE ANSWER.

At a certain stage of cell cycle chromosomes reach cellular poles, undergo despiralization;
nuclear membranes are being formed around them; nucleolus is restored. What stage of
mitosis is it?
Telophase
Prophase
Prometaphase
Metaphase
Anaphase
LOOK INTO THIS ACRONYM ---IPMAT
I-INTERPHASE -RESTING AND SYNTHESIS STAGE
P- PROPHASE – WHEN THE DNA DISNTERATE AND THE NUCLEAR MEMBRANE
GET DESROYED
M- METAPHASE – WHEN THE SISTER CHROMATID LIES AT THE CENTRE
A- WHEN THE DAUGHER CHROMATIDS ARE NOW MOVING TO THE POLES
WITHOUTH FORMATION OF NEW NUCLEAR MEMBRANE
T- TELOPHASE – CHROMOSOMES ARE NOW AT THE CELLULAR POLES,
UNDERGO DESPIRALIZATION; NUCLEAR MEMBRANES ARE BEING FORMED
AROUND THEM; NUCLEOLUS IS RESTORED

Cardiac arrest occurred in a patient during a surgery of the small intestine. What regulatory
mechamisms resulted in the cardiac arrest in this case?
A. Unconditioned parasympathetic reflexes
Unconditioned sympathetic reflexes
Conditioned parasympathetic reflexes
Conditioned sympathetic reflexes
Metasympathetic reflexes
THE VAGUS NERVE INNERVATES THE ABDOMINAL ORGANS. IRRITATION OF
THE ABDOMINAL VAGUS NERVE LEADS TO PARASYMPATHETIC REFLEX TO
THE CARIDIO-PULMONARY CENTRE IN THE MEDULA OBLONGATA THAT LED
TO THE CARDIAC ARREST. IN THIS CASE IT’S AN UNCONDTIONED SITUATION
AND NOT CONDITIONED BECAUSE THE BODY HASN’T GONE THROUGH ANY
TRIANING ABOUT THIS BEFORE. THIS IS JUST THE SIMPLE CONCEPT
Vestibular receptors of semicircular canals of an animal have been destroyed. What reflexes
will disappear as a result?
Statokinetic reflex during movements with angular acceleration
Statokinetic reflex during movements with linear acceleration
Head-righting reflex
Body-righting reflex
Primary orienting reflex
THE SEMICIRCULAR CANALS OR SEMICIRCULAR DUCTS ARE THREE SEMICIRCULAR,
INTERCONNECTED TUBES LOCATED IN THE INNERMOST PART OF EACH EAR,
THE INNER EAR. THE THREE CANALS ARE THE HORIZONTAL, SUPERIOR AND
POSTERIOR SEMICIRCULAR CANALS. AND THEY DETERMINE ANGULAR
ORIENTATIONS.

A patient working at a pig farm complai-ns of paroxysmal abdominal pain, liquid feces with
mucus and blood, headache, weakness, fever. Examination of large intestine revealed ulcers from 1
mm up to several cm in diameter, feces contained oval unicellular organisms with cilia. What di-
sease can be suspected?
Balantidiasis
Amebiasis
Toxoplasmosis
Lambliasis
Trichomoniasis
FECES CONTAINING OVAL UNICELLULAR ORGANISMS WITH CILIA – THIS IS
UIQUE FOR BALANTIDIASIS- CHECK GOOGLE FOR THE PICTURE TO MAKE IT
STICK BETTER

Blood group of a 30-year-old man has been determined before a surgery. The blood was Rhesus-
positive. Agglutination did not occur with standard 0 (I), А (II), and В (III) serums. The blood
belongs to the following group:
A. 0 (I)
B. А (II)
C. В (III)
D. АВ (IV)
AGGLUTINATION NOT OCCURNG WITH ANY OF THE BLOOD GROUOS LISTED
AUTOMATICALLY MADE THE BLOOD BEEN TESTED TO HAVE A SURFACE OF
ANTIGENS, AND ANTIBODIES PRESENT IN THE SERUMS. WHICH MAKES THE
ANSWER “A”

Histological specimen of a hemopoietic organ shows clusters of node- and band-shaped


lymphocytes that along with stroma elements compose cortical and medullar substances. Name this
organ:
A. Lymph node
Spleen
Red bone marrow
Thymus
Palatine tonsil
THIS WAS WELL EXPLAINED IN ONE OF THE QUESTIONS ABOVE- THE MAIN
CLUE HERE IS THE MEDULLARY SUBSTANCE. IT MADE IT DIFFERENT FROM
OTHER HAEMATOPOEITC ORGANS

It is known that in catecholamine metabolism a special role belongs to


monoamine oxidase (MAO). This enzyme inactivates mediators (noadrenalin, adrenalin,
dopamine) by:
A. Oxidative deamination
B. Adjoining amino groups
C. Removing methyl groups
Carboxylation
Hydrolysis
FROM THE NAME OF THE ENZYME – MONOAMINE OXIDASE – THEY
DEFINITELY UNDERGO OXIDATIVE DEAMINATION.
Cellular composition of exudate largely depends on the etiological factor of inflammation. What
leukocytes are the first to be involved in the focus of inflammation caused by pyogenic
bacteria?
A. Neutrophil granulocytes
Monocytes
Myelocytes
Eosinophilic granulocytes
Basophils
THIS IS A MEDICAL FACT – THAT NEUTROPHIL GRANULOCYTES RESPONDS
OFTEN TO BACTERIA INFECTIONS. TO ADD – LYMPHOCYTES RESPONDS TO
VIRAL INFECTIONS AND FUNGAL. MONOCYTES WILL INCREASE INCASE OF
INFECTIOUS MONONUCLEOSIS, EOSINOPHILS = HELMINTHS , MYELOCYTES
WILL INCREASE IN HE CASE OF ACUTE MYELOCYTIC LUKAEMIA

A surgeon has detected inflammation of the Meckel’s diverticulum in a patient. During surgical
invasion it can be located in the:
Ileum
Jejunum
Colon
Duodenum
Sigmoid colon
MECKEL'S DIVERTICULUM IS A SMALL ABNORMALITY IN THE SMALL
INTESTINE THAT IS PRESENT AT BIRTH.SPECIFICALLY IN THE ILLEUM

A patient complains of acute pain attacks in the right lumbar region. Duri-ng examination the
nephrolithic obturation of the right ureter in the region between its abdominal and pelvic segments
was detected. What anatomical boundary exi-sts between those two segments?
A. Linea terminalis
B. Linea semilunaris
C. Linea arcuata
D. Linea transversa
E. Linea inguinalis
THE LINEA TERMINALIS OR INNOMINATE LINE CONSISTS OF THE PECTINEAL LINE
(PECTEN PUBIS), THE ARCUATE LINE, THE PUBIC CREST, THE SACRAL ALA, AND
THE SACRAL PROMONTORY.
IT IS THE PELVIC BRIM, WHICH IS THE EDGE OF THE PELVIC INLET. THE PELVIC
INLET IS TYPICALLY USED TO DIVIDE THE ABDOMINOPELVIC CAVITY INTO AN
ABDOMINAL (ABOVE THE INLET) AND A PELVIC CAVITY (BELOW THE INLET).
SOMETIMES, THE PELVIS CAVITY IS CONSIDERED TO EXTEND ABOVE THE PELVIC
INLET, AND IN THIS CASE THE PELVIC INLET IS USED TO DIVIDE THE PELVIC
CAVITY INTO A FALSE (ABOVE THE INLET) AND A TRUE PELVIS (BELOW THE
INLET)

A patient died of cancerous cachexia with primary localization of cancer in the stomach.
Autopsy revealed acutely enlarged liver with uneven surface and numerous protruding nodes;
the nodes had clear margins in the section, rounded shape, gray-pink color, varying density, someti-
mes contained necrotic foci. Histologically: there are atypical cells in the nodes. What pathologic
process occurred in the liver?
A. Cancer metastases
Abscesses
Regeneratory nodes
Infarction
Hepatic cancer
THE PRIMARY LOCALIZATION OF THE CANCER WAS IN THE STOMACH BUT
DEFINTELY THROUGH HEMATOGENOUS WAY (HEPATIC PORTAL VEIN) OF
METASTASES THE CANCER CELLS FOUND ITSELF IN THE LIVER.
A patient has insufficient blood supply to the kidneys, which caused the development of pressor
effect due to the constriction of arterial resistance vessels. This is the result of the vessels being
greately affected by the following substance:

A. Angiotensin II
Angiotensinogen
Renin
Catecholamines
Norepinephrine
THE INSUFFICIENT BLOOD SUPPLY LEADS TO THE SENSITIVITY OF THE
MACULA DENSA WHICH ACTIATES THE PRODODUCTION OF RENIN. THIS
INTURN ACTIVATES THE RENIN- ANGIOTENSIN SYSTEM WHICH END PRODUCT
IS ANGIOSTENSIN II WHICH CASUSE CONSTRICTION OF THE ARTERY

106. Experimental stimulation of sympathetic nerve branches that innervate heart caused an increase
in force of heart contractions because membrane of typical cardiomyocytes permitted an increase
in:
A. Calcium ion entry
B. Calcium ion exit
C. Potassium ion exit
D. Potassium ion entry
E. Calcium and potassium ion exit
INFLUX OF CALCIUM LEADS TO INCREASE IN CONTRACITILITY OF HEART
MUSCLE

Parents of a 5-year-old child report him to have frequent colds that develop into pneumonias,
presence of purulent rashes on the skin. Laboratory tests have revealed the following: absence of
immunoglobuli-ns of any type; naked cells are absent from the lymph nodes punctate. What kind of
immune disorder is it?
X-linked hypogammaglobulinemia (Bruton type agammaglobulinemia)
Autosomal recessive agammaglobuli-naemia (Swiss type)
Hypoplastic anemia
Agranulocytosis
Louis-Barr syndrome
THE UNIQUENESS OF THIS DISORDER IS THE ABSENCE OF ANYTPE OF
IMMUNOGLOBULIN WHICH ARE FORMED FROM B-LYMPHOCYTES

A microslide contains the specimen of a gland composed of several secretory saccule-shaped


parts that open in the common excretory duct. What gland is it?
A. Simple branched alveolar gland
B. Compound branched alveolar gland C. Simple unbranched alveolar gland
D. Compound unbranched alveolar gland E. Simple branched tubular gland
FIRST OF ALL – IT HAS SEVRAL SECRETORY SACULE- SHAPPED = THIS MADE
IT BRANCHED ALVEOLAR; SECONDLY – IT OPENED INTO COMMON
EXCRETORY DUCT = MAKES IT SIMPLE BRANCHED ALVEOLAR GLAND

Microelectrode technique allowed to register a potential following ”all-or-none” law and capable
of undecremental spreadi-ng. Specify this potential:
A. Action potential
B. Excitatory postsynaptic potential C. Rest potential
D. Inhibitory postsynaptic potential E. Receptor potential
ACTION POTENTIAL - THE CHANGE IN ELECTRICAL POTENTIAL
ASSOCIATED WITH THE PASSAGE OF AN IMPULSE ALONG THE
MEMBRANE OF A MUSCLE CELL OR NERVE CELL.
Examination of a patient revealed hypertrophy and inflammation of lymphoid tissue, edema of
mucous membrane between palatine arches (acute tonsillitis). What tonsil is normally situated in
this area?
A. Tonsilla palatina
B. Tonsilla pharyngealis
C. Tonsilla tubaria
D. Tonsilla lingualis
-
THE EXAMINER ALREADY GAVE YOU A HINT ABOUT THE LOCATION –
BETWEEN THE PALATINE ARCHES. THEREFORE THE ANSWER IS TONSILLA
PALATINA

Histological specimen of an ovary demonstrates a spherical structure composed of large


glandular cells contai-ning lutein. What hormone is produced by the cells of this structure?
Progesterone
Estrogens
Testosterone
Corticosterone
Aldosterone
LUTEIN FROM THE WORD LEUTUS MEANS YELLOW. THE GLANDULAR CELLS
WON’T TURN TO YELLOW UNTIL AFTER OVULATION. IMMEDAITELY AFTER
OVULATION THE LARGE GLANDULAR CELLS WLL START PRODUCING
PROGESTRONE.
N.B PRIOR TO OVULATION THEY PRODUCE OESTROGEN

A patient, who has been subsisting exclusively on polished rice, has developed polyneuritis due
to thiamine deficiency. What substance is an indicator of such avi-taminosis, when it is excreted
with urine?
A. Pyruvic acid
Malate
Methylmalonic acid
Uric acid
Phenyl pyruvate

THIAMINE IS NEEDED AS A COFACTOR TPP (THIAMINE PYROPHOSPHATE) FOR


PYRUVATE DEHYDROGENASE COMPLEX. THIS COMPLEX CONVERTS PYRUVATE
INTO ACETYL -COA

When blood circulation in the damaged tissue is restored, lactate accumulation stops and glucose
consumption decelerates. These metabolic changes are caused by acti-vation of the following process:
A. Aerobic glycolysis
B. Anaerobic glycolysis
Lipolysis
Gluconeogenesis
Glycogen biosynthesis
RESTORATION OF BLOOD CRICULATION DENOTES SUPPLY OF OXYGEN IN THE
SITUATION WHERE OXYGEN WAS ONCE ABSENT WHICH LED TO THE
ACCUMULATION OF LACTATE(ANAEROBIC GLYCOLYSIS ). THEREFORE , AEROBIC
GLYCOLYSIS CAN COMENCE SINCE THE OXYGEN HAS BEEN RESTORED

A 67-year-old patient complains of periodic heartache, dyspnea during light physical activities.
ECG reveals extraordi-nary contractions of heart ventricles. Such arrhythmia is called:
Extrasystole
Bradycardia
Tachycardia
Flutter
Fibrillation
EXTRAORDINARY CONTRACTIONS IS THE CLUE TO THIS ANSWER
In investigation of serum proteins vari-ous physical and physicochemical methods can be used. In
particular, serum albumi-ns and globulins can be separated by the method of:
Electrophoresis
Polarography
Dialysis
Spectrography
Refractometry
ELECTROPHORESIS IS A TECHNIQUE USED IN LABORATORIES IN ORDER TO
SEPARATE MACROMOLECULES BASED ON SIZE

Pupil dilation occurs when a person steps from a light room into a dark one. What reflex causes
such a reaction?
A. Sympathetic unconditioned reflex
Sympathetic conditioned reflex
Metasympathetic reflex
Parasympathetic unconditioned reflex
Parasympathetic conditioned reflex
FROM A LIGHT ROOM – THE PUPIL WILL CONSTRICS ( MIOPIC) AND IN A DARK
ROOM THE PUPILL WILL DILATE TO ALLOW MORE LIGHT TO ENTER
(MYDRIASIS). THIS IS A FUNCTION OF THE SYMPATHETIC SYSTEM. AND IT’S AN
UNCONDITIONAL REFLEX.

Cells of a person working in the Chornobyl Exclusion Zone have undergone a mutation in DNA
molecule. However, with time the damaged interval of DNA molecule has been restored to its initial
structure with a specific enzyme. In this case the following occurred:
Repair
Replication
Transcription
Reverse transcription
Translation
ANOTHER WORD FOR DNA RESTORATION IS DNA REPAIR

Autopsy of a 5-year-old child revealed in the area of the vermis of cerebellum a soft grayish-pink
node 2 cm in diameter with blurred margins and areas of haemorrhage. Histologically this tumour
consisted of atypical monomorphous small round cells with large polymorphous nuclei. What
tumour is it?
Medulloblastoma
Meningioma
Glioblastoma
Astrocytoma
Oligodendroglioma
MEDULLOBLASTOMA IS A CANCEROUS TUMOR—ALSO CALLED CEREBELLAR
PRIMITIVE NEUROECTODERMAL TUMOR (PNET)—THAT STARTS IN THE
REGION OF THE BRAIN AT THE BASE OF THE SKULL, CALLED THE POSTERIOR
FOSSA.**** THE CERREBELUM IS SITUATED AT THE BASE OF THE SKULL

When studying the signs of pulmonary ventilation, reduction of forced expiratory volume has
been detected. What is the li-kely cause of this phenomenon?
A. Obstructive pulmonary disease B. Increase of respiratory volume
C. Increase of inspiratory reserve volume D. Increase of pulmonary residual volume E. Increase of
functional residual lung capacity
FORCED EXPIRATORY VOLUME (FEV) MEASURES HOW MUCH AIR A PERSON CAN
EXHALE DURING A FORCED BREATH.
REDUCTION IN FEV > 80% IS A MEDICAL CONCERN IN CASES OF COPD E.G
EMPHYSEMA AND BRONCHITIS
A specimen of a parenchymal organ shows poorly delineated hexagonal lobules surrounding a
central vein, and the interlobular connective tissue contains embedded triads (an artery, a vein and
an excretory duct). What organ is it?
Liver
Pancreas
Thymus
Spleen
Thyroid
HEXAGONAL STRUCTURE OF LOBULES AND TRIADS IS UNIQUE TO LIVER

A patient had a trauma that caused dysfunction of motor centers regulating activity of head
muscles. In what parts of cerebral cortex can these centers normally be located?
A. Inferior part of precentral gyrus
B. Superior part of precentral gyrus
C. Supramarginal gyrus
D. Superior parietal lobule
E. Angular gyrus
NOTE!!!- THE INFERIOR PART PRECENTRAL GYRUS CONTROLS THE MUSCLES OF
THE HEAD AND THE SUPERIOR PART OF PRECENTRAL GYRUS CONTROLS THE
LOWER LIMBS.

At the post-mortem examination the stomach of a patient with renal failure was found to have a
yellow-brown coating on the thickened mucosa. The coating was fi-rmly adhering to its surface and
had si-gnificant thickness. Microscopy revealed congestion and necrosis of mucosal and submucosal
layers, fibrin presence. What is the most likely diagnosis?
A. Fibrinous gastritis
B. Croupous gastritis
C. Gastric abscess
Esogastritis
Corrosive gastritis
THE MAIN CLUE IN THIS IS THAT FIBRIN IS PRESENT

A 60-year-old man suffering from chronic hepatitis frequently observes nasal and gingival
hemorrhages, spontaneous hemorrhagic rashes on the skin and mucosa.
Such presentations result from:
A. Decreased synthesis of prothrombin and fibrinogen
B. Increased blood content of ami-notransferases
C. Decreased synthesis of serum albumins D. Increased blood content of macroglobuli-ns and
cryoglobulins
Decreased blood content of choli-nesterase
THE HIGHLIGHTED SYMPTOMS SHOWS THE ABSENCE OF CLOTING FACTORS
AND METABOLITES THAT CONTROLS HAMEORHAGE WHICH ARE PROTHROMBIN
AND FIBRONOGEN.

Leading symptoms of primary hyperparathyroidism are osteoporosis and renal damage


resulting in urolithiasis development. What substances are the basis of uroliths in such cases?
A. Calcium phosphate
B. Uric acid
Cystine
Bilirubin
Cholesterol
HYPERPARATHYROIDISM SIMPLY PUT IS THE PARATHYROID GLAND OVER
WORKING BY MAKING TOO MUCH CALCIUM PRESENT IN THE BLOOD. NOTE!!!
THE BLOOD ENTERS THE KIDNEY, THEREFORE THE CALCIUM ENDS UP FORMING
STONES (UROLITHIASIS).
An oncology department has admitted a patient with suspected pulmonary tumor. On
examination a pathology localized wi-thin the lower lobe of the right lung was detected. How many
bronchopulmonary segments are there in this lobe?
5
6
4
3
2
IN THE RIGHT LUNG- FIVE LOBES ARE PRESENT IN THE LOWER LOBE , 3 LOBES
IN THE SUPERIOR LOBE , 2 LOBES IN THE MIDDLE LOBE,
IN THE LEFT LUNG – 5 IN THE SUPERIOR LOBE , 4 OR 5 IN THE LOWER LOBE

A 45-year-old woman suffers from arterial hypertension with high blood concentration of
angiotensin II. What antihypertensive drug is the most recommended in the given case?
Lisinopril
Prazosin
Metoprolol
Reserpine
Verapamil
LISINOPRIL IS A DRUG OF THE ANGIOTENSIN-CONVERTING ENZYME INHIBITOR
CLASS USED PRIMARILY IN TREATMENT OF HIGH BLOOD PRESSURE, HEART
FAILURE, AND AFTER HEART ATTACKS
An alcoholic suffers from alcoholic psychosis with evident psychomotor agitati-on. What
neuroleptic drug should be admi-nistered for emergency aid?
Aminazine
Diazepam
Sodium bromide
Reserpine
Halothane
AMINAZINE HYDROCHLORIDE BLOCKS DOPAMINE RECEPTORS AND STOPS
THEM BECOMING OVER-STIMULATED, THEREBY HELPING TO CONTROL
PSYCHOTIC ILLNESS

In one of Polessia regions there was an outbreak of helminthiasis manifested by cramps and
facial edemas. The developed preventive measures in particular included ban for eating infested
pork even after heat processing. What helminthiasis was the case?
Trichinosis
Taeniarhynchosis
Teniasis
Echinococcosis
Alveococcosis
TRICHINOSIS IS INFECTION CAUSED BY THE ROUNDWORM TRICHINELLA
SPIRALIS OR ANOTHER TRICHINELLA SPECIES. SYMPTOMS INCLUDE
DIARRHEA, ABDOMINAL CRAMPS, MUSCLE PAIN, AND FEVER. PEOPLE
ACQUIRE THE INFECTION BY EATING RAW OR UNDERCOOKED
CONTAMINATED MEAT.

A patient demonstrates functional loss of nasal halves of the retinas. What area of visual
pathways is affected?
A. Optic chiasm
B. Left optic tract
C. Right optic tract
D. Left optic nerve
E. Right optic nerve
NASAL HALVES – THIS MAKES IT TO BE DOMINTAED ON BOTH EYE. THE OPTIC
CHIASM IS A POINT OF DECUSATION OF THE NASAL OPTIC NERVE FIBRES.
A 26-year-old woman with bronchitis has been administered a broad spectrum antibiotic as a
causal treatment drug. Speci-fy this drug:
Doxycycline
Interferon
BCG vaccine
Ambroxol
Dexamethasone
DOXYCYCLINE IS AN ANTIBIOTIC THAT IS USED IN THE TREATMENT OF A
NUMBER OF TYPES OF INFECTIONS CAUSED BY BACTERIA AND PROTOZOA. IT
IS USEFUL FOR BACTERIAL PNEUMONIA, ACNE, CHLAMYDIA INFECTIONS,
EARLY LYME DISEASE, CHOLERA AND SYPHILIS.

A 16-year-old young man sufferi-ng from seasonal allergic rhinitis has been prescribed a highly
active second-generation H1 blocker, which can be characterized by absence of marked sedati-ve
action. Name this drug:
Loratadine
Pipolphen
Chloropyramine (Suprastin)
Indometacin
Erythromycin
THIS MEDICATION IS AN ANTIHISTAMINE THAT TREATS SYMPTOMS SUCH AS
ITCHING, RUNNY NOSE, ALLERGIC RHINITIS, WATERY EYES, AND SNEEZING
FROM "HAY FEVER" AND OTHER ALLERGIES.

Examination of a 56-year-old woman with a history of type 1 diabetes mellitus revealed a


disorder of protein metaboli-sm that is manifested by aminoacidemia in the laboratory blood test
values, and cli-nically by the delayed wound healing and decreased synthesis of antibodies.
Which of the following mechanisms causes the development of aminoacidemia?
A. Increased proteolysis
Albuminosis
Decrease in the concentration of amino acids in blood
Increase in the oncotic pressure in the blood plasma
Increase in low-density lipoprotein level
THE LABORATORY RESULT GIVES US AMINOACIDEMIA (INCREASE IN
AMINOACID IN BLOOD). THE SYMPTOM OF DELAYED WOUND HEALING DENOTES
THAT THE BODY LACKS ENOUGH PROTEIN TO HEAL, ALSO THERE WAS
DECREASED SYNTHESIS OF ANTIBODIES THAT ARE MADE FFROM PROTEIN. OF
ALL THE OPTION THE ONLY REASONABLE ONE IS THE DESTRUCTION OF
PROTEIN – PROTEOLYSIS.

A patient with injury sustained to a part of the central nervous system demonstrates disrupted
coordination and movement amplitude, muscle tremor during volitional movements, poor
muscle tone. What part of the central nervous system was injured?
Cerebellum
Medulla oblongata
Oliencephalon
Mesencephalon
Prosencephalon
THE HIGHLIGHTED SYMPTOMS ARE UNIQUE TO CEREBELLUM DISORDERS

A 36-year-old patient has been admini-stered a depolarizing muscle relaxant duri-ng a surgery.
Name this drug:
Dithylinum
Proserin
Pipecuronium bromide (Arduan)
Diazepam
Aminazine
DITHYLINUM-DARNITSA. MUSCLE RELAXANT. IT IS USED FOR INTUBATION OF
TRACHEA, ENDOSCOPIC PROCEDURES, SHORT-TERM SURGERIES, REQUIRING
MUSCLE RELAXATION (REDUCTION OF DISLOCATIONS, REPOSITION THE
BONE FRAGMENTS), TO MAINTAIN MYOPLEGIA DURING SURGERIES
(GYNAECOLOGIC, THORACIC, ABDOMINAL), ELIMINATION OF THE CRAMPS IN
TETANUS.
A man has suffered multiple bone fractures of his lower extremities duri-ng a traffic accident.
During transportati-on to a hospital his condition was further aggravated: blood pressure decreased,
there were signs of pulmonary artery embolism. What kind of embolism is the most likely in the
given case?
A. Fat embolism
B. Air embolism
C. Gas embolism
D. Tissue embolism
Thromboembolism
N.B WHENEVER THERE IS BONE FRACTURE THERE IS RELEASE OF ADIPOSE
TISSUE/ FAT TISSUE INTO THE CIRCULATORY SYSTEM.

UN volunteers have arrived in Nigeria to assist the locals in aftermath of earthquakes. What
drug should they prescribe for individual chemoprophylaxis of malaria?

Chingamin
Pyrantel
Pyrimethamine (Chloridinum)
Primaquine
Interferon (Laferon)
CHINGAMIN (CHLOROQUINE) IS PREFFERED TO PRIMAQUINE IN PREVENTING
MALARIA.

After an extended treatment wi-th sulfanamides a patient has developed macrocytic anemia.
Production of active forms of the following vitamin is disrupted in such a condition:
A. Folic acid
Thiamine
Riboflavin
Pyridoxine
Cyanocobalamin
MACROCYTIC ANEMIA MEANS THAT THE RED BLOOD CELLS ARE LARGER
THAN NORMAL. WE USE THIS CLASSIFICATION BECAUSE IT HELPS US TO
DETERMINE THE CAUSE OF THE ANEMIA. THE MOST COMMON CAUSES OF
MACROCYTIC ANEMIA ARE VITAMIN B-12(CYANOCOBALAMIN) AND FOLATE
(FOLIC ACID) DEFICIENCY.
THE REASON THE ANSWER IS FOLIC ACID WAS BECAUSE THERE WAS EXTENDED
TREATMENT WITH SULFANIMDES WHICH AFFECT THE NORMAL PATHWAY OF
FOLIC ACID CONTRIBUTION IN RBC FORMATION.

A woman resting in the countrysi-de has been stung by a bee. Immediately after she developed
pain in the stung area. In a few minutes there developed a vesi-cle, erythema and intense itch; later
- urti-caria and expiratory dyspnea. What factors resulted in the patient developing expi-ratory
dyspnea?
Histamine
Hageman’s factor
Lysosomal enzymes
Noradrenaline
Adrenaline
WHAT WE ARE DEALING WITH HERE IS THE TYPE 1 HYPERSENSITIVITY!!! FROM THE
OPTION THE SYMPTOMS SURFACED IMMEDIATELY AFTER FEW MINUTES.
THEREFORE, IF IT’S TYPE 1 IT IS ALLERGIC REACTION, IF IT IS ALLERGIC
REACTION THE BODY WILL ACTIVATE PRODUCTION OF HISTAMINE THAT LEADS
TO DILATION OF BLOOD VESSELS THEN WE SEE THE UNDERLINED SIGNS ON THE
SKIN.

Autopsy of a 28-year-old patient, who had been suffering from rheumatism and died of heart
failure, revealed pancardi-tis. Histological investigation of myocardi-um of the left ventricle
posterior wall and interventricular septum detected peri-vascular cellular focal infiltrates composed
of macrophages and creating palisade structures surrounding areas of fibrinoid necrosis.
Determine the type of myocarditis:
Granulomatous
Diffuse interstitial productive
Diffuse interstitial exudative
Focal interstitial exudative
-
THE INFILTRATION BY MACROPHAGES WITH FIBRINOID NECROSIS IS
UNIQUE TO GRANULOMATOUS MYOCARDITIS

When ascending to the top of Elbrus, a mountain climber experiences oxygen starvation,
dyspnea, palpitations, and numbness of the extremities. What kind of hypoxia has developed in the
mountain cli-mber?
Hypoxic
Circulatory
Hemic
Tissue
Cardiac
OXYGEN STARVATION – HYPOXIC HYPOXIA

A 2-year-old boy is diagnosed wi-th Down syndrome. What chromosomal changes can cause
this disease?
A. Trisomy 21
B. Trisomy 13
C. Trisomy X
D. Trisomy 18
E. Monosomy X
DOWN SYNDROME (DS OR DNS), ALSO KNOWN AS TRISOMY 21, IS A GENETIC
DISORDER CAUSED BY THE PRESENCE OF ALL OR PART OF A THIRD COPY OF
CHROMOSOME 21. IT IS TYPICALLY ASSOCIATED WITH PHYSICAL GROWTH
DELAYS, CHARACTERISTIC FACIAL FEATURES AND MILD TO MODERATE
INTELLECTUAL DISABILITY.

A 62-year-old patient has been hospi-talized due to massive cerebral hemorrhage. Blood
pressure is 70/30 mm Hg, heart rate is 120/min., respiratory rate is 4/min., unconscious, no
response to external sti-muli. Such condition can be determined as:
Coma
Shock
Collapse
Stress
Agony
OF ALL THE HIGHLIGHTED SIGNS AND SYMPTOMS, - PATIENT BEEN
UNCONSCIOUS, HAS A RESPIRATORY RATE 4/MIN (NORM 15-21) AND CAN’T
RESPOND TO EXTERNAL STIMULI DEFINITELY THE PATIENT IS IN COMA!!!

A 3-year-old girl with mental retardati-on has been diagnosed with sphingomyelin lipidosis
(Niemann-Pick disease). In this condition synthesis of the following substance is disrupted
Sphingomyelinase
Glycosyltransferase
Sphingosine
Ceramides
Gangliosides
LIPIDOSIS WITH ACCUMULATION OF SPHINGOMYELIN IN HISTIOCYTES IN
THE LIVER, SPLEEN, LYMPH NODES, AND BONE MARROW DUE TO A
DEFICIENCY OF SPHINGOMYELINASE; ASSOCIATED WITH
HEPATOSPLENOMEGALY, PHYSICAL AND MENTAL RETARDATION, AND
NEUROLOGIC MANIFESTATIONS; MACULAR CHERRY-RED SPOTS MAY OCCUR
AT A LATER STAGE
What condition may develop 15-30 mi-nutes after re-administration of an antigen as a result
of the increased level of anti-bodies, mainly IgE, that are adsorbed on the surface of target cells,
namely tissue basophils (mast cells) and blood basophi-ls?

Anaphylaxis
Antibody-dependent cytotoxicity
Delayed-type hypersensitivity
Immune complex hyperresponsiveness
Serum sickness
ANAPHYLAXIS IS A SERIOUS, LIFE-THREATENING ALLERGIC REACTION. THE MOST
COMMONANAPHYLACTIC REACTIONS ARE TO FOODS, INSECT STINGS, MEDICATIONS. IF
YOU ARE ALLERGIC TO A SUBSTANCE, YOUR IMMUNE SYSTEM OVERREACTS TO THIS
ALLERGEN BY RELEASING CHEMICALS THAT CAUSE ALLERGY SYMPTOMS

Local anesthetics (novocaine, lidocaine and others) decreases pain sensitivity of ti-
ssues by blocking N a+ and K+ ions from permeating membranes of nerve fibers and endings.
Such mechanism of drug action is called:
A. Membrane ionic
Receptor
Enzyme
Antienzyme
Direct chemical
FROM THE HIGHLIGHTED AND UNDERLINED POINT THE ANSWER IS CLEARLY
STATED

After a road accident a victim has tachycardia, arterial blood pressure 130/90 mm Hg,
tachypnoe, the skin is pale and dry, excitation of central nervous system is observed. What
shock stage is the patient most likely in?
Erectile
Terminal
Torpid
Preshock (compensation stage)
Agony
ERECTILE PHASE OF SHOCK- there is a strong motor
excitation, sweating, tremorskeletal muscles, staggering gait, frequent
urination, defecation.. The body is flooded with products of protein breakdown.
Increased body temperature, quickens the pulse and breathing.

A woman complaining of sharp pain in her lower abdomen has been delivered into an admission
room. A gynecologist on examination makes a provisional diagnosis of extrauterine pregnancy.
What anatomical structure should be punctated to confirm di-agnosis?
A. Rectouterine pouch
B. Utriculosaccular chamber
Recto-vesical pouch
Retropubic space
Intersigmoidal recess
THIS ENTIRE CLINICAL SITUATION IS ABOUT “ECTOPIC PREGNANCY” AND ITS
RUPTURE. IF THERE IS A RUPTURE, THERE WILL BE BLOOD IN THE
RECTOUTERINE POUCH OR DOUGLAS POUCH. THE DOCTOR WILL CARRY OUT A
PROCEEDURE CALLED CULDOCENTESIS .

A passenger of a fixed-run taxi has a sudden and expressed attack of tachycardia. A doctor
travelling by the same taxi has managed to slow down his heart rate by pressing upon the eyeballs
and thus causing the following reflex:
Dagnini-Aschner reflex
Bainbridge reflex
Holtz’s reflex
Hering-Breuer reflex
Frank-Starling mechanism
FROM EARTH TO MOON!... PRESSING OF THE EYEBALLS ACTIVATES THE
PARASYMPATHETIC NERVOUS SYSTEM- DANINI ASHNER REFLEX
A 47-year-old man developed intesti-nal colic against the background of essenti-al
hypertension. In this situation it would be the most efficient to arrest the colic by administering
drugs of the following group:
A. Myotropic antispasmodics
B. Anticholinesterase agents
Sympathomimetics
M-cholinomimetics
Adrenomimetics
THE INTESTINAL COLIC WAS CAUSED BY THE ESSENTIAL HYPERTENSION(
UNKOWN CAUSE OF HYPERTENSION). SO TREATING THE PATIENT GIVING A
MYOTROPIC ANTISPASMODICS WILL REDUCE THE HYPERTONICITY OF THE
SMOOTH MUSCLES OF THE INTESTINE.

A therapeutist has an appointment with a 40-year-old patient complaining of recurrent pain


attacks in his hallux joints and their swelling. Urine analysis revealed its marked acidity and pink
color. What substances can cause such changes in uri-ne?

A. Uric acid salt


Chlorides
Ammonium salts
Calcium phosphate
Magnesium sulfate
FROM THE SYMPTOMS THE PATIENT HAS GOUT. THEREFORE THE ACIDICITY OF
THE URINE POINTS US TO THE PRESENCE OF URIC ACID.

Exophthalmus observed during thyrotoxicosis is caused by accumulation of highly water-binding


substances within the retrobulbar tissues. Name these substances:
Glycosaminoglycans
Cholesterol
ATP
Kreatine
Phospholipids
GLYCOSAMINOGLYCANS (GAGS) OR MUCOPOLYSACCHARIDES ARE LONG UNBRANCHED
POLYSACCHARIDES CONSISTING OF A REPEATING DISACCHARIDE UNIT

A patient presents with dry peeli-ng skin, frequent cases of acute respi-ratory diseases,
xerophthalmia. What vi-tamin preparation should be prescribed in this case?

A. Retinol acetate
Thiamine
Cyanocobalamin
Menadione (Vikasolum)
Ergocalciferol
VITAMIN A DEFICIENCY ARE INADEQUATE INTAKE, FAT MALABSORPTION, OR
LIVER DISORDERS. DEFICIENCY: IMPAIRS IMMUNITY (the patient was having frequent
acute respiratory diseases) AND HEMATOPOIESIS AND CAUSES RASHES(dry peeling skin)
AND TYPICAL OCULAR EFFECTS (E.G., XEROPHTHALMIA, NIGHT BLINDNESS)
Parenchyma of an organ is composed of pseudounipolar neurons localized under the capsule of
connective tissue. Central place belongs to nerve fibers. Name this organ:
A. Spinal ganglion
B. Sympathetic ganglion
C. Intramural ganglion
D. Nerve trunk
E. Spinal cord
A pseudounipolar neuron (pseudo – false, uni – one) is a kind of sensory neuron in the
peripheral nervous system. This neuron contains an axon that has split into two branches; one
branch runs to the periphery and the other to the spinal cord

A patient consulted a physician about chest pain, cough, fever. Roentgenography of lungs
revealed eosinophilic infiltrates that were found to contain larvae. What ki-nd of helminthiasis are
these presentations typical of?
Ascariasis
Echinococcosis
Fascioliasis
Cysticercosis
Trichinosis
ASCARIASIS IS VERY UNIQUE IN ITS ABILITY TO MIGRATE THROUGH THE
LUNGS AND THEREBY CAUSING THE UNDERLINED SYMPTOMS ABOVE.

During appendectomy a patient had the a. appendicularis ligated. This vessel branches from
the following artery:
A. A. ileocolica
B. A. colica dextra
C. A. colica media
D. A. sigmoidea
E. A. mesenterica inferior
CHECK OUT THE APPENDICULARIS ARTERY IN YOUR ANATOMY ATLAS OR
GOOGLE FOR VISUAL UNDERSTANDING!

A patient with signs of osteoporosis and urolithiasis has been admitted to an


endocrinology department. Blood test revealed hypercalcemia and hypophosphatemia. These
changes are associated with abnormal synthesis of the following hormone:
A. Parathyroid hormone
Calcitonin
Cortisol
Aldosterone
Calcitriol
PARATHYROID HORMONE (PTH) REGULATES THE AMOUNT OF CALCIUM AND
PHOSPORUS IN THE BLOOD. THE HIGHLIGHTED SYMPTOMS SHOWS THAT THE
PT-GLAND WAS HYPERACTIVE I.E HYPER-PARATHYOIDISM.

Prescription of penicillin G sodium salt has caused development of neurotoxic effects


(hallucinations, convulsions). Such reaction is the result of antagonism with the following
neurotransmitter:
GABA
Dopamine
Serotonin
Adenosine
Acetylcholine
Gamma-Aminobutyric Acid (GABA) is one of your body's primary neurotransmitters, and
acts to calm your central nervous system (CNS). Research has shown the GABA supplement
to be effective in natural treatment for depression, anxiety, insomnia, and addiction.

A 30-year-old woman exhibits signs of virilism (growth of body hair, balding temples, menstrual
disorders). This condi-tion can be caused by overproduction of the following hormone:
Testosterone
Oestriol
Relaxin
Oxytocin
Prolactin
VIRILISM OCCURS WHEN THERE IS EXCESIVE PRODUCTION OF TESTORONE. A
MAJOR MALE HORMONE THAT GIVES MALES THEIR SEXUAL CHARACTERISTICS.

During a surgery for femoral hernia a surgeon operates within the boundaries of femoral
trigone. What structure makes up its upper margin?
A. Lig. inguinale
B. Arcus iliopectineus
C. Lig. lacunare
D. Lig. pectinale
E. Fascia lata
CHECK THIS UP ON GOOGLE FOR THE PICTURE - femoral trigone

Activation of a number of hemostatic factors occurs through their joining with calcium ions.
What structural component allows for adjoining of calcium ions?
Gamma-carboxyglutamic acid
Gamma-aminobutyric acid
Gamma-oxybutyric acid
Hydroxyproline
Monoamine-dicarboxylic acids
CARBOXYGLUTAMIC ACID IS AN UNCOMMON AMINO ACID INTRODUCED
INTO PROTEINS BY A POST-TRANSLATIONAL CARBOXYLATION OF GLUTAMIC
ACID RESIDUES. THIS MODIFICATION IS FOUND, FOR EXAMPLE, IN CLOTTING
FACTORS AND OTHER PROTEINS OF THE COAGULATION CASCADE

A patient has arterial hypertension. What long-acting calcium channel blocker should be
prescribed?
Amlodipine
Octadine
Pyrroxanum
Atenolol
Reserpine
AMLODIPINE IS A CALCIUM CHANNEL BLOCKER THAT DILATES (WIDENS)
BLOOD VESSELS AND IMPROVES BLOOD FLOW. AMLODIPINE IS USED TO
TREAT CHEST PAIN (ANGINA) AND OTHER CONDITIONS CAUSED BY
CORONARY ARTERY DISEASE. AMLODIPINE IS ALSO USED TO TREAT HIGH
BLOOD PRESSURE (HYPERTENSION)

Material obtained from a patient contains several types of microorganisms (staphylococci and
streptococci) causative of the patient’s disease. Name this type of infection:
A. Mixed infection
Superinfection
Reinfection
Consecutive infection
Coinfection
THIS IS STRAIGHT FORWARD!!!
A laboratory has been investigati-ng virulence of a diphtheria agent. In the process of the
experiment the infection was introduced intraperitoneally into test ani-mals. The dosage of bacteria
resulting in 95% mortality of test animals was found. What unit of virulence measurement was
determined?
DLM
DCL
LD50
ID
LD5
DLM IS UNIQUE FOR THIS QUESTION

164. A patient complains of palpitationsafter stress. Pulse is 104/min., P-Q=0,12 seconds, there are
no changes in QRS complex. What type of arrhythmia does the patient have?
A. Sinus tachycardia
B. Sinus bradycardia
C. Sinus arrhythmia
D. Ciliary arrhythmia
Extrasystole
THE PALPITATION IS RELATED TO STRESS, THE PULSE RATE INCREASED (NORM
70- 90), THE P-Q INTERVAL IS NORMAL. The Only Challenge Here Is The Increase In
Heart Rate Which Makes It Sinus Tachycardia

A patient consulted a dentist about restricted mouth opening (trismus). He has a history of a
stab wound of the lower extremity. What infection can cause these symptoms?
Tetanus
Brucellosis
Whooping cough
Wound anaerobic infection
Tularemia
NOTE: HISTORY OF STAB WOUND POINTS THAT THE WOUND MIGHT HAVE BEEN
INFECTED WITH CLOSTRIDIUM TETANY THEREFORE, TRISMUS OCCURRED.
Patient’s systolic blood pressure is 90mm Hg, diastolic - 70 mm Hg. Such blood pressure is
caused by decrease of the following factor:
A. Pumping ability of the left heart
B. Pumping ability of the right heart
C. Aortic compliance
D. Total peripheral resistance E. Vascular tone
THE SYSTOLIC PRESSURE IS A FUNCTION OF THE LEFT VENTRICLE.

A 29-year-old man with a knife wound of the neck presents with bleeding. Duri-ng the initial d-
bridement of the wound a surgeon revealed the injury of a vessel situated along the lateral edge of
the sternocleidomastoid muscle. Specify this vessel:

A. V. jugularis externa
B. V. jugularis anterior
C. A. carotis externa
D. A. carotis interna
E. V. jugularis interna
CHECK OUT EXTERNAL JUGULAR VEIN IN-SITU IN REALTATION TO
STERNOCLEIDOMASTOID MUSCLE.

A 6-year-old child suffers from delayed growth, disrupted ossification processes,


decalcification of the teeth. What can be the cause?
Vitamin D deficiency
Decreased glucagon production
Insulin deficiency
Hyperthyroidism
Vitamin C deficiency
WITHOUT GOING TOO FAR IT CAN ONLY BE VITAMIN D DEFICIENCY!!!
A patient addressed a hospital with complaints of lost sensitivity of the skin of the little finger.
What nerve is the most li-kely to be damaged?
Ulnar
Median
Radial
Musculocutaneous
Medial cutaneous nerve of the forearm
CHECK OUT THE SUPPLY OF ULNAR NERVE ON GOOGLE FOR A VISUAL IDEA!!!

A 30-year-old patient’s blood test has revealed the following: erythrocyte count
12
is 6 · 10 /l, hemoglobin is 10,55 mmol/l. Vaquez’s disease was diagnosed. Name the leading part
of pathogenesis:
A. Neoplastic erythroid hyperplasia
Iron-deficiency
B12-deficiency
Hypoxia
Acidosis
VAQUEZ’S DISEASE: Primary Polycythemia or Vaquez disease is characterised by an
excess of red cells caused by over-production by the bone marrow.
WE COULD SEE HIGH AMOUNT OF RBC.

Pancreas is known as a mixed gland. Endocrine functions include production of insulin by beta
cells. This hormone affects metabolism of carbohydrates. What is its effect on the activity of
glycogen phosphorylase (GP) and glycogen synthase (GS)?
A. It inhibits GP and activates GS B. It activates both GP and GS C. It inhibits both GP
and GS
D. It activates GP and inhibits GS
E. It does not affect the activity of GP and GS
GLYCOGEN PHOSPHORYLASE(GP)- USED IN THE FORMATION OF GLUCOSE BY
BREAKING DOWN GLYCOGEN.
GLYCOGEN SYNTHASE(GS)- USED IN THE SYNTHESIS OF GLYCOGEN.
THEREFORE, INSULIN WILL SUPPORT MORE OF THE PRODUCTION OF
GLYCOGEN THAN ITS LYSIS.

This year influenza epidemic is characterised by patients’ body temperature varying from 36,
9 C to 37, 9oC. Such fever is called:
o

Subfebrile
High
Hyperpyretic
Apyretic
Moderate
SUBFEBRILE: OF, RELATING TO, OR CONSTITUTING A BODY TEMPERATURE
VERY SLIGHTLY ABOVE NORMAL BUT NOT FEBRILE.

Fructosuria is known to be connected with inherited deficiency of fructose 1-phosphate


aldolase. What product of fructose metabolism will accumulate in the organism resulting in toxic
action?
Fructose 1-phosphate
Glucose 1-phosphate
Glucose 6-phosphate
Fructose 1,6-biphosphate
Fructose 6-phosphate
FRUCTOSE 1-PHOSPHATE ALDOLASE- IS USED FOR THE CLEAVAGE OF
FRUCTOSE-1-PHOSPHATE
A woman complains of visual impai-
rment. Examination revealed obesity
in the patient and her fasting plasma
glucose level is hyperglycemic. What
diabetes complicati-on can cause
visual impairment/blindness
A. Microangiopathy
B. Macroangiopathy
C. Atherosclerosis
D. Neuropathy
E. Glomerulopathy
MICROANGIOPATHY IS ONE OF
THE PATHOLOGIES THAT IS
PRESENT IN A DIABETIC
PATIENT.
175. Administration of doxycycline hydrochloride has caused an imbalance of the symbiotic
intestinal microflora. Specify the kind of imbalance caused by the antibi-otic therapy:
Dysbacteriosis
Sensibilization
Idiosyncrasy
Superimposed infection
Bacteriosis
THE DEFICIENCY OR ABSENCE OF INTESTINAL FLORA IS CALLED
DYSBACTERIOSIS. IN THIS QUESTION , THE EXCESS USE OF DOXICYCLINE LED
TO DYSBACTERIOSIS

Cholesterol content in blood serum of a 12-year-old boy is 25 mmol/l. Anamnesis states


hereditary familial hypercholesterolemia caused by synthesis disruption of receptor-related proteins
for:
Low-density lipoproteins
High-density lipoproteins
Chylomicrons
Very low-density lipoproteins
Middle-density lipoproteins
FAMILIAL HYPERCHOLESTEROLEMIA IS A GENETIC DISORDER. IT IS CAUSED
BY A DEFECT ON CHROMOSOME 19. THE DEFECT MAKES THE BODY UNABLE
TO REMOVE LOW DENSITY LIPOPROTEIN (LDL, OR BAD) CHOLESTEROL
FROM THE BLOOD. THIS RESULTS IN A HIGH LEVEL OF LDL IN THE

During recording of a spirogram a pati-ent calmly exhaled. How do we call the volume of air
remaining in the lungs?
A. Functional residual capacity
B. Pulmonary residual volume
C. Expiratory reserve volume
D. Tidal volume
E. Vital capacity of lungs
FUNCTIONAL RESIDUAL CAPACITY (FRC) IS THE VOLUME OF AIR PRESENT IN
THE LUNGS AT THE END OF PASSIVE EXPIRATION. AT FRC, THE OPPOSING
ELASTIC RECOIL FORCES OF THE LUNGS AND CHEST WALL ARE IN
EQUILIBRIUM AND THERE IS NO EXERTION BY THE DIAPHRAGM OR OTHER
RESPIRATORY MUSCLES.
A 40-year-old woman was diagnosed with glomerulonephritis based on her cli-nical symptoms and
the results of urine analysis. Anamnesis states chronic tonsilli-tis. What microorganisms are the
most likely cause for the kidney damage in this case?
Streptococci
Staphylococci
Escherichia
Mycoplasma
Meningococci
FROM THE ANAMNESIS, TONSILITIS WAS PRESENT AND NOW THE PATIENT IS
HAVING GLOMERULONEPHRITIS. THE ONLY THING YOU AS A DOCTOR YOU
CAN THINK IS STREPTOCOCCAL INFECTION

A man is suffering from diarrhea. In summer he spent his vacation in the south at the sea coast.
Bacteria with the following properties were detected in his feces: gram-negative curved mobile
monotrichous baci-lli that do not produce spores or capsules. Bacilli are undemanding to nutrient
medi-um but require alkaline reaction (рН 8,5-9,5). Described are the agents of the followi-ng
enteric infection:
A. Cholera
B. Shigellosis
C. Typhoid fever
D. Colienteritis
E. Pseudotuberculosis
This is the main HINT - gram-negative curved mobile monotrichous baci-lli that do not
produce spores or capsules.

An athlete (long-distance runner) duri-ng a contest developed a case of acute cardi-ac


insufficiency. This pathology resulted from:

A. Cardiac volume overload


B. Disrupted coronary circulation
C. Direct damage to myocardium
D. Pericardium pathology
E. Cardiac pressure overload
VOLUME OVERLOAD REFERS TO THE STATE OF ONE OF THE CHAMBERS OF THE
HEART IN WHICH TOO LARGE A VOLUME OF BLOOD EXISTS WITHIN IT FOR IT
TO FUNCTION EFFICIENTLY. VENTRICULAR VOLUME OVERLOAD IS
APPROXIMATELY EQUIVALENT TO AN EXCESSIVELY HIGH PRELOAD. IT IS A
CAUSE OF CARDIAC FAILURE.

Parkinson’s disease is caused by di-sruption of dopamine synthesis. What brain structure


synthesizes this neurotransmitter?
A. Substantia nigra
B. Globus pallidus
C. Corpora quadrigemina
D. Red nucleus
Hypothalamus
DOPAMINE IS SYNTHESIZED IN THE SUBSTANTIA NIGRA

Name the halogen-containing anti-septic with fungicidal properties, which is used to treat
dermatomycosis:
A. Iodine solution
B. Formalin solution
C. Methylene blue
D. Brilliant green
E. Boric acid solution
ACCORDING TO THE PERIODIC TABLE THE HALOGENS ARE FLOURINE, CHLORINE,
BROMINE , IODINE AND ASTATINE.
Due to severe pain syndrome a patient was prescribed a narcotic analgesic. Name this drug:
Morphine
Metamizole (Analgin)
Nimesulide
Dimethyl sulfoxide
Indometacin
Morphine is a pain medication of the opiate variety which is found naturally in a number of
plants and animals. It acts directly on the central nervous system to decrease the feeling of
pain. It can be taken for both acute pain and chronic pain.

During pathomorphological renal investigation of a patient, who for a long time had been
suffering from osteomyeli-tis and died of progressing renal fai-lure, the following was revealed:
deposi-ts of homogeneous eosinophilic masses in glomerular mesangium, arterial and arteriolar
walls, and stroma, which colored red when stained with Congo red. What pathological process is
this?
Amyloidosis
Mucoid swelling
Calcinosis
Carbohydrate degeneration
Hyalinosis
CONGO RED IS SPECIFIC HISTOLOGICAL DYE USED TO STAIN ANY TISSUE FOR
THE DETERMINATION OF THE AMYLOIDOSIS.

During experiment a dog has developed conditioned digestive reflex in


response to a sound stimulus. This conditi-oned reflex will not be exhibited anymore after the
extirpation of the following areas of the cerebral hemispheres:
A. Temporal lobe on both sides
B. Occipital lobe on one side
C. Parietal lobe on both sides
D. Temporal lobe on one side
E. Occipital lobe on both sides
SOUND STIMULI ARE BEEN INTERPRETED IN THE TEMPORAL LOBE. TEMPORAL
LOBES ON BOTH SIDES WHEN SEVERED WILL DEFINITELY LEAD TO THE
INHIBITION OF THE STIMULI.

Coenzym A participates in numerous important metabolic reactions. It is a deri-vative of the


following vitamin:
A. Pantothenic acid
Thiamine
Niacin
Calciferol
Ubiquinone
PANTOTHENIC ACID, ALSO CALLED VITAMIN B5 (A B VITAMIN), IS A WATER-SOLUBLE
VITAMIN. PANTOTHENIC ACID IS AN ESSENTIAL NUTRIENT. ANIMALS REQUIRE
PANTOTHENIC ACID IN ORDER TO SYNTHESIZE COENZYME-A (COA), AS WELL AS TO
SYNTHESIZE AND METABOLIZE PROTEINS, CARBOHYDRATES, AND FATS..

A patient diagnosed with acute dysentery has been treated for 3 days in an infectious diseases
hospital. On admission there were complaints of high temperature, stomachache and fluid
excrements with mucus up to 8-10 times a day. What sample should be taken for analysis?
Feces
Urine
Bile
Liquor
Blood
ITS DYSENTERY THEREFORE ITS FAECES

A patient with arthritis has been prescribed an anti-inflammatory selective COX-2 inhibitor.
Select this drug among those given below:
Celecoxib
Phenylbutazone (Butadion)
Dimethylsulfoxide (Dimexid)
Indometacin
Metamizole (Analgin)
Celecoxib is a nonsteroidal anti-inflammatory drug (NSAID) that is used to treat arthritis,
pain, menstrual cramps, and colonic polyps.

A person with the fourth blood group (genotype IAIB) has in erythrocytes both antigen A
controlled by allele IA and antigen B controlled by allele IB. This phenomenon is an example of the
following gene interation:

Codominance
Complementarity
Semidominance
Polymery
Epistasis
Codominance is a form of dominance wherein the alleles of a gene pair in a heterozygote are
fully expressed. This results in offspring with a phenotype that is neither dominant nor
recessive. A typical example showing codominance is the ABO blood group system
A patient with diabetes mellitus suffers from persistently nonhealing surgi-cal wound, which is
a sign of disrupted ti-ssue trophism. What is the cause of such disorder?
A. Disruption of protein metabolism regulation
Hypoglycemia
Ketonemia
Increased lipid catabolism
Anemia
AS A DOCTOR NEVER FORGET THIS THAT PATIENTS WITH DIABETES MELITUS
HAVE ISSUE WITH WOUND HEALING BECAUSE OF DIRUPTION OF PROTEIN
METABOLISM REGULATION

A patient with signs of intestinal infection (vomiting, diarrhea, abdominal pain) has been
presenting with increasing symptoms of intoxication for three days. Papular rash appeared on the
uncovered skin areas and spread to the torso. A doctor suspected pseudotuberculosis. What
laboratory test allows confirming this di-agnosis within the first week from the onset of disease?
Bacteriological
Microscopic
Serological
Allergic
Biological
FOR PSEUDOTUBERCULOSIS, BACTERIOLOGICAL METHOD IS THE BEST
APPROACH BECAUSE IT’LL ALSO GIVE ROOM FOR IDENTIFYING THE EXACT
MICROBE. AFTER THE IDETIFICATION OF THE MICROBE, SENSITIVITY TO
THE BEST ANTIBIOTIC CAN ALSO BE CARRIED OUT.

A microslide demonstrates an organ with its wall consisting of three membranes. The inner
membrane has tubular glands and undergoes cyclic changes. Name this organ:
Uterus
Esophagus
Vagina
Ureter
Urinary bladder
OF ALL THE OPTION THE ONLY ONE THAT FITS INTO THE UNDERLINED AND
HIGHLIGHTED CHARACTERISTICS IS UTERUS. WITH THE THREE WALLS
(ENDOMETRIUM , MYOMETRIUM AND PERIMETRIUM).
A patient with femoral neck fracture, who for a long time had to remain in bed in a forced
(supine) position, has developed dark-brown lesions along the backbone; soft tissues are
swollen, in the areas of macerati-on there is a foul-smelling liquid. Name the clinicopathologic
type of necrosis:
Bedsore
Infarction
Sequestrum
Coagulation necrosis
Dry gangrene
PRESSURE ULCERS, ALSO KNOWN AS PRESSURE SORES, PRESSURE INJURIES,
BEDSORES, AND DECUBITUS ULCERS, ARE LOCALIZED DAMAGE TO THE SKIN
AND/OR UNDERLYING TISSUE THAT USUALLY OCCUR OVER A BONY
PROMINENCE AS A RESULT OF PRESSURE, OR PRESSURE IN COMBINATION
WITH SHEAR AND/OR FRICTION.
A patient is 20 years old, an athlete. He addressed a doctor with complaints of fati-gue, fever up to
38oC - 40oC. Objectively: the liver and spleen are enlarged, lymph nodes on palpation are slightly
enlarged, dense, painless. Blood test: Нb- 100 g/l; erythrocytes - 2, 9 · 1012/l; leukocytes -
4, 4 · 109/l. Leukogram: 68% of blast cells. Cytochemical investigation of blast cells revealed negative
reactions to glycogen, peroxidase, non-specific esterase, lipids. Name this disease:
A. Acute undifferentiated leukemia B. Acute myeloid leukemia
C. Acute monoblastic leukemia
D. Acute lymphoblastic leukemia
E. Acute megakaryoblastic leukemia
BLAST CELLS OR IMMATURE CELLS ARE UNDIFFERNTIATED CELLS FOUND IN THE
BONE MARROW IN THE HIGHER PERCENTAGE THAN IN THE PERIPHERAL BLOOD.
IN THIS QUESTION THE BLAST CELLS ARE REALLY HIGH, THEREFORE, THE ANSWER
IS ACUTE UNDIFFERENTIATED LEUKEMIA .

Impression smear of mucosa biopsy material has been obtained from a patient with peptic ulcer
disease of the stomach. Gram-negative arcuate bent microorgani-sms were detected, urease
activity test was positive. What microorganisms were detected in the patient?
Helicobacter
Spirochete
Spirilla
Leptospira
Treponema
FROM MICROBIOLOGY VIEW- KEEP THIS IN MIND UREASE POSITIVE TEST AND
PEPTIC ULCER SYMPTOMS = HELICOBACTER PYLORI

A patient after disrupted cerebral ci-rculation has developed paralysis. Choose the
anticholinesterase drug to be prescribed in this case:
Proserin
Cordiamin
Aceclidine
Methacin
Hexamethonium (Benzohexonium)
ANTICHOLINESTERASE, ANY OF SEVERAL DRUGS THAT PREVENT
DESTRUCTION OF THE NEUROTRANSMITTER ACETYLCHOLINE BY THE
ENZYME ACETYLCHOLINESTERASE WITHIN THE NERVOUS SYSTEM. E.G
PROSERINE(NEOSTIGMINE), PHYSIOSTYGMINE

30 minutes after drinking mango juice a child suddenly developed a local swelli-ng in the area
of the soft palate, which impeded swallowing and, eventually, respi-ration. Mucosa of the swollen
area was hyperemic and painless. Blood test revealed moderate eosinophilia. Body temperature was
normal. Anamnesis states that the elder sister of the child has been suffering from bronchial asthma
attacks. What kind of edema has developed in the child?
Allergic
Inflammatory
Cardiac
Alimentary
Hepatic
THE EXAMINER IS TESTING YOU ON TYPE 1 HYPERSENSITIVITY (ANAPHYLAXIS).
THE ALLERGIC REACTION OCCURRED WITHIN 30 MINUTES. ANOTHER CLUE IN
THE QUESTION IS THE ELDER SISTER HAVING A FORM OF ALLERGIC RELATED
BRONCHIAL ASTHMA.
During examination of a patient a doctor should use anatomical division of anterior abdominal
wall into regions for more precise diagnostics. How many regi-ons can abdomen be divided into?
9
8
6
5
4
CHECK GOOGLE FOR THE PICTURE OF THE 9 ABDOMINAL REGION FOR VISUAL
UNDERSTANDING.

A child has a wound located posterior to the mastoid process. Bright red blood flows from the
wound. Damaged are the branches of the following artery:
A. A. occipitalis
B. A. temporalis superior
C. A. maxillaris
D. A. carotis externa
E. A. carotis interna
CHECK YOUR ATLAS OR GOOGLE FOR OCCIPITAL ARTERY FOR VISUAL
UNDERSTANDING

When investigating human saliva it is necessary to assess its hydrolytic properti-es. What
substance should be used as a substrate in the process?
Starch
Proteins
Fats
Fiber
Amino acids
FROM PHYSIOLOGY, STARCH DIGESTION STARTS FROM THE MOUTH. STARCH IS
BEEN ACTED UPON BY PTYALIN AND AMYLASE PRESENT IN THE SALAIVA

CREDITS: This was solved by Poopola Ebenezer


KROK 2017.

1. A 35-year-old man has been delivered into a surgical ward with a suppurating wound in the neck,
anterior to the trachea (previsceral space). If a surgical operation is not performed urgently, there is a
risk of infection spreading to the:
A. Thoracic cavity - middle mediastinum
B. Thoracic cavity - posterior mediastinum
C. Retrovisceral space
D. Interaponeurotic suprasternal space
E. Thoracic cavity - anterior mediastinum
ANS E
Trachea (previsceral space), the pretracheal portion of the visceral space extends to the anterior
mediastinum at the level of the arch of the aorta and fibrous pericardium, infections in the periveseral
space can rapidly extend to anterior mediastinum.

2. In the life cycle of a cell during mitosis a natural change in the amount of genetic material occurs.
The DNA doubles at the following stage:
A. Prophase
B. Metaphase
C. Interphase
D. Anaphase
E. Telophase
Ans C

lnterphase is the longest stage in the eukaryote cell cycle. During interphase, the cell acquires
nutrients, creates and uses proteins and other molecules, and starts the process of cell division by
replicating the DNA

3. A woman with seasonal vasomotor rhinitis, who works as a train dispatcher and is an outpatient,
should be prescribed an antihistaminic drug that does not suppress central nervous system. Name this
drug:

A. Dimedrol (Diphenhydramine)
B. Diprazine (Promethazine)
C. Suprastin (Chloropyramine)
D. Tavegyl (Clemastine)
E. Loratadine

Ans E
Loratadine is a tricyclic antihistamine, which acts as a selective inverse agonist of peripheral histamine
H1-receptors, a non-sedating antihistamine, First-generation antihistamines readily cross the blood-
brain barrier leading to significant drowsiness, altered mood, reduced wakefulness, and impaired
cognitive and psychomotor performance

4. During sanitary and bacteriological testing of water with the membrane filter technique there were
revealed two red colonies on the membrane filter (Endo medium) through which 500 ml of water was
filtered. Calculate the coli index and coli titer of this water:

A. 4 and 250
B. 2 and 500
C. 250 and 4
D. 500 and 2
E. 250 and 2
Ans A

The coli titter is determined as the smallest volume of water in which the presence of Escherichia coli
is established.
The coli index is determined as the number of Escherichia coli cells in 1l water sample, and it is
calculated based on the coli titter value with the equation:
Coli titer 500/2=250

Coli index =1000/coli titer


1000/250 =4

5. Protective function of saliva is based on several mechanisms, including the presence of enzyme that
has bactericidal action and causes lysis of complex capsular polysaccharides of staphylococci and
streptococci. Name this enzyme:

A. Alpha-amylase
B. Oligo-1,6-glucosidase
C. Lysozyme
D. Collagenase
E. Beta-glucuronidase

Ans C
Human lysozyme is commonly associated with innate immunity, which forms the first line of defense
against infections. In the human body, it functions primarily as an antimicrobial by breaking down the
carbohydrates in bacterial cell walls, killing the bacteria.
6. What kind of muscle contraction occurs in an upper limb during an attempt to lift a load beyond
one’s strength?
A. Isotonic
B. Auxotonic
C. Isometric
D. Phasic
E. Single

Ans C

Isometric Contraction
• In general in this form of contraction the muscle length remains constant. It occurs when
muscle force balances resistance and no joint movement occurs
• there is generally no movement resulting from this type of contraction
• pushing against a fixed object
• planking
isotonic contraction
an increase in tension (load) results in changes in skeletal muscle length. i.e. lengthening and
shortening of the muscle.

7. The patient’s pyramids of the medulla oblongata are damaged by tumor growth. As a result the
conduction of nervous impulses will be impaired in the following pathway:

A. Tr. corticospinalis
B. Tr. corticonuclearis
C. Tr. corticopontinus
D. Tr. dentatorubralis
E. Tr. Spinocerebellaris
Ans A

The medullary pyramids contain motor fibers that are known as the corticobulbar and corticospinal
tracts. The corticospinal tracts are on the anterior surface of the pyramids. These tracts transport
motor signals that originated in the precentral gyrus and travelled through the internal capsule to the
medulla oblongata and pyramids

8. A patient complains of pain in the upper umbilical region. On palpation there is a mobile painful
intestine. What intestine is being palpated by the doctor?

A. Transverse colon
B. Jejunum
C. Duodenum
D. Ileum
E. Sigmoid colon
ANS A

9. After inoculation of investigated material (feces) on 1% alkaline peptone water and 8-hour-long
incubation in the thermostat under 37 o C there is growth of pale bluish film observed. Such cultural
properties are characteristic of the agent of the following disease:

A. Plague
B. Cholera
C. Typhoid fever
D. Paratyphoid A fever
E. Dysentery
Ans B
On an alkaline agar the cholera vibrio grows with the formation of round, smooth, flat, bluish,
homogeneous colonies which are 1-2 mm in diameter, transparent in the transmitted light and have
smooth edges

10. Histological investigation of the uterine scrape of the 45-year-old woman with disturbed ovarian
menstrual cycle revealed increased number of endometrial glands, some of which are serrated, while
others are dilated and cyst-like. Make the diagnosis:

A. Endometrial cystic glandular hyperplasia


B. Placental polyp
C. Atypical endometrial hyperplasia
D. Glandular endometrial polyp
E. Endometrial adenocarcinoma
Ans A

Endometrial Hyperplasia is a condition that occurs due to the excessive growth of the lining and cells
of the endometrium i.e. the inner lining of the uterus, due to hick estrogen stimulation giving
Irregularity and cystic expansion of glands. This condition is benign, but may lead to cancer at times.
Women who experience irregular menstrual cycles are more prone to this disease, as they do not
ovulate properly

11. A pregnant woman with several miscarriages in anamnesis is prescribed a therapy that includes
vitamin preparations. What vitamin facilitates carrying of a pregnancy?
A alpha tocopherol
B folic acid
C cyanocobalamin
D pyridoxal phosphate
E rutin

Ans A
alpha tocopherol aids by the prevention of miscarriages in pregnant

12 a patient has decreases concentration of magnesium ions that are required for ribosomes
connection to granular endoplasmic reticulum. This condition is known to disturb the process of
protein biosynthesis. Disturbance occurs at the following stage:
A. Translation
B. Transcription
C. Replication
D. Amino acids activation E. Processing
Ans A

Magnesium is an essential cofactor for the synthesis and salvage of purine and pyrimidine
nucleotides. It plays important roles in the structure of nucleic acids and affects their interaction with
proteins and other ligands. Magnesium is required for DNA replication, transcription into RNA and
translation into protein

13. A patient with chronic heart failure presents with increased blood viscosity. Capillaroscopy
detected damage to the vessel walls of the microcirculation system. What disorder is possible in the
given case?
A. Blood ” sludge” phenomenon
B. Thrombosis
C. Embolism
D. Arterial hyperemia
E. Venous hyperemia

ANS a
Sludge is therefore a condition which is of great clinical importance. Sludge is a vital microscopical
phenomenon, caused by an increased blood viscosity and tendency to red cell aggregation, and it is
aggravated by low flow states. Sludge thereby impairs micro- circulation

14. A 3-year-old boy with pronounced hemorrhagic syndrome has no anti- hemophilic globulin A
(factor VIII) in the blood plasma. Hemostasis has been impaired at the following stage:

A. Internal mechanism of prothrombinase activation


B. External mechanism of prothrombinase activation
C. Conversion of prothrombin to thrombin D. Conversion of fibrinogen to fibrin
E. Blood clot retraction
Ans A
Hemophilia is a rare disorder in which your blood doesn't clot normally because it lacks sufficient
blood-clotting proteins (clotting factors) factor VIII, also called antihemophilic factor( intrinsic clotting
factor)

15. Paronychia of the patient’s little finger was complicated with phlegmon of the hand and forearm.
In this case the suppuration had spread through the:
A. Vagina synovialis communis mm. flexorum
B. Vagina tendinis m. flexor pollicis longi
C. Canalis carpalis
D. Vagina tendinis m. flexor carpi radialis
E. Interfascial compartments
Ans a

Synovial sheath of the little finger:


-It covers the flexor digitorum superficialis and profundus muscles. Superiorly this sheath protrudes 1-
2 cm proximal to the flexor retinaculum.
- Interiorly it extends on the flexors until the base of the distal phalanx of the little finger.
Phlegmon in the thumb or little finger may pass along the ulnar and radial bursae to the Pirogov’s
space in the forearm. From there it passes up to the cubital fossa and other regions

16. During cholecystectomy besides CYSTIC ARTERY another artery was pulled into the ligature.
Ligation of this artery resulted in right-sided necrosis of the liver which led to the death of the patient.
What artery was mistakenly ligated along with CYSTIC ARTERY?

A. Ramus dexter a. hepatica propria


B. А. hepatica communis
C. А. gasro-duodenalis
D. Ramus sinister a. hepatica propria
E. А. pancreato-duodenalis sup

Ans A
The cystic artery branches from the hepatic artery proper.
Right hepatic artery injury is most commonly associated with injury to the right hepatic duct or with
dissection under the mistakenly identified common bile duct for the cystic duct, ligation may lead to
ischemia, liver failure.
ligating the branch of the right lobe leads to necrosis of that part of the liver

17. During treatment with bismuth preparations a patient with syphilis developed gray spots on his
oral mucosa and nephropathy symptoms. What drug is used as an antidote to bismuth preparations
poisoning?
A. Unithiol
B. Nalorphine
C. Bemegride
D. Naloxone
E. Methylene blue
Ans a

18. A patient with periodontitis of the lower molar came to the doctor. It was determined that the
inflammatory process spread to the lymph nodes. What lymph nodes were the first to be affected by
the inflammatory process?
A. Lateral cervical
B. Submandibular
C. Anterior cervical
D. Submental
E. Facial
Ans B

infection in lower molar, initial sign of the infection would be in the submandibular nodes; if it were
not successfully combated there, it would spread secondarily to the upper deep cervical nodes
because submandibular nodes Drains the Mandibular canines, preolars, molars, except for wisdom
teeth.

19. A 63-year-old man suffers from esophageal carcinoma, presents with metastases into the
mediastinal lymph nodes and cancerous cachexia. What pathogenetic stage of neoplastic process is
observed in the patient?
A. Promotion
B. Transformation
C. Progression
D. Initiation
E.-

Ans c
Cancer phase- Initiation, Promotion, and Progression. Tumor progression is the third and last phase in
tumor development. This phase is characterised by increased growth speed and invasiveness of the
tumor cells. As a result of the progression, phenotypical changes occur and the tumor becomes more
aggressive and acquires greater malignant potential
As esophageal cancer begins to progress by metastases into the mediastinal lymph nodes, symptoms
become more apparent, and the cancer becomes more difficult to treat.

20. A person is in a room with air temperature of 38oC and relative air humidity of 50%. What type of
heat transfer ensures maintenance of constant body core temperature under these conditions?
A. Radiation
B. Conduction and convection
C. Convection
D.-
E. Evaporation
Ans E
Evaporation takes place at room temperature which is often well below the boiling point of the liquid.
Evaporation happens from the surface of the liquid. As the temperature increases the rate of
evaporation increases. Evaporation is also assisted by windy conditions which help to remove the
vapour particles from the liquid so that more escape.

21. A patient has hoarseness of voice. During laryngoscopy a gray-white larynx tumor with papillary
surface has been detected. Microscopic investigation has shown the following: growth of connective
tissue covered with multilayer, strongly keratinized pavement epithelium, no cellular atypia. What is
the most likely diagnosis?
A. Fibroma
B. PolyP
C. Papilloma
D. Angioma
E. Angiofibroma

ANS C
squamous papilloma is defined as a benign or noncancerous tumor that grows like small warts . The
growths are commonly found on different areas of the body but are usually seen in the mouth or
genital regions. papilloma lesion is soft that has a narrow stalk with some pointed finger-like
projections. These projections can either be long, short and rounded if the keratin protein has formed
around the lesion. It can also be white in color, similar to that of a cauliflower head if the lesions are
heavily keratinized.

22. A shepherd, who tended to the flock of sheep with his dogs, gradually developed pain in the chest
and bloody expectorations. X-ray revealed spherical helminth larvae in the patient’s lungs. Specify the
helminth that could be the causative agent of this disease:
A. Echinococcus
B. Hymenolepis nana
C. Diphylobotrium latum
D. Fasciola hepatica
E. Taenia soliu
Ans A
Echinococcus granulosus, also called the hydatid worm, hyper tape-worm or dog tapeworm,
The tapeworm eggs are typically ingested during play with dogs or through consumption of garden
vegetables or water contaminated by dog feces. The most common sites of tapeworm cyst formation
within the body are the liver and lungs

23. A 67-year-old patient with clinical diagnosis of chronic bronchitis, pneumosclerosis, and
cardiopulmonary decompensation has the biopsy material taken from the suspicious area in his right
bronchus mucosa. Cellular and tissue atypism along with pearly bodies can be histologically detected.
What pathologic process is characterized by the described histological changes?
A. Polypoid chronic bronchitis
B. Squamous cell carcinoma of bronchus with keratinization
C. Bronchiectasis
D. Acute bronchitis
E. Squamous cell metaplasia of bronchial mucosa
ANS B

Squamous cell carcinoma of bronchus with keratinization has more irregular tumor islands infiltration,
a greater degree of cellular atypia, atypical mitotic figures, and individual cell keratinization as well as
keratin pearl formation.

24. A child with point mutation presents with absence of glucose 6-phosphatase, hypoglycemia, and
hepatomegaly. What pathology are these signs characteristic of?
A. Von Gierke’s disease (Glycogen storage disease type I)
B. Cori’s disease (Glycogen storage disease type III)
C. Addison’s disease (Primary adrenal insufficiency)
D. Parkinson’s disease
E. McArdle’s disease (Glycogen storage disease type V)
Ans A

von Gierke disease, is the most common of the glycogen storage diseases. This genetic disease results
from deficiency of the enzyme glucose-6-phosphatase,
The deficiency impairs the ability of the liver to produce free glucose from glycogen and from
gluconeogenesis.
-Because no gluconeogenesis = no glucose = hypoglycemia
-Impairment of glycogenolysis also causes the characteristic enlargement of the liver (hepatomegaly)
due to accumulation of glycogen.
-Lactic acid competes with Uric acid for renal excretion in urine. And that is why eventually years later
patients start showing symptoms of Hyperuricemia.

25. At the 2-3 day after the gastric resection the patient’s intestinal peristalsis failed to restore. What
should the patient be prescribed to stimulate the function of his gastrointestinal tract?
A. Platyphyllin
B. Cyclodol (Trihexyphenidyl)
C. Atropine
D. Proserin
E Dithyline (Suxamethonium chloride)

Ans D
Proserin (neostigmine) is a Synthetic reversible cholinesterase inhibitor. , are known to increase upper
and lower gastrointestinal (GI) motility and are used to treat acute colonic pseudo-obstruction

26. Examination of the patient with traumatic brain injury revealed that he has lost the ability to
discern the movement of an object on the skin. What part of the cerebral cortex is damaged?
A. Occipital lobe
B. Parietal lobe
C. Posterior central gyrus
D. Frontal lobe
E. Anterior central gyrus
Ans C
The postcentral gyrus is a prominent gyrus in the lateral parietal lobe of the human brain. It is the
location of the primary somatosensory cortex, the main sensory receptive area for the sense of touch

27. Blood test of the patient revealed albumine content of 20 g/l and increased activity of lactate
dehydrogenase isoenzyme 5 (LDH5). These results indicate disorder of the following organ:
A. Liver
B. Kidneys
C. Heart
D. Lungs
E. Spleen
Ans A

28. Section shows significant enlargement of the patient’s right kidney. There is a nephrolith at the
place of incision. Renal pelvic lumen is distended with accumulating urine. Renal parenchyma is
acutely thinned out. What is the most correct diagnosis?
A. Pyelectasis
B. Hydroureteronephrosis
C. Renal cyst
D. Nephroblastoma
E. Hydronephrosis
Ans E

Hydronephrosis is the swelling of a kidney due to a build-up of urine. It happens when urine cannot
drain out from the kidney to the bladder from a blockage or obstruction.

29. A patient demonstrates sharp decrease of pulmonary surfactant activity. This condition can result
in:
A. Alveolar tendency to recede
B. Decreased airways resistance
C. Decreased work of expiratory muscles
D. Increased pulmonary ventilation
E. Hyperoxemia
Ans A

Pulmonary surfactant is a mixture of lipids and proteins which is secreted into the alveolar space by
epithelial type II cells. The main function of surfactant is to lower the surface tension at the air/liquid
interface within the alveoli of the lung
Without normal surfactant, the tissue surrounding the air sacs in the lungs (the alveoli) sticks together
(because of a force called surface tension) after exhalation, causing the alveoli to collapse.
30. After a case of cold the patient developed a lacrimation disorder. This disorder was caused by
functional disturbance of the following autonomic ganglion:
a. Ciliary
b. Pterygopalatine
C. Otic
D. Submandibular
E. Sublingual

Ans B
The pterygopalatine ganglion is a parasympathetic ganglion found in the pterygopalatine fossa. It is
largely innervated by the greater petrosal nerve); and its axons project to the lacrimal glands and
nasal mucosa.

31. A patient is diagnosed with diabetic coma. Blood sugar is 18,44mmol/l. What
glucose-regulating drug should be prescribed in the given case?
A. Rapid-acting insulin
B. Intermediate-acting insulin
C. Long-acting insulin
D. Biguanide
E. Sulfonylurea derivative

Ans A
Fast-acting insulin (also called rapid-acting) is absorbed quickly and starts working in about 15
minutes to lower blood sugar
Rapid insulin is used for correction of hyperglycemia in DKA coma

32. A man came into the admission room with complaints of edemas, rapid heart rate, dyspnea, and
cyanotic mucosal tunics. He was diagnosed with chronic heart failure. What drug should be prescribed
to improve the patient’s general state?

A. Papaverine hydrochloride
B. Mesaton (Phenylephrine)
C. Cordiamin
D Digoxin
E. Nitroglycerine

Ans D
A cardiotonic glycoside obtained mainly from Digitalis lanata it consists of three sugars and the
aglycone digoxigenin. Digoxin has positive inotropic and negative chronotropic activity. It is used to
control ventricular rate in atrial fibrillation and in the management of congestive heart failure with
atrial fibrillation

33. Exposure to colchicine resulted in metaphase plate of a human containing 23 chromosomes more
than it is normal. Name this mutation:
A. Aneuploidy
B. Polyteny
C. Inversion
D. Translocation
E. Polyploidy
Ans E

Colchicine promotes a change in chromosome structure without loss of sister chromatid cohesion in
prometaphase, it is widely used to separate chromosomes at the metaphase and to induce polyploidy.
Polyploid cells and organisms are those containing more than two paired (homologous) sets of
chromosomes

34. A 30-year-old man complains of suffocation, heaviness in the chest on the right, general weakness.
Body temperature is 38,9oC. Objectively the right side of the chest lags behind the left side during
respiration. Pleurocentesis yielded exudate. What is the leading factor of exudation in the patient?

A. Increased permeability of the vessel wall


B. Increased blood pressure
C. Hypoproteinemia
D. Erythrocyte aggregation
E. Decreased resorption of pleural fluid
Ans A

Increase vascular permeability will leads to a redistribution of water into the lung tissue
the earliest symptom of pulmonary edema is shortness of breath, in the late stage – suffocation.
Thoracentesis ( pleurocentesis )a procedure to remove fluid from the space between the lungs and
the chest wall called the pleural space

35. A 46-year-old woman suffering from cholelithiasis developed jaundice. Her urine became dark
yellow, while feces are light- colored. What substance will be the most increased in concentration in
the blood serum in this case?
A. Conjugated bilirubin
B. Unconjugated bilirubin
C. Biliverdine
D. Mesobilirubin
E. Urobilinogen
Ans A
Conjugated hyperbilirubinemia occurs in several disorders associated with bile duct obstruction, with
subsequent obstruction of biliary flow.in this case due to bile stone (cholelithiasis)

36. A traumatology unit received a patient with crushed muscular tissue. What biochemical indicator
of urine will be raised in this case?
A. Creatinine
B. Total lipids
C. Glucose
D. Mineral salts
E. Uric acid
Ans a
Creatinine is a chemical waste product produced by muscle metabolism, curshed muscular tissue will
increase it in urine

37. A 30-year-old woman first developed pain, swelling, and skin redness in the area of joints about a
year ago. Provisional diagnosis is rheumatoid arthritis. One of the likely causes of this disease is
change in the structure of the following connective tissue protein:
A. Mucin
B. Myosin
C. Ovalbumin
D. Collagen
E. Troponin
Ans D
Rheumatoid arthritis (RA) is an autoimmune disease characterized by chronic inflammation of the
joints, which gradually erodes the cartilage and bone ,it is also known as connective tissue disorders,
collagen vascular diseases (CVD) are primarily characterized by connective tissue pathologies.

38. A 15-year-old teenager complains of lack of air, general weakness, palpitations. Heart rate is
130/min., BP is 100/60 mm Hg. ECG: QRS complex has normal shape and duration. The number of P
waves and ventricular complexes is equal, T wave merges with P wave. What type of cardiac
arrhythmia is observed in the teenager?
A. Sinus tachycardia
B. Sinus extrasystole
C. Atrial fibrillation D. Atrial thrill
E. Paroxysmal atrial tachycardia
Ans A

Sinus Tachycardia (ST) is sinus rhythm at a rate equal to or greater than 100 bpm i.e.
Rate: Fast (100 bpm)
Rhythm: Regular
P Waves: Normal (upright and uniform)
PR Interval: Normal (0.12–0.20 sec)
QRS: Normal (0.06–0.10 sec)

Causes:
1 Increased physiologic demand for oxygen
2 Hyperthyroidism, heart failure, myocardial infarction, pulmonary embolism,
3 Physiologic ST commonly observed in neonate

39. A patient complaining of dizziness, thirst, difficult swallowing, and impaired vision of close objects
has addressed a doctor. Objectively: respiratory rate is increased, pupils are dilated, general agitation,
talkativeness, though the speech is indistinct. BP is 110/70 mm Hg, heart rate is 110/min. Given
symptoms can indicate over dosage of the following drug:

A. Morphine
B. Ephedrine
C. Aminazine
D. CaffeinE
E. Atropine

Ans E

Symptoms of atropine overdose include headache; nausea; vomiting; dry mouth; difficulty
swallowing; blurred vision; dilated pupils; hot, dry skin; dizziness; drowsiness; confusion; anxiety;
seizures; weak pulse; and an irregular heartbeat.

40. On autopsy of a 40-year-old woman, who had been suffering from rheumatoid arthritis, her liver is
found to be dense and enlarged. On dissection its tissue is red-brown colored, with enlarged follicles
resembling semi-transparent grayish-white granules. What is the most likely pathological process?
A. Sago spleen
B. Sugar-coated spleen
C. Lardaceous spleen
D. Splenic hyalinosis
E. Porphyry spleen

Ans A
sago spleen one with amyloid infiltration, the malpighian corpuscles looking like grains of
sand.
Spleen. Amyloidosis of the spleen has two different anatomical patterns. Most commonly, the
amyloid deposition is limited to the splenic follicles, resulting in the gross appearance of a moderately
enlarged spleen dotted with gray nodules (so called "sago" spleen).
Alternatively, the amyloid deposits may spare the follicles and mainly infiltrate the red pulp sinuses,
producing a large, firm spleen mottled with waxy discolorations ("lardaceous" spleen) OPTION C

41. On autopsy the dissector determined that the lungs are enlarged, pale, soft, do not deflate, crunch
when cut. Microscopically there are dilated alveolar ducts, alveolar septa are thin, and signs of
intracapillary sclerosis are observed. What pulmonary disorder are these presentations characteristic
of?
A. Pneumosclerosis
B. Emphysema
C. Pneumothorax
D. Atelectasis
E. Pneumonia

Ans B
Emphysema is a disease characterized by dilation of the alveolar spaces and destruction of the
alveolar walls. With their loss, much of the elastic recoil of the lung is also lost.

42. During removal of the hyperplastic thyroid gland of a 47-year-old woman, the parathyroid gland
was damaged. One month after the surgery the patient developed signs of hypoparathyroidism:
frequent convulsions, hyperreflexia, laryngospasm. What is the most likely cause of the patient’s
condition?
A. Hyponatremia
B. Hypocalcemia
C. Hyperchlorhydria
D. Hypophosphatemia
E. Hyperkalemia

Ans B
Parathyroid glands control the calcium levels in our blood, in our bones, and throughout our body.
Parathyroid glands regulate the calcium by producing a hormone called Parathyroid Hormone (PTH)

43. On examination the patient presents with hirsutism, moon-shaped face, stretch marks on the
abdomen. BP is 190/100 mm Hg, blood glucose is 17,6 mmol/l. What pathology is such clinical
presentation characteristic of?
A. Adrenocortical hyperfunction
B. Hyperthyroidism
C. Hypothyroidism
D. Gonadal hypofunction
E. Hyperfunction of the insular apparatus

Ans A
Cushing Syndrome. Adrenal hyperfunction (Cushing syndrome) causes excess cortisol secretion by
adrenal gland
44. Histological specimen of the ovary shows large hollow structures. Primary oocyte within these
structures is surrounded with transparent membrane and radiating crown and is situated in the
cumulus oophorus, the wall is made of follicular cell layer and theca. What ovarian structure can be
characterized by these morphological features?
A. Primordial follicle
B. Mature (tertiary) follicle
C. Primary follicle
D. Сorpus luteum
E. Corpus atreticum

Ans B

45. Histological specimen demonstrates a parenchymal organ with cortical and medullary substances.
The cortical substance is composed of bands of epithelial cells with capillary blood vessels between
them. The bands form three zones. The medullary substance consists of сhromaffin cells and venous
sinusoids. What organ can be characterized by these morphological features?
A. Kidney
B. Lymph node
C. Adrenal gland
D. Thymus
E. Thyroid gland

Ans C

The adrenal medulla is the source of norepinephrine (noradrenalin) and epinephrine (adrenalin).
These secretory cells are also called chromaffin cells because OF the secretory granules. The cells are
often more basophilic than the cells of the cortex. The organization of the tissue and nuclei are also
more distorted. Neuron cell bodies of sympathetic ganglia are also commonly found in the medulla.
Another identifiable characteristic of the adrenal medulla is the large central vein.
The adrenal cortex can be divided into 3 zones, each one named after the organization of the cells
within that zone –
the zona glomerulosa
the zona fasciculate ,
and the zona reticularis

46. A child with suspected coli enteritis was delivered to the infectious diseases hospital. Coli bacillus
was obtained from the child’s feces. How to determine whether this bacillus is of pathogenic variety?
A Based on its biochemical properties
B By means of bacteriophage typing
C Microscopy of stained smears
D Based on the nature of its growth in Endo medium
E Agglutination reaction with serum 0
Ans E
organisms can be isolated from blood, bone marrow, urine, and tissue biopsy from the rose spots if
present; antibodies to O, Vi, and H antigens in patient's serum can be detected by agglutination
(Widal test)

47. A patient undergoes right-sided pulmonectomy due to lung cancer. Name the anatomical
structures of the right lung radix (downward order):
A. Bronchus, artery, veins
B. Artery, bronchus, veins
C. Artery, veins, bronchus
D. Veins, artery, bronchus
E. Veins, bronchus, artery
Ans a

The root is formed by the bronchus, the pulmonary artery, the pulmonary veins.

48. In a township there was registered an outbreak of hepatitis, which was attributed to water supply.
What hepatitis virus could be the cause of the outbreak in this township?
A. Hepatitis E virus
B. Hepatitis C virus
C. Hepatitis D virus
D. Hepatitis G virus
E. Hepatitis B virus
Ans a

49. A 64-year-old woman presents with di- turbed fine motor function of her fingers, marked muscle
rigidity, and tremor. The neurologist diagnosed her with Parkinson’s disease. What brain structures
are damaged resulting in this disease?
A. Substantia nigra
B. Thalamus
C. Red nuclei
D. Cerebellum
E. Reticular formation

Ans a
Parkinson's disease (PD) is a neurodegenerative disorder that affects predominately dopamine-
producing (“dopaminergic”) neurons in a specific area of the brain called substantia nigra. Symptoms
generally develop slowly over years.
50. Due to prolonged taking of a drug the patient can develop osteoporosis, gastric mucosal erosions,
hypokalemia, sodium and water retention, and decreased blood content of corticotropine. Specify
this drug:
A Prednisolone
B. Hydrochlorothiazide
C. Digoxin
D. Indometacin
E. Reserpine
Ans a

Prednisolone is a steroid have major effects on the metabolism of calcium, vitamin D and bone. This
can lead to bone loss, osteoporosis, and broken bones. it increases the loss of potassium in the urine
causing hypokalemia,sodium and water retention and The most common cause of suppression of
ACTH(corticotropine) is the use of glucocorticoid medications to treat a large variety of illnesses

51. Nitrogen is being excreted from the body mainly as urea. When activity of a certain enzyme in the
liver is low, it results in inhibition of urea synthesis and nitrogen accumulation in blood and tissues.
Name this enzyme:
A. Aspartate aminotransferase
B. Urease
C. Amylase
D. Carbamoyl phosphate synthetase
E. Pepsin

Ans D

Carbamoyl phosphate synthetase I deficiency is an autosomal recessive metabolic disorder that


causes ammonia to accumulate in the blood due to a lack of the enzyme carbamoyl phosphate
synthetase I. Ammonia, which is formed when proteins are broken down in the body, is toxic if the
levels become too high. The nervous system is especially sensitive to the effects of excess ammonia.

52. After pancreatic surgery the patient developed hemorrhagic syndrome with disturbed 3rd stage of
blood clotting. What will be the most likely mechanism of the hemostatic disorder?
a. Decrease of prothrombin synthesis
b Fibrinolysis activation
C. Decrease of fibrinogen synthesis
D. Qualitative abnormalities of fibrinogenesis
E. Fibrin-stabilizing factor deficiency

Ans B

primary activation of the fibrinolytic system, most commonly in the setting of SURGERIES, IN THIS
CASE The patient has primary fibrinolysis, which may occurred due to the PANCREATIC surgery. In this
setting, urokinase is released from the tissue causing activation of plasminogen and the release of
plasmin.
Plasmin degrades multiple coagulation factors (e.g., V, VIII, fibrinogen).
This increases the PT and PTT, because factors V and VIII are in the final common pathway.
The degradation products of fibrinogen interfere with platelet aggregation causing an increase in the
bleeding time. The platelet count is not affected.

53. A patient with jaundice has high total bilirubin that is mainly indirect (unconjugated), high
concentration of stercobilin in the feces and urine. The level of direct (conjugated) bilirubin in the
blood plasma is normal. What type of jaundice can be suspected?
A Parenchymal (hepatic)
B. Mechanical
C. Neonatal
D. Gilbert’s disease
E. Hemolytic

ANS E
Pre-hepaticular jaundice is caused by anything which causes an increased rate of hemolysis
(breakdown of red blood cells). Unconjugated bilirubin comes from the breakdown of the heme
pigment found in red blood cells' hemoglobin.

54. Histological specimen shows organ parenchyma to consist of lymphoid tissue that forms lymph
nodules; the nodules are located diffusely and have a central artery. What anatomical structure has
such morphological characteristics?
A Tonsil
B. Lymph node
C. Thymus
D. Spleen
E Red bone marrow

ANS D

The spleen is the largest mass of lymphatic tissue, The spleen is covered by a dense capsule, and there
are connective tissue trabeculae, which provide internal support for the spleen, and carry the blood
vessels into the spleen. IT consist of lymphoid tissue AND A CENTRAL ARTERY

55. During fibergastroscopy of a patient with ulcer disease of the stomach, the mucosal biopsy
material is taken from the area of an ulcer. Impression smear is prepared from the biopsy material
and stained by Gram method; the rest of the biopsy material is tested for urease activity. Microscopy
of the
impression smear revealed gram-negative spiral-shaped microorganisms, urease activity test is
positive. What bacteria were detected?
A. Spirilla minor
B. Helicobacter pylori
C. Shigella flexneri
D. Treponema pallidum
E. Campylobacter jeuni

ANS B
H. pylori also neutralizes the acid in its environment by producing large amounts of urease, which
breaks down the urea present in the stomach to carbon dioxide and ammonia.

56. A patient suffering from gout was prescribed allopurinol. What pharmacological property of
allopurinol provides therapeutic effect in this case?
A. Competitive inhibition of xanthine oxidase
B. Acceleration of nitrogen-containing substances excretion
C. Acceleration of pyrimidine nucleotides catabolism
D. Deceleration of pyrimidine nucleotides salvage
E. Acceleration of nucleic acids synthesis
ANS A
Allopurinol is a XANTHINE OXIDASE inhibitor that decreases URIC ACID production

57. A woman, who has been suffering from marked hypertension for 15 years, has lately developed
dyspnea, palpitations, slightly decreased systolic pressure, while diastolic pressure remains the same.
What is the main mechanism of heart failure development in this case?

A. Cardiac overload due to increased blood volume


B. Damage to the myocardium
C. Cardiac overload due to increased vascular resistance
D. Disorder of impulse conduction in the myocardium
E Dysregulation of cardiac function

ANs C
an increase in systemic resistance results in a rise in diastolic blood pressure

58. Old burial ground for animal refuse, which has not been in use for the last 50 years, is planned to
be given for housing development. However, the soil analysis detected viable spores of a causative
agent of an extremely dangerous disease. What microorganism is the most likely to remain in the soil
for such a long period of time?
A. Bacillus anthracis
B. Francisella tularensis
C. Brucella abortus
D. Yersinia pestis
E. Mycobacterium bovis
ANS A
anthracis is a Gram-positive, endospore-forming, rod-shaped bacterium. Anthrax has the unexpected
ability to grow and reproduce while lurking in soil , spores remain dormant in soil until eaten by
cattle, then germinate and cause the deadly disease

59. Blood of the patients with diabetes mellitus shows increased content of free fatty acids. Name the
most likely cause of this:
A. Increased activity of adipose triglyceride lipase
B. Accumulation of palmitoyl-CoA in cytosol
C. Activation of ketone bodies utilization
D. Activation of apoА1, apoА2, and apoА4 apolipoprotein synthesis
E. Decreased activity of plasma phosphatidylcholine-cholesterol- acyltransferase
ANS A

Adipose triglyceride lipase (ATGL) is a triacylglycerol hydrolase that promotes the catabolism of stored
fat.

60. During autopsy of a man, who had been suffering from mitral stenosis, the lungs are revealed to
be dense and brown-colored. What pathologic process had occurred in the lungs?
A. Hemosiderosis
B. Hemochromatosis C. Jaundice
D. Hemomelanosis
E. Lipofuscinosis
ANS A
iron is seen as a coarse brown hemosiderin pigment deposited

61. Examination of the coronary arteries revealed atherosclerotic plaques with calcinosis that close
the arterial opening by 1/3. In the muscle there are numerous whitish layers of connective tissue.
Name the process detected in the myocardium:
a. Tiger heart
b. Post infarction cardiosclerosis
c. Diffuse cardiosclerosis
D. Myocarditis
E. Myocardial infarction
Ans c

cardiosclerosis Depending on the location and intensity of pathology, isolated focal and diffuse
cardiosclerosis. The first is characterized by the formation of well-defined lesions of various sizes. The
main reasons for their appearance - a myocardial infarction, or any inflammatory processes.
Unlike focal, diffuse cardiosclerosis distributed over the entire surface of the myocardium, i.e., it
equally affects various portions of the heart chamber and has no obvious boundaries. It develops,
usually after the defeat of the heart muscle.
62. T-lymphocytes are determined to be affected with HIV. In this case viral enzyme reverse
transcriptase (RNA-dependent DNA-polymerase) catalyzes the synthesis of:
A. DNA based on the viral RNA matrix
B. Viral RNA based on the DNA matrix
C. Viral protein based on the viral RNA matrix
D. Viral DNA based on the DNA matrix
E. Informational RNA based on the viral protein matrix

Ans A
Reverse transcriptase catalyzes the formation of double-stranded deoxyribonucleic acid (DNA) from a
single-stranded ribonucleic acid (RNA) genome. It is called "reverse" transcriptase because it reverses
the usual direction of information flow, from DNA to RNA. Reverse transcriptase is characteristic of
retroviruses, including HIV (human immunodeficiency virus), the virus responsible for AIDS (acquired
immunodeficiency syndrome)

63. A woman with dense area in her mammary gland came to a surgeon. To minimize the trauma to
the mammary gland lobule during the operation the surgeon should make the incision:
A. Vertically
B. Transversely
C. Arcuately
D. Radially
E. –
ANS D

Radically because It is important to place the incision in a position that will obtain the optimal
cosmetic result

64. Electrical activity of neurons is being measured. They fire prior to and at the beginning of
inhalation. Where are these neurons situated?
A. Diencephalon
B. Mesencephalon
C. Spinal cord
D. Cerebral cortex
E. Medulla oblongata
Ans E

The core neuronal network responsible for the generation and shaping of the respiratory rhythm, as
well as transmitting this rhythm to the spinal motor neurons controlling respiratory muscles, is
located in the lower brain stem – within a bilaterally organized dorsal respiratory group and ventral
respiratory column (VRC) of neurons in the pons and the medulla oblongata
65. Investigation of an isolated cardiac myocyte determined that it does not generate excitation
impulses automatically, which means this cardiac myocyte was obtained from the following cardiac
structure:
A. Sinoatrial node
B. Ventricles
C. Atrioventricular node
D. His’ bundle
E. Purkinje’s fibers
ANS B

Automaticity is the property of cardiac cells to generate spontaneous action potentials The sinoatrial
(SA) node normally displays the highest intrinsic rate. Atrial and ventricular myocardial cells do not
display spontaneous diastolic depolarization or automaticity under normal conditions

66. To lose some weight a woman has been limiting the amount of products in her diet. 3 months
later she developed edemas and her diuresis increased. What dietary component deficiency is the
cause of this?
A. Fats
B. Carbohydrates
C. Vitamins
D. Minerals
E. Proteins
ANS E

Low protein levels in the blood caused by malnutrition, kidney and liver disease can cause edema. The
proteins help to hold salt and water inside the blood vessels so fluid does not leak out into the tissues.
If a blood protein, called albumin, gets too low, fluid is retained and edema

67. The patients with organic brain disorder can take the following drug to improve their memory:
A Nitrazepam
B. Medazepam
C Diazepam
D. Caffeine
E. Piracetam

ANS E

Piracetam was the first nootropic discovered, it has been proven to act as a powerful memory
enhancer

68. A 40-year-old man developed skin redness and swelling in the neck area, where eventually a small
abscess appeared. On section the focus is dense and yellow- green colored. In the purulent masses
there are white granules. Histologically there are fungal druses, plasma and xanthome cells, and
macrophages detected. Specify the most correct etiological name of this pathological process:
A. Actinomycosis
B. Furuncle
C. Carbuncle
D. Syphilis
E. Leprosy
Ans A

A xanthoma (xanthos), meaning 'yellow', is a deposition of yellowish cholesterol-rich material that can
appear anywhere in the body it can be caused by actinomycosis

69. In preparation for business trip abroad the doctor was prescribed a histoschizontocidal
antimalarial drug as a personal means of disease prevention. What drug was given to the doctor?
A. Biseptol (Co-trimoxazole)
B Doxycycline
C Mefloquine
D Chingamin
E. Quinine

ANS D

Chingamin is an antimalarial agent. While the drug can inhibit certain enzymes, its effect is believed to
result, at least in part from its interaction with DNA. However, the mechanism of plasmodicidal action
of Chingamin is not completely certain.

70. A patient, who has been suffering from bronchial asthma for a long time, developed acute
respiratory failure. What is the main mechanism of pathology development in this case?
A. Restrictive disorders of pulmonary ventilation
B Obstructive disorders of pulmonary ventilation
C. Pulmonary blood supply disturbance
D. Pulmonary enzyme system disturbance
E. Decreased elasticity of the pulmonary tissue
ANS B

Asthma - It is generally characterized by inflamed and easily collapsible airways, obstruction to


airflow, problems exhaling

71. A 40-year-old woman has undergone thyroidectomy. Histological study of thyroid gland found the
follicles to be of different size and contain foamy colloid, follicle epithelium is high and forms papillae,
there is focal lymphocytic infiltration in the stroma. Diagnose the thyroid gland disease:
A. Basedow’s disease
B. Hashimoto’s thyroiditis
C. Riedel’s thyroiditis
D. De Quervain’s disease
E. Nodular goiter
ANS A

In basedow disease there is presence of increased numbers of lymphoid aggregates, including some
lymphoid follicles ,foamy colloid

72. A patient has been hospitalized with provisional diagnosis of virus B hepatitis. Serological reaction
based on complementation of antigen with antibody chemically bound to peroxidase or alkaline
phosphatase has been used for disease diagnostics. What is the name of the applied serological
reaction?
A.Radioimmunoassay technique
B Enzyme-linked immunosorbent assay
C. Immunofluorescence test
D. Complement fixation test
E. Immobilization test
ANS B

Enzyme-linked immunosorbent assay (ELISA), also known as an enzyme immunoassay (EIA), is a


biochemical technique used mainly in immunology to detect the presence of an antibody or an
antigen in a sample.it uses the enzyme peroxidase or alkaline phosphatase

73. Due to blood loss the circulating blood volume of a patient decreased. How will it affect the blood
pressure in this patient?
A. Only systolic pressure will decrease
B Systolic and diastolic pressure will decrease
C. Only diastolic pressure will decrease
D. Systolic pressure will decrease, while diastolic will increase
E. Diastolic pressure will decrease, while systolic will increase
ANS B

Hypovolemia due to blood loss will cause a decrease in both systolic and diastolic blood pressure.

74. A bacteriological laboratory tests canned meat for botulinum toxin. Extract of the tested material
and ABE botulinum antitoxin serum was introduced into the test group of mice; the control group of
mice received the extract without anti botulinic serum. What serological reaction was used?
A. Neutralization
B. Precipitation
C. Complement binding
D. Opsonophagocytic
E. Double immunodiffusion
Ans a
Toxins are determined by neutralization of the toxin with specific antitoxins
75. A 50-year-old man is diagnosed with ischemic heart disease and cardiosclerosis with hypertensive
syndrome. What drug should be prescribed in this case?
A. Metoprolol
B. Acetylsalicylic acid
C. Corglycon
D. Strophanthine
E. Potassium chloride

Ans A
Metoprolol belongs to a class of drugs known as beta blockers. It works by blocking the action of
certain natural chemicals in your body, such as epinephrine, on the heart and blood vessels. This
effect lowers the heart rate, blood pressure, and strain on the heart

76. Poisoning caused by mercury (II) chloride (corrosive sublimate) occurred in the
result of safety rules violation. In 2 days the patient’s diurnal diuresis became 620 ml. The patient
developed headache, vomiting, convulsions, dyspnea; moist crackles are observed in the lungs. Name
this pathology:
A. Acute renal failure
B. Chronic renal failure
C. Uremic coma
D. Glomerulonephritis
E. Pyelonephritis
Ans a
Acute renal failure is a sudden episode of kidney failure or kidney damage that happens within a few
hours or a few days. AKI causes a build-up of waste products in your blood and makes it hard for your
kidneys to keep the right balance of fluid in your body

77. Autopsy of a 9-yer-old child shows numerous irregular defects of varying depth with uneven
margins and gray-white films tightly attached to the underlying tissue on the rectal mucosa of the
body. What disease can be suspected?
A. Dysentery
B. Salmonellosis
C. Cholera
D. Typhoid fever
E. Amebiasis

ANS A

78. A man presents with glomerular filtration rate of 180 ml/min., while norm is 125±25 ml/min. The
likely cause of it is the decreased:
A. Plasma oncotic pressure
B. Effective filtration pressure
C. Hydrostatic blood pressure in the glomerular capillaries
D. Renal blood flow
E. Permeability of the renal filter
Ans A
oncotic pressure pulls fluid into the capillaries. In conditions where plasma proteins are reduced, e.g.
from being lost in the urine (proteinuria) or from malnutrition, there will be a reduction in oncotic
pressure and an increase in filtration across the capillary

79. A young man came to a hospital with complaints of disturbed urination. Examination of his
external genitalia revealed the urethra to be split on the top, with urine flowing out of this opening.
What type
of external genitalia maldevelopment is observed in this case?
A. Epispadia
B. Phimosis
C. Hermaphroditism
D. Paraphimosis
E. Hypospadia

ANS A

Hypospadias is a congenital defect, primarily of males, in which the urethra opens on the underside
(ventrum) of the penis.
Epispadias (also called bladder exstrophy) is a congenital defect of males in which the urethra opens
on the upper surface (dorsum) of the penis.

80. A 10-year-old child had cut his leg with a glass shard, when playing, and was delivered to the
outpatient department to receive anti-tetanus serum. To prevent development of anaphylactic shock
the serum was introduced by Bezredka method. This method of organism hyposensitization is based
on the following mechanism:

A. Blocking of mast cell mediators synthesis


B. Stimulation of immune tolerance to antigen
C. Binding of mast cell-fixed Ig E
D. Stimulation of antigen-specific IgG2
E. Stabilization of mast cell membranes

ANS C
In the immunologic mechanism of anaphylatic shock , immunoglobulin E (IgE) binds to the antigen
(the foreign material that provokes the allergic reaction). Bezredki method – the method of
desensitization, necessarily used to prevent anaphylactic reactions
81. A 38-year-old man, who has been suffering from systemic lupus erythematosus for 3 years,
developed diffuse renal lesions accompanied by massive edemas, marked proteinuria, hyperlipidemia,
and dysproteinemia. What is the most likely mechanism of proteinuria develoment in this case?
A. Autoimmune damage to the nephrons
B. Inflammatory damage to the nephrons
C. Ischemic damage to the tubules
D. Increased blood proteins
E. Morbid affection of the urinary tracts
ANS A
Systemic lupus erythematosus (SLE) is an autoimmune disease characterized by a breakdown of self-
tolerance, production of auto-antibodies and immune-mediated injury, resulting in damage accrual in
multiple organs. Kidney involvement, termed lupus nephritis nephrons are damaged leading to edema
due to loss of proteins (Albumin) prevents the fluid from leaking out and accumulating in the tissues.

82. During experiment a part of the brain was extracted, which resulted in asynergy and dysmetria
development in the test animal. What part of the brain was extracted in the animal?
A. Frontal lobe
B. Parietal lobe
C. Mesencephalon
D. Cerebellum
E. Reticulum

Ans D

Pathology of the cerebellum or its pathways typically results in an impairment of coordination


(asynergy, ataxia), misjudgment of distance (dysmetria) , and intention tremor

83. A woman with polyarticular rheumatoid arthritis was prescribed a non-steroidal anti-
inflammatory drug - diclofenac sodium. After the patient has been taking it for some time, her
concomitant disease exacerbated, which forced the doctor to cancel the prescription of this drug.
What concomitant disease could necessitate cancellation of this drug prescription?
A Ischemic heart disease
B. Diabetes mellitus
C Essential hypertension
D. Bronchial asthma
E. Ulcer disease

ANS E
a non-steroidal anti-inflammatory drug Diclofenac is a potent inhibitor of prostaglandin,
prostaglandins protect the gastric mucosa against the erosive action of ulcerogenic drugs, but in this
case any drug that can inhibit prostaglandin will be causing ulcer.
84. Histologic preparation stained with orcein demonstrates from 40 to 60 fenestrated elastic
membranes within the middle coat of the vessel. Name this vessel:
A. Elastic artery
B. Muscular artery
C. Mixed type artery
D. Muscular vein
E. Nonmuscular vein
ANS A

Elastic arteries are those adapted to receive blood at moderate to high pressure near to the outflow
tracts of the heart.
Their walls consist of:
- tunica intima
- tunica media:
thickest layer
at least 50 sheets of interconnected elastin fibres arranged in a concentric, coiled manner
between each sheet is a layer of smooth muscle cells
spiral around wall of vessel, each cell connected to other smooth muscle cells via gap junctions
secrete elastin and collagen fibres (types I,III and IV
- tunica adventitia

85. A woman with the III (В), Rh (-) blood group gave birth to a child with the II (А)
blood group. The child is diagnosed with hemolytic disease of newborn caused by rhesus
incompatibility. What blood group and Rh are likely in the father?
A. II (А), Rh (+)
B. I (0), Rh (+)
C. III (B), Rh (+)
D. I (0), Rh (-)
E. II (A), Rh (-)
ANS A
Rh incompatibility develops only when the mother is Rh-negative , father Rh-positive and the infant
is Rh-positive

86. A 40-year-old woman with Cushing’s disease presents with steroid diabetes. On biochemical
examination she has hyperglycemia and hypochloremia. What process activates in the first place in
such patients?
A. Glycogenolysis
B. Glucose reabsorption
C. Glucose transportation into a cell
D. Glycolysis
E. Gluconeogenesis
Ans E
Gluconeogenesis is a metabolic pathway that results in the generation of glucose from non-
carbohydrate carbon substrates
Cushing's disease is used exclusively to describe the condition of excessive cortisol arising from a
pituitary tumor secreting the hormone ACTH.
Hyperglycemia BECAUSE cortisol is gluconeogenic

87. A 40-year-old prisoner died of tuberculosis in the corrective labor camp. Autopsy of the body
revealed deformation and diminishing of both lung apices; in the both upper lobes there are multiple
cavities with dense walls 2-3 mm thick; in the lower lung lobes there are disseminated foci of caseous
necrosis varying from 5 mm to 2 cm in diameter. Diagnose the type of tuberculosis:

A. Secondary fibro-cavitary tuberculosis


B. Secondary fibrous-focal tuberculosis
C. Hematogenous macro focal pulmonary tuberculosis
D. Primary tuberculosis, primary affect development
E. Secondary cirrhotic tuberculosis

Ans A
Fibrous-cavernous pulmonary tuberculosis. is a secondary form of tuberculosis, the substrate is a
cavity (or more) with hard walls and massive fibrosis in adjacent tissue

88 Postmortem study revealed the following: macroscopically there were acutely enlarged various
groups of the lymph nodes, thymomegaly, hepatosplenomegaly, and bright red bone marrow;
microscopically there was hypercellular bone marrow with monomorphic infiltrations composed of
blasts and diffuse-focal tumor infiltrations in the liver, spleen, lymph nodes, brain substance and
tunics. Make the diagnosis:

A. Acute lymphoblastic leukemia


B. Acute myeloblastic leukemia
C. Acute undifferentiated leukemia
D. Acute monoblastic leukemia
E. Acute plasmablastic leukemia
ANS A

There are 4 main types of leukemia, based on whether they are acute or chronic, and myeloid or
lymphocytic:( PRESENCE OF BLAST MAKES IT ACUTE)

-Acute myeloid (or myelogenous) leukemia (AML) -presence of BLAST AND AUER RODS
-Chronic myeloid (or myelogenous) leukemia (CML) -NO BLAST
-Acute lymphocytic (or lymphoblastic) leukemia (ALL)- presence BLAST , WITHOUT AUER RODS
-Chronic lymphocytic leukemia (CLL) – NO BLAST
89. Autopsy of a man, who served on a nuclear submarine, revealed the following pathologies: bone
marrow atrophy (panmyelophthisis), anemia, leukopenia, thrombocytopenia, lymphocytes
disintegration in the lymph nodes, spleen, gastrointestinal lymphatic system, and hemorrhages into
the adrenal glands. What disease had developed in this case?

A. Decompression sickness
B. Acute leukemia
C. Acute radiation sickness
D. Acute anemia
E. Vibration disease

Ans C

Acute radiation syndrome or acute radiation sickness is classically subdivided into three
subsyndromes: the hematopoietic, gastrointestinal and neurovascular syndrome but many other
tissues can be damaged.

90. After sensitization a test animal received subcutaneously a dose of antigen. At the site of injection
a fibrinous inflammation developed with alteration of vessel walls, basal substance, and fibrous
structures of connective tissue. The inflammation took form of mucoid and fibrinoid degeneration,
fibrinoid necrosis. What immune response occurred in the test animal?

A. Delayed hypersensitivity
B. Immediate hypersensitivity
C. Transplantation immune reaction
D. Normergic reaction
E. Granulomatosis

Ans B
In type 1 hypersensitivity, B-cells are stimulated (by CD4+TH2 cells) to produce IgE antibodies specific
to an antigen. The immediate hypersensitivity reaction occurs minutes after exposure and includes
release of vasoactive amines(histamine) and lipid mediator. Histamine cause bronchoconstriction,
mucus secretion, vasodilatation, vascular permeability

91. Preoperative examination revealed prothrombin deficiency in the blood of the patient. What drug
should be preliminarily prescribed to mitigate blood loss in the patient during the surgery?

A Thrombin
B Vicasol (Menadione)
C. Aminocapronic acid
D. Phenylin (Phenindione)
E. Contrykal (Aprotinin)
ANS B
Vitamin K(VICASOL) is an essential fat soluble vitamin that is important in maintaining normal
coagulation, serving as a cofactor in the activation of several clotting factors and anticoagulant
proteins

92. A patient complaining of pain in the left shoulder-blade region has been diagnosed with
miocardial infarction. What kind of pain does the patient have?

A. Radiating
B. Visceral
C. Phantom
D. Protopathic
E. Epicritic

Ans A
If you have radiating pain that means that your pain begins in one place and travels to another
location along the path of a nerve.

93. During regular check-up a child is determined to have interrupted mineralization of the bones.
What vitamin deficiency can be the cause?

A. Riboflavin
B. Tocopherol
C. Folic acid
D. Cobalamin
E. Calciferol

Ans E
Low blood calcifediol (25-hydroxy-vitamin D) can result from avoiding the sun. Deficiency results in
impaired bone mineralization and bone damage which leads to bone-softening diseases, including
rickets and osteomalacia.

94. On examination the patient was determined to have strong, balanced, inert type of higher nervous
activity according to Pavlov’s classification. What temperament according to Hippocrates is it?

A. Phlegmatic
B. Sanguine
C. Choleric
D. Melancholic
E. -

95. A specimen shows an organ covered with connective tissue capsule with trabeculae radiating
inward the organ. The organ’s cortex contains lymph nodules; there are medullary cords made of
lymphoid cells. What organ is under study?
A. Lymph node
B. Thymus
C. Spleen
D. Red bone marrow
E. Tonsils
ANS A

The capsule and trabeculae of lymph nodes are formed by connective tissue cortex has lymphatic
nodules that mostly contain B-cells. medullary cords contain B-cells and plasma cells

96. Brain autopsy revealed an edema, hyperemia, and small hemorrhages in the medulla oblongata.
Microscopically chromatolysis, hydropia and nerve cell necrosis are observed; within the cytoplasm of
hippocampal nerve cells there are eosinophilic structures (Negri bodies) detected. What diagnosis
corresponds with the described morphological signs?

A. Meningococcal meningitis
B. Encephalitis
C. Encephalomyelitis
D. Rabies
E. Brucellosis
Ans D

The Negri bodies, as the etiological agent in rabies,are eosinophilic, sharply outlined, pathognomonic
inclusion bodies (2–10 µm in diameter) found in the cytoplasm of certain nerve cells containing the
virus of rabies, especially in pyramidal cells within Ammon's horn of the hippocampus.

97. A married couple came for a genetic counseling. The husband suffers from insulin-independent
diabetes mellitus, while the wife is healthy. What is the probability of their child developing insulin-
independent diabetes mellitus?
A. Higher than in the population
B. The same as in the population
C. Lower than in the population
D. 100%
E. 50%
ANS A
98. A 46-year-old patient suffering from ulcer disease of the stomach is diagnosed with rheumatoid
arthritis. What anti inflammatory drug would be the most advisable in this case?
A. Prednisolone
B. Celecoxib
C. Analgin (Metamizole)
D. Promedol (Trimeperidine)
E. Paracetamol
Ans B

A highly selective reversible inhibitor of the COX-2 isoform of cyclooxygenase, celecoxib inhibits the
transformation of arachidonic acid to prostaglandin precursors. Therefore, it has antipyretic, analgesic
and anti-inflammatory properties

99. Electron micrograph of the kidney shows fenestrated endothelium lying on the basement
membrane; the external surface of the membrane has adjacent dendritic epithelial cells. What do
these structures form in the kidney?

A. Juxtaglomerular apparatus
B. Filtration barrier
C. Distal nephron
D. Henle’s loop
E. Proximal nephron

Ans B
The kidney's glomerular filtration barrier consists of two cells—podocytes and endothelial cells and
the glomerular basement membrane (GBM), a specialized extracellular matrix that lies between them

100. A worker of an agricultural enterprise had been suffering from an acute disease with aggravating
intoxication signs, which resulted in his death. On autopsy: the spleen is enlarged, flaccid, dark cherry-
red on section, yields excessive pulp scrape. Soft meninges of the fornix and base of the brain are
edematous and saturated with blood (cardinal’s cap”). Microscopically: serous-hemorrhagic
inflammation of meninges and cerebral tissues. Make the diagnosis:

A. Anthrax
B. Tularemia
C. Plague
D. Cholera
E. Brucellosis

Ans a

cardinal's cap is a postmortem appearance unique to internal anthrax in which the proliferating
bacteria have caused the blood vessels in the brain to rupture, spreading veil of blood over the
cerebrum.

101. A woman suffers from tonsillitis complicated with retropharyngeal abscess that is localized in the
spatium retroviscerale. In this case the suppurative process can spread to the:

A. Mediastinum posterius
B. Spatium interaponeuroticum suprasternale
C. Mediastinum anterius
D. Spatium pretracheale
E. Spatium interscalenum
ANS A

102. A patient with chronic bronchitis was prescribed a drug with mucolytic action. Name this drug:
A. Anaprilin (Propranolol)
B. Atropine sulfate
C. Magnesium sulfate
D. Paracetamol
E. Ambroxol
ANS E

Ambroxol hydrochloride is a medication indicated to alleviate chest congestion associated with


conditions that include bronchitis, pneumonia and bronchospasm asthma. This medication is part of a
class of drugs called mucolytics and works by thinning chest mucus in affected patients.

103. A patient suffers from acute cardiopulmonary failure with pulmonary edema. What diuretic
should be prescribed in the given case?
A. Furosemide
B. Triamterene
C. Spironolactone
D. Dichlothiazidum (Hydrochlorothiazide)
E. Diacarb (Acetazolamide)
ANS A
Diuretics (Furosemide) Mechanism of action
• Highly potent loop diuretics, inhibits Na+-K+-Cl- symport in Loop of Henle of nephrons
Pharmacological action
• diuresis action + venodilation action + decrease in LV filling pressure
• Can cause salt and water excretion & reduction of preload
• benefits patients with pulmonary edema even before diuresis ensues
Route of administration: available as oral as well as IV.

104. Therapeutics unit of a hospital received a man suffering from ulcer disease of the stomach with
hyperacidity. Which of the listed group of drugs must be used as a part of the complex therapy of this
patient?

A. Histamine H 2 -receptor antagonists

B. Calcium channel blockers

C. Nonsteroidal antiinflammatory drugs


D. Steroidal antiinflammatory drugs

E. Histamine H 1 -receptor antagonists


AnS A

Histamine H 2 -receptor antagonists example


- Cimetidine, ranitidine, Famotidine, nizatidine
Reversible block of histamine H2 receptors in stomach. Leads to decreased acid secretion by parietal
cells.

105. A 45-year-old woman gave birth to a boy with cleft maxilla (cleft lip and palate). On additional
examination there are significant disturbances of the boy’s nervous, cardiovascular, and visual
systems. Karyotype investigation allowed diagnosing the patient with trisomy 13. What syndrome is
present in the boy?
A. Down
B. Kleinfelter
C. Turner
D. DiGeorge
E. Patau
ANS E

106. Pathologic material (mucosal excretion from the nasal passages) obtained from a patient
provisionally diagnosed with influenza was delivered to the virological laboratory. What quick test
allows detecting specific viral antigen in the investigated material?
A. Direct and indirect enzyme-linked immunosorbent assay (ELISA)
B. Direct and indirect immunofluorescence (IF)
C. Hemagglutination inhibition assay (HAI)
D. Reverse indirect haemagglutination (RIHA)
E. Radioimmunoassay (RIA)
ANS B

Serology - Immunofluorescence assay (IFA) IFA to detect specific antibody, antigen or virus offers a
simple, convenient and quick test, but is not quantitative

107. A 5-year-old child is diagnosed with Bruton syndrome (X-linked agammaglobulinemia) that
manifests itself in severe clinical course of bacterial infections and absence of B lymphocytes and
plasma cells. What changes of immunoglobulin content can be observed in blood serum of the child
with immunodeficiency

A. Decreased IgA, IgM

B. Increased IgA, IgM

C. Decreased IgD, IgE

D. Increased IgD, IgE

E. No changes
ANS A

Primary humoral immunodeficiency characterized by decreased immunoglobulins


-Genetics
X-linked recessive
-seen in male children
Pathogenesis
defect in Bruton tyrosine kinase (BTK)
defective maturation of B-cells
impaired signaling from pre-B-cell receptor
↓ B-cells
↓ production of all classes of Ig
impaired antibody immune response
 Perform initial studies measuring quantitative IgG, IgM, immunoglobulin E (IgE), and
immunoglobulin A (IgA). IgG levels below 100 mg/dL are usually indicative of X-linked
agammaglobulinemia (XLA). The detection of IgG, IgA, IgM, and IgE levels is related to age.
Typically, IgM and IgA are undetectable. All levels are reduced in males with XLA. Age-specific
reference range values are available to compare with the patient's level.

108. A 37-year-old man, who was working in a caisson, after being lifted to the surface suddenly
developed signs of acute cerebral circulation disturbance and loss of consciousness. Several days later
he died. On autopsy in the left cerebral hemisphere there was detected a gray soft irregular focus
5х6х3,5 cm in size. What process had occurred in the brain?

A. Ischemic stroke
B. Hemorrhagic infarction
C. Abscess
D. Cyst
E. Tumor

109. A short-term physical load resulted in reflex increase of heart rate and systemic arterial pressure
in a person. What receptor activation was the most contributory to inducing the pressor reflex?

A. Proprioceptors of the working muscles


B. Vascular chemoreceptors
C. Vascular volume receptors
D. Vascular baroreceptors
E. Hypothalamic thermoreceptors
ANS A

stimulation of proprioceptors in joints causes hyperpnea Respiratory—hyperpnea Cardiovascular—


increased heart rate, vasoconstriction, etc

110. During experiment a skeletal muscle is being stimulated with a series of electrical impulses. What
type of muscular contraction will develop, if each following impulse occurs within the relaxation
period after the previous single contraction of the muscle?

A. Smooth tetanus
B. Series of single contractions
C. Muscle contracture
D. Incomplete tetanus
E. Asynchronous tetanus

ANS D
Incomplete tetanus has a relaxation period during contractions
Complete tetanus has no relaxation period during contractions
111. Depression and emotional disturbances result from the lack of noradrenaline, serotonin, and
other biogenic amines in the brain. Their content in the synapses can be increased through
administration of antidepressants that inhibit the following enzyme:
A. Sympathetic unconditioned reflex
B. Sympathetic conditioned reflex
C. Metasympathetic reflex
D. Parasympathetic unconditioned reflex
E. Parasympathetic conditioned reflex
ANS A

112. A 2-year-old child presents with acute psychomotor retardation, vision and hearing impairment,
sharp enlargement of the liver and spleen. The child is diagnosed with hereditary Niemann-Pick
disease. What genetic defect is the cause of this disease?

A. Sphingomyelinase deficiency
B. Glucose 6-phosphatase deficiency
C. Amylo-1,6-glucosidase deficiency
D. Acid lipase deficiency
E. Xanthine oxidase deficiency
ANS A
Niemann-Pick disease IS a lipid storage disorder due to sphingomyelinase deficiency
Symptoms
-failure to thrive
-neurologic symptoms from demyelination
Physical exam
-hepatosplenomegaly
-cherry red pot on macula

Labs
residual acid sphingomyelinase activity is <10% of controls via
peripheral blood leukocytes or cultured skin fibroblasts

113. Clinical presentations of a woman allowed provisionally diagnosing her with X polysomy.
Cytogenetic method is applied to clarify the diagnosis. The diagnosis will be confirmed if the patient’s
karyotype is:

A. 48, XXXY
B. 47, ХХХ
C. 48, XXYY
D. 47, XXY
E. 46, XX
ANS B

characterized by addition of one extra X chromosome to a male 46, XY karyotype. Consistent clinical
features include increased mean height and infertility. The extra X chromosome results from non-
disjunction of sex-chromosomes during the first (or less frequently the second) meiotic division in
either parent.

114. During dehelmintization there was a 3,5-meter-long tapeworm produced from the patient’s
intestine. There are 4 suckers and hooks on the tapeworm’s scolex. Mature segments of the
tapeworm are immobile and have up to 12 uterine branches. What disease is it?

A. Echinococcosis
B. Beef tapeworm infection
C. Diphyllobothriasis
D. Teniasis
E. Opisthorchiasis
ANS D
The scolex is usually covered with hooks or suckers and is used by helminth taxonomists as a key
characteristic for identifying species of tapeworms. The pork tapeworm, Taenia solium has a scolex

115. A patient presents with steatorrhea. This disorder can be linked to disturbed supply of the
intestine with the following substances:

A. Bile acids
B. Carbohydrates
C. Tripsin
D. Chymotrypsin
E. Amylase

ANS A
Bile acids support the emulsification of triglycerides and form micelles with fatty acids and
monoglycerides to enable absorption from the intestinal lumen (compare below). Thus, decreased
luminal availability may result in or contribute to steatorrhea which is the excretion of abnormal
quantities of fat with the faeces owing to reduced absorption of fat by the intestine.

116. Cytochrome oxidase is a hemeprotein that is an end component of the mitochondrial respiratory
chain. What reaction is catalyzed with this enzyme?

A. Cytochrome synthesis
B. Transfer of reduced equivalents to ubiquinone
C. Transfer of reduced equivalents to molecular oxygen
D. Cytochrome splicing
E. Adenosine triphosphate synthesis

ANS C
Cytochrome c oxidase is the last enzyme in the respiratory electron transport chain of mitochondria.
Its main function is to convert molecular oxygen to water and aid in establishing mitochondrial
membrane potential. Cytochrome c oxidase locates to the inner membrane which separates the
mitochondrial matrix from the intermembrane space

117. A 16-year-old girl fainted when she tried to quickly change her position from horizontal to
vertical. What caused the loss of consciousness in the girl?

A. Increased venous return


B. Decreased venous return
C. Increased central venous pressure
D. Decreased oncotic plasma pressure
E. Increased arterial pressure

ANS B
Sudden posture change from a supine to an upright posture creates a strong vertical gradient of
gravitational pull on the fluid (blood) inour circulatory column.
There are several things that occur in your body when you change from a supine to a standing position
quickly:
(1) there is a decrease in venous return of blood to the heart. Blood pools in the elastic-walled
venous vessels due to the sudden increased gravitational pull on blood within your lower limbs;
(2) there is a decrease in cardiac blood volume
(end diastolic volume) due to the decreased venous return; and
(3) there is a decrease in arterial blood volume and
blood pressure within your head and neck. These changes are sometimes accompanied by feelings of
dizziness or lightheadedness, and might even lead to fainting (syncope).

118. An unconscious patient was delivered by ambulance to the hospital. On objective examination
the patient was found to have no reflexes, periodical convulsions, irregular breathing. After laboratory
examination the patient was diagnosed with hepatic coma. Disorders of the central nervous system
develop due to the accumulation of the following metabolite:

A. Urea
B. Glutamine
C. Bilirubin
D. Ammonia
E. Histamine
ANS D
The liver produces the enzymes involved in the urea cycle, which converts ammonia to urea. Elevated
ammonia levels can result in hepatic encephalopathy
The neurologic symptoms accompanying severe liver disease have been termed hepatic coma. This
syndrome is characterized by personality changes, inappropriate behavior, stupor, and eventually
coma. The most frequent biochemical abnormality is an elevation of the blood ammonia. This results
from the breakdown of nitrogenous products

119. A patient complains of acute pain attacks in the right lumbar region. During examination the
nephrolithic obturation of the right ureter in the region between its abdominal and pelvic segments
has been detected. What anatomical boundary exists between those two segments?

A. Linea terminalis
B. Linea semilunaris
C. Linea arcuata
D. Linea transversa
E. Linea inguinalis

ANS A
The linea terminalis or innominate line consists of the pectineal line, the arcuate line, the pubic crest,
the sacral ala, and the sacral promontory. It is the pelvic brim, which is the edge of the pelvic inlet

120. After mushroom poisoning the patient developed signs of acute hepatic failure leading to his
death. On autopsy the liver is diminished, flaccid; the capsule is wrinkled; the tissue is ochre-yellow on
section. Microscopically: fatty degeneration of hepatocytes, necrotic central segments of the hepatic
lobes. These changes are characteristic of:

A. Massive progressive necrosis


B. Fatty hepatosis
C. Acute exudative hepatitis
D. Acute productive hepatitis
E. Hepatolenticular degeneration
Ans a

121. Blood test of an athlete shows the following:


erythrocytes - 5, 5 · 10 12 /l, hemoglobin - 180 g/l, leukocytes - 7 · 10 9 /l, neutrophils - 64%, basophils
- 0,5%, eosinophils - 0,5%, monocytes - 8%, lymphocytes - 27%. These values primarily indicate the
stimulation of:

A. Leukopoiesis
B. Lymphopoiesis
C. Erythropoiesis
D. Granulocytopoiesis
E. Immunogenesis
ANS C
ERYTHROPOIESIS is the process which produces red blood cells (erythrocytes). It is stimulated by
decreased O2 in circulation, which is detected by the kidneys, which then secrete the hormone
erythropoietin.

122. Experimental stimulation of the sympathetic nerve branches that innervate the heart caused an
increase in the force of heart contractions because the membrane of typical cardiomyocytes
permitted an increase in:

A. Calcium ion exit


B. Calcium ion entry
C. Potassium ion exit
D. Potassium ion entry
E. Calcium and potassium ion exit
ANS B
excitation contraction (EC) coupling in cardiac myocytes is mediated by the entry of calcium ions
(Ca2+) from the bathing medium into the cell cytoplasm.
action potential subsequently triggers muscle contraction by increasing the concentration of calcium.

123. As a result of past encephalitis, a man has developed an increase in cerebrospinal fluid pressure
in the right lateral ventricle. What can be the cause of this condition?

A. Closure of the right interventricular foramen

B. Closure of the left interventricular foramen

C. Atresia of the tubus medullaris

D. Atresia of the sylvian aqueduct

E. Atresia of the fourth ventricle foramina


ANS A
CEREBOSPINAL FLUID CIRCULATE THROUG THE VENTRICLES ASS FOLLOWS:
FROM LATERAL VENTRICULE through interventricular foramen into 3RD VENTRICLE through
CEREBRAL AQUEDUCT OF SYLIVUS to reach 4TH VENTYRICLE and out of the FORAMEN LUSCHKA and
FORAMEN MAGENDIE TO SUBACHNOID SPACE where it is absorbed through arachnoid villi into dural
sinus,where it mixes with the venous blood. THEREFORE BLOCKAGE OF RIGHT INTRAVENTRICULAR
FORAMEM WILL INCREASE PRESURE IN RIGHT LATERAL VENTRICULR

124. A patient with pneumonia has body temperature of 39,2 o C . What cells are the main producers
of endogenous pyrogen that had caused such temperature rise?

A. Eosinophils
B. Monocytes
C. Neutrophils
D. Endotheliocytes
E. Fibroblasts
ANS B
pyrogen. the mediator of fever, produced by polymorphonuclear leukocytes, monocytes and
macrophages

125. A patient is diagnosed with compression fracture of the lumbar vertebra. The patient presents
with acutely increased lumbar lordosis. What ligament was damaged in this patient resulting in such
deformation of vertebral column curvature?

A. Anterior longitudinal ligament

B. Posterior longitudinal ligament

C. Yellow ligament

D. Iliolumbar ligament
E. Interspinal ligament

126. Gram-positive spore-forming bacilli were extracted in anoxic environment from the patient’s
wound contaminated with soil. Cultivation on a blood-glucose agar resulted in growth of the colonies
surrounded with hemolysis zone. What agent was extracted from the wound?
A. Staphylococcus aureus
B. Clostridium botulinum
C. Escherichia coli
D. Сlostridium perfringens
E. Pseudomonas aeruginosa

ANS D
Сlostridium perfringens Colony morphology on Sheep Blood agar:
- Large, smooth, regular, convex, slightly opaque, flat radially striated transparent border
- Can be rough flat vine leaf colony
-Typical Double zone of haemolysis:
-Zone of complete haemolysis due to Beta haemolytic Theta toxin (one of the minor toxins)
-Wider zone of partial haemolysis caused by Alpha haemolytic Alpha toxin (one of the major toxins)

127. The patient’s ECG shows that in the second standard lead from the extremities the P waves are
positive, their amplitude is 0,1 mV (norm is 0,05-0,25 mV), duration 0,1 seconds (norm is 0,07-0,10
seconds). It can be concluded that the following process occurs normally in the cardiac atria:

A. Depolarization
B. Repolarization
C. Activation
D. Contraction
E. Relaxation
ANS A
The first deflection is the P wave associated with right and left atrial depolarization. Wave of atrial
repolarization is invisible because of low amplitude.

128. Autopsy of a patient, who died of bilateral bronchopneumonia, shows in the left lung lower lobe
a cavity 5 cm in diameter, filled with liquid yellowish-white substance. What complication of the
patient’s pneumonia had developed?

A. Gangrene
B. Granuloma
C. Abscess
D. Sequestrum
E. Tuberculoma

ANS C
An abscess is a collection of pus. Pus is a thick fluid that usually contains white blood cells, dead tissue
and germs (bacteria). The pus may be yellow or green and may have a bad smell. The usual cause of
an abscess is an infection with bacteria

129. A patient, who has been subsisting exclusively on polished rice, has developed polyneuritis due
to thiamine deficiency. What substance is an indicator of such avitaminosis, when it is excreted with
urine?

A. Malate

B. Methylmalonic acid

C. Uric acid

D. Pyruvic acid
E. Phenyl pyruvate
ANS E

Vitamin B1 Deficiency and Beri Beri. Primary vitamin B1 (thiamine) deficiency is caused by a lack of
vitamin B1 in the diet, and is often seen in countries where the main diet is polished rice (white rice
with the outer layers removed) The excretion of pyruvic acid in urine increases in thiamine deficiency
and in beriberi

130. It is known that pentose-phosphate pathway actively functions in the erythrocytes. What is the
main function of this metabolic pathway in the erythrocytes?

A. Counteraction to lipid peroxidation

B. Activation of microsomal oxidation

C. Neutralization of xenobiotics

D. Oxidation of glucose into lactate

E. Increase of lipid peroxidation


ANS A

131. Pupil dilation occurs when a person steps from a light room into a dark one. What reflex causes
such a reaction?
A. Sympathetic conditioned reflex
B. Sympathetic unconditioned reflex
C. Metasympathetic reflex
D. Parasympathetic unconditioned reflex
E. Parasympathetic conditioned reflex

ANS B
Pupillary response is a physiological response that varies the size of the pupil, via the optic and
oculomotor cranial nerve. ... Dilation of the pupil occurs when the smooth cells of the radial muscle,
controlled by the sympathetic nervous system (SNS),and it is unconditional because is not under our
control.

132. A patient complaining of heartburn has undergone biopsy of the gastric mucosa. In the sample
there are numerous cells with oxyphilic cytoplasm in the glandular epithelium. Name these cells:

A. Exocrine parietal cells

B. Exocrine chief cells

C. Mucous cells

D. Epithelial cells
E. Endocrine cells
ANS A
Parietal cells are the epithelial cells that secrete hydrochloric acid (HCl) and intrinsic factor. These cells
are located in the gastric glands found in the lining of the fundus and in the body of the stomach.
The best-known component of gastric juice is hydrochloric acid, the secretory product of the parietal,
or oxyntic cell. It is known that the capacity of the stomach to secrete HCl is almost linearly related to
parietal cell numbers
too much parietal cell lead to high HCL that causes ulcer (Heartburn) is a painful burning feeling in
your chest or throat. It happens when stomach acid backs up into your esophagus

133. Inherited diseases, such as mucopolysaccharidoses, manifest in metabolic disorders of connective


tissue, bone and joint pathologies. The sign of this disease is the excessive urinary excretion of the
following substance:

A. Glycosaminoglycans
B. Amino acids

C. Glucose

D. Lipids

E. Urea
ANS A
Hunter syndrome, or mucopolysaccharidosis Type II, is a rare disease that it shows X-linked recessive
inheritance. Mucopolysaccharidoses are a different type of lysosomal storage disease in which the
substrates that accumulate in the lysosomes are extracellular matrix molecules called
glycosaminoglycans (which were previously known as mucopolysaccharides).
Diagnosis often can be made through clinical examination and urine tests (excess
mucopolysaccharides are excreted in the urine).

134. A woman was delivered to a gynecological unit with signs of acute abdomen and suspected
extrauterine pregnancy with oviduct rupture. Where will the blood accumulate in this case?
A. Vesicouterine pouch
B. Retrovesical pouch
C. Right lateral canal
D. Left lateral canal
E Rectouterine pouch
ANS E

The rectouterine pouch, also known as the rectovaginal pouch, cul-de-sac or pouch of Douglas, is an
extension of peritoneum between the posterior wall of uterus and the rectum in females. It is the
most dependent part of the peritoneal cavity and is analogous to the rectovesical pouch in males
Culdocentesis is a medical procedure involving the extraction of fluid from the pouch of Douglas
through a needle. It can be one diagnostic technique used in identifying pelvic inflammatory disease

135. A patient had a trauma that caused dysfunction of motor centers regulating activity of head
muscles. These centers can normally be located in the following area of the cerebral cortex:
A. Superior part of the precentral gyrus
B. Supramarginal gyrus
C. Inferior part of the precentral gyrus
D. Superior parietal lobule
E. Angular gyrus
ANS C

The precentral gyrus, which may also be called the primary motor area, the region that controls head
and face movement is found in the inferior precentral gyrus, hand and arm in the middle region of
the precentral gyrus and the leg actions are located in dorsalmedial area
136. At the post-mortem examination the stomach of a patient with renal failure was found to have a
yellow-brown coating on the thickened mucosa. The coating was firmly adhering to its surface and
had significant thickness. Microscopy revealed congestion and necrosis of mucosal and submucosal
layers, fibrin presence. What is the most likely diagnosis?
A. Diphtheritic gastritis
C. Phlegmonous gastritis
D. Catarrhal gastritis
E. Corrosive gastritis
ANS A

The dead tissue forms a thick, gray coating that can build up. This thick coating is called a
“pseudomembrane. Removal of the membrane reveals a bleeding cos is firmly adhering to its surface.
Pseudomembrane is A thick, gray, leathery pseudomembrane composed of a mixture of dead cells,
fibrin, RBCs, WBCs, and organism forms locally

137. Cystinuria manifests itself in a human as cystine stones in the kidneys (homozygous individuals)
or increased cystine content in the urine (heterozygous individuals). Cystinuria is a monogenic
disorder. Determine the type of interaction between the genes of cystinuria and normal urine cystine
excretion:

A. Epistasis
B. Complete dominance

C. Complementarity
D. Semidominance
E. Codominance

ANS D
Semi dominance is a form of intermediate inheritance in which one allele for a specific trait is not
completely expressed over its paired allele

138. A patient with obliterating atherosclerosis has undergone sympathectomy of the femoral artery
in the area of femoral triangle. What type of arterial hyperemia was developed in the patient due to
this surgery?

A. Reactive
B. Metabolic
C. Neurotonic
D. Functional
E. Neuroparalytic
ANS E

Neuroparalytic arterial hyperemia are caused by damage or blockage of α-adrenoreceptors


sympathetic nervous system. As usual tone of vessels is supplied by the impulses from sympathetic
nerves (1-3 impulses for a second) to smooth muscle cell of vascular wall.
• It is characterized by reduction or absence (paralysis) of the sympathetic nervous system effect on
the walls of the arteries

139. On bronchoscopy there is a polypoid growth 1,0 cm in diameter with ulcer in its center in the
upper lobe of the right lung. Histological investigation revealed a tumor composed of lymphocyte-like
cells with hyperchromic nuclei, the cells form layers and bands. What is the most likely tumor type?

A. Undifferentiated small cell carcinoma

B. Undifferentiated large cell carcinoma

C. Squamous cell carcinoma

D. Adenocarcinoma

E. Glandular squamous cell carcinoma


ANS A

140. The brain trauma unit received a patient with damaged greater wing of the sphenoid bone. The
fracture line crosses the spinous foramen of the sphenoid. What vessel was damaged?

A. Middle meningeal artery

B. Superficial temporal artery


C. Lateral pterygoid artery

D. Anterior deep temporal artery

E. Posterior deep temporal artery

ANS A

It ascends between the sphenomandibular ligament and the pterygoideus externus (lateral pterygoid
muscle), and between the two roots of the auriculotemporal nerve to the foramen spinosum of the
sphenoid bone, through which it enters the cranium; it then runs forward in a groove on the great
wing of the sphenoid bone, and divides into two branches, anterior and posterior

141. In one of Polessye regions there was an outbreak of helminthiasis manifested by cramps and
facial edemas. The developed preventive measures in particular included ban for eating infested pork
even after heat processing. What helminthiasis was the case?

A. Trichinosis
B. Taeniarhynchosis
C. Teniasis
D. Echinococcosis
E. Alveococcosis
ANS A
Trichinosis is infection caused by the roundworm Trichinella spiralis or another Trichinella species.
Symptoms include diarrhea, abdominal cramps, muscle pain, and fever. People acquire the infection
by eating raw or undercooked contaminated meat

142. A person with vitamin A deficiency develops twilight vision disturbance. Name the cells that
fulfill this photoreceptor function:

A. Rod cells

B. Horizontal cells of retina

C. Bipolar neurons

D. Cone cells
E. Ganglionic nerve cells
ANS A

Rods are rod like structures and are required for dim light (twilight/ night) vision. Both rods and cones
contain light sensitive pigments. Rod cells contain a purplish pigment known as visual purple or
rhodopsin and it is formed from vitamin A.

143. Presented is the biopsy material of an organ consisting of saccule-shaped rounded structures of
varying size. Inside these structures there is a gel-like non-cellular substance - colloid; structure walls
are composed of one layer of cuboidal cells that lay on the basement membrane. Between the
saccules there is connective tissue with vessels. Name this organ:

A. Pancreas
B. Thyroid gland
C. Parotid gland
D. Thymus
E. Parathyroid gland

ANS B
Microscopic structure of thyroid gland Structural unit is follicle or
gel-like
viscous iodine-rich material called colloid. Interfollicular spaces are filled by reticular connnective
tissue, adipose tissue and blood vessels.
144. A 26-year-old woman with bronchitis has been administered a broad spectrum antibiotic as a
causal treatment drug. Specify this drug:

A. Interferon
B. BCG vaccine
C. Ambroxol
D. Doxycycline
E. Dexamethasone

ANS D
Doxycycline is a broad-spectrum antibiotic of the tetracycline class.
From d question doxycycline is the only antibiotic in the option

145. Water affects the mucosa of lower nasal passages resulting in diving reflex. This responce
manifests itself as:
A. Reflex apnea
B. Reflex dyspnea
C. Reflex hyperpnea
D. Cough
E. Bronchial spasm
ANS A

An associated reflex is apnea--the instinct to hold the breath when immersed in water.
Water stimulation of the laryngeal chemoreflex result in apnea

146. Examination of a 56-year-old woman with a history of type 1 diabetes revealed a disorder of
protein metabolism that is manifested by aminoacidemia in the laboratory blood test values, and
clinically by the delayed wound healing and decreased synthesis of antibodies. Which of the following
mechanisms causes the development of aminoacidemia

A. Albuminosis
B. Decrease in concentration of blood amino acids
C. Increased proteolysis
D. Increase in plasma oncotic pressure
E. Increase in low-density lipoproteins level
ANS C

147. One of the factors that cause obesity is the inhibition of fatty acids oxidation due to:

A. Low carnitine content


B. Impaired phospholipid synthesis

C. Excessive consumption of fatty foods

D. Choline deficiency

E. Lack of carbohydrates in the diet

ANS A
The amino acid carnitine is required for the transport of long-chain fatty acyl coenzyme A (CoA) esters
into myocyte mitochondria, where they are oxidized for energy. Carnitine is obtained from foods,
particularly animal-based foods, and via endogenous synthesis.
low carnitine can lead to fat accumulation.
Raising your carnitine levels will fight this visceral fat gain because it increases fat burning

148. Streptomycin and other aminoglycosides prevent the joining of formylmethionyl-tRNA by


bonding with the 30S ribosomal subunit. This effect leads to disruption of the following process:

A. Translation initiation in eucaryotes


B. Translation initiation in procaryotes
C. Transcription initiation in procaryotes
D. Transcription initiation in eucaryotes
E. Replication initiation in procaryotes

ANS B
N-formyl-methionyl-tRNA (tRNAfMet) is the aminoacyl-tRNA that initiates bacterial protein synthesis
aminoglycosides are known to target rRNA in prokaryotes prevent the joining of formylmethionyl-
tRNA by bonding with the 30S ribosomal subunit and thereby alter protein translation

149. In hot weather ventilators are often used to normalize the microclimate in the heated rooms. It
leads to intensified heat transfer from the human body by means of:
A. Convection
B. Conduction and convection
C. Conduction
D. Radiation
E. Evaporation
Convection is heat transfer by mass motion of a fluid such as air or water when the heated fluid is
caused to move away from the source of heat, carrying energy with it. Convection above a hot surface
occurs because hot air expands, becomes less dense, and rises (see Ideal Gas Law)

150. During gastric resection the patient received mixed anesthesia with tubocurarin chloride muscle
relaxant; to restore spontaneous respiration the patient received proserin. What pharmacological
group does this drug belong to?

A. Angiotensin-converting-enzyme inhibitors
B. Calcium channel blockers
C. Muscarinic antagonists
D. Muscarinic agonists
E. Cholinesterase inhibitors
ANS E

Proserine. A cholinesterase inhibitor used in the treatment of Myasthenia Gravis and to reverse the
effects of Muscle relaxants such as gallamine and Tubocurarine

151. A 67-year-old man consumes eggs, pork fat, butter, milk and meat. Blood test results: cholesterol
- 12,3 mmol/l, total lipids - 8,2 g/l, increased low-density lipoprotein fraction (LDL). What type of
hyperlipoproteinemia is observed in the patient?
A. Hyperlipoproteinemia type I
B. Hyperlipoproteinemia type IIb
C. Hyperlipoproteinemia type IIa
D. Hyperlipoproteinemia type IV
E. Cholesterol, hyperlipoproteinemia
152. To treat bronchitis the patient was prescribed a beta-lactam antibiotic. Its mechanism of action is
based on inhibition of murein production, which results in death of the causative agent. Name this
drug:

C. Bijochinol (Quinine bismuth iodide)


B. Penicillin G Sodium Salt
C. Ciprofloxacin
D. Azithromycin
E. Streptomycin

ANSB
Penicillin belongs to the beta-lactam family of antibiotics, the members of which use a similar
mechanism of action to inhibit bacterial cell growth, Lactam antibiotics inhibit the formation of
peptidoglycan cross-links in the bacterial cell wall
Peptidoglycan, also called murein, is a polymer that makes up the cell wall of most bacteria

153. Cell membrane rest potential changed from -85 to -90 mV. It can be caused by activation of the
following cell membrane channels:

A. Potassium

B. Sodium

C. Potassium and sodium

D. Calcium

E. Potassium and calcium

ANS A

the resting membrane potential would be determined by the K+ concentration and be equal to the
equilibrium potential for K+ ions (approx. : - 90 mV)
154. Ionizing radiation or vitamin E deficiency affect the cell by increasing lysosome membrane
permeability. What are the possible consequences of this pathology?

A. Partial or complete cell destruction

B. Intensive protein synthesis

C. Intensive energy production

D. Restoration of cytoplasmic membrane

E. Formation of maturation spindle


ANS A
Ionizing radiation or vitamin E deficiency affect the cell by increasing lysosomal membranes
permeability which leads to cell lysis Partial or complete cell destruction

155. A 22-year-old woman ate some seafood. 5 hours later her torso and distal parts of her limbs
developed small itchy papules which were partially fused together. One day later the rash
disappeared spontaneously. Specify the hypersensitivity mechanism underlying these changes:

A. Systemic anaphylaxis
B. Atopy (local anaphylaxis)
C. Cellular cytotoxicity
D. Immune complex hypersensitivity
E. Antibody-dependent cell-mediated cytolysis
ANS B
In type 1 hypersensitivity, B-cells are stimulated (by CD4+TH2 cells) to produce IgE antibodies specific
to an antigen. The immediate hypersensitivity reaction occurs minutes after exposure and includes
release of vasoactive amines(histamine) and lipid mediator. Histamine cause bronchoconstriction,
mucus secretion, vasodilatation, vascular permeability

156. During acute hemorrhage the body loses not only fluid but also electrolytes. What substance
solution can be used as a simple blood substitute?

A. Sodium chloride
B. Sodium bromide
C. Albumin
D. Sodium nucleotide
E. Calcium chloride
ANS A

The electrolytes —sodium, chloride, and potassium—are the same electrolytes found within the
blood. which is highly water-soluble. This means it can easily bond with water molecules in your body,
leading to an increase in blood volume

157. A patient has been admitted to the contagious isolation ward with signs of jaundice caused by
hepatitis virus. Which of the symptoms given below is strictly specific for hepatocellular jaundice?

A. Hyperbilirubinemia
B. Increase of ALT, AST level
C. Bilirubinuria
D. Cholemia
E. Urobilinuria
ANS A
The ALT and AST tests measure enzymes that your liver releases in response to damage or disease.

158. In a body of a 37-year-old woman, who died with signs of pulmonary edema, there was detected
acute deformation of the aortic valve: it is shortened, thickened, ulcerated, has areas of stone-like
density. On its external surface there are large, up to 2 cm in diameter, thrombotic plaques. Left
ventricle wall is 2,2 cm thick. Cardiac muscle is dull, matt, and flaccid. What type of endocarditis
corresponds with described alterations of the aortic valve?

A. Ulcerative polypoid endocarditis


B. Diffuse endocarditis
C. Acute verrucous endocarditis
D. Recurrent verrucous endocarditis
E. Fibroplastic endocarditis
Ans A

159. Students study the stages of gametogenesis. They analyze a cell with haploid number of
chromosomes, with each chromosome consisting of two chromatids. The chromosomes are located in
the equatorial plane of the cell. Such situation is typical of the following stage of meiosis:

A. Metaphase of the first division


B. Metaphase of the second division
C. Anaphase of the first division
D. Anaphase of the second division
E. Prophase of the first division
ANS B
ln metaphase II, the second stage of meiosis II, in each of the two daughter cells produced by the first
meiotic division (which are known as secondary germ cells), the spindle again draws the
chromosomes to the metaphase plate

160. A 38-year-old woman developed a bronchial asthma attack. Which of the listed bronchial
spasmolytics is effective for emergency aid and belongs to beta-2adrenergic agonists?
A. Salbutamol
B. Adrenaline
C. Ipratropium bromide
D. Platyphyllin
E. Atropine
ANS A

Salbutamol is a short-acting, selective beta2-adrenergic receptor agonist used in the treatment of


asthma and COPD

161. After emotional upset a woman has been suffering from disturbed sleep for several days. What
soporific drug would be preferable for this type of insomnia?

A. Nitrazepam

B. Phenobarbital

C. Ethaminal sodium (Pentobarbital)

D. Barbamylum (Amobarbital)

E. Chloral hydrate
ANS A

Nitrazepam belongs to the class of medications called benzodiazepines. It is used for short-term
treatment of sleeping problems (insomnia), such as difficulty falling asleep, frequent awakenings
during the night, and early-morning awakening

162. During training session in the laboratory the students were performing spirography on
themselves. What indicator CANNOT be measured with this method?

A. Functional residual capacity

B. Vital capacity

C. Respiratory minute volume

D. Respiration rate

E. Maximal breathing capacity


ANS A

Functional Residual Capacity (FRC) is the volume of air present in the lungs at the end of passive
expiration
residual volume is a predicted value because it is the amount of air left in lungs after a maximum
exhale. The spirometer can't measure inside the lungs.

163. In the South and Central America there can be found a species of trypanosomes that is the
causative agent of Chagas disease. What animal is the infection carrier specific to this disease?

B. Cockroach
C. Tsetse fly
A. Triatomine bug
D. Mosquito
E. Gnat
ANS A

Chagas disease, also known as American trypanosomiasis, is a tropical parasitic disease caused by the
protist Trypanosoma cruzi.
It is spread mostly by insects known as Triatominae, or "kissing bugs

164. A 54-year-old woman was brought to the emergency department after a car accident. A
traumatologist diagnosed her with multiple fractures of the lower extremities. What kind of embolism
is the most likely to develop in this case?
A. Tissue
B. Thromboembolism
C. Gaseous
D. Adipose
E. Air

ANS D
A fat embolism occurs after major trauma, it is a type of embolism in which the embolus consists of
fatty material. They are often caused by physical trauma such as fracture of soft tissue trauma, and
burns.

165. A 30-year-old man with diabetes mellitus type I was hospitalized. The patient is comatose.
Laboratory tests revealed hyperglycemia and ketonemia. What metabolic disorder can be detected in
this patient?

A. Metabolic alkalosis
B. Metabolic acidosis
C. Respiratory acidosis
D. Respiratory alkalosis
E. Acid-base balance is normal

ANS B
Metabolic acidosis is a clinical disturbance characterized by an increase in plasma acidity.
Diabetic ketoacidosis is an acute metabolic complication of diabetes characterized by hyperglycemia,
hyperketonemia, and metabolic acidosis

166. Typical manifestations of food poisoning caused by C. botulinum are double vision, abnormal
functioning of the swallowing and breathing. These symptoms develop as the result of:

A. Exotoxin action

B. Enterotoxin action

C. Enterotoxic shock development

D. Activation of adenylate cyclase

E. Pathogen adhesion to the enterocyte receptors

ANS A
Botulism is a serious illness that causes flaccid paralysis of muscles. It is caused by a neurotoxin,
generically called botulinum toxin, produced by the bacterium Clostridium botulinum
botulinum toxin cause dangerous swallowing and breathing problems

167. An infant, who was on synthetic formula feeding, developed signs of vitamin B 1 deficiency. What
reactions does this vitamin take part in?

A. Amino acids transamination


B. Keto acids oxidative decarboxylation
C. Amino acids decarboxylation
D. Proline hydroxylation
E. Redox reactions

Vitamin B1, as the coenzyme thiamin pyrophosphate, is involved in the oxidative decarboxylation of
α-ketoacids, and functions in transketolase reactions. Thiamin is critical for decarboxylation of
pyruvate in preparation for entry into the tricarboxylic acid cycle

168. A 30-year-old woman developed the signs of virilism (body hair growth, balding temples,
disturbed menstrual cycle). What hormone can cause this condition when hyperproduced?

A. Testosterone

B. Estriol

C. Relaxin

D. Oxytocin

E. Prolactin
ANS A

Women with virilization often have an imbalance in sex hormones, such as estrogen and male sex
hormones, or androgens, like testosterone. An overproduction of androgens can cause virilization.
Androgens are primarily produced by the adrenal glands, which are present in both females and
males, and the testicles.

169. A histological specimen shows significant amount of mucous connective tissue (Wharton’s jelly),
vessels, as well as residual yolk and allantois. Name this organ:

A. Umbilical cord

B. Esophagus
C. Ureter

D. Urethra

E. Vermiform appendix

ANS A
Wharton's jelly is a gelatinous substance within the umbilical cord

170. On examination of a 6-year-old child the doctor noticed grayish film on the child’s tonsils.
Microscopy of the smears stained by Neisser method detected there Corynebacterium diphtheriae.
What morphologic feature was the most indicative for determining the type of the agent?

A. Localization of the causative agent within macrophages


B. Polar placement of volutin granules
C. Spores that exceed cells in diameter
D. Fence-like position of the agent’s cells
E. Presence of the capsule
ANS B

Volutin granules is used as one of the identifying criteria when attempting to isolate Corynebacterium
diphtheriae on Löffler's medium
171. During the sports competition a boxer received a strong blow to the abdomen, which caused a
knockout due to a brief drop in blood pressure. What physiological mechanisms are the cause of this
condition?

A. Stimulation of parasympathetic nerves

B. Alteration of transcapillary exchange

C. Ischemia of the central nervous system

D. Abrupt change in body fluid volume

E. Stimulation of sympathetic nerves

ANS A
The vagus nerve (parasympathetic) is the longest cranial nerve. It contains motor and sensory fibers
and, because it passes through the neck and thorax to the abdomen.
the process is called a vasovagal response. You can either stimulate the vagus nerve directly by
physical contact (punching them in the right place), or by the sheer stress brought on by the pain
resulting from the punch.
The vagus nerve stimulates the parasympathetic nervous system and inhibits the sympathetic nervous
system. This combination can lead to a drastic decrease in heart rate (negative chronotropy),
heart contraction (negative inotropy), and/or blood pressure which leads to syncope
(fainting).

172. After a severe stress the patient presents with eosinopenia in the blood test. In this case the
decreased number of eosinophils can explain changes in the level of the following hormones:

A. Adrenaline
B. Glucocorticoids
C. Insulin
D. Mineralocorticoids
E. Vasopressin

ANS B

Eosinophilia =increase number of eosinophil


Eosinopenia =decrease number of eosinophil

Eosinophilia is encountered only with specific infectious diseases. With active bacterial or viral
infections, eosinopenia is characteristic. This suppression of blood eosinophils is due in part to
heightened endogenous corticosteroid production as well as to inflammatory mediators released
during these infections.

173. A 30-year-old patient’s blood test revealed the following: erythrocyte count is 6 · 10 12 /l,
hemoglobin is 10,55 mmol/l. Vaquez’s disease was diagnosed. Name the leading part of pathogenesis
in this case:

A. Neoplastic erythroid hyperplasia

B. Iron-deficiency

C. B 12 -deficiency

D. Hypoxia

E. Acidosis
ANS A

Polycythemia Vera (VAQUEZ DISEASE) A myeloproliferative disorder of unknown etiology,


characterized by abnormal proliferation of all hematopoietic bone marrow elements and an absolute
increase in red cell mass and total blood volume, associated frequently with splenomegaly,
leukocytosis, and thrombocythemia

174. Deaf parents with genotypes DDee and ddEE gave birth to a child with normal hearing. Specify
the interaction of D and E genes:
A. Complete dominance
B. Epistasis
C. Polymery
D. Overdominance
E. Complementary interaction
ANS E
A pair of genes can often work together to create a specific phenotype, We call this complementary
interaction. With this type of interaction we see 2 different phenotypes instead of the 4 seen in 2
genes 1 phenotype.

175. Corticosteroid hormones regulate the adaptation processes of the body as a whole to
environmental changes and ensure the maintenance of internal homeostasis. What hormone
activates the hypothalamopituitary-adrenal axis?
A. Corticoliberin
B. Somatoliberin
C. Somatostatin
D. Corticostatin
E. Thyroliberin
ANS A
Corticoliberin Hormone regulating the release of corticotropin from pituitary gland

176. A patient with signs of emotional lability that result in troubled sleep has been prescribed
nitrazepam. Specify the sleep-inducing mechanism of this drug:

A. Blockade of opiate receptors


B. Inhibition of stimulating amino acids
C. H 1 -histamine receptors stimulation
D. GABA-ergic system activation
E. Supression of serotonergic neurotransmission

ANS D

nitrazepam act on benzodiazepine receptors in the brain which are associated with the GABA
receptors, causing an enhanced binding of GABA to GABA receptors

177. A 50-year-old inpatient during examination presents with glucosuria and blood glucose of 3,0
mmol/l, which are the most likely to be caused by:

A. Diabetes insipidus
B. Myxedema
C. Renal disorder
D. Essential hypertension
E. Pellagra
ANS C
In Renal disorder glucose is abnormally eliminated in the urine due to improper functioning of the
renal tubules, which are primary components of the filtering units of the kidneys (nephrons).

178. A man is suffering from diarrhea. summer he spent his vacation in the south at the sea coast.
Bacteria with the following properties were detected in his feces: gram negative curved mobile
monotrichous bacilli that do not produce spores or capsules. They are undemanding to nutrient
medium but require alkaline reaction (рН - 8,5-9,5). Described are the agents of the following enteric
infection:

A. Shigellosis
B. Cholera
C. Typhoid fever
D. Colienteritis
E. Pseudotuberculosis

ANS B
V. cholerae is Gram-negative and comma-shaped. Initial isolates are slightly curved. The bacterium
has a flagellum at one cell pole as well as pili.it causes Symptoms that include abrupt onset of watery
diarrhea (a grey and cloudy liquid), occasional vomiting, and abdominal cramps
Vibrio cholerae, the pathogenic agent of cholera, grows best at the slightly basic pH of 8.0; it can
survive pH values of 11.0 but is inactivated by the acid of the stomach

179. A pregnant woman was detected to have IgM to rubella virus. An obstetrician-gynecologist
recommended therapeutic abortion due to the high risk of teratogenic affection of the fetus.
Detection of IgM was of great importance as it is these specific immunoglobulins that:

A. Indicate recent infection


B. Penetrate placental barrier
C. Have the largest molecular weight
D. Are associated with anaphylactic reactions
E. Are the main factor of antiviral protection

ANS A

mmunoglobulin G (IgG), the most abundant type of antibody, is found in all body fluids and protects
against bacterial and viral infections.it indicates chronic infection
Immunoglobulin M (IgM), which is found mainly in the blood and lymph fluid, is the first/
ACUTE/recent antibody to be made by the body to fight a new infection

180. During examination of a teenager with xanthomatosis the family history of hypercholesterolemia
is revealed. What transportable lipids are increased in concentration in case of such a disease?

A. Chylomicrons
B. Very low-density lipoproteins
C. High-density lipoproteins
D. Intermediate-density lipoproteins
E. Low-density lipoproteins
ANS E

Xanthoma is due to deposition of cholestrol in the tissues .


- familial hypercholesterolemia is due to defect in LDL receptor gene which code for LDL(Low-density
lipoproteins) receptors that bind LDL ( lipoprotein that transfer cholesterol ester , no triglycerides ) ,
so the main pathology of this case is due to high Low-density lipoproteins, LDL will deposit in eyelid (
xanthelasma ) , deposit in achilles tendon ( Tendon xanthoma ) , deposit in the margin of the cornea (
arcus senilis ) , high LDL level will also cause premture atherosclerosis and MI

181. On examination the patient is found to have low production of adrenocorticotropic hormone.
How would this affect production of the other hormones?

A. Decrease adrenocorticotropic hormones synthesis

B. Decrease hormone synthesis in the adrenal medulla

C. Decrease insulin synthesis

D. Increase sex hormones synthesis

E. Increase thyroid hormones synthesis

ANS A

182. Parkinson’s disease is caused by disturbance of dopamine synthesis. What brain structure
synthesizes this neurotransmitter?

A. Substantia nigra

B. Globus pallidus

C. Corpora quadrigemina

D. Red nuclei

E. Hypothalamus

Ans A
Parkinson's disease (PD) is a neurodegenerative disorder that affects predominately dopamine-
producing (“dopaminergic”) neurons in a specific area of the brain called substantia nigra. Symptoms
generally develop slowly over years.

183. Determining a patient’s blood group with monoclonal test-reagents revealed positive
agglutination reaction to anti-A and anti-B reagents, and negative reaction to anti-D. What blood
group does this patient have?

A. II (А) Rh (+)
B. III (В) Rh (-)
C. IV (АВ) Rh (-)
D. IV (АВ) Rh (+)
E. I (0) Rh (+)

ANS C
NEGATIVE REACTION TO ANTI D INDICATES RHESUS NEGATIVE(RH-) OPTION A
POSITIVE REACTION TO ANTI D INDICATES RHESUS POSITIVE (RH+) OPTION B TO E

184. A patient visited a dentist to extract a tooth. After the tooth had been extracted, bleeding from
the tooth socket continued for 15 minutes. Anamnesis states that the patient suffers from active
chronic hepatitis. What phenomenon can extend the time of hemorrhage?

A. Decrease of fibrinogen content in blood

B. Thrombocytopenia

C. Hypocalcemia

D. Increased activity of anticoagulation system

E. Decrease of albumine content in blood


Ans A

Fibrinogen is made and secreted into the blood primarily by liver hepatocyte cells
Hemostasis is intimately related to liver function, because most coagulation factors are synthesized by
liver parenchymal cells. The extent of coagulation abnormalities depends upon the degree of
disturbed liver function. Acute or chronic hepatocellular diseases may display decreases in the vitamin
K-dependent factors (prothrombin; factors VII, IX, and X; proteins C and S), Patients with hepatitis
may present with the entire spectrum of factor deficiencies and may even develop disseminated
intravascular coagulation (DIC) which cause continuous bleeding

185. During ascent into mountains a person develops increased respiration rate and rapid heart rate.
What is the cause of these changes?

A. Decrease of O 2 partial pressure

B. Increase of CO 2 partial pressure

C. Increase of blood pH
D. Increase of nitrogen content in air

E. Increase of air humidity


ANS A

As you go up a mountain, the air becomes less compressed and is therefore thinner. the pressure is
lower, you’re more likely experiencing altitude sickness. the thinner air means there is less oxygen to
breathe so the body tries to compensate by increasing respiratory rate and rapid heart rate. if you
stay in high mountain for a long time your body compensate by producing more red blood
cells(polycythemia)

186. To stop the bleeding the patient was prescribed a direct coagulant. During introduction of the
solution the patient was complaining of pain along the vein, hot sensation, and palpitations. Name
the drug that causes such symptoms:

A. Calcium chloride

B. Hirudine

C. Ergocalciferol

D. Pentoxyl

E. Streptokinase

ANS A

Rapid I.V. Injections of Calcium Chloride are accompanied by peripheral vasodilatation, the patient
may complain of tingling local “burning” sensation and there may be a moderate fall in blood
pressure.

187. A student, whose educational achievements throughout the semester were poor, feels
emotionally tense during the final test. What is the primary cause that induced leading mechanism of
emotional tension in this case

A. Lack of information

B. Tight time

C. Tight time and lack of energy

D. Lack of energy
E. Lack of energy and information

Ans A

188. A young family came for a genetic counseling to identify the father of their child. The husband
insists that the child does not resemble him at all and cannot possibly be his. Polymerase chain
reaction method for person identification is based on the following:

A. Nucleotide deletion
B. Gene amplification
C. Genetic recombination
D. Missense mutation
E. Transduction

ANS B

Gene amplification may refer to: Polymerase chain reaction, a synthetic gene amplification use to
determine whether two individuals are biologically parent and child. A paternity test establishes
genetic proof whether a man is the biological father of an individual, and a maternity test establishes
whether a woman is the biological mother of an individual. Tests can also determine the likelihood of
someone being a biological grandparent to a grandchild

189. A 52-year-old man presents with fever and pain in the joints. Both of his first
metatarsophalangeal articulations are deformed, swollen, and reddened. Blood urea is high. The
patient is diagnosed with gout. What is the main developmental factor in the pathogenesis of this
disease?

A. Argininosuccinic aciduria
B. Hyperuricemy
C. Hyperazotemia
D. Hyperaminoacidemia
E. Citrullinuria

ANS B

Gout is a form of arthritis in which excess uric acid forms crystals in joints and other tissues causing
painful inflammation.
the root cause of gout is hyperuricemia and it is characterized by recurrent attacks of acute
inflammatory arthritis
190. A 67-year-old man was delivered to the cardiology unit with complaints of periodical pain in the
heart, dyspnea after even insignificant physical exertion, cyanosis, and edemas. ECG revealed
additional contractions of the heart ventricles. Name this type of rhythm disturbance:

A. Bradycardia
B. Extrasystole
C. Tachycardia
D. Flutter
E. Fibrillation
ANS B
Extrasystole is also known as extra systole, premature beat, premature contraction, premature
systole, premature ventricular beat or PVB, premature ventricular contraction

191. During narcosis the patient developed a risk of cerebral edema. What drug should be
administered in this case?

A. Furosemide
B. Dopamine
C. Phenazepam
D. Triamterene
E. Sodium bromide
ANS A

Diuretics (Furosemide) Mechanism of action


• Highly potent loop diuretics, inhibits Na+-K+-Cl- symport in Loop of Henle of nephrons
Pharmacological action
• diuresis action + venodilation action + decrease in LV filling pressure
• Can cause salt and water excretion & reduction of preload
• benefits patients with pulmonary edema even before diuresis ensues Route of administration:
available as oral as well as IV.

192. A patient with pulmonary tuberculosis is prescribed the most effective anti-tuberculous
antibiotic. Name this drug:

A. Tetracycline
B. Streptocide
C. Furasolidone
D. Bactrim (Co-trimoxazole)
E. Rifampicin
ANS E
Antituberculosis agents: isoniazid, rifampin, streptomycin, ethambutol. Effective antituberculosis
drugs have radically improved the prognosis of the patient with active tuberculosis
193. During surgery performed in the abdominal cavity a surgeon located ligament of liver stretching
from anterior abdominal wall (navel) to inferior surface of liver. What ligament is it?

A. Round ligament of the liver


B. Falciform ligament of the liver
C. Coronary ligament of the liver
D. Venous ligament of the liver
E. Triangular ligament of the liver

ANS A
the round ligament of the liver (or ligamentum teres, or ligamentum teres hepatis) is a degenerative
string of tissue that exists in the free edge of the falciform ligament of the liver. The round ligament
divides the left part of the liver into medial and lateral sections. The round ligament represents the
remnant of the fetal umbilical vein. The umbilical vein/round ligament inserts around the umbilicus
and is an important landmark of the inner surface of the anterior abdominal wall.

194. On examination of a patient with disease onset 5 days ago the doctor suspected tularemia and
prescribed the patient tularin intracutaneously. What is the purpose of this drug administration in the
patient?

A. Prevention
B. Treatment
C. Allergy diagnostics
D. Treatment evaluation
E. Prognosis for the disease

ANS C
At diagnosis of a tularemia apply an intracutaneous test with tularin.
diagnostic methods of allergic diseases and diseases with an allergic component in a pathogeny.
Tests are based on local or general reaction of a sensibilized organism in response to administration of
specific allergen.

195. A patient developed increased blood content of HCO3 against the background of repeated and
uncontrollable vomiting. What will be the leading mechanism in compensation of developed acid-
base imbalance?

A. Increased pulmonary ventilation


B. Increased renal reabsorption of bicarbonate
C. Decreased pulmonary ventilation
D. Increased renal reabsorption of ammonia
E. -
ANS C

196. A 13-year-old boy presents with eczematous rashes on his shins and torso. Anamnesis states
cases of otitis, pneumonia, and furuncles in the patient. Blood test: platelets - 70 · 10 9 /l, low activity
of T helper and T suppressor cells, low IgM, with normal IgA and IgG. What immunodeficient disease
does this boy have?

A. Wiskott-Aldrich syndrome

B. Louis-Bar syndrome (Ataxia-telangiectasia)

C. Severe combined immunodeficiency (Swiss type)

D. DiGeorge syndrome

E. Chediak-Higashi syndrome

ANS A

Wiskott-Aldrich syndrome MnemoniC

Wiskott- Aldrich Syndrome = W-A-S Triad

W- Weeping skin lesions- eczema


A- Absent platelets- thrombocytopenia - leading to petichiae
S- Severe immunodeficiency...... (low IgM and high IgA & IgE levels

197. During the exam a student was unable to correctly answer all the questions in his question card,
which was accompanied by the reddening of his face and hot sensation. What type of arterial
hyperemia did the student develop in this case?

A. Metabolic
B. Neurotonic
C. Postischemic

D. Pathologic

E. Neuroparalytic
ANS B
- Neurotonic mechanism. It is in the predominance of the effects of parasympathetic nervous
influences (in comparison with sympathetic) on the walls of arterial vessels.
Causes: activation of sympathetic influences on tissues and organs (for example, in various types of
stress, action on low temperature tissue, mechanical trauma, chemicals)
- Neuroparalytic mechanism. Characterized by a decrease or absence ("paralysis") of sympathetic
nerve effects on the walls of arteries and arterioles.

198. An injured person with wound of the anterior cervical region presents with hemorrhage. The
outflowing blood is dark. What vessel is damaged?

A. V. jugularis anterior

B. V. jugularis externa

C. V. jugularis interna

D. A. carotis externa

E. A. thyroidea superior
ANS A
199. Due to trauma the patient presents with disturbed function of the parotid gland. What nerve
ensures its secretion function?

A. N. petrosus major
B. N. petrosus profundus
C. N. auricularis minor
D. N. petrosus minor
E N. auricularis major
ANS D

The lesser petrosal nerve (also known as the small superficial petrosal nerve) is the General visceral
efferent component of the glossopharyngeal nerve (CN IX), carrying parasympathetic pre-ganglionic
fibers from the tympanic plexus to the parotid gland

200. A patient developed pyo inflammatory process of periodontal tissues caused by activation of the
microorganisms inherent in the body, which are a part of oral mucosal microflora. What type of
infection is it?

A. Exogenous infection
B. Reinfection
C. Superinfection
D. Relapse
E. Autoinfection

ANS E
Autoinfection reinfection by microbes or parasitic organisms that are present on or within the body.
Self-infection by direct contact with a contagious agent, as with parasite eggs in the infectious state
transmitted by fingernails also called autoreinfection

CREDITS: This was solved by Obuna Lilian

You might also like

pFad - Phonifier reborn

Pfad - The Proxy pFad of © 2024 Garber Painting. All rights reserved.

Note: This service is not intended for secure transactions such as banking, social media, email, or purchasing. Use at your own risk. We assume no liability whatsoever for broken pages.


Alternative Proxies:

Alternative Proxy

pFad Proxy

pFad v3 Proxy

pFad v4 Proxy